cost accounting question icwai inter

752
;kn esa ijefirk ijes'oj f'ko dh tks bl lf"V dk jfp;srk Hkh gS vkSj ikyugkj Hkh lefiZr vius ekrk firk ds pj.kksa esa ftUgksaus eq>s fl[kk;k fd pkykdh vkSj >wB cksyus ls vktrd dksbZ cM+k dke ugha gqvk vkSj vxj gks Hkh x;k rks ckn esa d"V] i'pkrki vkSj vieku dk dkj.k cuk- eka ljLorh ds pj.kksa esa lknj ueu ds lkFk eSa ;g iqLrd izkjaHk djrk gwa ;g fy[kh rks ml ek= ek/;e eq>s cuk;k gSA Problems and Solutions on Cost and Management Accounting By Anand Kumar Shrivastava B.E.(Civil) , AICWA How to get the best out of this book ? 1 2 3 4 Dedicated to the two most beautiful females in my life, my mom and daughter mini. It is only because of them that I survive today. I remember Manjula Bhabhi and Satish Bhaiya who gave me shelter duri days of desert, destitute and distress. 388 solved problems on 16 Topics of Cost and management accounting in Intermediate and Final Examinations conducted by ICWAI and ICAI Develop a deep religious outlook towards studies. Thank God for He given you opportunity, capability, facility and capacity to study. of those who just don't have the chance to read and write. Thank H selecting you for granting this privilege of being in the proximit Thank your parents and guardians for financing your studies. Thank teachers for providing guidance and assistance. Be liberal and fre in granting the credit of your success to your parents, guardians Read a good book on theory before you begin to solve the numerical problems from this book. Refer to theory portion quite often. Stud materials supplied by the Institute cover the theory portion Read the question twice. This is a good habit. In second reading y should skip the irrelevant narrative portion of the question and m

description

cost accounting question icwai inter and final group must try all.................best of luck

Transcript of cost accounting question icwai inter

Page 1: cost accounting question icwai inter

;kn esa ijefirk ijes'oj f'ko dh

tks bl lf"V dk jfp;srk Hkh gSvkSj ikyugkj Hkh

lefiZrvius ekrk firk ds pj.kksa esa

ftUgksaus eq>s fl[kk;k fdpkykdh vkSj >wB cksyus ls vktrd dksbZ cM+k dke ugha gqvkvkSj vxj gks Hkh x;k rksckn esa d"V] i'pkrki vkSj vieku dk dkj.k cuk-

eka ljLorh ds pj.kksa esa lknj ueu ds lkFkeSa ;g iqLrd izkjaHk djrk gwa ;g fy[kh rks mlh us gh gS ek= ek/;e eq>s cuk;k gSA

Problems and Solutions onCost and Management AccountingBy Anand Kumar Shrivastava B.E.(Civil) , AICWA

How to get the best out of this book ?

1

2

3

4

5

Dedicated to the two most beautiful females in my life, my mom and my daughter mini. It is only because of them that I survive today. I remember Manjula Bhabhi and Satish Bhaiya who gave me shelter during my days of desert, destitute and distress.

388 solved problems on 16 Topics of Cost and management accounting, asked in Intermediate and Final Examinations conducted by ICWAI and ICAI.

Develop a deep religious outlook towards studies. Thank God for He has given you opportunity, capability, facility and capacity to study. Think of those who just don't have the chance to read and write. Thank Him for selecting you for granting this privilege of being in the proximity of books.

Thank your parents and guardians for financing your studies. Thank your teachers for providing guidance and assistance. Be liberal and frequent in granting the credit of your success to your parents, guardians and teachers.

Read a good book on theory before you begin to solve the numerical problems from this book. Refer to theory portion quite often. Study materials supplied by the Institute cover the theory portion sufficiently.

Read the question twice. This is a good habit. In second reading you should skip the irrelevant narrative portion of the question and make some jottings of relevant portion.

If you are a bit confused about how and where to begin to frame the solution, begin with making working notes.

Page 2: cost accounting question icwai inter

6

7 If you want to learn thoroughly any topic or subject, the best way is to explain it to others.

8 Learn to use calculator with left hand. This keeps your brain more active and attentive.

9

10

11 If you think education is expensive and hard, try ignorance.

12

After reading the question, pause, ponder over and attempt to solve the question. Do not go to the solution immediately after reading the question. Make your own solution whatever it may be. Compare it with the solution provided in the book. Notice the difference and learn from it.

Learn to study by minutes than by hours. Small intervals of time in routine activities may be utilised for study purposes. Bathroom occupied, breakfast preparation taking a bit longer or waiting for a friend, a call or car are small lapses of time which go unnoticed. Use them for study purposes.

Make ample working notes. Any rough calculations may be transformed into working notes. Working notes carry marks in examination.

It is natural to ask why this book does not cover the theoretical aspects of the topics. The answer is simple. The study materials supplied by the institute to the students cover the theory portion in lucidly explanatory manner. The study materials however, lack in sufficiency and variety in respect of number of solved problems, hence the need and importance of this book.

Page 3: cost accounting question icwai inter

eSa ;g iqLrd izkjaHk djrk gwa ;g fy[kh rks mlh us gh gS

Dedicated to the two most beautiful females in my life, my mom and my daughter mini. It is only because of them that I survive today. I remember Manjula Bhabhi and Satish Bhaiya who gave me shelter during my days of desert, destitute and distress.

388 solved problems on 16 Topics of Cost and management accounting, asked in Intermediate and Final Examinations conducted by ICWAI and ICAI.

Develop a deep religious outlook towards studies. Thank God for He has given you opportunity, capability, facility and capacity to study. Think of those who just don't have the chance to read and write. Thank Him for selecting you for granting this privilege of being in

Thank your parents and guardians for financing your studies. Thank your teachers for providing guidance and assistance. Be liberal and frequent in granting the credit of your

Read a good book on theory before you begin to solve the numerical problems from this book. Refer to theory portion quite often. Study materials supplied by the Institute cover the

Read the question twice. This is a good habit. In second reading you should skip the irrelevant narrative portion of the question and make some jottings of relevant portion.

If you are a bit confused about how and where to begin to frame the solution, begin with

Page 4: cost accounting question icwai inter

If you want to learn thoroughly any topic or subject, the best way is to explain it to others.

Learn to use calculator with left hand. This keeps your brain more active and attentive.

If you think education is expensive and hard, try ignorance.

After reading the question, pause, ponder over and attempt to solve the question. Do not go to the solution immediately after reading the question. Make your own solution whatever it may be. Compare it with the solution provided in the book. Notice the difference and learn

Learn to study by minutes than by hours. Small intervals of time in routine activities may be utilised for study purposes. Bathroom occupied, breakfast preparation taking a bit longer or waiting for a friend, a call or car are small lapses of time which go unnoticed. Use them for

Make ample working notes. Any rough calculations may be transformed into working notes.

It is natural to ask why this book does not cover the theoretical aspects of the topics. The answer is simple. The study materials supplied by the institute to the students cover the theory portion in lucidly explanatory manner. The study materials however, lack in sufficiency and variety in respect of number of solved problems, hence the need and

Page 5: cost accounting question icwai inter

Chapter Heading cwa inter cwa final ca inter1 Material control and EOQ 11 11 72 Labour 16 15 113 Overheads 15 0 114 Job, Batch and Contract Costing 6 0 55 Process Costing 9 0 126 Joints products and bye products 4 2 97 Reconciliation of Cost and financial accounts 4 0 28 Operating, service and relevant costing 3 7 09 Marginal Costing 32 9 1

10 Cost, Volume and Profit Analysis 15 11 111 Budgets and Budgetary Control 18 3 112 Standard Costing 25 3 013 Activity Bases Costing 4 4 714 Transfer Pricing 5 5 315 Ratio Analysis 16 5 316 Learning Curve 3 5 2

186 80 75

Page 6: cost accounting question icwai inter

ca final Total4 331 431 271 120 210 150 67 176 489 360 223 311 16

13 261 250 10

47 388

Page 7: cost accounting question icwai inter

You must revise the following before your examinationChapter 1 : Materials Control and EOQ

Re-order level = Safety stock + max.usage x max. delivery periodRe-order level = Normal usage x (Minimum stock period + Average delivery time)

Re-order level = Safety stock + Lead time consumption

Efficiency ratio = Standard hour for production / Actual hoursActivity ratio = Std. hour for production / Budgeted hoursCapacity ratio = Actual hours / Budgeted hoursEfficiency ratio = Activity ratio / Capacity ratioReorder level = Maximum usage x Maximum reorder periodMinimum level = Reorder level -- ( normal usage x normal reorder period )

Maximum level = ( Re-order level + re-order quantity ) -- ( Min.usage x Min. re-order period)Average Stock level = ( Minimum level + Maximum level ) / 2Average Stock level = Minimum level + half of Reorder quantity

Idle capacity ratio = (Practical capacity in standard hours -- Planned capacity in std. hrs.)Practical capacity in standard hours

Economic batch quantity = under root 2 x Annual Demand x Set up costRate of interest x Cost per unit

Economic order quantity = under root 2 x Annual Demand x Cost of placing one orderCost of carrying one unit for one year

Following are the assumptions of Economic Ordering Quantity :1. The ordering and carrying costs are accurately known;2. The sales and production can be predicted accurately;3. There is no safety stock, the stock comes to zero and is replenished immediately ;4. The unit price remains same irrespective of order size;5. The supply is instant, there is no delivery time in supply.Students are expected to write the above assumptions along with the computation of EOQ.

Chapter 2 : LabourTime rate system: Earnings = Hours worked x Rate per hour Straight Piece Rate System Earnings = Hours worked x Piece Rate per unitDifferential Piece Rate System

F.W. Taylor's system Earnings = 80% of Piece rate when below standardEarnings = 120% of Piece rate at or above standard

Merrick Differential Piece rate systemEfficiency EarningsUp to 83.3% Ordinary Piece rate83.3% to 100% 110% of ordinary piece rateAbove 100% 130% of ordinary piece rate

Rowan plan Earnings = Rate x hours worked + (time saved / time allowed) x hour worked x rateHasley plan earnings = Hour worked x rate/hour + 50% of time saved x rate/hour

Comparison of Halsey and Rowan Plan

Students should note the distinction between material control and inventory control. Reducing consumption of material at production site is material control and not inventory control.

Material cost is required for decision making and replacement cost is preferable for this purpose. However, FIFO and Average cost methods are selected for reporting to external parties.

Note : Any one of the above formulae can be used, depending upon the parameters supplied in the question.

The Eoq formula should not be used when purchasing cost per unit varies with the quantities ordered. In this situation, a schedule of costs of different quantities ordered should be prepared.

To determine whether or not to avail the offered discount, incremental benefits should be compared with incremental costs relating to holding.

When large size order comes out to be optimum, the management should be advised to take the opportunity cost of additional funds required, into consideration.

Page 8: cost accounting question icwai inter

Labour turnover rate Labour turnover rate is generally expressed in percentage.

Flux method LT rate = No. of Separation + No. of Replacement ) Average no. of employees

Separation method LT rate = (no. of workers left + no. of workers discharged)Average no. of workers

Replacement method = No. of workers replaced / Average no. of workersChapter 3: Overheads

Overhead absorption rate or overhead recovery rate = Amount of overhead / Basis of absorption

Predetermined overhead rate = Budgeted overhead for the period / Budgeted basis for the period

Blanket overhead rate = Overhead cost for the entire company / Basis of absorptionBasis of absorption is generally total labour hours or machine hours

Variable portion in Semi variable overhead = Change in amount / change in activity or quantityThere are three ways to treat the unabsorbed overheads.

1. Carry the unabsorbed overheads forward to the next year,

Journal Manufacturing Overhead suspense a/c Dr.Or Manufacturing overhead reserve a/c

To Manufacturing overhead a/c(For the absorption of unabsorbed overhead)2. Write off to costing profit and loss account :

The unabsorbed portion is treated as expense and is transferred to costing profit and loss a/c.

Since there is no carry forward, the profit will be less by an amount equal to unabsorbed overhead.JournalUnabsorbed overhead a/c Dr.

To manufacturing overhead a/c3. Use of supplementary rates for absorption:

JournalUnabsorbed overhead a/c Dr.

Work-in-progressFinished goods Cost of goods sold

Chapter 4 : Contract Costing

Profit to be taken = 1/3 x Notional profit x ( Cash received / Work Certified)When the work is between 50% to 90%Profit to be taken = 2/3 x Notional profit x ( Cash received / Work Certified)

1. When the worker completes his work within half the time allowed, the bonus under both the plans will be same.

2. If the time saved is less than 50% of the time allowed, the Rowan plans gives more bonus to workers. Otherwise Halsey plan is more beneficial to workers.

The unabsorbed overhead is carried to the next year and is added to the overheads of next year for absorption. A suspense account or Overhead reserve account is opened to accommodate the unabsorbed overhead. Thus there would be no impact on profit earned by the firm.

In this method the under absorbed overhead is absorbed in the same year and there is no carry forward, The under absorbed overhead is adjusted or charged to (i) work-in-progress (ii) Finished goods and (iii) Cost of sales, by adding back.

The amount of unabsorbed overhead is added by applying supplementary rate and is charged to work-in-progress, finished goods, and cost of sales in the ratio of their cost.

When the work certified is less than 25% of the total value of the contract, no profit is calculated.

When the work certified is more than 25% but less than 50%, the profit to be taken to profit and loss account is determined by the following formula

Page 9: cost accounting question icwai inter

1 Estimated total profit x ( Works certified / Contract Price)

2 Estimated total profit x (Cash received / Contract Price)

3 Estimated total profit x Cost of work till date / total cost of contract

4 Estimated total profit x (cost of work till date) x (cash received )(total cost of contract) (Work certified)

Chapter 5: Process Costing

6.1

6.2 Accounting treatment for normal loss:

6.3 Accounting treatment for Abnormal loss / gain:

6.4 Process costing when work done includes unfinished units or work-in-progress:

6.41

6.42

6.43

When the work certified is sufficiently advanced, an estimated total profit is determined by deducting the overall estimated cost of the contract from the total value of the contract. A portion of this estimated profit is credited to the profit and loss account.

The profit to be credited to profit and loss account is determined by any of the following formulae;

While preparing process accounts, first of all, accounts for the number of units should be prepared as working notes. This gives abnormal loss/gain arising during the process. All abnormal loss/gain should be valued as good units. The good units should be valued at normal loss figures irrespective of actual number of good units produced.

The cost of normal loss is absorbed by good units. If normal units have any realisable value as scrap, the process account is credited with the amount, conversely, if the normal loss requires some expenditure for its disposal, the process accounting should be debited with this amount.

Students should make a particular note of treatment given to abnormal loss / gain. In preparing process accounts the abnormal loss / gain is treated as good units and valued accordingly at par with normal output. While preparing abnormal loss / gain account it is treated as waste and the difference between its value as good units and value as waste, is taken to profit and loss account.

For finding out equivalent units for work done on unfinished units in closing stock, the stage of completion with regard to different elements of cost viz. material, labour and overhead is kept in view.

For finding out equivalent units for work done on unfinished units in opening stock, following two points should be kept in view:

1. Stage of completion at the time of introduction into the process. The completion of balance work is included in cost of process.

2. The opening stock will not bear the subsequent cost transferred to the process with newly introduced units. To facilitate the matter, units completed in the process are divided into two categories: Two columns Material A and Material B are used in the statement of equivalent production. Column Material A represents the units which will bear the cost transferred to the process, thus the opening stock units will not appear in this column.

3. The newly introduced units will bear both the costs viz. cost transferred to the process as well as the material cost relating to the process. These units will appear in both the columns of Material A and Material B.

4. The column Material B in statement of equivalent production will include (i) units equal to work done on opening stock (ii) units introduced and completed and (iii) units equal to work done on closing stock.

Page 10: cost accounting question icwai inter

6.44

6.45

6.5 Process costing under different methods of inventory valuation:The inventory costing method will influence the cost per unit of the process.

6.51

6.52

6.53

6.54

Chapter 6: Costing of Joint Products

Joints costs are apportioned on the basis of weight, sales value, net realisable value at split off point etc.

Regarding by products, the following points should be particularly taken care of:

Equivalent completed units are not computed for normal loss as it is shared by good units produced in the process.

Units representing abnormal loss/gain should be treated like goods units finished and transferred to the next process for the purpose of preparation of statement of equivalent production.

If FIFO method is used, it is presumed that units are completed in the order of their introduction to the process. Thus the units in opening stock are completed first followed by newly introduced units and lastly the work in done on closing stock.

Under average cost method, the stage completion of opening stock is considered as of no consequence. In the equivalent production statement, the opening stock units are not shown separately, but are included in the total units completed and transferred to next process/ finished goods. The value of opening stock or work-in-process is added to the cost incurred in the current accounting period, and the total cost is divided by the total number of completed units in order to arrive at the average cost of equivalent units.

Following two points should be kept in mind while following the average cost method inventory costing, (i) Units completed will not be divided in two categories i.e. opening inventory and units introduced and completed during the period as done in fifo method and (ii) Cost of opening work-in-process is added element wise with cost incurred during the period.

In average cost method of inventory valuation the cost of opening inventory is added element wise to cost incurred during the process and the aggregate of these two is divided by sum of (i) completed units during the period and (ii) equivalent completed units of closing inventory.

Joints costs are those costs which are common to the processing of joint products and by products. The joint costs are different from common costs which are the costs of facilities and services employed in the production of joint products.

Apportionment of joint costs to joint products are relevant for stock valuation purposes, but such apportionment and analysis is unlikely to be relevant and valid for decision making. For decision making purposes, the further cost of processing with additional revenues becomes significant.

When by product is also there, total cost at split off point should be reduced by realisation value of by product to arrive at Net cost at split off point.

Joint products are the result of same raw materials and same processing operations like by-products, the joint products are generally equally important while the by products are secondary result of operation having less economic value and importance than joint products.

It is advised that entire process upto split off point and beyond should be clearly shown by a way of flow chart showing details of costs, revenues and apportioned costs etc.

(a) No costs are assigned to by products; (b) net realisable value of by products is not attempted at all (c) the accounting treatment should be settled before the apportionment of the joint costs.

Page 11: cost accounting question icwai inter

Theoretical questions are seldom set at the examination from this chapter, that too limited to some short notes.

(i) Estimated profit margins (b) Selling and distribution expenses, if any, and (c) post separation expenses.

Chapter 9 :Marginal CostingSales -- Variable Cost = Contribution = Profit + Fixed cost Pv ratio = Contribution / Sales ; Sales = Contribution / Pv ratioPv ratio = Change in profit / Change in salesBreak even point is the sales at which the contribution is just equal to fixed cost.Margin of Safety = Total Sales -- Break even SalesMargin of Safety ratio = Total Sales -- Break even Sales / Total Sales Margin of safety x Pv ratio = Profit

Chapter 12 : Standard Costing

Summary of Model Steps

For material cost variances : M1 Actual cost of material usedM2 Standard cost of material usedM3 Standard cost of material if it had been used in standard proportionM4 Standard material cost of output

Calculation of Material cost variances :Material Price Variance M1 -- M2Material Mix Variance M2 -- M3Material Yield Variance M3 -- M4Material Cost Variance M1 -- M4 Material Usage Variance or Volume Variance M2 -- M4For Direct Wage Variances

L1 Actual payment made to workers for actual hours worked

Although there is no scientifically established method for apportionment of joint costs, following methods are usually used for the purpose:

(a) Quantitative method attempts to distribute total joint costs on the basis of some units of measurement, such as Liters, meters, kg, pieces etc. All products must have common unit of measurement and all the joint products carry proportionate joint cost as per the measurement.

(b) In Average unit cost method the joint costs are apportioned on the total number of units produced. This is suitable only if the products do not differ greatly.

(c) In weighted average method improves the average cost method by assigning weights to all joint cost products. Weight may be assigned on the basis of selling price, quantity produced, sales value or some arbitrarily but logically assigned points.

(d) Market value method is quite popular. In this method the costs are apportioned on the basis of market value of the products. This is another form of weighted average method. This method can be applied at the point of separation or after further processing or at both.

(e) Under the Net Value Method, the net realisable value is determined. From the sales value of finished joint products, the calculation are reversed and deductions are made to arrive at the net realisable value which is nothing but the apportioned joint costs. Following deductions are made from the sales value of final joint products.

(f) This method is on the basis of concept of marginal costing. Joint costs are segregated into two parts variable and fixed. The contribution made by the product is considered as the basis of apportionment.

Cost Indifference point is the point at which total cost (fixed cost + variable cost) is the same under two alternatives.

In the following formulae, it should be noted that if the value of the preceding step is MORE than the value of following step, it will be case of Adverse or Unfavourable variance and vice versa.

Page 12: cost accounting question icwai inter

L2 Standard payment for actual hours

L3

Payment involved, if the workers had been used in standard proportion with standard rate

L4L5 Standard labour cost of output achieved

Calculation of Direct wages variances :Wage Rate Variance L1 -- L2Wage Gang Variance L2 -- L3Wage Idle Time Variance L3 -- L4Wage Yield Variance L4 -- L5Wage Cost Variance L1 -- L5Wage Efficiency Variance L2 -- L5For Variable Overhead Variances

VO1 Actual Variable Overhead

VO2 Actual Hours worked at standard variable overhead rate

VO3 Standard variable overhead for the production Calculation of Variable Overhead Variance Variable Overhead Expenditure variance VO1 -- VO2Variable Overhead Efficiency Variance VO2 -- VO3Variable Overhead Variance VO1 -- VO3For Fixed Overhead Variances

FO1 Actual Fixed overhead incurred

FO2 Budgeted fixed overhead for the period or Standard FOH allowance

FO3 Fixed overhead for the days/hours available at std. rate during the period

FO4 Fixed overhead for actual hours worked at standard rate

FO5 Standard fixed overhead for production Calculation of Fixed Overhead variancesFOH Expenditure Variance FO1 -- FO2FOH Idle time Variance FO2 -- FO3FOH Capacity Variance FO3 -- FO4FOH Efficiency Variance FO4 -- FO5FOH Variance FO1 -- FO5FOH Volume Variance FO2 -- FO5For Sales Margin Variances

SM1 Actual sales margin on actual salesSM2 Standard sales margin on actual sales

SM3

SM4

Calculate of VariancesSales Margin Price Variance SM1 -- SM2Sales Margin Mix Variance SM2 -- SM3 Sales Margin Quantity Variance SM3 -- SM4 Sales Margin Variance SM1 -- SM4Sales Margin Volume Variance SM2 -- SM4For Total Sales Value VariancesSV1 Actual sales value realisedSV2 Standard sales value of actual sales

Standard labour cost of labour hours utilised. Will be zero if hours utilised is equal to hours available.

In the following formulae, it should be noted that if the value of the preceding step is LESS than the value of following step, it will be case of Adverse or Unfavourable variance and vice versa.

Standard sales margin, if the actual sales had been in the ratio of standard sales mix

Standard sales margin on Standard sales mix or budgeted sales margin as per standard or budget

Page 13: cost accounting question icwai inter

SV3

SV4 Standard sales value of sales as per standard or budgetCalculation of VariancesSales value price variance SV1 -- SV2Sales value mix variance SV2 -- SV3Sales value quantity variance SV3 -- SV4 Sales value variance SV1 -- SV4Sales value volume variance SV2 -- SV4

Chapter 14: Transfer Pricing

There are basically two methods for fixation of transfer price; Cost based price and market based pricing

Chapter 15 : Ratio Analysis

The basic purpose of various accounting ratios :Ratio Purpose Gross profit ratio Efficiency of the production and trading operationsNet Profit ratio Net margin on salesOperation Ratio Efficiency of the operation in relation to salesReturn on Shareholders funds Earning of shareholders on share capital

Proprietary ratio

Debt Equity ratioEarnings per equity share Amount attributable to every equity shareDividend Yield Dividend on market value of sharePrice earning ratio Earning on market price of shareFixed interest cover Coverage of interest requirementInventory turnover Evaluation of liquidity of inventory and inventory controlsCurrent ratio Short term liabilities payment capabilityQuick or Acid test ratio Measure of short term liability and its payment.

Ratio FormulaLiquidity RatiosCurrent ratio Current assets / Current liabilities Quick or Acid test ratio ( Current assets -- Stock ) / Current liabilities Super Quick Ratio (Cash + Marketable securities)/ Current liabilities

Standard sales value of actual sales; if the sales had been in the ratio of standard sales mix.

A Transfer price is that notional value at which goods and services are transferred between divisions in decentralized organization. fixation of transfer price is essential for measurement of performance of a division.

In cost based pricing, Actual cost of production, Full cost plus a mark up, standard cost, marginal cost, are basic parameters to arrive at transfer price.

Under market pricing methods, the market price will be determined according to the market price prevailing in the market. As no selling efforts are required, usually selling and distribution overheads should be deducted and price thus arrived at should be charged as transfer price.

It may also happen that the selling division may notice that the price offered by the buying division is less than the price at which the product can be sold in the open market. Alternatively the buying division may find that it can procure the product from the market at price lower than that charged by selling division. In all such conflicts, it is in the interest of the organization that the buying division and selling division should negotiate the price. Such prices are known as 'negotiated price'.

Opportunity cost pricing: This pricing recognises the minimum price that the selling division is ready to accept and the minimum price the buying division is ready to pay. The transfer price is fixed between these two limits.

Relation of shareholder's funds with total assets employed in the business

Measure of extent of trading on equity. Percentage of funds being financed through borrowings.

Page 14: cost accounting question icwai inter

Payables Turnover Cost of goods sold / Accounts payablesAverage payment period Accounts payable / Cost of goods sold per day

Leverage RatiosDebt Equity ratio

Total Debt / Equity Share capital Debt Assets Ratio Total Debt / Total AssetsInterest Coverage ratio EBIT / Interest ChargesFixed charges coverage ratio EBIT

Activity / Turnover RatioInventory Turnover Ratio Sales / Inventory OR

Cost of goods sold / Average InventoryInventory turnover ratio is sometimes termed as Stock velocity.Average Collection period Receivables / Average sales per dayDebtor's Turnover Ratio Net Credit Sales / Average DebtorsFixed Assets Turnover Ratio Net Sales / Fixed assets OR

Cost of goods sold / Fixed assets Total Assets Net Sales / Total assets

Profitability RatiosGross profit ratio Gross profit / Net SalesNet profit ratio Net profit / Total salesOperating profit ratio Operating profit / Total salesReturn on Assets Net profit after tax / Total Tangible assetsReturn on Capital employed Net profit after tax / Total Capital employedNet operating profit rate of return EBIT / Total tangible assetsPrice Earning ratio

Long term debt / Equity Share capital OR

[Interest Charges + Rent + Lease + Principal payment after tax]

Page 15: cost accounting question icwai inter

Chapter 1 : Material Control and Economic Order Quantitycwa inter cwa final ca inter ca final Total

11 11 7 4 33

SolutionMaximum delivery period 3 weeks Maximum usage 800 units Safety stock : 300 units

Re-order level = Safety stock + max. Usage x max.delivery period) 300 + 3 x 800 = 2,700 units

Solution : Students should learn the following formulae.Efficiency ratio = Std.hour for production / Actual hoursActivity ratio = Std. hour for production / Budgeted hoursCapacity ratio = Actual hours / Budgeted hoursEfficiency ratio = Activity ratio / Capacity ratio = 0.99 / 0.90 = 110%

Particulars A BNormal usage per week nos. 60 60Maximum usage nos. 80 80Minimum usage nos. 30 30Reorder quantity nos. 400 600Reorder period 4 to 6 weeks 2 to 4 weeksYou are required to calculate for each component:(i) Reorder level (ii) Minimum level (iii) Maximum level (iv) Average stock levelSolutionThis is formula based question. The students should memories the following formulae:Reorder level = Maximum usage x Maximum reorder periodMinimum level = Reorder level -- ( normal usage x normal reorder period )Maximum level = ( Re-order level + re-order quantity ) -- ( Min.usage x Min. re-order period)Average Stock level = ( Minimum level + Maximum level ) / 2AlternativelyAverage Stock level = Minimum level + (Re-order quantity ) / 2Based on above, the fixation of various levels is as follows:

A units B unitsReorder level 80 x 6 480 80 x 4 320Minimum level 480 -- ( 60 x 5) 180 320 --( 60 x 3) 140Maximum level 480 + 400 -- 30 x 4 760 320 + 600 -- 30 x 2 860Av. stock level ( 760 + 180 ) / 2 470 (860 + 140) / 2 500or alternativelyAverage stock level 180 + (400 / 2) = 380 140 + ( 600 / 2) = 440

yksxksa esa dkfcfy;r dh deh ugha gksrh] fnDdr ;s gS fd oks eu yxkdj dke ugh djrsAProblem 1:(cwa inter I June 05)

Zee limited uses material A for the production of product M, the safety stock of material A is 300 units, the supplier quotes a delivery delay of two or three weeks, if the company uses 500 to 800 units a week according to the activity levels, Find the re-order level of A.

Problem 2:(cwa inter I June 04)

If the capacity usage ratio of a production department is 90% and the activity ratio is 99%, then the efficiency ratio of the department is………………

Problem 3:(cwa inter I June 04)

For the manufacture of a certain product two components A and B are used. The following particulars about these components are available.

Page 16: cost accounting question icwai inter

Solution :you must remember the formula of re-order level.Re-order level = Safety stock +(Maximum. delivery period x Maximum usage)Re-order level = 200 + ( 3 x 700 ) = 2.300 units.

Solution: u fp<+sa u xqLlk gksa] 'kkar jgus dk vH;kl djsaAReorder level = Maximum usage x management reorder period = 450 x 5 = 2250 unitsMinimum level = Reorder level -- ( normal usage x normal lead time ) = Minimum level = 2,250 -- { ( 300 + 450)/2 x 2 } = 1,500 units..

The following data relate to the operations of a manufacturing company :Standard hours of output Actual hours

Theoretical capacity 100 100Theoretical capacity less unavoidable time 95 95Planned activity for the period 81 90Actual activity for the period 68 85Calculate : (i) Efficiency ratio (ii) Production volume ratio ( or activity ratio)

(iii) idle capacity ratio of the firm.Solution : Students should learn the following formulae.

Efficiency ratio = Std.hour for production / Actual hours (68/85) = 80%Activity ratio = Std. hour for production / Budgeted hours (68/81) = 84%Efficiency ratio = Activity ratio / Capacity ratio

Idle capacity ratio =(Practical capacity in standard hours -- Planned capacity in standard hrs.)Practical capacity in standard hours

Idle capacity ratio = ( 95 -- 81 ) / 95 = 14.7%

The following particulars are collected for a product 'ZED'.(a) Monthly demand : 1,000 units. (b) Cost of placing an order : Rs. 100(c) Annual carrying cost per unit Rs. 15 (d) Normal usage 50 units per week.(e) Minimum usage : 25 units per week (f) Maximum usage 75 units per week(g) Re-order period 4 to 6 weeks.Compute from the above :1. Reorder quantity 2. Re-order level 3. Minimum level 4.Maximum level 5. Ave.stock level.Solution: Tutorial Notes:

For the computation of eoq please see the relevant chapter.

EOQ = 186 units approximately. using the formula)Re-order level = Maximum reorder period x maximum usage = 6 x 75 = 450 units.Minimum level = Reorder level -- normal usage x Average reorder period = 450 -- 50 x 5 Minimum level 200 unitsMaximum level 536 units

Problem 4 (cwa inter I dec.02)

The safety stock is 200 units, the supplier quotes a delivery delay of two to three weeks; the factory uses 400 to 700 units a week according to safety levels. What is the re-order level ?

Problem 5: (cwa inter I Dec 06)

In a factory, current uses of a component varies between 300 and 450 units per week and replenishment time is normally two weeks but can go up to five weeks. The minimum stock level of the component is …………….

Problem 6:(cwa inter II dec.02 repeat June 05, dec.06)

Problem 7: (ca inter may 87)

Monthly demand is given and normal usage per week is also given. It is the consumption and not the demand that matters in EOQ formula. Thus monthly demand of 1,000 units is not relevant for computation of EOQ. Normal usage of 50 units per week is given. Thus annual consumption would be 50 x 52 = 2,600 units.

Page 17: cost accounting question icwai inter

Average level 368 units.

A company has the option to procure a particular material from two sources.

Source 1 assures that defectives will not be more than 2% of the supplied quantity.

You are required to find out which of the two sources is more economical ?Solution:Comparative statement of procurement of units from source 1 and source 2

Source 1 Source 2Units supplied per lot 1000 1000% of defective units 2% 2.80%Defective units in no. 20 28Rectification cost of defective units @ Rs. 5/unit 100 140Extra price paid per lot 100 --Total additional cost per lot 200 140Source 2 requires less cost and is therefore recommended for supply.

His accountant has brought the following :(i) We are following FIFO system for the purpose of issues.(ii) Costs are going up faster than 12% and they will go up further in 1999.

C Manufacturing Ltd. Income Statement (Rs. 000)1998 1997

Sales 1,008 Cost of goods sold

Opening Inventory 320 250Raw materials 500 400Labour 200 174Depreciation 150 150End Inventory (390) (320)

780 654Gross Margin 228 246Administrative expenses 100 92EBIT 128 154Interest 50 50

Problem 8: (ca inter may 2001)

Source 2 does not give any assurance, but on the basis of past experience of supplies received from it, it is observed that defective percentage is 2.8%.

The material is supplied in lots of 1000 units. Source 2 supplies the lot at a price, which is lower by Rs. 100 compared to source 1. The defective units of material can be rectified for use at a cost of Rs. 5 unit.

Problem 9: (ca final may 99)

C developed original specification of a product and founded C Manufacturing Ltd. In 1997 the firm manufactured 980 Nos. at an average price of Rs. 900 each. In 1998 due to continuous price rise of the inputs, he raised his prices at an average of 12%, since he knew he could sell plant's full capacity of 980 nos.

In spite of price rise for the product, which sold for over Rs. 1,000 for the first time, C was surprised to learn in the late 1998 (as may be seen from the financial statements) that C Manufacturing Ltd. show a decline in earning and still worse decline in cash flow.

(iii) We are not setting aside enough to replace the machinery, we need to set aside Rs. 165,000, not Rs. 150,000 so as to be able to buy new machinery.

(iv) It is still not late to switch to LIFO for 1998. This will reduce closing inventory to Rs. 330,000 and raise cost of goods sold.

Page 18: cost accounting question icwai inter

78 104Income tax 39 52Profit after tax 39 52Add : Non-cash expenses 150 150Inventory change (70) (70)Cash flow 119 132Required :(i) What is the weighted average inflation factor for the firm using LIFO?

(ii) If the firm desires a 15% margin on sales, how much should the firm charge for the product per unit.

Solution : Tutorial Notes:

1. Determination of weighted average inflation factor for the firm in 1998Cost of goods sold 1998 1997

Opening Inventory 320 250Raw materials 500 400Labour 200 174Depreciation 165 150

Less : End Inventory (330) (320)Cost of goods sold 855 654Add: Administrative expenses 100 92Cost of goods sold 955 746Hence weighted average inflation factor for the firm in 1998 : 955 / 746 = 1.28 i.e. 28% over 19972. Determination of price of the product : Rs.Total cost in 1998 for 980 Nos. 955,000Cost of goods sold per unit: 974.5Price with 15% margin on Selling price : 974.50/ 0.85 1,146.5Total sales for 980 units (Rs.'000) 1,123.5

Particulars Cast Wheel Gun Metal Bush(i) Weight of each raw casting (kg) 25 3(ii) Material Cost / kg (Rs.) 20 60(iii) Sale price of scrap / kg. (Rs.) 4 40(iv) Finished weight / unit after machining (kg) 20 2(v) Machine hours required per unit

(a) Boring M/c 6(b) Lathe 1.5

Machine hours rates are Rs. 40/hour for Boring and Rs. 20/hour for Turning.Assembly takes half hour, costs Rs. 10/hour and consumes hardware worth Rs. 5.

Solution : blh oDr vPNk eglwl dhft;s A vHkh eqLdqjkb;sAMaterial cost = Cost of raw material + Cost of hardware -- Scrap sale = (25 x 20) + (2 x 3 x 60) + 5 -- [ 4 x (25 -- 20) + 2 x 40 x (3 -- 2)] = Rs. 765

1. You should note that if LIFO method is followed as suggested, the end inventory would be Rs. 330,000 and not Rs. 390,000.

2. The machinery should cost Rs. 165,000 and not Rs. 150,000, consequently the depreciation charged would be Rs. 165,000 and not Rs. 150,000.

3. This is a peculiar type of question which is not related with any specific topic. It is difficult to know where to begin the solution. As the question itself mentions that the prices are going up we can consider the inflation between the two periods so as to bring the figures at common datum.

Problem 10: (cwa final June 06)

A crane wheel assembly is made of cast steel and two gun metal bushes. Find the cost of the assembly assuming no

Page 19: cost accounting question icwai inter

Machining Cost = Machine hour rate x machine hour consumed = (40 x 6) + (2 x 20 x 1.5) = Rs. 300

Assembly cost @ Rs. 10/hr for half hour = Rs. 5Total cost = Material cost + Machining cost + Assembly cost = Rs. 765 + Rs. 300 + Rs. 5 = Rs. 1070.

The current replacement price of the material is Rs. 0.80 per kg.Determine the relevant cost of material required for the job.Solution :The original price of the material is a sunk cost is not relevant. The current resale value is relevant.500 kgs of stock will be valued at current resale value Rs.Balance 300 kgs will be acquired on current market price of Rs. 0.80/kg Rs.

Relevant cost Rs.

2001--02 2002--03 fig. in Rs. Sales 1,00,00,000 1,00,00,000Scrap 400,000 300,000Rework 600,000 400,000Product inspection 100,000 125,000Product warranty 800,000 600,000Quality training 40,000 80,000Materials inspection 60,000 40,000You are required to : (i) Classify the costs as prevention, appraisal, internal failure or external failure.(ii) Compute the profit that has increased because of quality improvements?Solution:

Prevention Costs Quality trainingAppraisal Costs Product inspection and material inspectionInternal Failure costs Scrap and reworkExternal Failure costs Warranty

2001--02 2002--03Rs. Lacs Rs. Lacs

Scrap 4.00 3.00Rework 6.00 4.00Product inspection 1.00 1.25Product warranty 8.00 6.00Quality training 0.40 0.80Materials inspection 0.60 0.40Total Cost 20.00 15.45Reduction in cost due to quality improvements 4.55

The profit has increased by Rs. 20 lakhs --Rs. 15.45 lakhs = Rs. 4.55 lakhs.

Problem 11: (cwa final June 07)

A particular job requires 800 kgs of a material. 500 kgs of the particular material is currently in stock. The original price of the material was Rs. 300 but current resale value of the same has been determined as Rs. 200.

Problem 12: (cwa final June 03)

ABC ltd. initiated a quality improvement program at the beginning of the year. Efforts were made to reduce the number of defective units produced. By the end of the year, reports from the production manager revealed that scrap and rework had both decreased. Though pleased with the success, the President of the company wanted some assessment of the financial impact of the improvements. To make this assessment, the following financial data were collected for the current and preceding year:

Problem 13: (ca final nov 03)

Page 20: cost accounting question icwai inter

Calculate capacity ratio, activity ratio and efficiency ratio. Also establish their relationship.Solution : Tutorial Notes:

Capacity ratio = Actual working hours / Budgeted working hours = 25 days x 50 workers x 8 days / ( 102,000 / 12) = 1.17 or 117%

Activity Ratio = Actual production in standard hours / Budgeted hours= [(1,000 units of X x 5 hours ) + ( 600 units of Y x 10 hours)] / (102,000 / 12)= 1.29 or 129%

Efficiency ratio = Standard hours in actual production / Actual hours = [(1,000 units of X x 5 hours ) + ( 600 units of Y x 10 hours) ]/ (25 x 50 x 8)= 1.10 or 110%

Inter-relationship among ratios :Capacity ratio x Efficiency ratio = Activity ratio

Calculate capacity ratio, activity ratio and efficiency ratio. Also establish their relationship.Solution ; Solve on the lines of above problem. Key answers are given below.

Capacity ratio = 113%, Efficiency ratio = 109% and Activity ratio = 124%.

Problem 15 :(CA Nov. 2007)Calculate capacity ratio and efficiency ratio. Actual hours worked are 500Budgeted production : 80 units ; Actual production : 60 units ; Std. time / unit : 8 hours

Problem 16 : (CA IPCC Nov.09)Calculate efficiency ratio and activity ratio from the following data:Activity ratio : 75%; Budgeted hours :6,000 units, Actual out put : 5,000 units. Standard time per unit : 4 hours.Answer : Efficiency ratio 111% Activity ratio : 83.3%, Actual hours : 18,000

Problem 17: (cwa inter Dec.05)

Annual Demand 32,000 units Set up cost per batch Rs. 120Annual rate of interest 12% Cost of production per unit Rs. 16Find out the economic batch quantity.

A company manufactures two products X and Y. Product X requires 5 hours to produce while Y requires 10 hours. In June 03, of 25 effective working days of 8 hours a day, 1,000 units of X and 600 units of Y were produced. The company employs 50 workers in the production deptt. to produce X and Y. The budgeted hours are 102,000 for the year.

Don't pray for an easy life; pray for being stronger person.1. Budgeted hours are given per annum basis. Budgeted hours per month can be obtained by dividing it by 12. You can also divide it by 365 and then multiply by 31 day for July month also. Budgeted hours are relevant for computing the capacity ratio and activity ratio.

Problem 14: (ca final may 04)

A company manufactures two products X and Y. Product X requires 8 hours to produce while Y requires 12 hours. In April, 2004, of 22 effective working days of 8 hours a day, 1,200 units of X and 800 units of Y were produced. The company employs 100 workers in production department to produce X and Y. The budgeted hours are 186,000 for the year.

Answer : Efficiency ratio : 96% and Capacity ratio : 78%

A television company manufactures seera components in batches. The following data relates to component :

Page 21: cost accounting question icwai inter

Solution:Economic batch quantity = under root 2 x Annual Demand x Set up cost 2 x 32,000 x 120

Rate of interest x Cost per unit 0.12 x 162,000 units

Problem 18: (cwa inter Dec.04)

Solution: Use eoq formula, the eoq comes as 800 units.

What other factors should also be considered before the decision is taken?Solution:Using EOQ formula the EOQ comes to be 480 units.Comparative statement of three options proposed by the supplier:Particulars Option 1 Option 2 Option 3

At eoq 2 orders 1 orderAnnual consumption units 9,600 9,600 9,600Order quantity or size of order units 480 4,800 9,600No. of orders no. 20 2 1Total ordering cost @ Rs. 60/order 1,200 120 60Average stock 240 2,400 4,800Carrying cost per unit Rs. 5 5 5Total carrying cost Rs. 1,200 12,000 24,000Cost of component per unit Rs. 50 48 47Total cost of components Rs. 480,000 460,800 451,200

Statement showing cost comparison of the three optionsParticulars Option 1 Option 2 Option 3Total ordering cost Rs. 1,200 120 60Total carrying cost Rs. 1,200 12,000 24,000Total cost of components Rs. 480,000 460,800 451,200

482,400 472,920 475,260Recommendation:

Placing two orders each consisting of 4,800 units of components is recommended.Other considerations1. Funds requirement is raised. Opportunity cost of funds required must be considered.

A firm requires 16,000 nos. of a certain component, which it buys at Rs. 60 each. The cost of placing an order and following it up is Rs. 120 and the annual storage charges works out to 10% of the cost of the item. To get maximum benefit the firm should place order for how many units at a time?

Problem 19: (cwa inter June 03)

The annual demand of a certain component bought from the market is 1,000 units. The cost of an order is Rs. 60 and the carrying cost is Rs. 3 p.a. The EOQ is ……………. Units. (Ans: 200 units)

Problem 20: (cwa inter Dec 03)

A manufacturer requires 9,600 units of certain components annually. This is currently purchased from a supplier at Rs. 50 per unit. The cost of placing an order is Rs. 60 per order and the annual carrying cost is Rs. 5 per piece. What is the EOQ for placing order?

Recently the supplier has expressed his willingness to reduce the price to Rs. 48, if the total requirements are obtained from him in two equal orders and to Rs. 47, if the entire quantity required is purchased in one lot. Analyze the cost of the three options and recommend the best course.

From the above analysis it is evident that the most economical order quantity is 4,800 units of components.

2. Average stock is raised from 480 units to 4,800 units i.e. 10 times. This would call for more space along with arrangement for safety, security and maintenance of units.

Page 22: cost accounting question icwai inter

Which policy would you recommend?

Solution: blh oDr vPNk eglwl dhft;s A vHkh eqLdqjkb;sAStatement showing costs of various no. of orders for procurement of componentNo. of orders 1 2 3 4 5 6Order Qty. or order size in units 24,000 12,000 8,000 6,000 4,800 4,000Average stock in units 12,000 6,000 4,000 3,000 2,400 2,000

Value @ Rs. 500 (Rs. In lacs) 60 30 20 15 12 10Stock holding cost(1% in Rs.) 60,000 30,000 20,000 15,000 12,000 10,000Ordering cost @ Rs. 2,500 2,500 5,000 7,500 10,000 12,500 15,000Total cost (ordering +holding) 62,500 35,000 27,500 25,000 24,500 25,000

The EOQ of the above requirement can be calculated as follows:Economic order quantity = under root 2 x Annual Demand x Cost of placing one order

Cost of carrying one unit for one year2 x 24,000 x 2,5000.01 x 500

Economic order quantity 4,899 unitsNo. of orders 24,000 / 4,899 = 4.9 say 5Average stock 2,450 unitsStock carrying cost 2,450 x 5 Rs. 12,250Ordering cost @ Rs. 2500/order Rs. 12,500Total Rs. 24,750Recommendation The company should place 5 orders.

Rs.Direct material purchased 250,000Sales revenues 750,000Direct manufacturing labour 222,250Prime cost 397,750Cost of goods available for sale 555,775Indirect manufacturing costs 40% of conversion costWork in process inventory,1..9.03 40,000Direct materials inventory, 1.9.03 20,000Finished goods inventory, 1.9.03 37,750Gross margin % based on sales 30%

3. Components are to be stocked for 6 months. Their perishability and possible deterioration need to be kept in mind.

Problem 21: (cwa inter Dec 02)

A manufacturing company purchases 24,000 pieces of a component from a sub-contractor at Rs. 500 per piece and uses them in its assembly department at a steady rate. The cost of placing an order and following it up is Rs. 2,500. The estimated stock holding cost is approximately 1% of the value of average stock held. The company is at present placing orders which at present vary between an order placed every two months i.e. six orders per annum.

In consideration of the above statement, it is recommended that the management should place five orders to keep the cost to the minimum.

Problem 22: (CA PE-II Nov.03)

A fire occurred in the factory premises on October 31,2003. The accounting reports have been destroyed. Certain accounting records were kept in another building. They reveal the following for the period Sept.1, 2003 to October 31,2003.

Page 23: cost accounting question icwai inter

Required : (i) Finished goods inventory, 31.10.2003(ii) Work-in-progress inventory 31.10.2003(iii) Direct materials inventory 31.10.2003

Solution: This problem requires certain thinking before attempting to solve.

Working notes: Rs.1. Direct material used during the period: Prime cost 397,750

Less: Direct manufacturing cost 222,250Direct material used during the period 175,500

2. Computation of indirect manufacturing cost:Conversion cost = Direct manufacturing cost + Indirect manuf.cost

But indirect manuf.cost is 40% of conversion cost or conversion cost is 2.5 x indirect manuf.cost1.5 x Indirect manufacturing cost = direct manufacturing cost = Rs. 222,250Indirect manufacturing cost = 222,250 / 1.5 = Rs.148,167

3. Cost of goods manufactured: Cost of goods available for sale 555,775Less: Finished goods 1.9.2003 37,750Cost of goods manufactured 518,025

1. Finished goods inventory 31.10.2003 Rs. Sales revenue 750,000Less Gross profit 30% 225,000Cost of goods sold 525,000Cost of goods available for sale 555,775Finished goods inventory 31.10.03 30,775

2. Work-in-progress inventory 31.10.2003Prime cost 397,750Add: Indirect manufacturing cost 148,167Add: Opening work-in-progress 40,000Manufacturing cost to account for 585,917Less: Cost of goods manufactured 518,025Work-in-progress inventory 31.10.03 67,892

The loss is fully covered by the insurance company. The insurance company wants to know the historical cost of the inventories as a basis for negotiating a settlement, although the settlement is actually to be based on replacement cost and not the historical cost.

The finished goods inventory as on 31.10.2003 is easy to compute. Cost of goods available for sale is already given. Cost of goods sold is 70% of sales revenue. The difference between cgas and cogs represents the finished goods inventory as on 31.10.2003.

Similarly the prime cost minus direct manufacturing cost gives the material used during the period. Opening stock plus purchase less material consumed will give the closing inventory of direct materials as on 31.10.2003.

Cost of goods manufactured is cost of goods available for sale minus opening stock of finished goods. Manufacturing cost is sum of prime cost plus indirect manufacturing cost plus opening stock of work-in-process. The difference between these two will give the closing inventory of work-in-process.

Indirect manufacturing costs are given indirectly. The conversion cost is sum of direct and indirect manufacturing costs. The indirect manufacturing cost is 40% of conversion cost. Worded differently the conversion cost is 2.5 times the indirect manufacturing cost. Thus the direct manufacturing cost (given as Rs. 222.250) is 1.5 times the indirect manufacturing cost.

The students are advised to try to solve this problem on the lines given above before going through the solution which goes as follows:

Page 24: cost accounting question icwai inter

3. Direct materials inventory 31.10.2003Direct materials 1.9.2003 20,000Add: Direct materials purchased 250,000Less: Consumption during the period 175,500Direct materials 31.10.2003 94,500

RST Ltd. has received an offer of quantity discount on its order of materials as underPrice per tonne Rs. Tones number

9,600 Less than 509,360 50 and less than 1009,120 100 and less than 2008,880 200 and less than 3008,640 300 and above

Annual requirement 500 tonnesOrdering cost per order Rs. 12,500Stock holding cost:25% of material cost p.a.Required: (i) Compute the most economical purchase level.(ii) Compute EOQ if there are no quantity discounts and the price per tonne is Rs. 10,500.Solution:

Statement showing costs of various no. of orders for procurement of material:Order quantity or size of order tonnes 40 50 100 200 300No. of order (500/ order size) 12.5 10.0 5.0 2.5 1.7Ordering cost @ Rs. 12,500 156,250 125,000 62,500 31,250 20,833Average stock tonnes 20 25 50 100 150Cost per tonne Rs. 9,600 9,360 9,120 8,880 8,640Stock holding cost@ 25% Rs. 48,000 58,500 114,000 222,000 324,000Procurement cost for 500 tonne Rs. Lacs 48.00 46.80 45.60 44.40 43.20Total cost Rs. Lacs 50.04 48.64 47.37 46.93 46.65

(ii) Second part is easy. The answer is 69 tonnes.

Statement showing costs of various no. of orders for procurement of material:Order quantity or size of order tonnes 40 50 100 200 300No. of order (500/ order size) 13.0 10.0 5.0 3.0 2.0Ordering cost @ Rs. 12,500 162,500 125,000 62,500 37,500 25,000Average stock tonnes 20 25 50 100 150Cost per tonne Rs. 9,600 9,360 9,120 8,880 8,640Stock holding cost@ 25% Rs. 48,000 58,500 114,000 222,000 324,000Procurement cost for 500 tonne Rs. Lacs 48.00 46.80 45.60 44.40 43.20Total cost Rs. Lacs 50.11 48.64 47.37 47.00 46.69

Problem 23: (CA PE-II Nov.04)

Tutorial notes; When a number of purchase alternatives are compared; generally the number of orders may not be a whole number and it should be taken as such and should not be rounded off to complete number notwithstanding the fact that the order cannot be placed in fractions.

In consideration of the above statement, it is recommended that the management should place order of size of 300 tonnes. Two orders should be placed.

Alternatively : If no. of order is rounded off to whole number, following table is relevant

In consideration of the above statement, it is recommended that the management should place order of size of 300 tonnes. Two orders should be placed.

Note : Both the analyses are correct. You can opt for any one. However, it is rational to round off the number of order to the nearest whole no.

Page 25: cost accounting question icwai inter

Rs. Rs.Sand paper 5,000 Direct material 1.10.04 100,000Material handling cost 175,000 Direct material 31.10.04 125,000Lubricants and coolants 12,500 Finished goods 1.10.04 250,000Misc.Indirect manufacturing labour 100,000 Finished goods 31.10.04 375,000Direct manufacturing labour 750,000 Work-in-progress 1.10.04 25,000Plant leasing cost 135,000 Work-in-progress 31.10.04 35,000Depreciation plant equipment 90,000 Direct material purchased 11,50,000Property tax on plant equipment 10,000 Sales revenues 34,00,000fire insurance on plant equipment 7,500 Marketing promotions 150,000Marketing salaries 250,000 Customer service cost 250,000Distribution costs 175,000

Prepare an income statement with a separate supporting schedule of cost of goods manufactured.

Solution: This is very easy. Try to solve before going through the solution as below.Popeya Company Ltd.

Schedule of Cost of goods manufactured for the month of October 2004 fig. in Rs.Opening stock of direct materials 100,000Add :Direct material purchased 11,50,000Less; Direct material 31.10.04 125,000Direct materials used 11,25,000Add: Direct manufacturing labour 750,000Prime cost 18,75,000Indirect manufacturing cost:Sand paper 5,000 V Material handling cost 175,000 V Lubricants and coolants 12,500 V Misc.Indirect manufacturing labour 100,000 V Plant leasing cost 135,000 F Depreciation plant equipment 90,000 F Property tax on plant equipment 10,000 F fire insurance on plant equipment 7,500 F Total indirect manufacturing cost 5,35,000Manufacturing cost incurred during the month 24,10,000Add; Work-in-progress 1.10.04 25,000Less: Work-in-progress 31.10.04 35,000Cost of goods manufactured 24,00,000Schedule of Cost of goods sold for the month of October 2004

Finished goods 1.10.04 2,50,000Add: Cost of goods manufactured 24,00,000Less: Finished goods 31.10.04 3,75,000Cost of goods sold 22,75,000Schedule of Gross profit and operating income:

Sales Revenue 34,00,000Cost of goods sold 22,75,000

Problem 24: (CA PE-II Nov.04)

Popeye company is a metal and wood cutting manufacturer, selling products to home construction market. Consider the following data for the month of October 2004:

For all manufacturing items, indicate by V or Following whether each is basically a variable cost or fixed cost (where the cost object is a product unit)

Page 26: cost accounting question icwai inter

Gross margin 11,25,000Marketing, Distribution and customer service costs :Marketing promotions 150,000Marketing salaries 250,000Distribution costs 175,000Customer service cost 250,000 825,000

Total cost of sales (22,75,000 + 8,25,000) 31,00,000Operating income (11,25,000 -- 8,25,000) 3,00,000

Price per tonne Rs. Tonnes1,400 Less than 5001,380 500 and less than 1,0001,360 1,000 and less than 2,0001,340 2,000 and less than 3,0001,320 3,000 and above

The purchase quantity options to be considered are 400, 500, 1000, 2000, and 3000 tonnes.Solution: Solve yourself on the lines of problem 7 above. The answer is given here:Answer: Order size 400 500 1,000 2,000 3,000

Total cost in Rs,000 7,071 6,981 6,942 6,971 6,998

Average monthly demand in units 2,000 tubes Lead time to supply : 6 -- 8 weeks.Ordering cost : Rs. 100 per order. Inventory carrying cost : 20% per annumCost of tubes : Rs. 500 per tube Normal usage : 100 tubes per week. Minimum usage : 50 tubes per week Maximum usage : 200 Tubes per weekCompute from the above: 1. Maximum, Minimum and reorder levels of stock.

Solution:

Inventory carrying cost is 20% of Rs. 500 i.e. Rs. 100 per unit per annum.From the formula the EOQ comes to be approximately 102 units.Evaluation of supplier's proposal of 5% discount:Order size units 102 1,500Annual demand units units 5,200 5,200No. of orders nos. 51 3.47Ordering cost @ Rs. 100 Rs. 5,098 347

Problem 25: (CA PE-II Nov.00, similar nov. 04)

The purchase department of your organization has received an offer of quantity discount on its order of material as under:

The annual requirement of the material is 5,000 tonnes. The delivery cost per order is Rs. 1,200 and the annual stock holding cost is estimated at 20% of the average inventory.

The purchase department wants you to consider the following purchase options and advise which among them will be most economical ordering quantity, presenting the relevant information in a tabular form.

Problem 26: (CA PE-II may 98, repeat may 00)

M/s Tubes ltd. are the manufacturer of picture tubes for TV. The following are the details of their operation during 1997.

2. Economic order quantity. If the supplier is willing to supply quarterly 1,500 units at discount of 5%, is it worth accepting ?

Monthly demand is given and normal usage per week is also given. It is the consumption and not the demand that matters in EOQ formula. Thus monthly demand of 2,000 units is not relevant for computation of EOQ. Normal usage of 100 units per week is given. Thus annual consumption would be 100 x 52 = 5,200 units.

Page 27: cost accounting question icwai inter

Average inventory units 51 750Price per unit Rs. Rs. 500 475 5% disct.Inventory carrying cost @ 20% of price/ unit Rs. 100 95Inventory carrying cost ( Ave.inventory x cost/unit) 5,098 71,250Cost of procurement Rs. Rs. 26,00,000 24,70,000Total cost Rs. 26,10,196 25,41,597Saving if supplier's proposal is accepted Rs. 68,600

Other considerations1. Funds requirement is raised. Opportunity cost of funds required must be considered.

Maximum, Minimum and reorder levels of stock.Reorder level = Maximum usage x Maximum reorder period 1,600 unitsMinimum level = Reorder level -- ( normal usage x normal reorder period ) 900 units Maximum level :Reorder level + reorder quantity -- (Min.usage x Min. reorder period) 1,402 units

You are required to calculate :1. Economic order quantity.2. If the minimum lot size is 4,000 units, what is the extra cost, the company has to incur?3. What is the minimum carrying cost the company has to incur?Solution: 1 EOQ = 2,400 units. Use formula and compute yourself.2. Comparative statement of order size of 2400 and 4000 unitsOrder size units 2,400 4,000Annual demand units 48,000 48,000No. of orders no. 20 12Ordering cost @ Rs. 120 Rs. 2,400 1,440Average inventory units 1,200 2,000Inventory carrying cost @ 10% i.e. Rs. 2/unit Rs. 2,400 4,000Total cost of ordering and carrying Rs. 4,800 5,440Extra cost of order Rs. 640

Recommendation: In view of the above presentation, it is evident that order size of 1500 units per quarter with discount of 5% would result in net saving of Rs. 68,600 and is therefore recommended.

2. Average stock is raised from 51 units to 750 units i.e. 15 times. This would call for more space along with arrangement for safety, security and maintenance of units.

3. Components are to be stocked for 3 months. Their perishability and possible deterioration need to be kept in mind.

Problem 27: (ca iner-II may 99)

G Ltd. produces a product which has a monthly demand of 4,000 units. The product requires a component which is purchased at Rs. 20. For every finished product, one unit of component is required. The ordering cost is Rs. 120 per order and the holding cost is 10% p.a.

3. Minimum carrying cost : Carrying cost is minimum when order size is minimum. In this case the minimum order is of size 2400 units. The carrying cost as calculated above is Rs. 2400.

Problem 28: (cwa final June 04)

Three'' Ltd. is a popular manufacturer of different kinds of lifting tools. These lifting tools are made of polypropylene fiber. The company installed a plant for backward integration of its raw material polypropylene fiber. It produces these polypropylene fiber at the rate of 5000 mt per hour. The usage rate of these fiber for the production of lifting tool is 20000 mt per day considering 8 hours working per day. The cost of fiber is Rs. 5 per meter. The inventory carrying cost is 25% and the set up cost are Rs. 4050 per set up. Compute the optimal number of cycle required in a year for the manufacture of this special fiber. ( Take 365 days as no. of days for the year in your computations.)

Page 28: cost accounting question icwai inter

Solution:Production rate p = 5000 meter/ hourUsage rate r = 20,000 / 8 meter / hr = 2500 mt./hourInventory carrying cost = 25% of Rs. 5 = Rs. 1.25/ meterSetup cost per set up = Rs. 4050Annual requirement = 365 x 20,000 = 73,00,000 meterOptimum quantity Q = Under root [( 2 x 4,050 x 73,00,000) / 1.25 x (5,000 -- 2,500) / 5,000]Optimum quantity = 370,584 meters.Optimum no. of cycles = 73,00,000 / 370,484 = 19.7 say 20 cycles.

Rs. Rs.Purchase personnel salary and expenses 200,000 Cost of bill payment 80,000Stores personnel salary and expenses 170,000 Interest charges 15%Obsolescence, spoilage etc. 65,000 Insurance charges 1%Rent and machine handling in store 140,000 Cost of collection material 50,000Evaporation loss and pilferage 25,000 Cost of receiving material 45,000Floor rental for purchase section 50,000 Cost of inspection 50,000

Follow-up expenses 55,000The company buys an item whose price is Rs. 12.

Solution:Cost of procurement and cost of holding are to be segregated.Cost / Expenses related to procurement Cost/Expenses related to holding

Rs. Rs.Purchase personnel salary 200,000 Stores personnel salary and expenses 1,70,000Floor rental for purchase 50,000 Obsolescence, spoilage etc. 65,000Cost of bill payment 80,000 Rent and machine handling in store 1,40,000Cost of collection material 50,000 Evaporation loss and pilferage 25,000Cost of receiving material 45,000 Interest (15% of Rs. 100 lakh) 15,00,000Cost of inspection 50,000 Insurance (1% of Rs. 100 lakh) 1,00,000Follow-up expenses 55,000 Total holding cost 20,00,000

530,000 Total Inventory Rs. 100 lakhNo. of orders placed 100 20%Cost of purchase order Rs. 530

Computation of EOQ: Unit cost : Rs. 12, annual demand : 20,000 unit.Carrying cost 20% and ordering cost Rs. 530/order The EOQ works out to 2972 units.

Problem 29: (cwa final dec.04)

The books of accounts of ABC Co. revealed that the following expenses were incurred on purchase and stores activities :(fig. in Rs.)

The annual requirement is 20,000 nos. The average inventory is Rs. 100 lakhs and it releases 1000 orders in a year. Calculate the cost of placing an order, cost of carrying inventory as a % of inventory and the EOQ.

Inventory carrying cost (Rs.20

Problem 30: (ca inter may.04)

IPL Limited uses a small casting in one of its finished products. The castings are purchased from a foundry. IPL limited purchases 54,000 castings per year at a cost of Rs.800 per casting.

Page 29: cost accounting question icwai inter

Delivery time in days 6 7 8 9 10Percentage of occurrence 75 10 5 5 5Required:(i) Compute the Economic order quantity (EOQ)

Solution: EOQ2 = (2 x 54,000 x 9,000) / 300 which gives EOQ = 1,800 units.(ii) Assuming the company is willing to take 15% risk of being out of stock::

Safety Stock = 1 day normal consumption = 54,000 / 360 = 150 castings.Reorder point = Safety stock + Average lead time x Average consumption

= 150 + ( 6 x 150) = 1050 Castings (iii) Assuming the company is willing to take 5% risk of being out of stock::

Safety Stock = 3 days normal consumption 3 x 150 = 450 castings.Reorder point = Safety stock + Average lead time x Average consumption

= 150 + ( 6 x 150) = 1050 Castings (iv) New EOQ with revised figures comes to be 300 units

The castings are used evenly throughout the year in the production process on a 360-day-per-year basis. The company estimates that it costs Rs. 9,000 to place a single purchase order and about Rs.300 to carry one casting in inventory for a year. The high carrying costs result from the need to keep the casting in carefully controlled temperature and humidity conditions, and from the high cost of insurance.

Delivery from the foundry generally takes 6 days, but it can take as much as 10 days. The days of delivery time and percentage of occurrence are shown in the following tabulation.

(ii) Assume the Company is willing to assume a 15% risk of being out of stock. What would be the safety stock? The reorder point ?

(iii) Assume the Company is willing to assume a 5% risk of being out of stock, What would be the safety stock? The reorder point?

(iv) Assume 5% stock out risk, what would be the total cost of ordering and carrying inventory for one year?

(v) Refer to the original data. Assume that using process re-engineering the company reduces its

cost of placing a purchase order to only Rs. 600. In addition company estimates that when the waste

and inefficiency caused by inventories are considered, the true cost of carrying a unit in stock is Rs.

720 per year.

(a) Compute the new EOQ. (b) How frequently would the company be placing an order, as compared to the old purchasing policy?

It implies that the company should receive the ordered quantity in 85% of time. The tabulation shows that if company receives ordered stock on seventh day, it takes the risk of being out of stock of 15%. Assuming further that the ordered quantity is delivered at the end of the day and given that the normal delivery is 6 days, the company should have safety stock for seventh day only. Thus safety stock would be one day normal consumption.

It implies that the company should receive the ordered quantity in 95% of time. The tabulation shows that if company receives ordered stock on ninth day, it takes the risk of being out of stock of 5%. Assuming further that the ordered quantity is delivered at the end of the day and given that the normal delivery is 6 days, the company should have safety stock for three day i.e. seventh, eighth and ninth days only. Thus safety stock would be three days normal consumption i.e. 150 x 3 = 450 castings.

Page 30: cost accounting question icwai inter

(ii) If the actual transportation incurred per order is Rs. 30/- determine the cost of error.Solution:EOQ at the given figures = 1,000 units (Use formula and compute yourself.)Variable cost (excluding cost of acquisition) = ordering cost + carrying costVariable cost = (2,500/1,000) x 40 + 20% x 1 x (1,000 / 2) = Rs. 200

Variable cost = (3,000/1,000) x 40 + 20% x 1 x (1,000 / 2) = Rs. 220If the correct estimate of demand were made, the eoq would have been :EOQ 2 = (2 x 3,000 x 40 ) /1 x 0.20 ; Giving EOQ = 1,095 unitsThe actual cost would have been :Variable cost = (3,000/ 1,095) x 40 + 20% x 1 (1,095/2) = Rs. 219.1Thus the cost of error = Rs. 220 -- Rs. 219.10 = Rs. 0.90

Variable cost = (2,500/1,000) x 30 + 20% x 1 x (1,000 / 2) = Rs. 175If the correct estimate of ordering cost i.e. transport cost were made, the eoq would have been :EOQ 2 = (2 x 2,500 x 30 ) /1 x 0.20 ; Giving EOQ = 866 unitsRelevant Variable cost = (2,500/866) x 30 + 20% x 1 x (866 / 2) = Rs.173.20So, the cost of error = Rs. 175 -- Rs. 173.20 = Rs. 1.80

(i) What is the optimal order quantity and the total relevant cost of this order quantity?

Solution: Solve on the lines of problem 15 above. The answers are given below: (i) EOQ = 300, the relevant cost of ordering and holding cost: Rs. 3600(ii) Revised EOQ = 231 units Cost of prediction error = (2867 -- 2772) = Rs. 95(iii) Additional cost (368,000 -- 363,600) = Rs. 4,400. Not recommended.Problem 33 (ca final June 2009)

(v) Total number of orders to be placed under revised figures comes to be 180. Thus orders to be placed at an interval of 2days (taking one year as 360 days) as against 12 days in earlier case.

Problem 31: (cwa final dec.06)

A manufacturer uses a component at the rate of 2,500 units/year. The basic cost of the component is Rs.1/-each and the marginal cost of carrying inventory is 20% of the cost of the item. The supply cost of the components have to be collected from the supplier involving an overall transport cost of Rs. 40 per trip.

(i)Assuming that the actual consumption of the components during the year was 3.000, to what extent will the manufacturer lose by working out his order quantity for a demand of 2,500?

Case 1: When the demand is 3,000 units, The variable cost would be

Case 2 :The actual cost of transport is Rs. 30/trip

Problem 32: (cwa final June 04)

In a firm, the annual demand for raw material R is 4000 units and the purchase price is expected to be Rs. 90 per unit. The incremental cost of processing an order is Rs. 135 and the cost of storage is estimated to be Rs. 12 per unit.

(ii) Suppose that the Rs. 135 estimate of the incremental cost of processing an order is incorrect and should have been Rs. 80. Assume that all other estimates are correct. What is the cost of this prediction error assuming the solution to (i) is implemented for one year?

(iii) Assume at the start of the period a supplier offers 4,000 units at a price of Rs. 86. The material will be delivered immediately and placed in the stores. Assume that the incremental cost of placing this order is zero and the original estimate of Rs. 135 for placing an order for the economic batch size is correct. Would the order be accepted?

A firm maintains a separate account for cash disbursement. Total disbursements are Rs. 2,62,500 per month. Administrative and transaction cost of transferring cash to disbursement account is Rs. 25 per transfer. Marketable securities yield is 7.5% per annum.

Page 31: cost accounting question icwai inter

Determine the optimum cash balance according to William J Baumol model.Solution: Eoq = Under root of (2 A B / C )Here : EQQ = Optimum cash balance A = Annual disbursements B = Administrative and transaction cost C = Marketable securities yield

Eoq = under root of [ 2 x ( 12 x 2 x 262,500) x 25 ] / 0.075 = Rs. 45826 /-

·       Associate yourself with people of good quality. It is better to be alone than to be in bad company.

Page 32: cost accounting question icwai inter

Safety stock : 300 units

300 + 3 x 800 = 2,700 units

yksxksa esa dkfcfy;r dh deh ugha gksrh] fnDdr ;s gS fd oks eu

Zee limited uses material A for the production of product M, the safety stock of material A is 300 units, the supplier quotes a delivery delay of two or three weeks, if the company uses 500 to 800 units a week according to

If the capacity usage ratio of a production department is 90% and the activity ratio is 99%, then the efficiency

For the manufacture of a certain product two components A and B are used. The following particulars about

Page 33: cost accounting question icwai inter

u fp<+sa u xqLlk gksa] 'kkar jgus dk vH;kl djsaA

Actual hours

(68/85) = 80%(68/81) = 84%

For the computation of eoq please see the relevant chapter.

The safety stock is 200 units, the supplier quotes a delivery delay of two to three weeks; the factory uses 400 to

In a factory, current uses of a component varies between 300 and 450 units per week and replenishment time is normally two weeks but can go up to five weeks. The minimum stock level of the component is …………….

Monthly demand is given and normal usage per week is also given. It is the consumption and not the demand that matters in EOQ formula. Thus monthly demand of 1,000 units is not relevant for computation of EOQ. Normal usage of 50 units per week is given. Thus annual consumption would be 50 x 52 = 2,600 units.

Page 34: cost accounting question icwai inter

Source 1 assures that defectives will not be more than 2% of the supplied quantity.

Source 2 does not give any assurance, but on the basis of past experience of supplies received from it, it is

The material is supplied in lots of 1000 units. Source 2 supplies the lot at a price, which is lower by Rs. 100 compared to source 1. The defective units of material can be rectified for use at a cost of Rs. 5 unit.

C developed original specification of a product and founded C Manufacturing Ltd. In 1997 the firm manufactured 980 Nos. at an average price of Rs. 900 each. In 1998 due to continuous price rise of the inputs, he raised his

In spite of price rise for the product, which sold for over Rs. 1,000 for the first time, C was surprised to learn in the late 1998 (as may be seen from the financial statements) that C Manufacturing Ltd. show a decline in earning

(iii) We are not setting aside enough to replace the machinery, we need to set aside Rs. 165,000, not Rs.

(iv) It is still not late to switch to LIFO for 1998. This will reduce closing inventory to Rs. 330,000 and raise cost of

Page 35: cost accounting question icwai inter

(ii) If the firm desires a 15% margin on sales, how much should the firm charge for the product per unit.

Gun Metal Bush

1. You should note that if LIFO method is followed as suggested, the end inventory would be Rs. 330,000 and

2. The machinery should cost Rs. 165,000 and not Rs. 150,000, consequently the depreciation charged would be

3. This is a peculiar type of question which is not related with any specific topic. It is difficult to know where to begin the solution. As the question itself mentions that the prices are going up we can consider the inflation

A crane wheel assembly is made of cast steel and two gun metal bushes. Find the cost of the assembly assuming no

Page 36: cost accounting question icwai inter

200240440

Product inspection and material inspection

A particular job requires 800 kgs of a material. 500 kgs of the particular material is currently in stock. The original price of the material was Rs. 300 but current resale value of the same has been determined as Rs. 200.

ABC ltd. initiated a quality improvement program at the beginning of the year. Efforts were made to reduce the number of defective units produced. By the end of the year, reports from the production manager revealed that scrap and rework had both decreased. Though pleased with the success, the President of the company wanted some assessment of the financial impact of the improvements. To make this assessment, the following financial

Page 37: cost accounting question icwai inter

Activity ratio : 83.3%, Actual hours : 18,000

320,000

A company manufactures two products X and Y. Product X requires 5 hours to produce while Y requires 10 hours. In June 03, of 25 effective working days of 8 hours a day, 1,000 units of X and 600 units of Y were produced. The company employs 50 workers in the production deptt. to produce X and Y. The budgeted hours

1. Budgeted hours are given per annum basis. Budgeted hours per month can be obtained by dividing it by 12. You can also divide it by 365 and then multiply by 31 day for July month also. Budgeted hours are relevant for

A company manufactures two products X and Y. Product X requires 8 hours to produce while Y requires 12 hours. In April, 2004, of 22 effective working days of 8 hours a day, 1,200 units of X and 800 units of Y were produced. The company employs 100 workers in production department to produce X and Y. The budgeted

Page 38: cost accounting question icwai inter

What other factors should also be considered before the decision is taken?

Page 39: cost accounting question icwai inter

Prepare an income statement with a separate supporting schedule of cost of goods manufactured.

Page 40: cost accounting question icwai inter

The purchase quantity options to be considered are 400, 500, 1000, 2000, and 3000 tonnes.

Lead time to supply : 6 -- 8 weeks.

Page 41: cost accounting question icwai inter

1,600 units 900 units 1,402 units

Page 42: cost accounting question icwai inter

300,000 Scrap400,000 Rework125,000 Product inspection600,000 Product warranty

80,000 Quality training40,000 Materials inspection

Page 43: cost accounting question icwai inter

400,000 300,000 Scrap600,000 400,000 Rework100,000 125,000 Product inspection800,000 600,000 Product warranty

40,000 80,000 Quality training60,000 40,000 Materials inspection

Page 44: cost accounting question icwai inter

400,000 300,000 Scrap 400,000600,000 400,000 Rework 600,000100,000 125,000 Product inspection 100,000800,000 600,000 Product warranty 800,000

40,000 80,000 Quality training 40,00060,000 40,000 Materials inspection 60,000

Page 45: cost accounting question icwai inter

300,000 Scrap 400,000400,000 Rework 600,000125,000 Product inspection 100,000600,000 Product warranty 800,000

80,000 Quality training 40,00040,000 Materials inspection 60,000

Page 46: cost accounting question icwai inter

300,000 Scrap 400,000 300,000400,000 Rework 600,000 400,000125,000 Product inspection 100,000 125,000600,000 Product warranty 800,000 600,000

80,000 Quality training 40,000 80,00040,000 Materials inspection 60,000 40,000

Page 47: cost accounting question icwai inter

300,000 Scrap 400,000 300,000400,000 Rework 600,000 400,000125,000 Product inspection 100,000 125,000600,000 Product warranty 800,000 600,000

Quality training 40,000 80,000Materials inspection 60,000 40,000

Page 48: cost accounting question icwai inter

Scrap 400,000 300,000 ScrapRework 600,000 400,000 ReworkProduct inspection 100,000 125,000 Product inspectionProduct warranty 800,000 600,000 Product warrantyQuality training 40,000 80,000 Quality trainingMaterials inspection 60,000 40,000 Materials inspection

Page 49: cost accounting question icwai inter

400,000 300,000 Scrap600,000 400,000 Rework

Product inspection 100,000 125,000 Product inspectionProduct warranty 800,000 600,000 Product warrantyQuality training 40,000 80,000 Quality trainingMaterials inspection 60,000 40,000 Materials inspection

Page 50: cost accounting question icwai inter

400,000 300,000 Scrap600,000 400,000 Rework100,000 125,000 Product inspection800,000 600,000 Product warranty

40,000 80,000 Quality training60,000 40,000 Materials inspection

Page 51: cost accounting question icwai inter

400,000 300,000 Scrap600,000 400,000 Rework100,000 125,000 Product inspection800,000 600,000 Product warranty

40,000 80,000 Quality training60,000 40,000 Materials inspection

Page 52: cost accounting question icwai inter

400,000 300,000 Scrap 400,000600,000 400,000 Rework 600,000100,000 125,000 Product inspection 100,000800,000 600,000 Product warranty 800,000

40,000 80,000 Quality training 40,00060,000 40,000 Materials inspection 60,000

Page 53: cost accounting question icwai inter

300,000 Scrap 400,000400,000 Rework 600,000125,000 Product inspection 100,000600,000 Product warranty 800,000

80,000 Quality training 40,00040,000 Materials inspection 60,000

Page 54: cost accounting question icwai inter

300,000 Scrap 400,000 300,000400,000 Rework 600,000 400,000125,000 Product inspection 100,000 125,000600,000 Product warranty 800,000 600,000

80,000 Quality training 40,000 80,00040,000 Materials inspection 60,000 40,000

Page 55: cost accounting question icwai inter

300,000 Scrap 400,000 300,000400,000 Rework 600,000 400,000125,000 Product inspection 100,000 125,000600,000 Product warranty 800,000 600,000

Quality training 40,000 80,000Materials inspection 60,000 40,000

Page 56: cost accounting question icwai inter

Scrap 400,000 300,000 ScrapRework 600,000 400,000 ReworkProduct inspection 100,000 125,000 Product inspectionProduct warranty 800,000 600,000 Product warrantyQuality training 40,000 80,000 Quality trainingMaterials inspection 60,000 40,000 Materials inspection

Page 57: cost accounting question icwai inter

400,000 300,000 Scrap600,000 400,000 Rework

Product inspection 100,000 125,000 Product inspectionProduct warranty 800,000 600,000 Product warrantyQuality training 40,000 80,000 Quality trainingMaterials inspection 60,000 40,000 Materials inspection

Page 58: cost accounting question icwai inter

400,000 300,000 Scrap600,000 400,000 Rework100,000 125,000 Product inspection800,000 600,000 Product warranty

40,000 80,000 Quality training60,000 40,000 Materials inspection

Page 59: cost accounting question icwai inter

400,000 300,000 Scrap600,000 400,000 Rework100,000 125,000 Product inspection800,000 600,000 Product warranty

40,000 80,000 Quality training60,000 40,000 Materials inspection

Page 60: cost accounting question icwai inter

400,000 300,000 Scrap 400,000600,000 400,000 Rework 600,000100,000 125,000 Product inspection 100,000800,000 600,000 Product warranty 800,000

40,000 80,000 Quality training 40,00060,000 40,000 Materials inspection 60,000

Page 61: cost accounting question icwai inter

300,000 Scrap 400,000400,000 Rework 600,000125,000 Product inspection 100,000600,000 Product warranty 800,000

80,000 Quality training 40,00040,000 Materials inspection 60,000

Page 62: cost accounting question icwai inter

300,000 Scrap 400,000 300,000400,000 Rework 600,000 400,000125,000 Product inspection 100,000 125,000600,000 Product warranty 800,000 600,000

80,000 Quality training 40,000 80,00040,000 Materials inspection 60,000 40,000

Page 63: cost accounting question icwai inter

300,000 Scrap 400,000 300,000400,000 Rework 600,000 400,000125,000 Product inspection 100,000 125,000600,000 Product warranty 800,000 600,000

Quality training 40,000 80,000Materials inspection 60,000 40,000

Page 64: cost accounting question icwai inter

Scrap 400,000 300,000 ScrapRework 600,000 400,000 ReworkProduct inspection 100,000 125,000 Product inspectionProduct warranty 800,000 600,000 Product warrantyQuality training 40,000 80,000 Quality trainingMaterials inspection 60,000 40,000 Materials inspection

Page 65: cost accounting question icwai inter

400,000 300,000 Scrap600,000 400,000 Rework

Product inspection 100,000 125,000 Product inspectionProduct warranty 800,000 600,000 Product warrantyQuality training 40,000 80,000 Quality trainingMaterials inspection 60,000 40,000 Materials inspection

Page 66: cost accounting question icwai inter

400,000 300,000 Scrap600,000 400,000 Rework100,000 125,000 Product inspection800,000 600,000 Product warranty

40,000 80,000 Quality training60,000 40,000 Materials inspection

Page 67: cost accounting question icwai inter

400,000 300,000 Scrap600,000 400,000 Rework100,000 125,000 Product inspection800,000 600,000 Product warranty

40,000 80,000 Quality training60,000 40,000 Materials inspection

Page 68: cost accounting question icwai inter

400,000 300,000600,000 400,000100,000 125,000800,000 600,000

40,000 80,00060,000 40,000

Page 69: cost accounting question icwai inter

Chapter 2 : Labour cwa inter cwa final ca inter ca final total16 15 11 1 43

Solution: Think good, feel good, speak good and all good will follow.

Time required to complete 720 units @ 5 minute / unit 60 hours.Standard time for a week: 48 hoursTime saving 12 hours% of time saving with time allowed = (12 / 60) x 100 = 20% = bonus Bonus allowed for a week = 20% of 48 hours = 9.6 hoursTotal hours of payment for a week including bonus (48 + 9.6) 57.6 hoursTotal payment (57.6 x 15) in Rs. 864.0

Examine the scheme and state whether it can be accepted.Solution: Tutorial notes:

Solution goes as follows:Particulars Without the scheme With the schemeAnnual production 120,000 units 120,000 units Standard time 2.5 hours 2.5 hoursEfficiency 60% 75%Time required =(120,000 x 2.5) / 0.60 = (120,000 x 2.5) / 0.75

500,000 hours 400,000 hoursWages rate / hour Rs. 15 Rs. 16 Total payment Rs. 75 lakhs Rs. 64 lakhsCost of training nil Rs. 10 lakhsTotal expenditure Rs. 75 lakhs Rs. 74 lakhSaving Rs. 1 lakh.

vkids pkjksa vksj ,slh laHkkoukvksa dk <sj yxk gqvk gS ftudk ykHk mBkdj vki vius lkjs vjekuksa dks iwjk dj ldrs gSA vki D;k vkSj fdruk gkfly dj ldrs gS] ;g bl ij fuHkZj gS fd vki [kqn dks fdl gn rd dkfcy le>rss gSAProblem 1:cwa inter dec.05

A worker has a time rate of Rs. 15/hour. He makes 720 units of a component (standard time : 5 minutes / unit) in a week of 48 hours. Find his total wages including Rowan bonus for the week?

Rowan bonus plan envisages the percentage of saving of time would be the percentage of bonus, over standard rate.

Problem 2 (cwa inter dec. 04)

The budgeted annual production of a company is 120,000 units, each requiring 2.5 hours at an hourly wage rate of Rs. 15. Currently the average efficiency of the production workers is only 60%. The management has a scheme to raise this to 75%. The scheme involves realigning the machinery and intensive training of the production workers, at an one time cost of Rs. 10 lakhs. The scheme also proposes to raise the wage rate to Rs. 16 to enlist the full co-operation of the workers.

It is given that each unit requires 2.5 hours for its completion. It should be understood that this time is at 100% efficiency. The workers are presently working at 60% efficiency which implies that they take (120,000 x 2.5 ) / 0.6 = 500,000 hours for the annual production. It means that the presently the workers are being paid Rs. 75 lakhs @ Rs. 15 per hour for 500,000 hours.

If the efficiency of the worker is raised to 75% they would require (120,000 x 2.5 ) / 0.75 = 400,000 hours for the production. The workers would be paid a labour rate of Rs. 16 / hr. Thus the workers would now be paid annual wages of Rs.. 64 lakhs.

Page 70: cost accounting question icwai inter

In view of the saving of Rs. 1 lakh, the scheme is recommended to be introduced and implemented.

Alternatively:

Cost saving due to Time saved with the scheme 100,000 x 15 Rs. 15 lacs.

Less : Cost of training Rs. 10 lacs

(400,000 x 1) Rs. 4 lacs

Additional benefit due to training Rs. One lacs

Solution: This can be simply solved by using the flux method formulaAccording to Flux method the labour turnover is given by the following formulaLabour turn over = Average of ( Separation + Replacement ) / Average no. of employees

= 0.50 x ( 120 + 96 ) / 0.50 (1,200 + 1,180) = 9.08%

Efficiency Incentive as % of basic wagesBelow 100% No incentive100% i.e. 4 hours / unit 10%Above 100% 1% additional incentive for every 1% increase in

efficiency above 100%, fraction excluded.

The basic wages of all the four workers is Rs. 15 per hour.

Solution:Tutorial notes :

The efficiency can be computed in three ways

Less : Additional payment due to hike in labour rates from Rs. 15 to Rs. 16

Problem 3: (cwa inter dec.03)

In a company there were 1,200 employees on the rolls at the beginning of a year and 1,180 at the end. During the year 120 persons left service and 96 replacements were made. The rate of labour turnover according to flux method is …………

Problem 4: (cwa inter dec.03)

The standard labour time required for the production of a certain component has been fixed as 4 hours. An incentive scheme was introduced recently to raise the labour productivity. The relevant details of the scheme are as follows:

Four workers A, B, C and D produced 16, 12, 14 and 10 units respectively in a particular week of 48 hours.

Calculate the efficiency, incentive bonus, total earnings and labour cost per unit in respect of each of the four workers.

Let us consider worker A for instance. He has produced 16 units in a week of 48 hours. This can be expressed in three manners as given below:

1. He has produced 16 units whereas the standard output is 12 (48 / 4) units. Thus he has produced 4 additional units with respect to standard output of 12 units. He thus enjoys additional efficiency of 4/12 i.e. 33%.

2. The other way to look at his performance is with respect to standard time. He has produce 16 units in 48 hours time. Thus instead of taking a time of 16 x 4 = 64 hours he has actually taken a time of 48 hours. Thus he has saved 16 hours of time with respect to standard time of 48 hours. he thus enjoys an additional efficiency of 16/48 i.e. 33%.

3. The third way is to compute the actual time taken to produce the units. He has produced 16 units in 48 hours. He has thus taken 48/16 i.e. 3 hours for each unit where the standard is 4 hours/unit. He saves one hour for every three standard hour of working. He enjoys 1/3 i.e. 33%.

Page 71: cost accounting question icwai inter

The solution goes as follows:The statement showing the efficiency of the three workers : fig. in units.Worker Standard output Actual output Additional output Efficiency Bonus %A 12 16 4 133% 10 + 33B 12 12 0 100% 10C 12 14 2 116% 10 + 16D 12 10 0 83% 0Statement showing the total wages and labour cost per unit of componentWorker Wages Incentive Total Output Labour cost

Rs. % Amount Earnings units per unit Rs..A 720 43 309.6 1029.6 16 64.4B 720 10 72.0 792.0 12 66.0C 720 26 187.2 907.2 14 64.8D 720 0 0.0 720.0 10 72.0

He carried out the following work in a 48-hour week.Job 1 1500 items at 4 hours per 1000 .Job 2 1800 items at 3 hours per 1000Job 3 9000 items at 6 hours per 1000

Job 4

Job 5 2000 items at 8 hours per 1000, each item was estimated to be half finished.

Solution:Calculation of total hours of working by the worker:Job Description Time taken in hoursJob 1 1500 items at 4 hours per 1000 6.0Job 2 1800 items at 3 hours per 1000 5.4

Add; 1/9 as allowance 0.6 6.0Job 3 9000 items at 6 hours per 1000 54.0Job 4 Actual time taken plus 25% bonus 4.0

Bonus 1.0 5.0Job 5 2000 items at 8 hours per 1000, (half finished) 8.0Total time taken by the worker in hours 79.0Total time available for working in a week 48.0Less : Time lost due to power cut 4.0 44.0Time saving for a 48-hour week (79 -- 44) 35.0

4. In all cases the efficiency should be computed with respect to standard time allowed for the task or standard number of units to be produced. The efficiency should not be calculated with respect to actual time taken or actual number of units produced.

Problem 5: (cwa inter dec.01)

Rowan bonus plan envisages the percentage of saving of time would be the percentage of bonus, over standard rate.

A worker, whose day-work wages is Rs. 2.50 an hour, received production bonus under the Rowan Scheme.

1500 items for which no standard time was fixed and it was arranged that the worker would be paid a bonus of 25%. Actual time on the job was 4 hours.

Job 2 was carried out on a machine running at 90% efficiency and extra allowance of 1/9 of actual time was given to compensate the worker.

4 hours were lost due to power-cut. Calculate the earnings of the worker, clearly stating your assumptions for the treatment given by you for the hours lost due to power cut.

Page 72: cost accounting question icwai inter

Bonus entitlement ( 35/79 x 44) 19.49Total hours of payment for a week including bonus (48 + 19.5) 67.5Payment @ Rs. 2.50/hr for 67.50 hours Rs. 168.75

No. of workers employed 100Weekly working hours (including lunch break) 48Average no. of hours lost due to idle time per week per employee 8Standard time required per unit 2 hoursHourly rate of payment in Rs. 15Current level of efficiency 80%

Evaluate the impact of the decision on firm's profit.Solution:Tutorial notes:

A comparative statement needs to be prepared about the current operations and the status after the agreement.

Particulars Current After the agreementTotal hours available per week (48x100) 4,800 4,800Less : Idle time hr/employee/week 8 4Idle time in hours 800 400Actual hours of working 4,000 4,400Standard time required per unit 2 2Standard production at 100% efficiency (4000/2) 2,000 (4400/2) 2,200Level of efficiency 0.8 0.9Actual production at efficiency level (2000x0.8) ( 2200 x 0.9)

1,600 1,980Cash profit before labour cost (120 x 1600) (120 x 1980)

192,000 237,600Labour cost at Rs./hour 15 18Total labour cost (15 x 4800) (18 x 4800)

72,000 86,400Net cash profit 120,000 151,200

Problem 6: (cwa inter dec.00)

A manufacturing firm makes a standard electronic component used in television sets. The details of current operations of the firm are as follows:

For every unit sold, the company is getting a cash profit of Rs. 120 before charging labour cost {i.e. surplus of sales value over cost of production (only cash expenses ) excluding labour cost.}

In view of the increasing demand for the product, the firm came to an agreement with the Labour union to raise the wage rate by Rs. 3 per hour in return for the workers reducing the idle time by 4 hours and raising the operational efficiency to 90%.

Nothing has been mentioned about the lunch break. It remains same in both the cases, thus it becomes irrelevant for the evaluation purposes.

Recommendation : The cash profit goes up from Rs. 120,000 to Rs. 151,200 i.e. by Rs. 31,200 because of the agreement. It is recommended to implement the agreement.

Problem 7: (cwa inter june.00)

The employees in a manufacturing unit are paid wages at the rate of Rs. 7 per hour for an eight hour shift. Each employee produces 5 units per hour. The overhead in this department is Rs. 10 per direct labour hour. Employees and the management are considering the following piece rate wage proposal:

Page 73: cost accounting question icwai inter

Up to 45 units per day of 8 hours Rs. 1.30 per unit.From 46 units to 50 units Rs. 1.60 per unit.From 51 units to 55 units Rs. 1.65 per unit.From 56 units to 60 units Rs. 1.70 per unit.Above 60 units Rs. 1.75 per unit.

Solution:Statement indicating benefits to the employees on piece rate proposalWages = Rs.7/hour for 8 hours.

Output Wages Piece rate wages Benefit tounits Rs. per unit per day employee

40 56 1.30 52.00 (4)45 56 1.30 58.50 2.5055 56 1.65 90.75 34.7560 56 1.70 102.00 46.00

Statement indicating benefits to the management on piece rate proposalPresent conversion cost is Rs. 17 (7 +10) per hour. Employee produces 5 unit per hour.Present conversion cost per unit comes to be Rs. 17/5 = Rs. 3.40.

Output Overhead

units Rs.10/hr40 1.30 52.00 80 132.00 136 4.0045 1.30 58.50 80 138.50 153 14.0055 1.65 90.75 80 170.75 187 16.2560 1.70 102.00 80 182.00 204 22.00

The profitability position of Target Ltd. For the year ending 31.3. 06 is as underAnnual Turnover (fig. in lakhs) 200Variable costs : Direct material 60

Direct labour 40Variable Overhead 50 150Marginal contribution 50Fixed overhead 10Profit 40

Analysis of the data revealed the following: fig. in direct labour hoursPermanent worker worked during the year 960,000Apprentice workmen worked 80,000

10,40,000

You are required to calculate the loss of profit on account of loss of production from labour turnover.Solution:

The working hours are restricted to 8 hours per day. Overhead rate does not change with increased production.

Prepare a statement indicating advantages to the employees as well as to the management at production of 40,45,55 and 60 units.

Proposed piece rate

Piece wages

Proposed total cost

Present wages @

Rs. 3.40/unit

Savings per day

Problem 8: (cwa inter june.06)

The profit for the year did not match with company's expectations and works management attributed it to labour turnover.

The effectiveness of direct labour hours put in by apprentice workmen was 50% and delay in replacing against separation during the year resulted in loss of 20,000 direct labour hours.

Page 74: cost accounting question icwai inter

Calculation of total direct labour hours effective for production :Permanent worker worked during the year 960,000Effective direct labour hours for apprentice worker (50% efficiency ) 40,000

Total 10 lakh hoursAnnual turnover = Rs. 200 lakh. Sales per direct labor hour = 200 / 10 = Rs. 20Loss of production hours For replacement 20,000 hrs.

For apprentices 40,000 hrs.total 60,000 hrs.

Total loss of sales due to loss of production hours ( 20 x 60,000) Rs. 12 lakhs.Loss of potential sales 12,00,000 12,00,000Less Variable cost Material ( 60 / 200 ) x 12 3,60,000

Labour ( 40 / 10.40) x 20 76,923Overhead ( 50 / 200) x 12 3,00,000 736,923

Loss of contribution due to labour turnover Rs. 463,077Alternatively :Potential profit with no labour turnover : fig. in Rs. LakhsPotential Sales (200 + 12) 212.00Less variable cost Material ( 60 + 60/200 x 12) 63.60

Labour ( 40 + 40/10.4 x 20,000) 40.77Overhead ( 50 + 50/200 x 12) 53.00 157.37

Potential contribution 54.63Less Fixed cost 10.00Potential Profit 44.63Actual profit 40.00Loss of profit due to labour turnover 4.63

Solution: Rowan bonus = (time saved / time allowed) x hour worked x rateRowan bonus =[ { ( 300 x 10 ) / 60 -- 40 } / 50 ] x 40 x 15 = Rs. 120.Total earnings = Rs. 120 x 5 + 120 = Rs. 720.

(i) Standard production per month per worker : 2000 units with piece rate of Rs. 0.50/ unit.(ii) Production bonus to be given as follows: Upto 85% efficiency : Nil

Incentive bonus at Rs. 40 for every 5% increase above 85%

Above 100% efficiency :

Solution: Standard production per month : 2000 unitsStatement of total earnings for the month of January, 2001

Worker Output wages Efficiency Incentive Total earningsin units @ Rs. 0.50 Rs. Rs.

Problem 9: (cwa inter Dec.06)

In a factory where piece work system is followed with guaranteed minimum wages of Rs.120 (for 8 hrs.), incentive payments are made according to Rowan Bonus Scheme. The std. time per unit is 10 minutes. If in a five-day week of 40 working hours the actual production is 300 units, the total earnings of the worker is ...

Problem 10: (cwa inter June.01)

Calculate the total monthly remuneration of three workers P,Q and R who are working in a factory, based on the following data:

Between 85% and 100% efficiency :

Incentive bonus at Rs. 40 for every 5% increase above 85% plus 20% additional bonus on the incentive earned.

(iii) P, Q and R had a production of 1600 units, 2000 units and 2200 units respectively during January 01.

Page 75: cost accounting question icwai inter

P 1,600 800 80% 0 800Q 2,000 1,000 100% 120 1,120R 2,200 1,100 110% 240 1,340Working notes: Incentive of Q = 3 x 40 = Rs. 120.Incentive for R = 5 x 40 + 20% of ( 5 x 40 ) = Rs. 240.

If the customer is prepared to pay Rs. 800 for the order, should the order be accepted ?Solution: Try yourself.Labour : Rs. 150; Variable overheads : Rs. 225 and Opportunity cost : 50 x 10 = Rs. 500.Total cost : Rs. 875. Revenue receivable : Rs. 800. The order should not be accepted.

Solution: This is very easy question. Try to solve it without pen and calculator.Particulars Current level After incentive schemeSales 200 units @ Rs. 8 1,600 300 units @ Rs. 7 2,100Cost of material Rs. 400 600Cost of labour Rs. 200 300Cost of overheads Rs. 800 800Cost of training Rs. 0 100Total cost Rs. 1,400 1,800Profit: Rs. 200 300Hence the incentive scheme should be introduced.

Solution: blh oDr vPNk eglwl dhft;s A vHkh eqLdqjkb;sA

Calculation for price to be quoted: Rs.

Material and other processing cost in Rs. 11,250Labour cost @ Rs. 100 for 25 hours 2,500Total cost 13,750Mark up @ 20% of cost 2,750Add: Opportunity cost of 25 hours @ Rs. 200 5,000

Problem 11: (cwa inter II Dec.2000)

A machine shop in a factory is working to its full capacity and earning a contribution of Rs. 50 per hour. The management receives a high priority order which it wants to execute immediately. Material will be supplied by the customer and the order will take minimum of ten hours. Wages payable will be Rs. 15 per hour and variable overheads will be 150% of labour cost.

Problem 12 (cwa inter II June.2002)

A product is manufactured at the rate of 200 units per day and sold for Rs. 8 each. Direct materials and direct labour are Rs. 2 and Re. 1 respectively per unit. Overheads are Rs. 800 per day. If the selling price can be reduced by Re.1 per unit, it is expected that 50% more units can be sold. The workers are prepared to produce 50% more, only if there is proportional increase in their earnings. A suitable incentive scheme would cost Rs. 100 more per day to administer. With appropriate calculations, justify if the company should introduce such an incentive scheme.

Problem 13 (cwa inter II June.2002)

A highly skilled technician is paid Rs. 100 per hour and is fully engaged in the manufacture of a certain product which earns a contribution of Rs. 200 per hour to the firm.

The firm has received an order which will require the services of the technician for 25 hours. If the material and other processing costs amount to Rs. 11,250 and the mark up 20% on cost, what price should be quoted for the new order ?

By accepting the new order the firm is scarifying 25 hours of contribution which is Rs. 5000 This should be added to the cost. The opportunity cost of course, does not qualify for mark up of 20% on cost.

Page 76: cost accounting question icwai inter

Price to be quoted 21,500

Solution: Calculation of earning of workers : Day wages basisWorker Output Day wages Labour cost per 100 unitsAchyuta 180 6 3.33Ananta 120 6 5.00Govinda 100 6 6.00Total 400 18Average cost of labour per 100 unit: total wages / total output = (18/400 ) x100 = Rs. 4.50

Calculation of earning of workers : Piece rate basisWorker Output Piece rate Rs. Wages earned Rs. Cost per 100Achyuta 180 0.075 13.50 7.50Ananta 120 0.075 9.00 7.50Govinda 100 0.075 7.50 7.50Total 400 30.00

Computation of Earnings as per two incentive plans

Hasley Plan Rowan PlanAchyuta Ananta Govinda Achyuta Ananta Govinda

Output in units 180 120 100 180 120 100Std. time for actual output 18 12 10 18 12 10Actual time hrs. 8 8 8 8 8 8Time saved hrs. 10 4 2 10 4 2Bonus* time awarded 5 2 1 4.44 2.67 1.60Total time for payment 13 10 9 12.44 10.67 9.6Total payment @ Rs. 0.75/hr 9.75 7.5 6.75 9.33 8.00 7.2Earnings per 100 pieces 5.42 6.25 6.75 5.18 6.67 7.20Bonus 1. 50% of time saved in Hasley plan and

2. (time saved/time allowed) x time taken as per Rowan plan.

Solution:Let us first calculate the average number of workers on role:Labour turn over = Average of ( Separation + Replacement ) / Average no. of employees

= No. of replacements / Average no. of employeesThe labour turnover under replacement method has been given as 5%.No. of replacement in the quarter is given as 30. Putting these values in the formula above:Average no. of employees = 600.

Problem 14 (ca inter )

The case of three workers Achyuta, Ananta and Govinda who produce respectively 180, 120 and 100 units of the company's product in a normal day of 8 hours is taken up for study.

Assuming that day wages would be guaranteed at 75 paise per hour and the piece rate would be based on a standard hourly output of 10 units,

Calculate the earnings of each of the three workers and the labour cost per 100 pieces under (i) Day wages, (ii) Piece rate (iii) Halsey scheme (iv) Rowan scheme.

Also calculate under the above schemes the average cost of labour for the company to produce 100 pieces.

Problem 15 (ca inter may 97)

The cost accountant of Y Ltd. has computed labour turnover rates for the quarter ended 31st March, 1997 as 10%, 5% and 3% respectively under Flux method, Replacement method, and Separation method. If the number of workers replaced during that quarter is 30, Find out the number of (1) workers recruited and joined and (2) worker left and discharged.

Page 77: cost accounting question icwai inter

No. of separations = labour turnover under separation method x Ave.no. of worker.labour turnover under separation method has been given as 3%.Thus we have no. of separations = 3% of 600 = 18(1) Labour turnover rate (Flux method) is given as 10%. We can write Labour turnover = 10% = ( no. of separation + no. of accessions) / average no. of workersAverage no. of employees = 600; no. of separation = 18; Putting these values we get No. of accessions as 42.

Hourly wage rate Guaranteed Rs. 30 Actual production during the month in units 6,120 unitsAverage time for producing one unit by one worker at the previous performance ( hours)(This may be taken as time allowed) 1.975 hrs.No. of working days in a month 24 No. of working hours in a day 8Required:1. Calculate the effective rate of earnings under the Halsey scheme and the Rowan scheme.2. Calculate the savings to the ZED limited in terms of direct labour and cost per piece.3. Advise ZED limited about the selection of the scheme to fulfill their assurance.Solution:Working note : Computation of time saved in hours during the month of April 2004Standard time allowed for production of 6,120 units @ 1975 hrs./unit Standard time allowed = (6,120 x 1.975) 12,087Actual time take ( 24 days x 8 hours x 50 workers) 9,600Time saved 2,487Bonus under Halsey scheme = 50% x 2,487 hours x Rs. 30 = Rs. 37,305Bonus under Rowan scheme = (2,487 / 12,087) x 9,600 x Rs. 30 = Rs. 59,258.(i) Calculation of effective rate of earnings under the Halsey scheme:Time wages = 24days x 8 hours x 50 workers x Rs. 30 = 24 x 8 x 50 x 30 = Rs. 2,88,000

Halsey RowanTime wages (same in both schemes) Rs. 288,000 288,000Bonus as calculated above Rs. 37,305 59,258Total earnings 325,305 347,258Increase in % over the present earnings 13.0% 20.6%Actual time taken in hours 9,600 9,600Effective earnings per hour 33.89 36.17No. of units produced 6,120 6,120Effective earnings per unit 53.15 56.74Standard cost allowed ( Rs. 30 x 1.975 hours) 59.25 59.25Savings to ZED limited 6.10 2.51

Problem 16 (ca inter may 04)

ZED Limited is working by employing 50 skilled workers. It is considered the introduction of incentive scheme-either Halsey scheme (with 50% bonus) or Rowan scheme of wages payment for increasing the labour productivity to cope up the increasing demand for the product by 40%. It is believed that proposed incentive scheme could bring about an average 20% increase over the present earnings of the workers ; it could act as sufficient incentive by them to produce more.

Because of assurance, the increase in productivity has been observed as revealed by the figures for the month of April 2004

Advise to ZED limited: Rowan scheme should be implemented because it fulfills the promise of the management to give 20% increase over present earnings.

Problem 17 (ca inter may 02)

Page 78: cost accounting question icwai inter

Workers A B CActual hour worked in a week 38 40 34Hourly rate of wages in Rs. 6 5 7.2Production in units Product P 21 0 60

Product Q 36 0 135Product R 46 25 0

Standard time allowed per unit of each product is ( in minutes) P Q R12 18 30

For the purpose of piece rate, each minute is valued at Rs. 0.10.You are required to calculate the wages of each worker under :(i) Guaranteed hourly rates basis;

(iii) Premium bonus basis where the worker receives bonus based on Rowan scheme.Solution:(i) Computation of wages under guaranteed hourly rate basis:

Workers A B CActual hour worked in a week 38 40 34Hourly rate of wages in Rs. 6 5 7.2Total wages ( rate x hours) Rs. 228 200 244.8(ii) Computation of wages under piece rate basisStandard time allowed per unit of each product is ( in minutes) P Q R

12 18 30Standard rate per piece in Rs. ( time x Rs. 0.10) 1.2 1.8 3.0

A B CProduct Rate units wages units wages units wages

P 1.2 21 25.2 0 0 60 72Q 1.8 36 64.8 0 0 135 243R 3.0 46 138.0 25 75 0 0

(iii) Computation of Bonus with Rowan schemeYou must calculate time allowed and time saved to determine the bonus payable.

A B C Productme allowed units Time units wages units wages

P 12 21 252 0 0 60 720Q 18 36 648 0 0 135 2,430R 30 46 1,380 25 750 0 0

Total time in minutes 2,280 750 3,150Total time in hours 38 12.5 52.5Actual time taken 38 40.0 34.0Time saved hours 0 0 18.5Wage rate Rs. / hr. 6 5 7.2Earnings (rate x time) 228 200 244.8Bonus as per Rowan scheme* 86.3* (18.50/52.50) x 34 x 7.20 331.1

A company is undecided as to what kind of wage scheme should be introduced. The following particulars have been complied in respect of three systems, which are under consideration of the management.

(ii) Piece work earnings basis, but guaranteed at 75% of basic pay (guaranteed hourly rate) if his earnings are less than 50% of basic pay.

Problem 18 (ca inter may 02)

Page 79: cost accounting question icwai inter

Solution: Let the machine man work for T hours @ Rs. 37.50 per hour.Under Rowan plan, the total earnings are given byEarnings = Rate x hours worked + (time saved / time allowed) x hour worked x ratePutting the values we get 37.5 x T = T x Rs. 30 + [(4--T) / 4] x T x Rs. 30 This gives T = 3 hours.Under Halsey scheme (50%)Earnings = Rate x hours worked + 50% x hour saved x rateEarnings = 30 x 3 + 50% x 1 hour x Rs. 30 = Rs. 105 Thus Effective rate = 105/3 = Rs. 35/hr

Solution: blh oDr vPNk eglwl dhft;s A vHkh eqLdqjkb;sA

Time allowed per unit = 1.20 x 15 = 18 minutes. Time allowed for weekly production of 200 units @ 18 minutes per unit Time allowed to the worker for the job = 200 x 18 = 3,600 minutes i.e. 60 hoursTime actually taken by worker for the job 45 hoursTime saved hours 15 hoursEarnings under Halsey plan = 81 + 50% x 15 x (81/45) = Rs. 94.50Effective earnings under Halsey plan = 94.50/45 hours = Rs. 2.10 per hour.Earnings under Rowan plan = 81 + (15/60) x 45 x (81/45) = Rs. 101.25 i.e. Rs. 2.25/hr.

From the following information, calculate Labour turnover rate and labour flux rate:No. of workers as on 01. 01.2000 = 7,600 and on 31.12.200 = 8,400

Solution: Labour turnover rate under various methods is as follows:Separation method LT rate = (no. of workers left + no. of workers discharged)

Average no. of workers= ( 80 +320) / Average of (7,600 + 8,400) = 5%Replacement method LT rate = (no. of workers replaced) / Average no. of workers

= 300 / 8,000 = 3.75%Flux method LT rate = (separations + accessions) / Average no. of workers

= (400 + 1500) / 8,000 = 23.75%New recruitment = New recruitment / average no. of workers = 1200/8000 = 15%

A skill worker is paid a guaranteed wage rate of Rs. 30 per hour. The standard time per unit for a particular product is 4 hours. R a machine man has been paid wages under the Rowan plan and he had earned an effective hourly rate of Rs. 37.50 on the manufacture of that particular product.

What could have been his total earnings and effective hourly rate had he been put on Halsey incentive scheme (50%)

Problem 19 (ca inter may 95)

A worker produced 200 units in a week's time. The guaranteed weekly wage payment for 45 hours is Rs. 81. The expected time to produce one unit is 15 minutes which is raised further by 20% under the incentive scheme. What will be the earnings per hour of that worker under Halsey (50% sharing) and Rowan bonus schemes ?

For computation of earnings, time allowed and time taken for the job should be known. The time allowed is 20% more of the expected time of 15 minutes.

Problem 20 (ca inter may 01)

During the year, 80 workers left while 320 workers were discharged, 1,500 workers were recruited during the year of these,300 workers were recruited because of exits and the rest were recruited in accordance with expansion plans.

Problem 21 (ca inter nov.97)

Page 80: cost accounting question icwai inter

Solution:The hourly wage rate is same for both the workers. Let us assume it is Rs. K per hour.Earnings of A as per Hasley plan = 32 x K + 4 x K = 36KEarnings of B as per Rowan plan = 30 x K + (10/40) x 30 x K = 37.5 K.Cost of the job = Raw materials cost + labour + overheads Overheads are absorbed @ Rs. 7.5 per hour worked.Cost of the Job as per worker A = Raw materials + 36K + 7.5 x 32 = Rs. 2.600Cost of the Job as per worker B = Raw materials + 37.5K + 7.5 x 30 = Rs. 2,600Solving these two, one gets K = Rs. 10 per hour and Raw materials = Rs. 2,000.

Settlement cost due to leaving Rs. 27,420Recruitment cost Rs. 18,725Training costs Rs. 16,105Selection costs Rs. 12,750

Solution:The actual direct labour hours worked by the labour force 345,000 hoursLess: Unproductive training hours (half of 30,000 hours) 15,000 hours

Actual productive hours 330,000 hoursTotal sales is Rs. 66,00,000 Sales per productive hours :(66,00,000 / 330,000) 20Sales foregone due to loss of 75,000 productive hours (20 x 75,000) 15,00,000Contribution foregone @ 20% of sales 300,000Computation of Profit foregone due to labour turnover Rs. Contribution foregone 300,000Add: Cost incurred in consequent to labour turnover

Settlement cost due to leaving 27,420Recruitment cost 18,725Selection costs 12,750Training costs 16,105

A job can be executed either through A or B. A takes 32 hours to complete the job while B finishes it is 30 hours. The standard time to finish the job is 40 hours.

The hourly wage rate is same for both the workers. In addition workman A is entitled to receive bonus according to Hasley plan(50%) while B is paid bonus as per Rowan plan. The works overheads are absorbed on the job at Rs. 7.50 per labour hour worked. The factory cost of the job is Rs. 2,600 irrespective of the workman engaged.

Find out the hourly wage rate and cost of raw materials input. Also show cost against each element of cost included in factory cost.

Problem 22 (ca inter may 98, similar nov.04, cwa final dec.2000)

The management of Sunshine Ltd. wants to have an idea of the profit foregone as a result of labour turnover last year.

Last year sales accounted to Rs. 66,00,000 and the pv ratio was 20%. The total number of actual hours worked by the direct labour force was 3.45 lakhs. As a result of the delays by the Personnel department in filling vacancies due to labour turnover 75,000 potential productive hours were lost.

The actual direct labour hours included 30,000 hours attributable to training new recruits, out of which half of the hours were unproductive. The costs incurred consequent on labour turnover revealed on analysis the following :

Assuming that the potential production lost due to labour turnover could have been sold at prevailing selling price, ascertain the profit foregone/lost last year on account of labour turnover.

Page 81: cost accounting question icwai inter

Total profit foregone 375,000

Solution:Let us calculate the bonus payable under both the schemes viz.Halsey and RowanTotal available hours per week : (60 workers x 40 hours) 2,400Total standard hour required to produce 19,200 unit @ 6 units per hour. (19200/6) 3,200Total standard hour required to produce 19,200 unit @ 5 units per hour. 2,400Time saved due to introduction of incentive scheme 800Earnings as per Halsey scheme : 0.50 x 800 x Rs. 10 Rs. 4,000Earnings as per Rowan scheme (800/3200) x 2400 x Rs. 10 Rs. 6,000Profit statement under Incentive schemes

Current Halsey RowanSales revenue ( 19,200 units @ Rs. 11/unit) 211,200 211,200 211,200Direct materials ( 19,200 units @ Rs. 8/unit) 153,600 153,600 153,600Direct labour hours 3,200 2,400 2,400Direct wages @ Rs. 10/hr. 32,000 24,000 24,000Overtime premium (not allowed with incentive schemes) 4,000Variable overheads @ Rs. 0.50/hr. 1,600 1,200 1,200Fixed overheads 9,000 9,000 9,000Add Bonus 4,000 6,000Total cost 200,200 191,800 193,800Total profit 11,200 19,400 17,400

Machine Standard time in minutes M/c utilisation factor1. Sawing 3.0 0.802. Facing and Centering 2.5 0.803. Lathe 4.0 0.754. Milling 5.0 0.805. Drilling 6.0 0.90Tutorial notes :

Problem 23 (ca inter may 02)

The finishing shop of a company employs 60 direct workers. Each worker is paid Rs. 400 as wages per week of 40 hours. When necessary, overtime is worked upto a maximum of 15 hours per week per worker at time rate plus one-half as premium. The current output on an average is 6 units per man hour which may be regarded as standard output.

If bonus scheme is introduced, it is expected that the output will increase to 8 units per man hour. The workers will, if necessary, continue to work overtime upto the specified limit although no premium on incentives will be paid.

The company is considering introduction of either Halsey scheme or Rowan scheme of wages incentive system. The budgeted weekly output is 19,200 units. The selling price is Rs. 11 per unit and the direct materials cost is Rs. 8 per unit. The variable overheads amount to Rs. 0.50 per direct labour hour and the fixed overhead is Rs. 9,000 per week.

Prepare a statement to show the effect of the company's weekly profit of the proposal to introduce (a) Hasley scheme (b) Rowan scheme.

Problem 24 (cwa inter Dec.04)

An automobile component gets processed on a number of machines for which the standard times and the machine utilization factors are given below :

Find the capacity in the set-up per day, considering two 8-hour shifts per day and the average efficiency of the workmen being (1) 100% and (2) 80%.

Page 82: cost accounting question icwai inter

Solution:Machine Std.time Utilisation Actual time taken

100% 80%1. Sawing 3.0 0.80 3.75 4.692. Facing and Centering 2.5 0.80 3.13 3.913. Lathe 4.0 0.75 5.33 6.674. Milling 5.0 0.80 6.25 7.815. Drilling 6.0 0.90 6.67 8.33The longest time is required for drilling at both the efficiency levels.Set-up capacity at efficiency level of 100% (2 x 8 x 60)/ 6.67 144Set-up capacity at efficiency level of 80% (2 x 8 x 60)/ 8.33 115

Output target Incentive rate80% to 90% 10%90% to 100% 12.50%100%to 110% 15%110% to 120% 17.50%120% and above 20%

Solution: WEEKS1st 2nd 3rd 4th

Actual output in no. 383 442 350 318Standard output in no. 400 400 400 400Efficiency % (Actual / Standard) 95.8% 110.5% 87.5% 79.5%Basic wages @ Rs. 5/unit Rs. 1915 2210 1750 1590Dearness allowance in Rs. Rs. 120 120 120 120Total Rs. 2035 2330 1870 1710Incentive % 12.50% 17.50% 10% nilIncentive Rs. 254.375 407.75 187 nil

1. Determine the number of furnaces required.

1. The set-up can be commenced when all the operations are completed. Thus the operation taking the longest time would be decisive factor to determine the number of set-ups.

2. Standard time is at 100% efficiency and utilisation. This will go on increasing with decrease in efficiency as well as utilisation of machine.

Problem 25 (cwa final June.04)

The targeted weekly output of a manufacturing unit employing 20 workers is 400 pieces. The group is entitled to earn an incentive @ 10% on the aggregate of wages based on basic piece rate plus dearness allowance ( which is Rs. 120 per week) upon achievement of a minimum of 80% of the output target. This incentive rate is increased by 2.5% flat for every 10% increase in achievement of targets up to a maximum of 10% at the level of 120% of the output target in the following manner ;

During the four weeks in February, the actual output achieved by the workers are 383 pieces, 442 pieces, 350 pieces and 318 pieces respectively. The average basic piece rate is Rs. 5. Compute the amount of incentive earned by the group during each of the four weeks.

Problem 26 (cwa final dec.01, June.04)

On anticipating the current booming market, the Vajpayee Steels is planning to expand its capacity by adding more numbers of 10 ton capacity Electric Arc Furnace (EAF). Each batch of production (10 ton) involves 30 minutes furnace time including loading and unloading operations. Due to power restriction the furnace can be used only for 80% of the time. The required output for the capacity added is 160 ton per shift of 8 hours. The plant efficiency is 50%.

Page 83: cost accounting question icwai inter

2. Estimate the percentage of idle time in furnace.

Solution: blh oDr vPNk eglwl dhft;s A vHkh eqLdqjkb;sA1. Time available per shift ( 80%) in minutes (8 x 0.80 x 60) 384Batches available @ 30 minutes per batch (384/30) 12.8Output per batch @ 50% of efficiency (10 ton x 50%) 5.0Total output per furnace (12.8 x 5) 64No. of furnaces required for 160 ton output (160 / 64) 2.5 say 3.Alternatively :The required system capacity = 160 ton per shift / 50% 320 tonsCapacity per hour for (8 x 80%) hours = 320 / (8 x 0.80) 50 ton/ hourIndividual furnace capacity = 10 ton / 0.50 20 ton/hourNo. of furnaces required (50 ton per hour / 20 ton per hour 2.5 say 32. Determination of idle timeTotal hours available per shift 3 furnace x 8 hours 24 furnace-hoursTotal hours of actual use per shift 160 ton x 0.50 / 10 ton 8 furnace-hoursIdle time 16 furnace-hoursIdle time as percentage (24/16) 66.67%

P -- 400; Q -- 300; R -- 180; and S -- 150.(i) What are the earnings of each worker on this day ?(ii) What is the average labour cost of packing on this day ?

Solution: Statement showing wages payable (Minimum Rs. 40/day) in Rs. Worker Cakes packed Wage @ Rs. 0.25/cak Wage payable Productivity %P 400 100 100 83.3%Q 300 75 75 62.5%R 180 45 45 37.5%S 150 37.5 40 31.3%

1030 260Productivity = Actual cakes / Standard cakes of 480(ii) Average labour cost of packing = Wages paid / Actual packing = 260/1030 = Rs. 0.25.(iii) Productivity is shown in the table above.

Tutorial notes : This has one line solution as given below. Try to understand it.Productivity = [(380,000+13650x16) / 50x400x44x0.85]= 80%Solution: Working hours per week = 8 x 5 + 4 = 44 hours

Problem 27 (cwa final dec.06)

The packing section in a detergent factory operates under the guaranteed piece rate system of wage payment. The piece rate per cake packed is 25 paise. The guaranteed wage per day (of 480 minutes) is Rs. 40. On a typical day, the number of cakes packed by each of the four workers P, Q,R and S in the packing department is as follows :

(iii) What is the labour productivity of each worker if the standard time for packing each cake is one minute ?

Problem 28 (cwa final June 07)

A radial bearing manufacturing unit produces three types of bearings namely X,Y and Z. During the year 2006-07, the unit manufactured 20000,10000 and 13650 nos. of X,Y and Z respectively. The standard man-hour (SMH) of X,Y and Z are 12,14 and 16 hours respectively.

The unit worked for 8 hours per day single shift for Monday to Friday and 4 hour shift on Saturday. There were 50 working weeks during the year. The ratio of piece workers to Day workers among industrial employees was 1:04 and total no. of industrial employees were 560. Assuming the average absenteeism of 15% among industrial employees, what was the productivity of piece workers during the year 2006--07.

Page 84: cost accounting question icwai inter

No. of workers : industrial workers ratio 1:0.4 (560/1.4) 400Input hours with absenteeism (50 x 44 x 400 x 0.85) 748,000 hoursStandard hours required (20,000 x 12 + 10,000 x 14 + 13,650 x 16) 598,400 hoursProductivity = (598,400 / 748,000) = 80%.

Time in minutesTurning Grinding Milling Heat treating Sales O.Stock C.Stock

Drills 16 10 4 3 18,000 5,000 2,000Cutters 34 8 5 3 20,000 6,000 3,000Reamers 40 17 8 3 15,000 0 4,000

Using the above data : Calculate : (i) The production quantity.(ii) No. of operations required per annum and ;(iii) Annual direct labour cost on each type of tool.

Solution : HIPROD Company Ltd.(i) Statement of Quantity of production in units

Drills Cutters ReamersSales : 18,000 20,000 15,000Add : Closing stock 2,000 3,000 4,000Less Opening stock 5,000 6,000 0Production Quantity 15,000 17,000 19,000(ii) Statement of no. of operators per annum

Time required in hoursProduction Turning Grinding Milling Heat treat. Total

Drills 15,000 4,000 2,500 1,000 750 Err:522Cutters 17,000 9,633 2,267 1,417 850 Err:522Reamers 19,000 12,667 5,383 2,533 950 Err:522

26,300 10,150 4,950 2,550 Err:522Hours available for working 2,200 2,200 2,200 2,200No. of operators required 12 5 3 2 22(iii) Statement of Direct labour hour cost in Rs.

Operation Hours Amount @ Rs. 11Drills 8,250 90,750Cutters 14,167 155,833Reamers 21,533 236,867

483,450

Problem 29 (cwa final June 07)

HIPROD Company Ltd. manufactures three different items of tools. The time required to produce each tool on different operations is as given below :

The workers are trained in each trade as such their services are not inter-changeable. They are paid at Rs. 11.00 per hour. The workers are paid for 2500 hours per annum which includes 200 hours for leave in which time substitute operators are appointed and for 300 hours the machines are taken for overhaul.

Problem 30 (cwa final June 00)

A company selling goods worth about Rs. 100,000 annually has three factory employees. The foreman is paid Rs. 100 per week. The other two unskilled workers are each paid Rs. 80 per week. The three men work 40 hrs. per week. Production could rise considerably before a fourth man would be needed.For a certain special process, it is the practice to the work done through an outside contractor. The company is now considering purchasing a special equipment so that the operation could be performed within the plant.

Page 85: cost accounting question icwai inter

The relevant cost information is as follows:(i) Cost of outside processing per piece Re.0.50(ii) Expected volume of work on special equipment next year 10,000(iii) Estimated hours per piece spent by (a) Foreman 0.04

(b) Workers 0.075(iv) Volume of power used by new equipment Re.0.02 per piece(v) Average value of material used Re.0.02 per piece(vi) Burden rate per hour of labour 100%

Tutorial notes :

Solution : The relevant cost for this process is as follows : Rs. Cost of material (0.02 x 10,000) 200Cost of power (0.02 x 10,000) 200Cost of equipment 2,000Total cost of processing 2,400

Rest of the cost elements furnished with the question are irrelevant for decision making.Cost of outside processing @ Rs. 0.50/ piece (0.50 x 10,000 5,000It is economically viable to go for in-house processing. It will lead to a saving of Rs. 2,600.

The performance data of a steel plant indicates the following for the year 1998--99:(i) Steel Output : 4 million tons (ii) Raw materials consumed :6 million tons(iii) Average employment : 20,000 (iv) Energy consumed : 2 million MWhrs.(v) Average Capital employed : Rs. 20,000 millions.Compute the factor productivities of Manpower, Material, Energy and Capital employed.Solution :Manpower productivity = 40,00,000 / 20,000 = 200 ton/ man yearMaterial productivity = 40,00,000 / 60,00,000 = 0.67 ton/ ton of raw materialsEnergy productivity = 40,00,000 / 20,00,000 = 2 ton / MWhrCapital productivity = 40,00,000 / Rs. 20,000 million = 200 ton / Million Rs.

1 Work content : 6 Standard minutes2 No. of drums filled : 563 Attendance time : 8 hours4 No. of filled drums found underweight : 25 Authorised waiting time : 1 hour6 Authorised unmeasured time : 1 hour7 Job hourly rate : Rs. 5 per hour

Solution :

There is an extra room in which to locate the new equipment, which would cost Rs. 2,000 for installation. What should the company do ?

1. This is a tricky question. It relates with relevancy of costs. The foreman and workers are already working with the company. Their contribution to this job is irrelevant for decision making. The only pertinent costs are material and power required for the job.

2. Opportunity cost of foreman and workers does not come into consideration because the question itself says that 'production could rise considerably before a fourth man would be needed.'

Problem 31 (cwa final June 00)

Problem 32 (cwa final dec.01)

Calculate the operator performance and the total payment (wages + bonus, if any) due to an operator filling drums in a chemical factory with the help of the following performance data :

The performance bonus is payable on a pro-rata basis with a cut-off point as the 75% performance level and the bonus is 33.33% at the 100% performance level.

Page 86: cost accounting question icwai inter

Working time = 8 -- 1 -- 1 = 6 hours = 360 minutes.Work produced = (56 -- 2) x 6 = 324 minutes. Efficiency = 324/360 = 90%Bonus payable : at 75% : 0% and at 100% : 33.33%.Bonus payable at 90% performance level = (15/25) x 33.33% = 20%.Bonus per hour = 20% of wages per hour = 20% of Rs. 5 = Rs. 1/hour.Wages to paid @ Rs. 5/hour for 8 hours Rs. 40Bonus to be paid @ Rs. 1/hour for 6 hours (not 8 hours) Rs. 06

Rs. 46

Demand data Jan. Feb. Mar. Apr. May JuneBeginning inventory (units) 50Forecast demand (units) 300 500 400 100 200 300Working days 22 19 21 21 22 20

Cost data: Production dataHolding cost of inventory Rs. 2/unit/month Labour hours/ unit 10Stock-out cost Rs. 20/unit/month Working hours 8Hiring and training cost/ worker Rs. 300 Current workforce 20Lay off cost/hour Straight time Rs. 6Labour cost/hour Overtime Rs. 9You are advised to consider one of the following strategies for production :1. Vary the workforce size (assuming a starting workforce of 20) and have exact production.2. Maintain a constant workforce of 20 and use overtime and idle time to meet demand.3. Maintain a constant workforce of 20 and build inventory or incur stock-out cost.Which strategy would you choose ?Solution : Statement showing production requirementDemand data Jan. Feb. Mar. Apr. May JuneForecast demand (units) 300 500 400 100 200 300Safety stock (units) 60 100 80 20 40 60Opening inventory (units) 50Production Quantity 310 540 380 40 220 320

Strategy 1: Vary Work Force SizeDemand data Jan. Feb. Mar. Apr. May JuneProduction Quantity 310 540 380 40 220 320Production hrs @ 10 hr./unit 3,100 5,400 3,800 400 2,200 3,200No. of days 22 19 21 21 22 20Available hrs. 176 152 168 168 176 160(@ 8 hrs/day/worker)No. of workers required 17.6 35.5 22.6 2.4 12.5 20.0(prodn. hrs. / available hrs.)Rounding off no. of workers 18 36 23 3 13 20Workers hired 0 18 10Workers fired 2 13 20 7Hiring cost @ Rs.300/worker 5,400 3,000 2,100Layoff cost @ Rs. 6/hr/worker 2,112 0 13,104 20,160 0 0Total cost Rs. 2,112 5,400 13,104 20,160 3,000 2,100

Problem 33 (cwa final dec.01)

A firm producing computer floppies for domestic market engages you to develop an aggregate production plan for the next six months and supplies you with the following information :

Firm's policy is to retain a safety stock equal to 20% of the monthly forecast and each month's safety becomes the opening inventory for the next month.

Page 87: cost accounting question icwai inter

Total cost of strategy Rs. 45,876Strategy 2: Use Overtime and idle time (Constant Workforce=20)

Demand data Jan. Feb. Mar. Apr. May JuneProduction Quantity 310 540 380 40 220 320Production hrs @ 10 hr./unit 3,100 5,400 3,800 400 2,200 3,200No. of days 22 19 21 21 22 20Available hours 176 152 168 168 176 160

3,520 3,040 3,360 3,360 3,520 3,200Overtime hours required 0 2,360 440Overtime cost @ Rs. 9/hr 21,240 3,960Idle time in hours 420 2,960 1,320 0Idle time cost @ Rs. 6/hr* 2,520 17,760 7,920Total cost of strategy Rs. 53,400

Strategy 3: Use inventory and stock out cost (Constant Workforce=20)Demand data Jan. Feb. Mar. Apr. May JuneProduction Quantity 310 540 380 40 220 320Cumulative requirement units 310 850 1,230 1,270 1,490 1,810Production hrs @ 10 hr./unit 3,100 5,400 3,800 400 2,200 3,200

3,520 3,040 3,360 3,360 3,520 3,200Units produced 352 304 336 336 352 320Cumulative production units 352 656 992 1,328 1,680 2,000Units short 0 194 238 0 0 0

3,880 4,760Excess units 42 0 0 58 190 190Inventory costs 84 116 380 380Total cost of strategy Rs. 9,600Recommendation : Strategy 3 is the best as its cost is the least.

(i) Determine the number of furnaces required.(ii) Estimate the percentage of furnace idle time.

A factory has fixed the following labour time standards for its products :Products A B C D Std. time per unit in man-minutes 10 6 4 2Production data for 2 groups are as follows : 10 workers for 8 hours for each groupProduction units A B C D Group 1 100 200 0 200Group 2 50 250 50 100

Calculate the productivity of each group and also the overall productivity of both groups combined.Solution :

Total available hours for 20 workers

Total available hours for 20 workers

Shortage cost @ Rs. 20/unit/month

Problem 34 (cwa final dec.01)

Delta Manufacturing Company Ltd. is considering expansion of manufacturing by adding more number of one-tone capacity furnaces. Each batch of production (1 tone) involves 30 minutes of furnace time including loading and unloading operations. Due to power restrictions, the furnaces can be used only for 80% of the time. The required output for the capacity added is 16 tones/shift of 8 hours. Plant system efficiency is 50%.

Solution : Similar to problem solve earlier. No. of furnace is 2.5 say 3. Idle time % 66.67%.Problem 35 (cwa final dec.02)

Page 88: cost accounting question icwai inter

Production units A B C D Group 1 100 200 0 200Standard time in man-minutes 10 6 4 2Total Standard time 1,000 1,200 0 400 2,600 minutesActual time taken in minutes (10 x 8 hrs.x 60) 4,800 minutesEfficiency % (Actual / Standard) 54%Group 2 50 250 50 100Standard time in man-minutes 10 6 4 2Total Standard time 500 1,500 200 200 2,400 minutesActual time taken in minutes (10 x 8 hrs.x 60) 4,800 minutes

50%Overall productivity = (2,600 + 2,400) / (4,800 + 4,800) = 5,000/9,600 52%

The following data are the activities of man and machine in an operation:Operation Duration in minutes

Loading piece into the machine 0.20Starting the machine 0.10Running time of the machine 4.00Unloading piece from the machine 0.10Cleaning the piece with a brush 0.10Inspecting the component 0.30Packing it in box 0.20Calculate : (i) Work Cycle time (ii) % of machine utilisation and mantime utilisation.Solution:

Operation Engaged by Duration in minutesLoading piece into the machine Man and machine 0.20Starting the machine Man and machine 0.10Running time of the machine Machine 4.00Unloading piece from the machine Man and machine 0.10Cleaning the piece with a brush Man 0.10Inspecting the component Man 0.30Packing it in box Man 0.20Total Time 5.00(i) Work cycle time = 5 minutes(ii) % of man utilisation = time for which man is busy / total time or work cycle time

= (0.2 + 0.1 + 0.1 + 0.1 + 0.3 + 0.2) / 5.0 20%(iii) % of machine utilisation :(0.2 + 0.1 + 4.0 + 0.1) / 5.0 88%

Find : (i) Absenteeism % (ii) Labour utilisation (iii) Productive efficiency of labour (iv) Labour productivity in term of units produced/man-hour/month

Solution : (i) Absent man-days 15Total man-days available = 15 x 25 375Absenteeism ( 15 / 375 ) 8%

Problem 36 (cwa final dec.02)

The operations narrated above, may need only machine, man and machine both or only man. The operations are to be segregated into these three categories as shows below :

Problem 37 (cwa final dec.02)

An Engg. Organization having 15 operators, works 25 days in a month, with a single shift of 8 hours. The standard production per month is 300 units while the standard man-hours/unit are 8. In November 2002, 30 man-days were lost due to absenteeism. The company produced 240 units while the idle time logged by operators was 276 man-hours.

Page 89: cost accounting question icwai inter

(ii) Labour utilisation : in hrs.Total labour-hours available ( 15 x 25 x 8 ) 3,000Less : time lost due to absenteeism ( 30 man-days x 8 hrs.) 240

Total hours available for working 2,760Less Idle time logged by operators 276Actual hours of working 2,484Labour utilisation ( 2484 / 2760) 90%

(iii) Productive efficiency of labour = (Standard man-hours -- Actual man-hours) of Actual production = 240 x 8 -- 2484 = 77.30%

(iv) Labour productivity in terms of unit produced / man-hour/ monthTotal units produced in 25 days @ 240units/day 6,000Total no. of man days [(15 x 25) -- (30)] = 345Labour productivity ( 6,000 / 345) 17.39 units/man-month

The present output details of manufacturing department of Simple Ltd. are as follows :Average output/ week :48,000 units. Saleable value :Rs. 60,000.Contribution Rs. 24,000.

Solution : Existing strength 160 employeesExisting output per week 48,000 units

Existing average output/week / employee (48,000/160) 300 unitsProjected output (60% above the present output) 480 unitsProjected output with reduced employees of 120 nos. 57,600 unitsExisting wages per week ( @ Rs. 0.10 per piece for 48,000 units) 4,800

Revised piece work rate = 1.30 x 0.10 = Rs. 0.13.Existing Selling price : Rs. 60.000/ 48,000 = Rs. 1.25Projected selling price (4% less than the current selling price) (0.96 x 1.25) 1.20

Rs. Variable cost : Sales 60,000

Contribution 24,000Variable cost 36,000Direct wages 4,800Other variable cost 31,200 for 48,000 unitsOther variable cost /unit 0.65

Forecast of Profitability of Simple Ltd. (time horizon : One week)Per unit For 57,600 units

Rs. Rs.Sales 1.20 69,120Variable cost 0.65 37,440

Problem 38 (cwa final dec.01)

The management plans to introduce more mechanization in the department at a capital cost of Rs. 16,000. As an effect of this the number of employees will be reduced from the existing strength of 160 nos. to 120 nos. but the output of individual employee will increase by 60%. As an incentive to achieve the extra output, the management proposes to offer an one percent increase in the existing piece work price of Re.0.10 per article for every 2% increase in the individual output achieved. In order to sell the increased output, it will be necessary to reduce the sale price by 4%.

You are required to calculate extra weekly contribution resulting from the proposed changes, as above, and give your recommendation.

Revised piece work rate = existing rate + 1% of every 2% increase. The increase in output is 60%. Thus the piece rate would be 30% above the existing rate.

Page 90: cost accounting question icwai inter

Direct wages 0.13 7,488Contribution 0.42 24,192Present Contribution 24,000Increase in contribution per week 192

Solution:Bonus under Rowan Plan = (Time saved / Time allowed) x Time saved x rate per hour

For example let us assume that time allowed for a job is say 4 hours and labour rate is Rs. 50/hour.

Case 1 : With less efficient worker. Time taken by him is say 3 hoursTime saved = 4 -- 3 = 1 hr. Bonus = (3 hours / 4 hours) x 1 hour x Rs. 50 = Rs. 37.50Case 2 : With highly efficient worker. Time taken by him say 2 hours.Time saved = 4 -- 2 = 2 hr. Bonus = (2 hours / 4 hours) x 2 hour x Rs. 50 = Rs.50

Bonus = ( 1 hour / 4 hour ) x 3 hour x Rs.50 = Rs. 37.50.

Solution : 1st unit 100% 100%2nd unit 80% of 100% 80%4th unit 80% of 80% 64%8th unit 80% of 64% 51%16th unit 80% of 51% 41%

16th unit will require 41% of the time required by first unit.Problem 41 (ca final June 09)

Solution Total earnings of A (Rs. 24 x 60) + 20 x 40 1,920Total earnings of B = Rate x 80 + 20% of 80 hours x Rate = Rs. 1,920Hourly rate of B = 1920 / 96 Rs. 20

Problem 42 :(CWA Inter June 09)

Problem 39 (ca inter may 07)

Under the Rowan Premium Bonus Plan', a less efficient worker can obtain same bonus as a highly efficient worker.' Discuss with suitable examples.

The Rowan plan will come into scene when the time taken by the worker is less than the time allowed. The efficiency will be measured with respect to time saved, thus less efficient worker will save less time in comparison to highly efficient worker.

We are to prove that the less efficient worker and highly efficient worker can obtain same bonus under Rowan plan. Let us try that highly efficient worker takes one hour for the job for which less efficient worker takes 3 hours. Time saved by highly efficient worker is 3 hours. The Bonus will be :

Thus we see that the less efficient worker and the highly efficient worker can get same amount of bonus under Rowan plan. This occurs when time saved is 25% and 75% of the time allowed.

Problem 40 (cwa final June 03)

If the direct labour cost is reduced by 20% with every doubling of output, what will be the cost of labour for the sixteenth unit produced as an approximate percentage of the cost of first unit?

(i)         Two workmen, A and B, produce the same product using the same material. A is paid bonus according to Halsey plan, while B is paid bonus according to Rowan plan. The time allowed to manufacture the product is 100 hours. A has taken 60 hours and B has taken 80 hours to complete the product. The normal hourly rate of wages of workman A is Rs. 24 per hour. The total earnings of both the workers are same. Calculate normal hourly rate of wages of workman B.

Use good cheerful words in your language. Avoid negative words while speaking,

The standard hours for job X is 100 hours. The job has been completed by Amar in 60 hours, Akbar in 70 hours and Anthony in 95 hours. The bonus system applicable to the job is as follows :

Page 91: cost accounting question icwai inter

Percentage of time saved to time allowed BonusSaving up to 10% 10% of time savedFrom 11% to 20% 15% of time savedFrom 21% to 40% 20% of time savedFrom 41% to 100% 25% of time saved

Solution : It is very easy question.Statement showing the total earnings and rate of earnings per hour:Particulars Amar Akbar AnthonyStandard hours for the job 100 100 100Time taken for the job in hrs. 60 70 95Time saved 40 30 5% of time saved 40% 30% 5%Bonus as per table in % (on time saved) 20% 20% 10%Bonus hours 8 6 0.5Total hours to be paid (time taken + bonus) 68 76 95.5Total earnings @ Rs. 10 per hour 680 760 955Earnings per hour (earnings / time taken) 11.33 10.86 10.05Problem 43 :(CWA Inter Dec.08)

Solution : Bonus = 6 hours (60 - 48) / 2. Total payment = (48 + 6) x 10 = Rs. 540

The rate of pay is Rs. 10 per hour. Calculate the total earnings of each worker and also the rate of earnings per hour.

A worker is allowed 60 hours to complete a job on a guaranteed wage of Rs. 10 per hour. He completes the job in 48 hours. For the saving in time, how much he will get under Hasley Premium Plan (@ 50% bonus).

·       Don't count on luck for promotions, victories or other good things in life

Page 92: cost accounting question icwai inter
Page 93: cost accounting question icwai inter

Chapter 3: Overheadscwa inter cwa final ca inter ca final Total

15 0 11 1 27

Normal available hours per month 218Absenteeism (without pay) hours 18Stoppage for repairs and maintenance etc. hours 20Leave (with pay) 20Average rate of wages per day of 8 hours 80Production bonus estimated 15% on wagesValue of power consumed in Rs. 24,150Supervision and indirect labour in Rs. 9,900Lighting and electricity in Rs. 4,800These particulars are for a year :( in Rs.)Repairs and maintenance including consumables 36,000insurance 60,000Other sundry expenses 36,000General management expenses allocated 109,040

You are required to work out a comprehensive machine hour rate for the machine shop.

Solution: Tutorial notes: blh oDr vPNk eglwl dhft;s A vHkh eqLdqjkb;sA

Computation of total machine hoursNormal available hours per month per operator 218less: unutilised hours due to

Absenteeism 18Leave 20Stoppage for repairs etc. 20 58

Total hours utilized per month per operator 160Total hours utilized for six months for (160 x 6 x 6) 5,760Calculation for depreciationCost of machine in lacs 12.00Less scrap value in lacs 1.20Cost of machine to be depreciated in 9 years 10.80Depreciation per year 1.20

euksfoKku dk fu;e gS fd vxj vki vius fnekx esa ml ckr dh Nfo cuk ysrs gSa tks vki cuuk pkgrs gS vkSj fQj ml Nfo dks dkQh nsj rd ogka dk;e j[krs gSa rks vki tYn gh oSls cu tk,axsa tSlk fd vkius lkspk FkkAProblem 1(cwa inter I June 05)

A machine shop of Avon ltd. has six identical machines manned by 6 operators. The machine cannot be worked without an operator wholly engaged on it. The cost of all these 6 machines including installation charges works out to Rs. 12 lakhs and these machines are deemed to have a scrap value of 10% at the end of effective life of 9 years. these particulars are furnished for a six months period

It is given in the question that the machine cannot run without an operator being wholly engaged on it. What does it signify ? It implies that the total machine hours are equal to total number of hours of working of all the six operators. Thus hours utilized by all the six operators represent the total machine hours.

Comprehensive machine hour rate can be computed by adding up all the expenses, direct and indirect, incurred on or allocated to machine and then dividing the sum by total number of hours the machines are subjected to run.

Page 94: cost accounting question icwai inter

Depreciation per six month in Rs. 60,000Total wages for 6 operators for 6 monthsNormal wages for hour (80/8) 10Normal hours/month for which wages are to be paid (218 -18) 200Wages for 6 operators for 6 months ( 200 x 10 x 6 x 6) 72,000Computation for comprehensive machine hour rate Rs.Operator wages as given above 72,000Production bonus @ 15% of wages 10,800Power consumed 24,150Supervision and Indirect labour 9,900Lighting and Electricity 4,800Depreciation for 6 months 60,000Repairs and maintenance including consumables 18,000insurance 30,000Other sundry expenses 18,000General management expenses allocated 54,520Total overheads for six months 302,170Total number of machine hours 5,760comprehensive machine hour rate 52.46Problem 2 cs inter dec. 08

Solution: Solve yourself. Rate per hour of standing charges : Rs. 2.00.Rate per hour of machine expenses : Rs. 1.75Machine hour rate Rs. 3.75/hr.

Indirect Cost Amount Rs. Indirect Cost Amount Rs.Indirect expenses 9,000 Electric power 6,000Lighting and heating 1,200 Depreciation 24,000Rent and rates 12,000 Sundry expenses 7,800Following further details are collected for distribution of the above costs:Particulars Departments

X Y Z S CValue of machine (Rs. '000) 60 50 80 10Horse power of machines 40 45 60 5Light points in nos. 20 30 50 20 10Floor space in sq. meters 150 200 250 100 50

In a factory department, the cost of a machine is Rs. 11,500. It is expected that it will work for about 20,000 hours and its scrap value is estimated at Rs. 1,500. The rent of factory department is Rs. 400 per month and 25% of the area of the department is utilised for conducting the operation of the machine. One foreman and one attendant are employed on a salary of Rs. 200 and Rs. 100 per month respectively, to work on two machines of similar type. The other expenses of the month are as under in the department :

Light charges of the factory department is Rs. 160 having 32 points in all, out of which 8 points are used for both these machines. Power used for this machine Rs. 160; indirect labour for both machines Rs. 100; and repair and renewal for this machine Rs. 40.

You are required to find out the machine hour rate for one month of four weeks when it is expected to work for 40 hours a week.

Problem 3 (cwa inter I dec 05)

MM Ltd. has three production departments X,Y,Z and two service departments S and C. the following details are extracted from the books of accounts in respect of indirect expenses incurred during April 05

Page 95: cost accounting question icwai inter

Direct wages (Rs. '000) 30 20 40 4 6Machine hours worked 4,250 3,380 7,120 The costs of the service departments are apportioned percentage wise as follows:Departments X Y Z S C

S 20 30 40 10C 40 20 30 10

Calculate:(a) Overhead recovery rates showing the basis of apportionment.

Particulars X Y ZDirect materials used (Rs.) 268 131 102Direct wages (Rs..) 300 250 300Machine hours worked 10 12 12Solution: Tutorial notes:

2. Indirect costs and sundry expenses should be distributed on the basis of direct wages.

Distribution of indirect expenses to Production and Service Departments (Fig. in Rs.)Items Basis of Total Departments

Apportionment X Y Z S CIndirect expensesDirect wages 9,000 2,700 1,800 3,600 360 540Lighting and heat nos. 1,200 200 300 400 200 100Rent and rates Floor space 12,000 2,400 3,200 4,000 1,600 800Electric power HP of machine 6,000 1,600 1,800 2,400 200Depreciation Value 24,000 7,200 6,000 9,600 1,200Sundry expenses Direct wages 7,800 2,340 1,560 3,120 312 468Direct wages as given 10,000 4,000 6,000Total 70,000 16,440 14,660 23,120 7,872 7,908

DepartmentsX Y Z S C

Rs. Rs. Rs. Rs. Rs. Rs.Costs to be distributed 7,872 7,908Distribution of 7872 1,574 2,362 3,149 787

(b) Total cost of Job 321 with element wise and department wise cost breakdown, the job card of which contains the following details:

1. Apportionment of costs of service departments indicates that repeated distribution method is to be used for overhead distribution. Students are advised to revise the knowledge of this method prior to attempting this problem.

3. The methodology to solve this question is sequential in nature. First the indirect expenses are to be distributed to all the five departments on suitable basis and thereafter the service departments' total costs are to be distributed to production departments by repeated distribution method.

4. The most important point in this question (this is likely to be omitted by most of the students) is that not only the indirect expenses are to be apportioned, but the direct labour costs of service departments are also to be apportioned.

5.The second part is dependent of first part. First the recovery rate is to be calculated then it is to be multiplied to machine hours worked on job 321 to find out the overall overhead charges allocated to job 321. the overhead charges when added to direct material and labour charges furnish the total cost of the job.

Service departments costs of Rs.7,872 and Rs. 7,908 are to be apportioned as per the distribution percentage supplied in the question using the repeated distribution method.

Page 96: cost accounting question icwai inter

8,695Distribution of 8695 3,478 1,739 2,609 870Distribution of 870 174 261 347 87Distribution of 87 35 17 26 9Distribution of 9 2 3 4Total 5,263 4,382 6,134Add: Apportionment from indirect expenses 16,440 14,660 23,120Total Cost 21,703 19,042 29,254Machine hours worked 4,250 3,380 7,120Overhead recovery rate (Rs.) per hour 5.11 5.63 4.11

Computation of Total Cost of Job 321Particulars X Y Z Total costDirect materials used (Rs.) 268 131 102 501Direct wages (Rs..) 300 250 300 850Machine hours worked 10 12 12Overhead recovery rate Rs. per hour 5.11 5.63 4.11Overhead charges 51 68 49 168Total Cost 619 449 451 1,519

Following data is available :Month Oct.04 Nov.04 Dec.04Cost of running in Rs. 17,500 15,400 --Machine hours worked 7,500 5,400 6,200Find the cost of running for the month of Dec-04.Solution :Month Oct.04 Nov.04 Dec.04 Difference for Nov. and Dec.Cost of running in Rs. 17,500 15,400 -- Rs. 2,100 Machine hours worked 7,500 5,400 6,200 2,100 hours

Total cost of running 6,200 hours in the month of Dec-04 would be Rs. 16,200.

Particulars A B C X YPrimary distribution (Rs. ) 240,000 210,000 250,000 140,000 96,000Service renderedDept. X 30% 20% 35% -- 15%Dept.Y 25% 40% 25% 10% --

Solution :

Simultaneous equation is very easy and short method of distribution of overheads. It goes as follows:

Problem 4 (cwa inter I dec 04)

For the difference of 2,100 hours, the cost varies by Rs. 2,100 thus the variable portion is Rs. 1 per hour. Thus for 7,500 machine hours, the variable component is Rs. 7,500 and the fixed portion is Rs. 10,000.

Thus for Dec-04, the cost of running 6,200 machine hours would be Rs. 6,200 variable and Rs. 10,000 as fixed.

Problem 5 (cwa inter I dec 04)

A factory has three production departments A, B and C and also two service departments X and Y. The primary distribution of the estimated overheads in the factory has just been completed. These details and the quantum of service rendered by the service departments, to the other departments are given below:

Prepare a statement showing the distribution of service dept. overheads to the production departments by the simultaneous method.

Page 97: cost accounting question icwai inter

Suppose ' a ' and ' b ' are the total overheads of departments X and Y respectively.The simultaneous equations are as follows;a = 140,000 + 0.10 b and b = 96,000 + 0.15 a, which on solution give a = 151,878 and b = 118,782.a = 140,000 + 0.10 ( 96,000 + 0.15 a)Particulars A B CPrimary distribution (Rs. ) 240,000 210,000 250,000Service renderedDept. X 151,878 30% 20% 35%

45,563 30,376 53,157 129,096Dept.Y 118,782 25% 40% 25%

29,696 47,513 29,696 106,90475,259 77,888 82,853 236,000

Check: Total service received by Production department : Rs. 236,000Total service to be apportioned as per the question : Rs. 236,000

SolutionVolume variance = (Budgeted production -- actual production ) x Std. rateVolume variance = (10,000 -- 11,000 ) x (20,000 / 10,000) = 2,000 fav. (actual is more)

Particulars ValueCost including installation charges in Rs. 20,00,000Estimated useful life 10 yearsEstimated salvage value 10%No. of working days 300 daysNumber of shifts per day 2 no.Effective working hours per shift 7 hoursStoppage for repairs and maintenance etc. 200 hoursOperating and other costs :(i) Wages of two operators (one for each shift) @ Rs. 5,000 p.m.

(iii) Electric power 20 units per hour; each unit costing Rs. 3.20.(iv) Insurance charges Rs. 5,000 p.a.(v) Repairs and maintenance (estimated) Rs. 12,500 p.m.(vi) Rent, rates and taxes (allocated) Rs. 10,000 per annum.(vii) General lighting etc. (allocated) Rs. 750 per month(viii) Other factory overhead (allocated) Rs. 140,000 per annum.

Solution: Tutorial notes: blh oDr vPNk eglwl dhft;s A vHkh eqLdqjkb;sA

2. The working should be based on variable portion and fixed portion of the cost.3. Intermediate calculations should be explained by way of working notes. They carry marks.

Problem 6 (cwa inter I June 04)

The budgeted fixed overhead for a budgeted production of 10,000 units is Rs. 20,000. For a certain period the actual production was 11,000 units and the actual expenditure came to Rs. 24,000. Then the volume variance is ……………..

Problem 7 (cwa inter I June 03)

Compute a comprehensive machine hour rate for a machine in Production department 'A' of a factory from the following details :

(ii) Salary of supervisor ( one for each shift) @ Rs. 7,500. Only one-fifth of the supervisor's time is devoted to this machine.

1. Comprehensive machine hours should be calculated on the basis of actual run of the machine and not on the basis of total hours available for working or running.

4. All allocated costs form direct costs charged to machine. These are relevant for computation of machine hour rate.

Page 98: cost accounting question icwai inter

The solution goes as follows:……………………..Ltd.Production Department A

Computation of Comprehensive Machine Hour RateTotal working hours availa ( 300 x 2 x 7) 4,200 hoursLess Stoppage due to repairs and maintenance 200 hoursMachine hours effective for running of machine 4,000 hours(A) Variable ExpensesParticulars Per Annum Per Hour

Rs. Rs.1.Wages of operators in Rs. ( 5,000 x 2 x 12) 120,0002. Power in Rs. ( 4,000 x 20 x 3.20) 256,0003. Repairs and maintenance in Rs. ( 12,500 x 12 ) 150,000Total Variable Expenses 526,000 131.50(B) Fixed Expens Figures in Rs. 1. Salary of supervisor 1/5 th of ( 2 x 7,500 x 12) 36,0002. Depreciation ( 20,00,000 -- 2,00,000) / 10 180,0003. Insurance charges 5,0004. Rent, rates and taxes (allocated) 10,0005. General lighting etc.( allocated) (750 x 12) 9,0006. Other factory overheads (allocated) 140,000Total Fixed Expenses 380,000 95.00Machine hour rate A + B 226.5

The following cost information for a period is available for a small engineering unit:(a) Allocated expenditure All fig. in Rs.Particulars Allocated to production dept. Allocated to Service dept.

Total Assembly

Indirect wages 29,300 8,000 6,000 4,000 11,300Stores consumed 6,700 2,200 1,700 1,100 1,700Supervisor Salaries 14,000 0 0 14,000 0Other salaries 10,000 0 0 10,000 0(b) Expenditure to be apportioned All fig. in Rs.Power and fuel Rent Insurance Depreciation 15,000 15,000 3,000 100,000

(c) Additional information available:Particulars Floor area HP hours employees. Investments

sq. ft hrs no. Rs.Machine shop 2,000 3,500 30 640,000Assembly 1,000 500 15 200,000General Plant 500 0 5 10,000Stores and Mis. 1,500 1,000 10 150,000(d) Basis of distribution of service dept.costs :

Machine shop Assembly General Plant Stores and Mis.Stores and Mis. 50% 20% 30%General Plant In proportion to the no. of employees.

Problem 8 (cwa inter I Dec. 03)

Machine shop

General Plant

Stores & Misc.

You are required to prepare an overhead distribution statement in detail. Service department costs are to be distributed by continued distribution. Carry through three cycles. Calculations to be shown to the nearest rupee.

Page 99: cost accounting question icwai inter

Solution:Small Engineering Ltd.

Departmental Primary Overhead Distribution Summary1. Apportionment of expenditure Production Department Service Department

Assembly

Particulars Basis 15,000 10,500 1,500 3,000Power and fuel HP hrs. 15,000 6,000 3,000 1,500 4,500Rent Floor area 3,000 1,920 600 30 450Insurance Investment 100,000 64,000 20,000 1,000 15,000Depreciation Investment2. Allocated Expenditure 29,300 8,000 6,000 4,000 11,300Indirect wages 6,700 2,200 1,700 1,100 1,700Stores consumed 14,000 0 0 14,000 0Supervisor Salaries 10,000 0 0 10,000 0Other salaries 193,000 92,620 32,800 31,630 35,950Statement showing secondary distribution of overhead :Particulars Production Department Service Department

Expdt. Assembly

As per primary distribution 193,000 92,620 32,800 31,630 35,950Re-distribution of Stores 50:20:30 17,975 7,190 10,785 (35,950)

42,415Redistribution of plant 30:15:10 23,135 11,568 (42415) 7,712Re-distribution of Stores 50:20:30 3,856 1,542 2,314 (7712)Redistribution of plant 30:15:10 1,262 631 (2314) 421Re-distribution of Stores 50:20:30 210 84 126 (421)Redistribution of plant 30:15:10 69 34 23

193,000 139,127 53,850

Budgeted labour hours 8,500 Actual labour hours 7,928Budgeted overheads in Rs. 148,750 Actual overhead Rs. 146,200Solution:Overhead absorption rate = Budgeted overhead / Budgeted labour hour

= 148,750 / 8,500 = Rs. 17.50Actual absorption of overhead = Actual labour hours x Absorption rate

= 7,928 x Rs. 17.50 = Rs. 138,740Actual overhead Rs. 146,200Actual absorption Rs. 138,740Under absorption of overhead Rs. 7,460

Rs. Rs. Production Departments : Mixing 144,000

Curing 96,000 240,000Service Departments: Time Office 48,000

Expenditure

Machine shop

General Plant

Stores and mis.

Machine shop

General Plant

Stores and mis.

Problem 9 (cwa inter I Dec. 02)

Based on the data given below; compute overhead absorption rate and the amount of over or under absorption rate.

Problem 10 (cwa inter I Dec. 01)

Excellent manufacturing works have two production departments : Mixing and Curing and three service departments : Time office, Stores and Maintenance. The following details are available from the departmental distribution summary for the month of July 2001

Page 100: cost accounting question icwai inter

Stores 60,000Maintenance 36,000 144,000

The following relevant data are also available:Particulars Production Department Service Departments

Mixing Curing Time Stores Maintenanceoffice

No. of employees 20 15 10 8 5No. of stores requisitions 120 100 30Machine hours 3,600 2,400

Solution:Particulars Production Department Service Departments

Mixing Curing Time Stores Maint,office

As per primary distribution 144,000 96,000 48,000 60,000 36,000Time office cost (20:15:8:5) 20,000 15,000 (48,000) 8,000 5,000apportioned 68,000Stores cost (120:100:30) 32,640 27,200 (68,000) 8,160apportioned 49,160Maintenance cost (36:24) 29,496 19,664 (49,160)apportioned

226,136 157,864Notes :The basis of apportionment of service department costs are as per following:(i) time office : in the ratio of number of employees : 20:15:8:5(ii) Stores : no. of stores requisitions (120:100:30)(iii) Maintenance : Machine hours 3:2(iv) Following assumptions have been made in the apportionment of costs:1. The inter departmental distribution in service section is not relevant.

ABC Ltd. uses historical cost system and applies overheads on the basis of predetermined rates.The following data are made available by the company for the year ended 31.03.00:

Rs. lacs Manufacturing overheads 32.72Manufacturing overheads applied 32.00Work-in-progress (closing balance) 5.00Finished goods (closing balance) 15.00Cost of goods sold 220.00

Also indicate the relevant journal entries.

Solution: Rs. lacs Manufacturing overheads 32.72Manufacturing overheads applied 32.00Under absorbed overheads 0.72There are three ways to treat the unabsorbed overheads.

The company consistently follows the methods of secondary distribution on non-reciprocal basis. Show the apportionment of the cost of service departments to production departments stating the basis of computation in the form of a note at the end of the exercise.

2. Service departments have been considered in the order first time office second stores and last the maintenance. If this order is altered, the apportionment will also undergo alteration.

Problem 11 (cwa inter I Dec. 00)

Apply two methods for disposal of under absorbed overheads and state the impact of each method on the profit earned by the firm.

Page 101: cost accounting question icwai inter

1. Carry the unabsorbed overheads forward to the next year,

Journal Dr.Manufacturing Overhead suspense a/c 72,000Or Manufacturing overhead reserve a/c

To Manufacturing overhead a/c 72,000(For the absorption of unabsorbed overhead)2. Write off to costing profit and loss account :The unabsorbed portion is treated as expense and is transferred to costing profit and loss account.Since there is no carry forward, the profit will be less by an amount equal to unabsorbed overhead.Journal Dr.Unabsorbed overhead a/c 72,000

To manufacturing overhead a/c 72,0003. Use of supplementary rates for absorption:

Journal Dr.Work-in-progress 1,500Finished goods 4,500Cost of goods sold 66,000(distribution of Rs. 72,000 in the ratio of 1:3:44)

To manufacturing overhead a/c 72,000In this method the profit will be less by Rs. 66,000.

Items Amount Rs. Indirect labour 9,000Lighting 1,200Rent and rates 12,000Power 6,000Depreciation 24,000Sundry Expe. 7,800

Following data is available for distribution of overhead:Particulars Departments

P Q R M C Value of machine in Rs. 60,000 50,000 40,000 10,000 0H.P. of machine 40 45 60 15 0No. of light points 20 30 40 20 10Floor space sq.m. 150 200 250 100 50

The unabsorbed overhead is carried to the next year and is added to the overheads of next year for absorption. A suspense account or Overhead reserve account is opened to accommodate the unabsorbed overhead. Thus there would be no impact on profit earned by the firm.

In this method the under absorbed overhead is absorbed in the same year and there is no carry forward, The under absorbed overhead is adjusted or charged to (i) work-in-progress (ii) Finished goods and (iii) Cost of sales, by adding back.

The amount of unabsorbed overhead is added by applying supplementary rate and is charged to work-in-progress, finished goods, and cost of sales in the ratio of their cost. Thus in the present case the unabsorbed overhead of Rs. 72,000 will be charged to wip, finished good and cost of sales in the ratio of 5:15:220 or 1:3:44.

Problem 12 (cwa inter I Dec. 06)

A company has three production departments P,Q and R and two service departments M and C. The following information in respect of indirect expenses incurred are furnished for a typical month.

vxj vkidh lkal py jgh gS rks vkidk le; cgqr vPNk py jgk gSA

Page 102: cost accounting question icwai inter

Direct wages 30 20 40 4 6Machine hrs. worked 2,940 2,060 2,150The service costs are apportioned as follows;Department P Q R M C M (%) 20 30 40 0 10C (%) 40 20 30 10 0Calculate : (i) Overhead recovery rate showing the basis of apportionment.(ii) The cost of job no. 234, the job card of which has recorded the following data:Particulars Departments

P Q RMaterial consumed Rs. Rs. 228 136 100Direct wages Rs. Rs. 162 144 256Machine hours worked hours 10 8 12Solution : This can be solved on the lines of solution to problem no. 2 above.Answer :The primary distribution of overhead is as follows:

Department P Q R M C Rs. Rs. Rs. Rs. Rs.

Total overhead 18,140 16,048 19,370 8,534 7,908From M 1,884 2,825 3,766From C 3,540 1,770 2,655Total overhead 23,564 20,643 25,791Machine hours worked 2,940 2,060 2,150Overhead recovery rate Rs./hr. 8.0 10.0 12.0

Total cost of Job 234 is Rs. 1330.

The following data are available: Cutting Welding TotalDepatt. Depatt.

Actual usage in 2002--03 Machine hours 60,000 40,000 100,000Practical capacity for each departmen Machine hours 90,000 60,000 150,000Required

4. Comment on your results in requirements 1,2 and 3 above.

Solution: blh oDr vPNk eglwl dhft;s A vHkh eqLdqjkb;sAWorking notes :1. Calculation of budgeted rate based on practical capacity:

Problem 13 (CA PE-II May 03)

PQR Ltd has its own power plant, which has two uses, Cutting Department and Welding Department. When the plans were prepared for the power plant, top management decided that its practical capacity should be 150,000 machine hours. Annual budgeted practical capacity fixed cost are Rs. 9,00,000 and budgeted variable costs are Rs. 4 per machine hour.

1.Allocate the power plants cost to the cutting and the welding departments using a single rate method in which the budgeted rate is calculated using practical capacity and costs are allocated based on actual usage.

2. Allocate the power plant's cost to the cutting and the welding departments, using the dual-rate method in which the fixed costs are allocated based on practical capacity and variable costs are allocated based on actual usage.

3.Allocate the power plant's cost to the cutting and welding departments using the dual-rate method in which the fixed cost rate is calculated using practical capacity, but fixed costs are allocated to the cutting and welding departments based on actual usage. Variable costs are allocated based on actual usage.

Page 103: cost accounting question icwai inter

Practical capacity fixed costs in Rs. 900,000Practical capacity in machine hours 150,000Practical capacity fixed costs in Rs./ machine hour 900/150 6.0Budgeted variable cost per machine hour as given in Rs. / hour 4.0Budgeted rate per machine hour in Rs./ hr. 10.01. Statement showing Power Plant's cost allocation to the cutting and welding departments:Using single rate method on actual usage of machine hours All fig. in Rs.'000Particulars Cutting Deptt. Welding Deptt. TotalActual usage in 2002--03 Machine hours 60 40 100Budgeted rate per machine hour in Rs./ hr. 10 10Power cost allocation based on budgeted rate 600 400 1,0002. Statement showing Power Plant's cost allocation to the cutting and welding departments:Using dual rate method: All fig.in Rs.'000Particulars Cutting Deptt. Welding Deptt. TotalPractical capacity Machine hours 90 60 150Fixed costs (Rs. 900,000 in ratio 9:6) Rs. 540 360 900Variable cost/hour. in Rs. Rs. /hr 4 4Actual usage in 2002--03 Hours 60 40Variable cost Rs. 240 160 400Total cost Rs. 780 520 1,3003. Statement showing Power plant's cost allocation to cutting and welding departments:Using dual rate method: All fig.in Rs.'000Particulars Cutting Deptt. Welding Deptt. TotalActual usage in 2002--03 60 40fixed cost allocation @ Rs. 6 for actual usage 360 240 600Variable cost @ Rs. 4/hr for actual usage 240 160 400Total cost 600 400 1,000

The following data are available for October 2003:Department Revenues Rs. No. of employees Processing time in minutesCorporate sales 1667750 42 2,400Consumer sales 833875 28 2,000Administrative 0 14 400Information systems 0 21 1,400Cost incurred in each of the four departments in October 2003 are as follows:

Rs. Corporate sales 12,97,751Consumer sales 636,818Administrative 94,510

Comments : In single rate method under 1 above and under 3 above, the fixed cost are allocated on the basis of actual usage during the period. Thus entire fixed cost of Rs. 900,000 could not be absorbed and only fixed cost amounting to Rs. 600,000 could be allocated leaving fixed cost of Rs. 300,000 un-allocated. This highlights the unused capacity of the operating plant.

In a dual rate method, the entire fixed cost is allocated based on practical capacity, thus unused capacity cannot be associated with allocation of fixed cost.

Problem 14 (CA PE-II Nov. 03)

E-books is an online book retailer. The company has four departments. The two sales departments are Corporate sales and Consumer sales. The two support departments are Administrative (Human resources and Accounting) and Information systems. Each of the sales departments conducts merchandising and marketing operations independently.

Page 104: cost accounting question icwai inter

Information systems 304,720

Required:

(i) Allocate the support department costs to the sales departments using the direct method.

(iii) How would you have ranked support departments differently?

Solution:The students should revise the knowledge of distribution of overheads by various methods.

Following abbreviations are used hereunder for convenience.A = Corporate sales; B = Consumer sales ; C = Admin. Deptt. and D=Information systems.Statement showing apportionment of service department cost to production departments.Using the direct method: Department Particulars Basis of A B C D

allocation Rs. Rs. Rs. Rs.Cost incurred 1297751 636,818 94,510 304,720Reallocation of cost of C no. of employees* 56,706 37,804Reallocation of cost of D processing time** 166,211 138,509* In the ratio of 42:28 i.e.3:2 to A and B.** In the ratio of 24:20 i.e. 6:5 to A and B.Total cost allocated to A and B 222,917 176,313Total cost of A and B 1520668 813,131Check: Total cost allocated to Sales departments (222,917 + 176,313) 399,230

Total cost incurred in service departments (94,510 + 304,720) 399,230

Computation of % of services to other departments:Administrative deptt. provides [21 / ( 21+28+42)] i.e.23% of its services to information systems.

Statement showing apportionment of service department cost to production departments.Using the step down method: Department Particulars Basis of A B C D

allocation Rs. Rs. Rs. Rs.Cost incurred 1297751 636,818 94,510 304,720Reallocation of cost of C no. of employees* 43,620 29,080 21,810* In the ratio of 42:28:21 i.e.6:4:3 to A,B and D. 326,530Reallocation of cost of D processing time 178,107 148,423

The company uses no. of employees as a basis to allocate administrative costs and processing time as a basis to allocate information system costs.

(ii) Rank the support departments based on percentage of their services rendered to other support departments. Use this ranking to allocate support costs based on the step-down method.

(iv) Allocate the support department costs to two sales departments using the reciprocal allocation method.

(i) In the direct method, service department's costs are apportioned to production departments ignoring service rendered by one service department to the other service department.

(ii) In step down method also known as non-reciprocal method, there is one way distribution of cost. Normally the department which is serving largest no. of other departments should be taken first. Some are also of the view that department having largest amount of cost should be taken first. In this case service department C is arbitrarily taken as first.

Similarly information systems provides [400 / (400+2000+2400)] i.e 8.3% of its services to administrative departments.

If C is taken first, in step down method the cost of C would be apportioned on the basis of no. of employees to A, B and D in the ratio of 42:28:21 i.e 6:4:3.

Page 105: cost accounting question icwai inter

1519478 814,321

Statement showing apportionment of service department cost to production departments.Using the step down method: Department Particulars Basis of A B C D

allocation Rs. Rs. Rs. Rs.Cost incurred 1297751 636,818 94,510 304,720Reallocation of cost of D processing time 152360 126967 25393 (304,720)

119,903Reallocation of cost of C no. of employees* 71,942 47,961(iv) Allocation of costs of service department to two sales departments Reciprocal allocation methodThe % of distribution of cost among the departments is as follows:Department C: In the ratio of 6:4:3 or 46%:31%:23% to A,B and D.Department D: In the ratio of 6:5:1 or 50%:42%:8% to A,B and C.Using the reciprocal method: Department Particulars A B C D Cost incurred Rs. 1297751 636,818 94,510 304,720Distribution of cost of C % 46% 31% 23%Distribution of cost of D % 50% 42% 8%Total cost of service department is computed using simultaneous equation method:Let x and y be the total cost of departments C and D. Thenx = 94,510 + 0.08y and y = 304,720 + 0.23 x which give x = Rs. 122,243 and y = Rs. 332,922Statement showing apportionment of cost of C and D.Using the reciprocal method: Department Particulars A B C D Cost incurred Rs. 1297751 636,818 122,243 332,922Reallocation of C to A and B 46%:31% Rs. 56,232 37,895Reallocation of D to A and B 50%:42% Rs. 166,461 139,827

Rs. 1520444 814,541

Problem 15 (cwa inter I June 01)

From Service DepartmentsX Y

To Spinning 30% 25%To Weaving 20% 35%To X -- 40%To Y 50%

Variable cost of service department Rs. 120,000 260,000Output of service department in units units 10,000 20,000Calculate : (i) Cost per unit of service produced by X and Y.

(ii) Cost of service received by spinning and weaving departments.

Solution: blh oDr vPNk eglwl dhft;s A vHkh eqLdqjkb;sA

(iii) The alternative way is to take the administrative department first. In this case the cost of administrative department Rs. 304,720 would be distributed in the ratio of 2400:2000:400 i.e 6:5:1 to A,B and C

A textile mill has two production departments, spinning and weaving and two service departments, X and Y. Variable cost of operating two service departments, their outputs and quantum of service rendered by them to other departments are as follows:

Page 106: cost accounting question icwai inter

The solution goes as follows; Suppose x and y are the total costs of two service departments.Then as per the question: x = 120,000 + 0.4 y and Y = 260,000 + 0.5 xThe solution gives x = Rs. 280,000 and y = Rs. 400,000Cost per unit of service department X = Rs. 280,000 / 10,000 = Rs. 28Cost per unit of service department Y = Rs. 400,000 / 20,000 = Rs. 20.Service received by production departments : Rs. Service received by Spinning 30% of X 30% of Rs. 280,000 84,000

25% of Y 25% of Rs. 400,000 100,000184,000

Service received by Weaving 20% of X35% of Y 20% of Rs. 280,000 56,000

35% of Rs. 400,000 140,000196,000

Total cost of service received by production departments : 380,000Check : Total variable cost of service departments : 380,000

The company has furnished the following data relating to two jobs undertaken by it in period.

Job 101 Job 102Rs. Rs.

Direct materials 54,000 37,500Direct labour 42,000 30,000Selling price 166,650 128,250Profit percentage on total cost 10% 20%Required:(i) Computation of percentage recovery rates of factory and administrative overheads.

(iii) Using the above recovery rates fix the selling price of Job 103. The additional data being:

Direct materials : Rs. 24,000; Direct labour : Rs. 20,000 Profit % on selling price : 12.50%Solution: Job 101 Job 102Selling price Rs./unit 166,650 128,250Profit percentage on total cost 10% 20%Cost of sales Rs. 151,500 106,875

Factory cost consists of direct materials, direct labour and factory overheads.Factory cost of job 101 54,000 + 42,000 + 42,000 F = 96,000 + 42,000 F Factory cost of job 102 37,500 + 30,000 + 30,000 F = 67,500 + 30,000 F

1. Find the total costs of service departments by any suitable method. Simultaneous equation method or repeated distribution method or trial and error method can be employed for this purpose. Simultaneous method being quick is followed here.

2. Divide the total cost by number of units produced by service departments. This will give cost per unit which is answer no. 1

3. The spinning receives 30% cost of X and 25% cost of Y. The weaving department receives 20% cost of X and 35% cost of Y. This is answer no.2.

Problem 16 (ca inter may 95) repeat nov.02 similar.

In an engineering company, the factory overheads are recovered on a fixed percentage basis on direct wages and the administrative overheads are absorbed on a fixed percentage basis on factory cost.

(ii) Calculation of the amount of factory overheads and administrative overheads and profit for each of the two jobs.

Let factory overhead recovery rate, as percentage of direct labour be F and administrative overheads recovery rate as % of factory cost be A.

Page 107: cost accounting question icwai inter

Administrative overheads of job 101 (96,000 + 42,000 F) AAdministrative overheads of job 102 (67,500 + 30,000 F) ATotal cost = Factory cost + Administrative overheadsJob 101 96,000 + 42,000 F + (96,000 + 42,000 F) A = 151,500Job 102 67,500 + 30,000 F + (67,500 + 30,000 F) A = 106,875Solving (see the solution carefully) (96,000 + 42,000 F) (1+ A) = 151,500

(67,500 + 30,000 F) (1+ A) = 106,875Which yields F = 0.60 and A = 0.25

Hence % recovery rates of factory and admin. overheads are 60% and 25% respectively.Statement showing factory and administrative overheads and profit of jobs

Job 101 Job 102 Job 103Rs. Rs. Rs.

Direct materials 54,000 37,500 24,000Direct labour 42,000 30,000 20,000Prime cost 96,000 67,500 44,000Factory overheads @ 60% of direct labour 25,200 18,000 12,000Factory cost 121,200 85,500 56,000Administ. overheads @ 25% of factory cost 30,300 21,375 14,000Total cost 151,500 106,875 70,000Profit on total cost 10% 20% 1/7 of costProfit 0 0 10,000Selling price 166,650 128,250 0Note : For Job 103 the profit as given as 12.5% of sales. It is equal to 1/7 of cost.

(ii) Estimated cost of maintenance of the machine is Rs. 25,000 per annum.

Calculate the machine hour rate, if (i) Setting up time is unproductive and (ii) Setting up time is productive.Solution: Solve yourself. Answer is supplied for reference,

Computation of machine hour rateSetting up unproductive Setting up productive

Rs. Rs.Standing charges per machine hr. 9.05 8.69Depreciation 45.45 43.63Electricity 48.00 46.07

Problem 17 (ca inter)

A manufacturing unit has purchased and installed a new machine of Rs. 12,70,000 to its fleet of 7 existing machines. The new machine has an estimated life of 12 years and is expected to realize Rs. 70,000 as scrap at the end of its working life. Other relevant data are as follows:

(i) Budgeted working hours are 2,592 based on 8 hours per day for 324 days. This includes 300 hours for plant maintenance and 92 hours for setting up of plant.

(iii) The machine requires a special solution, which is replaced at the end of each week (6 days a week) at a cost of Rs. 400 each time.

(iv) Four operators control operation of 8 machines and the average wages per person amounts to Rs. 420 per week plus 15% fringe benefits.

(v) Electricity by the machine during the production is 16 units per hour at a cost of Rs. 3 per unit. No current is taken during maintenance and setting up.

(vi) Departmental and general works overhead allocated to the operation during last year was Rs. 50,000. During the current year it is estimated to increase 10% of this amount.

Page 108: cost accounting question icwai inter

Special chemical solution 9.82 9.42Maintenance 11.36 10.91Machine hour rate 123.68 118.72Total machine hours 2,200 2,292

The expense data relating to the machine are as under:Cost of machine is Rs. 500,000. Life 10 years. Estimated scrap value at the end of life is Rs. 20,000. Rs. Repairs and maintenance p.a. 60,480Consumable stores p.a. 47,520Wage of operator per machine 2,500Rent of building per machine (the machine occupies 1/6 th of the area) 72,000Supervisor's salary per month (Common to three machines) 6,000General lighting charges per month allocated to the machine 1,000Power 25 units per hour at Rs. 2 per unit

Required: Calculate a two-tier machine-hour rate (a) set up time (b) running time.Solution: Tutorial notes:

Computation of Standing Charges per hour :Effective hours (208 -- 8) : 200 hours

per month effec.hr. per hourRs. Rs. Rs.

Supervisor salary (6000/3) 2,000General lighting 1,000Rent[ (72,000 / 6) x 1 /12] 1,000Total Standing Charges 4,000 200 20.0Computation of machine Charges per hour :Depreciation [( 500,000 -- 20,000) / 10] x (1/12) 4,000 200 20.0Repairs and Maintenance ( 60,480 / 12) 5,040 180 28.0Consumable stores (47,520 / 12) 3,960 180 22.0Power ( 25 units x Rs. 2 x 180 hours) 9,000 180 50.0Wages 2,500 200 12.5Machine hour rate per hour of running 152.5

Setting up time rate Depreciation wages Standing charges Total Rs.. 20 12.5 20 52.5

Problem 18 (ca inter may 02)

In a factory, a machine is considered to work for 208 hours in a month. It includes maintenance time of 8 hours and set up time of 20 hours.

Power is required for productive purposes only. Set-up time, though productive, does not require power. The supervision and operator are permanent. Repairs and maintenance and consumable stores vary with machine.

Question says that repairs and maintenance and consumable stores vary with the running of machine. It implies that effective hours for these items will be running hours only. Running hours can be computed as ( total hours less maintenance time less setup time). Power is required for productive purposes only. Thus running hours are also relevant for computing effective hour rate due to power.

For set up time rate per machine hour the relevant items of expenses would be standing charges, wages and depreciation only. Other expenses do not relate to setting up of machine.

Problem 19 (ca inter may 97)

Page 109: cost accounting question icwai inter

X Ltd. having fifteen different types of automatic machines furnishes information as under for 1996-97:

(i) Overhead expenses : Factory rent Rs. 96,000 (Floor area 80,000 sq.ft.)Heat and gas Rs. 45,000 and Supervision Rs. 120,0000.

In respect of machine B (one of the above machines) the following particulars are furnished:(i) Cost of machine Rs. 45,000, life 10 years and scrap value at the end Rs. 5,000(ii) Annual expenses on special equipment attached to machine Rs. 3,000(iii) Estimated operation time of the machine is 3,600 hours while setup time p.a. is 400 hrs.(iv) The machine occupies 5,000 sq.ft. of floor area.(v) Power costs Rs. 2 per hour while machine is in operation.

Work-order 31 Work-order 32Machine set up time (hours) 10 20Machine operation time (hours) 90 180Solution: Computation of standing charges per hour (total hours 4000)

Rs.Factory rent (Rs. 96,000/8,000 sq.ft x 5,000 sq.ft.) 6,000Heat and Gas (Rs. 45,000/15 machines) 3,000Supervision (Rs. 120,000/15 machines) 8,000Depreciation (Rs. 45,000 -- 5,000)/10 years 4,000Annual expenses on special equipment 3,000Total Standing charges for 4000 machine hours 24,000Fixed cost per hour (24,000 / 4,000) 6Computation of setup rate and operation rate per machine hour

Setup rate per hr. Operation rate per hr.Rs. Rs.

Fixed cost 6 6Power 0 2Wages 6 3

Comprehensive machine hour rate per hour 12 11Machine costs to be absorbed in two work orders:

Work-order 31 Work-order 32Rs. Rs.

Machine set up time cost ( Rs. ) 10 x 12 120 20 x 12 240Machine operation time cost (Rs.) 90 x 11 990 180 x 11 1,980

1,110 2,220

A company manufacturing two products furnishes the following data for a year.Product Annual output Machine hours No.of purchase orders No. of setupsA 5,000 units 20,000 160 20B 60,000 units 120,000 384 44The annual overheads are as under :Volume related activity costs Rs. 550,000. Setup related costs Rs. 820,000Purchase related costs Rs. 618,000.You are required to calculate the cost per unit of each product A and B based on :(i) Traditional method of charging overheads and (ii) Activity based costing method.

(ii) Wages of the operator are Rs. 48 per day for 8 hours. He attends to one machine when it is under set up and two machines while they are under operation.

Find out the comprehensive machine hour rate of machine B. Also find out machine costs to be absorbed in respect of use of machine B on the following two work-orders

Problem 20 (ca inter nov 02)

Page 110: cost accounting question icwai inter

Solution:

1. Machine hour rate = Total overheads / Total machine hours= (550,000 + 820,000 + 618,000) / (20,000 + 120,000) = Rs. 14.20/ hr.

2. Overhead of volume related activities per hour:= Cost of volume related activities / Total hours= 550,000 / 140,000 = Rs. 3.93/hr.

3. Cost of one setup = Setup related costs / no. of setups= 820,000 / 64 = Rs. 12,812.50

4. Cost of a purchase orde = Cost of purchase related activities / no. of purchase orders= 618,000 / (160 +384) = 618,000 / 544= Rs. 1,136 per order.

Statement showing Overhead cost per unit : Traditional Method

Products

A 5,000 20,000 14.2 284000 56.8B 60,000 120,000 14.2 1704000 28.4Statement showing Overhead cost per unit : Activities Based costing Method

A B TotalAnnual output in units 5,000 20,000Total machine hours 20,000 120,000Cost related to volume activities @ Rs. 3.93/hr. 78,600 471,600 550,200No. of purchase orders 160 384 544Cost related to purchase activities @ Rs. 1,136/order 181,760 436,224 617,984No. of setups 20 44 64Cost related to setups @ Rs. 12,812.50/setup 256,250 563,750 820,000Total cost Rs. 516,610 1471574 1988184

Job 1102 Job 1108Rs. Rs.

Selling price 107,325 157,920Profit as % on cost 8% 12%Direct materials 37,500 54,000Direct labour 30,000 42,000

You are required to compute :(i) The rates of factory overheads and selling and administration overheads to be charged.(ii) The selling price of new order.

Solution: See problem 14 above. blh oDr vPNk eglwl dhft;s A vHkh eqLdqjkb;sAAnswer: Factory overhead : 40% of direct labour and

Selling and admin. Overhead : 25% of factory cost.Selling price of the new order : Rs. 209,375

Working notes: Let us calculate overhead cost per activity.

Annual output in units

Machine hours

Machine hour rate in Rs.

Overhead cost

Overhead cost per unit in Rs.

Problem 21 (ca inter nov 02)

In the current quarter, a company has undertaken two jobs. The data relating to these jobs are as under:

It is the policy of the company to charge factory overheads, as % on direct wages and Selling and Administration overheads as % of Factory cost.

The company has received a new order for manufacturing of a similar job. The estimate of direct materials and direct wages relating to the new order are Rs. 64,000 and Rs. 50,000 respectively. A profit of 20% on sales is required.

Problem 22 (ca inter nov 00)

Page 111: cost accounting question icwai inter

Solution:

Rs. Actual overhead expenses incurred 446,380Overhead absorbed @ Rs. 1.25 per hour for 293,104 hours 366,380This is under absorption of overhead. Unabsorbed overhead 80,000Reasons for unabsorbed overhead 50% on account of increase in indirect material and indirect labour 40,00050% due to factory inefficiency 40,000Treatment of unabsorbed overheads in cost accounts:1. Use of supplementary rates: Unabsorbed overhead is charged to produced units as a supplementary rateSupplementary rate = Unabsorbed overhead / Units produced = 40,000 / (7,800 + 200)

= 40,000/8,000 = Rs. 5 per unit.The supplementary rate should be charged to goods sold, finished goods and wip.

Rs.Cost of sales 7,000 units @ Rs. 5 35,000 Reduce the profit by this amountFinished goods 800 units @ Rs. 5 4,000 Increase the value of stockWork-in-progress 200 units @ Rs. 5 1,000 Increase the value of stockTotal 40,0002. The unabsorbed overhead should be charged to costing profit and loss account.

The production and sales data for the year 1998-99 is as under:Production : Finished goods 20,000 units

Work-in-progress (50% complete in all respects) 8,000 unitsSales : Finished goods 18,000 units

You are required to :Calculate the amount of under-absorption of production overheads during the year 98-99.Show the accounting treatment of under-absorption of production overheads.Solution: Solve it yourself on the lines of solution to problem 20 above.

The total overhead expenses of a factory are Rs. 446,380. Taking into account the normal working of the factory, overhead was recovered in production at Rs. 1.25 per hour. The actual hours worked were 293,104. How would you proceed to close the books of accounts, assuming that besides 7,800 units produced of which 7,000 were sold, there were 200 equivalent units in work-in-progress ?

On investigation it was found that 50% of the unabsorbed overhead was on account of increase in the cost of indirect material and indirect labour and the remaining 50% was due to factory inefficiency. Also give profit implication of the method suggested.

Note :Under-absorption due to factory inefficiency, should not be charged to production units but should directly be debited to profit and loss account/

Problem 23 (ca inter nov 99)

ABC ltd manufactures a single product and absorbs the production overheads at predetermined rate of Rs. 10 per machine hour.

At the end of financial year 1998-99, it has been found that actual production overheads incurred were Rs. 600,000. It included Rs. 45,000 on account of 'written off' obsolete stores and Rs. 30,000 being the wages paid for the strike period under an award.

The actual machine hours worked during the period were 48,000. It has been found that one-third of the under-absorption of the production overheads was due to lack of production planning and the rest was attributable to normal decrease in costs.

Page 112: cost accounting question icwai inter

Answer: Unabsorbed overhead (600,000 -- 45,000 -- 30,000 -- 480,000 = Rs. 45,000)

Supplementary rate = Unabsorbed overhead / Units produced =30,000 / 24,000 = Rs. 1.25.Accounting treatment :Work-in-progress control A/c Dr. Rs. 5,000Finished goods control A/c Dr. Rs. 2,500Cost of sales A/c Dr. Rs. 22,500Profit and loss account Dr. Rs. 15,000

To Overhead control A/c Rs. 45,000

Manufacturing overheads : Amount actually spent Rs. 170,000Amount absorbed Rs. 150,000Cost of goods sold Rs. 336,000Stock of finished goods Rs. 96,000Work-in-progress Rs. 48,000

Solution: u fp<sa] u xqLlk gksa] 'kkarfpRr jgus dk vH;kl djsaA

The apportionment may be carried out on the basis of value of the elements.

Absorption of under-absorbed overhead of Rs. 20,000additional overhead Rs. Cost Total cost

Cost of goods sold (20,000 x 336) / (336 + 96 + 48) 14,000 336,000 350,000Finished goods (20,000 x 96) / (336 + 96 + 48) 4,000 96,000 100,000Work-in-progress (20,000 x 48) / (336 + 96 + 48) 2,000 48,000 50,000Thus use of this method will reduce by Rs. 14,000.

Problem 25 (ca final June 2009)

Rs.(i) Factory overheads under absorbed 5,000(ii) Administration overheads over absorbed 3,000(iii) Depreciation charged in financial accounts 70,000

Note: The one-third of under-absorbed overheads was due to lack of proper production planning. This is abnormal loss, therefore should not be charged to production units. This should be debited to profit and loss account directly. Supplementary rate should be used for the 2/3rd of under-absorbed overheads i.e. for Rs. 30,000 only.

Problem 24 (ca inter nov 97)

Sweet Dreams Ltd. uses a historical cost system and absorbs overheads on the basis of predetermined rate. The following data are available for the year ended 31st March 1997.

Using two methods of disposal of under-absorbed overheads show the implication on the profits of the company under each method.

According to first method, a supplementary rate may be determined and used to accommodate the unabsorbed overhead. The under-absorbed amount in total may, at the end of accounting period would be apportioned on proportionate basis over cost of goods sold, stock of finished goods and work-in-progress.

Thus if this method is applied to the above case, The under-absorbed amount of Rs. 20,000 would be apportioned to Cost of goods sold, Finished goods and Work-in-progress in the ratio of 336:96:48.

According to second method, The entire amount of under-absorbed overheads of Rs. 20,000 would be written off to costing profit and loss account, thus use of this method will bring down the profit by an amount equal to under-absorbed overheads i.e. Rs. 20,000.

A manufacturing company has disclosed a net loss of Rs. 2,13,000 as per their cost accounting records for the year ended March 31, 2009. However, their financial accounting records disclosed a net loss of Rs. 2,58,000 for the same period. A scrutiny of data of both the sets of books of accounts revealed the following information :

Page 113: cost accounting question icwai inter

(iv) Depreciation charged in cost accounts 80,000(v) Interest on investments not included in cost accounts 20000(vi) Income-tax provided in financial accounts 65,000(vii) Transfer fees (credit in financial accounts) 2,000(viii) Preliminary expenses written off 3,000(ix) Over-valuation of closing stock of finished goods in cost accounts 7,000Prepare a Memorandum Reconciliation Account.Solution :Memorandum Reconciliation Account

Particulars Amount Rs.To net loss as per costing books 2,13,000 By Admin. ohs over absorbed in cost a/c To factory overhead under absorbed in cost a/c 5,000 By Depreciation over charged in cost a/cTo Income tax not provided in cost accounts 65,000 By interest on invest not included in cost a/To over-valuation of closing stock 7,000 By transfer fees in financial books To preliminary expenses written off 3,000 By net loss as per financial books

2,93,000The greatest foolishness is to reject something you know nothing about.

Problem 26 (CWA Inter June 09)

Departments A B C P Q From Primary distribution Rs. 3,150 3,700 1,400 2,250 1,000The expenses of the service departments are to be apportioned on the percentage basis as follows:Departments A B C P Q P % 40 30 20 – 10Q % 30 30 20 20 –Prepare secondary distribution summary by simultaneous equations method.Solution : It is very easy question.Let P and Q are total overheads of departments P and Q.Therefore P = 2,250 + 20% of Q and Q = 1,000 + 10% of P

Or P = 2,250 + 0.20 Q and Q = 1,000 + 0.10 PSolving we get P = Rs. 2,500 and Q = Rs. 1,250.

Statement of secondary distribution :Departments A B CFrom Primary distribution Rs. 3,150 3,700 1,400From P : 40% : 30% : 20% of Rs. 2,500 1,000 750 500From Q : 30% : 30% : 20% of Rs. 1,000 300 300 200

4,450 4,750 2,100Additional Knowledge :if you adopt repeated distribution method, the total cost would be as follows:Departments A B C P Q From Primary distribution Rs. 3,150 3,700 1,400 2,250 1,000Distribution of P Rs. 2250 900 675 450 0 225

1,225Distribution of Q Rs. 1225 368 368 245 245 0Distribution of P Rs. 245 98 73 49 0 25Distribution of Q Rs. 25 9 9 7 0 0

4,525 4,825 2,151If you decide to take deptt. Q first, the total cost would be as follows;Departments A B C P Q

A company has three production departments, A, B and C and two service departments, P and Question. The following figures are available from the primary distribution summary.

Page 114: cost accounting question icwai inter

From Primary distribution Rs. 3,150 3,700 1,400 2,250 1,000Distribution of Q Rs. 1000 300 300 200 200 0

2,450Distribution of P Rs. 2450 980 735 490 245Distribution of Q Rs. 245 74 74 49 48Distribution of P Rs. 48 20 16 9

4,524 4,825 2,148Problem 27 (CWA Inter Dec.08)

From Maintenance Scheduling Molding AssemblyMaintenance ― 10% 40% 50%Scheduling 20% ― 50% 30%Total overhead cost Rs. 750,000 400,000 378,000 276,000

Solution : M = 750,000 + 0.20 S and S = 400,000 + 0.10 M = 400,000 + 0.10 ( 750,000 + 0.20 S)Which give S = 475,000 / 0.98 = Rs. 484,694 and M = Rs. 846,939

Molding Rs. Assembly Rs.From Maintenance 40% of Rs. 846,939 338,776 50% of Rs.846,939 423,470From Scheduling 50% of Rs.484,694 242,347 30% of Rs. 484,694 145,408Direct cost 378,000 276,000Total cost 959,123 844,878

A company makes components for television sets using two service departments and two production departments. The inter-departmental relationship and overhead costs are given below:

You are required to show the amount of Scheduling department costs and Maintenance department costs to allocated to the Production Departments using Simultaneous equation method.

fdlh dke esa :fp ysaA fdlh pht esa iwjk eu yxk;saA mlesa vius vkidks iwjk >ksad nsaA dqN djsaA dqN cusaA

Page 115: cost accounting question icwai inter

blh oDr vPNk eglwl dhft;s A vHkh eqLdqjkb;sA

Page 116: cost accounting question icwai inter

blh oDr vPNk eglwl dhft;s A vHkh eqLdqjkb;sA

Page 117: cost accounting question icwai inter

1/7 of cost

Page 118: cost accounting question icwai inter

blh oDr vPNk eglwl dhft;s A vHkh eqLdqjkb;sA

Page 119: cost accounting question icwai inter

u fp<sa] u xqLlk gksa] 'kkarfpRr jgus dk vH;kl djsaA

A manufacturing company has disclosed a net loss of Rs. 2,13,000 as per their cost accounting records for the year ended March 31, 2009. However, their financial accounting records disclosed a net loss of Rs. 2,58,000 for the same period. A scrutiny of data of both the sets of books of accounts revealed the following information :

Page 120: cost accounting question icwai inter

Amount3,000

10,00020,000

2,0002,58,0002,93,000

The greatest foolishness is to reject something you know nothing about.

A company has three production departments, A, B and C and two service departments, P and Question. The

Page 121: cost accounting question icwai inter

A company makes components for television sets using two service departments and two production

You are required to show the amount of Scheduling department costs and Maintenance department costs to

Page 122: cost accounting question icwai inter

Chapter 4: Contract Costing cwa inter cwa fin. ca inter ca final6 0 5 1

Solution:First of all let us calculate the estimated profit: Rs. In lakhsExpenditure incurred up to 31.03.04 19.80Estimated additional expenditure (including contingencies) 1.20Estimated total cost 21.00Contract price 25.00Estimated profit (also known as notional profit) 4.00Profit to be taken to profit and loss account:(i) Estimated profit x Cash received x Work certified

Work certified Contract priceCash received Rs. 21.60 lakhs contract price Rs. 25 lacs.Work certified Rs. 24.00 lakhs= 4 x ( 21 / 24 ) x ( 24 / 25 ) = Rs. 3.45 lakhs

(ii) Estimated profit x Cash received x Cost of work to dateWork certified Estimated total cost

Cost of work to date Rs. 19.80 lakhsEstimated total cost Rs. 21.00 lakhs= 4 x ( 21 / 24 ) x ( 19.8/ 21 ) = Rs. 3.39 lakhs

(iii) Estimated profit x Cash received x ( 2/3)Work certified

= 4 x ( 21 / 24 ) x ( 2 / 3 ) = Rs. 2.40 lakhs

(iv) Estimated profit x Work certifiedContract price

= 4 x ( 24 / 25 ) = Rs. 3.84 lakhs.

A firm of contractors furnished the following information for the year ended 31.12.000 (Rs. In lakhs)

Rs. Lacs Rs. LacsContract value 10.00 Labour expended 2.00Material sent to site 2.00 Direct expenses 0.50Material returned from site 0.20 Overheads apportioned 0.25Material at site on 31.12.00 0.10 Work certified 6.00Cash received 4.80 Cost of work not certified 0.50

vxj vki bl ckr dks ysdj vk'oLr vkSj fuf'par gks tk,a fd vkidk [qk'k vkSj dke;kc gksuk rks r; gS vkSj tks Hkh :dkoVsa ;k ck/kk;sa vk jgh gSa] oks vkidks y{; gkf;y djus ds fy;s vko';d vuqHko fnykus ds fy;s vk jgh gS] rks fQj vkidks dksbZ ugha jksd ldrkA

Problem 1 (cwa inter dec.04)

An amount of Rs. 19,80,000 was incurred on a contract work up to 31.03.2004. Certificates have been received to date to the value of Rs. 24,00,000 against which Rs. 21,60,000 has been received in cash. The cost of work done but not certified amounted to Rs. 45,000. It is estimated that by spending an additional amount of Rs. 1,20,000 (including provision to contingencies) the work can be completed in all respects in another two months. The agreed contract price of the work is Rs. 25 lakhs. Compute a conservative estimate of the profit to be taken to profit and loss account.

Problem 2 (cwa inter II June 01)

Page 123: cost accounting question icwai inter

Plant installed at site 1.00 Value of plant on 31.12.0 0.30Material costing Rs. 10,000 were sold for Rs. 15,000. Material sold as scrap were for Rs. 5,000.The firm has decided to follow a transparent provisioning norm as indicated below:Stage of completion upto 25% 25%--50% 51%--75% above 75%Provision 100% 60% 30% 10%

Prepare (i) Contract account (ii) Work-in-progress accountWorking required: Provision for unexpired risk on contract.Solution: Rs. In lakhsWorking notes: 1. Computation of provision for unexpired risk on contract

Work certified 6.0Cost of work not certified 0.5Cost of work done 6.5% of work done of contract value of Rs. 10 lakhs 65%Provision for 65% completion of work 30%

Jhakajhak and Dhakadhak company Ltd.

Contract A/c for the year ended March 31,2000Rs. Lacs Rs. Lacs

To Material sent to site 2.00 By material returned from site 0.20To Labour expended 2.00 By material sold 0.15To Direct expenses 0.50 By scrap sold 0.05To Overheads apportioned 0.25 By work certified 6.00To Plant installed at site 1.00 By work not certified 0.50To Profit on sale of material 0.05 By Plant at site 0.30To Profit on contract 1.50 By material at site 0.10

7.30 7.30Provisions to made out of profit of Rs. 1,50,000 is as follows: Rs.

30% on account of work completed (30% of Rs. 150,000) 45,00010% on account of cash retention of Rs. 120,000 12,000

Total provision 57,000Appropriation of profit on contract account

Rs. Lacs Rs. LacsTo provision on unexpired contract 0.57 By Contract a/c 1.50To profit and loss account 0.93

1.50 1.50

1.11.03 to 31.10.04 1.11.04 to 31.03.05Actual Estimated

Rs. Rs.Material issued 675,000 1237500Labour Paid 450,000 562,500

Prepaid 25,000 0Outstanding 0 2,500

Plant purchased 375,000 0

Stage of completion is decided on the basis of work done till date. The above mentioned provision is to be increased by an additional provision of 10% on the cash retention part.

2. Cash is retained to the extent of Rs. 1.20 lakhs ( work certified -- cash received). 10% of it will be placed as provision.

Problem 3 (ca inter II nov.04. similar may 01, nov 06)

Brock Construction Ltd. commenced a contract on November 1,2003, The total contract was for Rs. 39,37,500. It was decided to estimate the total profit on the contract and to take to the credit of P/l A/c that proportion of estimated profit on cash basis, which work completed bore to the total contract. Actual expenditure for the period November 1, 2003 to October 31, 2004 and estimated expenditure for November 1, 2004 to March 31, 2005 are given below:

Page 124: cost accounting question icwai inter

Expenses Paid 200,000 350,000Outstanding 50,000 25,000

Plant returned to store (historical cost) on 31.3.04 75,000 300,000Work certified 2000000 FullWork uncertified 75,000Cash received 1750000Material at site 75,000 37,500

Solution:Computation of depreciation on plant :From 1.11.03 to 31.10.04 i.e. for one year :1/3 rd of (375000 -- 75000) Rs. 100,000From 31.10.04 to 31.3.05 i.e. for 5 months 1/3 rd of ( 200,000 x 5/12) Rs.27,778From 1.11.03 to 31.3.04 on 75,000 (plant returned to store) Rs. 10,417** 1/3rd of (75,000 x 5/12 )Value of plant returned to store Rs. Rs.on 31.3.04 Cost 75,000

Less: Depreciation 10,417 64,583on 31.3.05 Cost 200,000

Less: Depreciation 27,778 172,222Let us first compute the estimated profit on the entire contract

Brock Construction Ltd. Contract a/c from 1.11.03 to 31.03.05 Rs.in '000To Material sent to site By Contractee's A/c 3,937.5(675 + 1237.5) 1912.5 By plant returned to storeTo labour (450 + 562.5 +2.5) 1015.0 on 31.3.04 64.6To Plant purchased 375 on 31.3.05 172.2 236.8To Expenses(200 + 350 + 25) 575 By material at site 37.5Subtotal 3877.5

To notional profit 334.34,211.8 4,211.8

Transfer of notional profit to profit and loss account :

Transfer of notional profit to profit and loss account : = 334,305 x ( 17.50/20.00) x (20.75/ 39.375) Rs. 154,152

Brock Construction Ltd. Contract a/c from 1.11.03 to 31.10.04 Rs.in '000To Material sent to site 675.0 By work certified 2,000.00To labourpaid 450 By work not certified 75.00Less: Prepaid 25 425.0 By Plant at siteTo Plant purchased 375.0 Historical cost 300To Expenpaid 200 Less: Depreciation 100 200.00add: outstanding 50 250.0 By plant returned to store on 31.3.04To profit and loss account 154.2 Historical cost 75.00To profit in reserve 535.4 Less: Depreciation 10.41 64.6

By material at site 75.002,414.6 2,414.6

The plant is subject to annual depreciation @ 33.33% on written down value method. The contract is likely to be completed on March 31,2005.

You are required to prepare the contract a/c. Determine the profit on contract for the year Nov. 03 to Oct. 04 on prudent basis, which has to be credited to P/l A/c.

The question states that ''to take to the credit of P/l A/c that proportion of estimated profit on cash basis, which work completed bore to the total contract.''

Thus work completed is treated different from work certified. Thus work completed should be taken as work certified plus work uncertified.

Problem 4 (ca inter II may 01)

Page 125: cost accounting question icwai inter

Direct materials Direct labour Overheads Erection costs to dale Total 280 100 60 110 550

The contract price is Rs. 11 crores and cash received on account till 31.3.01 was Rs. 6 crores.The technical estimate of the contract indicates the following degree of completion of work.Direct materials 70%; Direct labour and overheads 60%; Erection 40%

Solution:

Statement showing estimated profit on completed contract and partly completed contract( in lakhs)Particulars Cost upto 31.3.01 Future costs Total costs

% completi Amount % completi Amount Rs. LakhsDirect materials 70 280 30 120 400Direct labour 60 100 40 66.67 166.7Overheads 60 60 40 40 100Erection 40 110 60 165 275

550 392 941.7Contract price 1100.00

Profit estimated on whole contract 158.3Proportional profit on partly completed contract: 158.33 x ( 550 / 941.57) 92.5Work Certified : Cost of work + estimated profit = 550 + 92.48 = Rs. 642.48 lakhs.

Degree of completion of contract on 31.3.01 : Work Certified / Contract Price: 642.48 / 1100 = 58.40%

Rs.'000 Rs.'000Material at site 5,000 Direct wages paid 3,800Plant hired 700 Site office costs 270Material returned from site 100 Direct expenses 500Work Certified 10,000 Progress payment receiv 7,200

Required : Prepare the contract account for the year ended 31.02.02.Compute the profit to be taken to the profit and loss account.

Solution: Solve by yourself. Clues are given below. (fig. in '000)

Cost of contract : [5000+(3800+110)+700+270+500+300(dep)--1800--100] = 8780Profit = Work certified -- cost of contract = 10,000 -- 8,780 = 1,280Completion of contract = 100/108 = 92.6%Profit to be taken to profit and loss account :

= Budgeted profit or Estimated profit x (Cash received / Contract price)= 1800 (given) x 7,200 / 10,800 = 1,200

Rs. Rs.

Paramount Engg. are engaged in construction and erection of a bridge under a long-term contract. The cost incurred upto 31.3.01 was as under ( Rs. In lakhs )

You are required to estimate the profit that could be taken to profit and loss account against this partly completed contract as at 31.3.01

Notional profit is to be computed on the basis of degree of completion of various components of contract and their costs.

Problem 5 (ca inter II nov. 02)

A construction company undertook a contract at an estimated price of Rs. 108 lacs, which includes a budgeted profit of Rs. 18 lacs. The relevant data for the year ended 31.3.02 are as under: (fig. in Rs. '000)

A special plant was purchased specifically for this contract at Rs. 8,00,000 and after use on this contract till the end of 31.02.02, it was valued at Rs. 5,00,000. The cost of material at site at the end of the year estimated at Rs. 18,00,000. Direct wages accrued as on 31.03.02 was Rs. 1,10,000.

Problem 6 (ca inter II may. 98)

Compute a conservative estimate of profit on a contract (80% complete) from the following particulars. Illustrate four methods of computing the profit.

Page 126: cost accounting question icwai inter

Total expenditure to date 170,000 Work certified 200,000Contract price 306,000 Work not certified 17,000Cash received 163,200Estimated further expenditure to complete the contract 34,000Solution: Students should note the distinction between notional and estimated profit.1.Computation of notional profit: Rs.Cost of work certified = Expenditure upto date -- work u(170000-17000) 163,000Notional profit = Value of work certified -- Cost of work c(200,000-163000) 37,0002. Computation of Estimated profit (306,000 -- 170,000 -- 34,000) 102,000Four methods of computation of conservative estimates of profit:1. Estimated profit x ( work certified / contract price )

'= 102,000 x ( 200,000 / 306,000) 66,6662. Estimated profit x ( cash received / contract price )

= 102,000 ( 163,200 / 306,000 ) 54,4003. Notional profit x ( work certified / contract price )

'= 47,000 x ( 200,000 / 306,000) 30,7184. Notional profit x ( cash received / work certified )

= 47,000 ( 163,200 / 200,000 ) 25,568

Materials purchased 10.0 General expenses 1.0Wages paid 4.5 Plant purchased 5.0Work certified 20.0 Materials on hand closing 2.5Cash received 15.0 Wages outstanding closing 0.5Work uncertified 1.5 Depreciation of plant 0.5The escalation clause in the contract was as follows:

Prepare the contract account. Show your workings.Solution :

You should compute the contract escalation, because contract account will remain incomplete without this.

Working Notes: 1 Computation of escalation: Rs. In lacs

Rs. Lacs

Material purchased 10.0

Less : Closing stock of material 2.5

Problem 7 (cwa inter June 2008)

A Limited undertook a contract for Rs. 50 lacs on 1st April 2006. On 31st March 2007 when accounts were closed the following details were available:

In the even price of materials and rate of wages increase by more than 5%, the contract price would be increased accordingly by 25% of the rise in the cost of materials and wages beyond in each case.

It was found that since the date of signing the agreement the prices of material and wages rates increased by 25%. The value of the work certified does not take into account the effect of the above clauses.

The escalation clause in the contract is important to understand. The escalation clause is applicable to materials consumed and not to the material purchased. You should notice that the figures supplied are inclusive of increase in prices of materials and wages rates.

To compute the effect of escalation clause, you must first determine the cost of material and labour exclusive of price hike. This is basic cost. Increase basic cost by 5% because no escalation is payable up to 5% increase. This increased cost is the datum beyond which escalation is payable @ 25% of the rise.

Since the contract is less than 50% complete but more than 25%, only one third of notional profit would be transferred to the profit and loss account. You should show the working thereof.

The material consumed is actual price which is more than base price by 25%. To get the basic cost, this should be multiplied with the factor 100/125 or 0.80.

Page 127: cost accounting question icwai inter

Material consumed 7.5

Material consumed without escalation (7.5 x 0.80) 6.0

Add : 5% increase 6.3

Escalation @ 25% beyond 6.30 i.e. 25% of (7.50 -- 6.30) 0.3

Working Notes: 1 Computation of escalation: Rs. In lacs : For Wages increase

Wages paid 4.5

Wages outstanding closing 0.5

Total wages 5.0

Total wages at basic price 4.0

Add : 5% increase 4.2

Escalation @ 25% beyond 4.2 i.e. 25% of (5.0 -- 4.20) 0.2

Contract Account of Diljale LtdFor the year ended 31st March,2007

Particulars Rs. Lacs Particulars Rs. LacsMaterials purchased 10.0 Work certified 20.0Wages paid 4.5 Work uncertified 1.5Wages outstanding closing 0.5 Materials on hand closing 2.5General expenses 1.0 Contract escalationDepreciation of plant 0.5 Material 0.3Notional profit 8.0 Wages 0.2 0.5

24.5 24.5Profit to be taken to profit and loss account :

Contract Account of Diljale LtdTo Profit and loss a/c 2.0 By Balance b/d 8.0To Work-in-progress a/c 6.0

Contract 1 Contract 2Contract price 27,00,000 60,00,000Materials 580,000 10,80,000Wages paid 11,24,000 16,50,000Other segments 28,000 60,000Plants at site 160,000 300,000Unused material at site 40,000 60,000Wages payable 36,000 54,000Other expenses due 4,000 9,000Work certified 16,00,000 30,00,000Cash received 12,00,000 22,50,000Work certified but not yet certif 80,000 90,000

Solution:

Contract 1 A/c

Notional profit is Rs. 8 lacs. The profit to be transferred to profit and loss account would be determined by the formula 1/3 x Notional profit x (cash received / work certified) i.e.1/3 x 8.0 x (15 / 20) = Rs. 2.00 lacs.

Problem 8 (ca inter may. 98,cs inter dec.97)

Modern construction Ltd. has taken two contracts on 1st October 1997, The position of contracts on 30th Sept.1997 is as follows:

The plant at site is to be depreciated at 10%. Prepare contract accounts in respect of each work showing the notional profit and also profit to be transferred to Profit and loss account.

Tutorial notes: 1. Use figures either in lacs or thousands.

Page 128: cost accounting question icwai inter

Rs.'000 Rs. '000To Materials 580 By work certified 1,600Less: Unused material (40) 540 By work not yet certified 80To wages 1124 By loss to P/ l a/c 68Add: Wages payable 36 1,160To Other expenses 28Add: expenses due 4 32Depreciation 10% of Rs. 160,000 16

1,748 0Note : Entire loss should be taken to profit and loss account in the same year.

Contract 2 A/cParticulars Rs.'000 Particulars Rs. '000To Materials 1,080 By work certified 3,000Less: Unused material (60) 1,020 By work not yet certified 90To wages 1,650Add: Wages payable 54 1,704To Other expenses 60Add: expenses due 9 69Depreciation 10% of Rs. 300,000 30To notional profit 267

3,090 3,090Transfer of notional profit to Profit and loss account :Work is 50% complete. The profit to be transferred to profit and loss account will be :

2/3 (Cash received / Work certified) x Notional profit= 2/3 x 22.50/30.00 x 2,67,000 = Rs. 133,500

Rs.Material and stores 187,000Wages paid 270,000Plant hire charges and other expenses 60,000Establishment expenses 54,000Material unused 11,000Work certified 600,000Cash received 540,000Work not yet certified at cost 20,000It is estimated that the following further expenses will be required to complete the work :

Solution :Tutorial Notes :

Problem 9 (cs inter Dec. 2008)

XYZ Contractors obtained a contract to construct a house for Rs. 8,00,000. Work was started on 1st January, 2007 and it was estimated that the contract would take 15 months to complete. Work is proceeding according to schedule and the details upto 31st Dec, 2007 were as follows :

Additional Material : Rs. 25,000 ; Wages : Rs. 20,000, Sub-contract cost : Rs. 50,000 ; Plant hire charges : Rs. 10,000; Establishment expenses : Rs. 11,800; and Provision for contingencies : 5% of total cost.

You are required to calculate the value of work-in-progress as on 31st December,2007 taking credit for a reasonable profit and also show the contract account.

1. Compute the percentage of completion of contract. The stage of completion will decide the profit, if any, to be taken to profit and loss a/c. According to AS --7, the percentage of completion would be estimated by comparing total cost incurred to date with total cost expected for the entire contract.

Page 129: cost accounting question icwai inter

3. You should revise your knowledge of Accounting Standards (AS) -- 7, Construction Contracts.Solution : In the books of XYZ Contractors Dr. Contract Account as on 31st December, 2007 Cr.

Rs. Rs.Material and stores 187,000 Material unused 11,000Wages paid 270,000 Work certified 600,000Plant hire charges and other 60,000 Work not yet certified at cost 20,000Establishment expenses 54,000Notional profit 60,000

631,000 631,000

Total expenses incurred upto date : (631,000 -- 60,000 -- 11,000) 560,000(Total expenses can also be obtained as 187,000 + 270,000 + 60,000 + 54,000 -- 11,000)

Profit to profit and loss a/c = (Estimated profit x work certified) / Contract price ORProfit to profit and loss a/c = (Estimated profit x cash received) / Contract priceComputation of Estimated profit : Rs.Total expenses incurred upto date 560,000Add : Additional Estimated cost

Materials (25,000 + 11,000) 36,000Wages 20,000Sub-contract cost 50,000Plant hire charges 10,000Establishment charges 11,800Estimated cost before provision 687,800Add : Provision for contingencies (5/95 x 687,800) 36,200Estimated total cost of contract 724,000Contract Price 800,000Estimated profit 76,000

Determination of stage of completion :

Costs incurred upto date 560,000

Estimated total cost of contract as computed earlier 724,000

Stage of completion of contract : (560,000 / 724,000) 77%Profit to be taken to profit and loss a/c = (Estimated profit x Work certified) / Contract price

Profit to profit and loss a/c = ( 76,000 x 600,000) / 800,000 57,000Alternatively Profit to profit and loss a/c = ( 76,000 x 540,000) / 800,000 51,300

Computation of profit to date as per AS-7:Notional profit 60,000Less : Profit to be taken to profit and loss a/c 57,000Transferred to Work-in-progress 3,000According to AS -- 7, Profit to date is calculated as follows :Profit to date = (Cost of work certified x estimated profit) / Estimated total costProfit to date = ( 600,000 x 76,000) / 724,000 62,983

Valuation of Work-in-progress : Rs.Value of work certified 600,000Cost of work uncertified 20,000

2. Expenses incurred upto Dec.07 are given. The estimated expenses to complete the contract are also given. You can find the total cost (without contingencies) just by summing up the incurred and estimated expenses. Problem will arise when you come to provision for contingencies. This is 5% of total cost and total cost with contingencies, is not known.

The proportion of notional profit of Rs. 60,000 as determined above to be taken to profit and loss a/c, will depend on stage of completion of contract.

The contract is 70% complete and estimate expenses are also known to complete the contract. The profit to profit and loss a/c will be determined by the following formula:

Page 130: cost accounting question icwai inter

620,000Less : Reserve for unrealised profit 3,000Less : Amount received from contractee 540,000 543,000Value of Work-in-progress 77,000

Problem 10 (ca final June 2009)

Rs.Cost of work-in-progress uncertified 8,000 96000

Profit transferred to Profit & Loss A/c at the end of year I on incomplete contract 60,000 120000

Cost of work to date 88,000Calculate the value of work-in-progress certified and amount of contract price.Solution :Contract is estimated to be 80% Completed.The Contractee pays 75% of value of work certified

When contract is 80% complete, the profit transferred to Profit and loss a/c is equal to2/3 x Notional Profit x (cash received) / (work certified).

Profit transferred = 60,000 = 2/3 Notional Profit x 0.75 which gives Notional profit = Rs. 120,000Contract A/c

Rs. Rs.

To Cost to date 88,000 By Work certified. (B/L) 2,00,000To Notional Profit 1,20,000 By Work uncertified. 8,000

2,08,000 2,08,000To P/ L A/c 60,000 By Notional Profit 1,20,000To Work in Progress 60,000

1,20,000 1,20,000The amount of contract price would be Rs. 250,000 (200,000 / 0.80)

Particulars Amounts Rs.Wages 140,000Plant 35,000Materials 105,000Head office expenses 12,500

(a) that the contract would be completed by 30th Sept, 2008;(b) the wages to complete would amount Rs. 84,750;(c) that the materials in addition to those in stock on 31st March, 2008 would cost Rs. 50,000.

(i)          A contract is estimated to be 80% complete in its first year of construction as certified. The contractee pays 75% of value of work certified, as and when certified and makes the final payment on the completion of contract. Following information is available for the first year :

The profit transferred is given as Rs. 60,000 and cash certified is 0.75 x work certified. Using these figures, the following equation appears;

If some one points out your mistakes, be happy that at least someone is interested in what you have done !Problem 11 (cwa inter June 2009)

Prabhu builders Ltd. commenced on 1st April,2007 on a contract of which the agree price was Rs. 5 lakhs. The following expenditure was incurred during the year up to 31st March, 2008.

Materials costing Rs. 10,000 proved unsuitable and were sold for Rs. 11,500 and a part of plant was scrapped and sold for Rs. 1,700. Of the contract price, Rs. 240,000 representing 80% of work certified had been received by 31st March, 2008 and on that date the value of plant on the job was Rs. 8,000 and the value of materials was Rs. 3,000. The cost of work done but not certified was Rs. 25,000.

It was decided to (a) Estimate what further expenditure would be incurred in completing the contract, (b) Compute from the estimate and the expenditure already incurred, the total profit that would be made on the contract and (c) Ascertain the amount of profit to be taken which the value of work certified bears to the contract price. Details of the estimates to complete the contract are given below:

Page 131: cost accounting question icwai inter

(e) that the head office expenses would be same rate as previous year;(f) that claims, temporary maintenance and contingencies would require Rs. 9,000.

Solution : Tutorial Notes :

Contract a/c for the year ended 31st March, 2008Particulars Rs. Particulars Rs.To Wages 140,000 By Plant in hand 8,000To Plant 35,000 By Material in hand 3,000To Materials 105,000 By Cash (material sold) 11,500To HO expenses 12,500 By Cash (plant sold) 1,700To Profit and loss a/c 1,500 By Work-in-progress (profit on sale of materials) work certified 300,000To Notional profit 55,200 work uncertified 25,000 325,000

349,200 349,200The notional profit, will be divided in profit transferred to profit and loss a/c and reserve to be taken to next year.Transfer to profit and loss a/c on 31.3.08 = estimated profit x (work certified / contract price)Transfer to Profit and loss a/c on 31.3.08 : 60,200 x (300,000 / 500,000) = Rs. 36,120Working Notes :1. Material and plant used during the year : Rs. Rs.Material purchased 105,000 Plant introduced 35,000Less : Material in hand 10,000 Less : Plant in hand 1,700

Material sold 30,000 13,000 Less : Plant sold 8,000 9,700Material used during the year 92,000 Plant used during the year 25,3002. Estimates of materials and plant to be used from 1.4.08 to 30.9.08

Rs. Rs.Material in hand at the beginning 3,000 Plant in hand 8,000Material further introduced during 6 months 50,000 Introduced 15,000Estimated materials to be used during 6 months 53,000 Less : residual value 6,000

Plant to be used 17,0003. Computation of Estimated profit till the completion of contract:

Rs. Rs. Total Rs. Expenses during 2007 - 08 Materials 92,000 53,000 145,000

Plant 25,300 17,000 42,300Wages 140,000 84,750 224,750HO expenses 12,500 6,250 18,750Contingencies 0 9,000 9,000

(HO expenses at the same rate as that of previous year) 269,800 170,000 439,800Contract Price 500,000Profit 60,200Transfer to Profit and loss a/c on 31.3.08 : 60,200 x (300,000 / 500,000) = Rs. 36,120

The following was the expenditure on a contract for Rs. 1200,000 commenced in January 2008.Materials : Rs. 240,000 ; Wages : Rs. 328,000 ; Plant : Rs. 40,000 and Overheads : Rs. 17,200.

(d) that further Rs. 15,000 would have to be spent on plant and the residual value of the plant on 30th sept. 2008 would be Rs. 6,000.

Prepare contract account for the year ended 31st March, 2008 and show calculations of the sum to be credited to Profit and loss a/c for the year.

Material costing Rs. 10,000 was sold for Rs. 11,500. The profit earned is Rs. 1,500. This profit is not related to contract and this entire transaction should be kept out of contract a/c. You can do this in two ways :

1. Remove the cost of material sold from the contract a/c. Material consumed is Rs. 105,000 − Rs. 10,000 = Rs. 95,000. Use material consumed as Rs. 95,000 in contract a/c.

2. Credit contract a/c with the sale proceeds of material i.e. Rs. 11,500 and debit it with amount of profit earned on sale of material i.e. Rs. 1,500.

Problem 12 (cwa inter dec.08)

Page 132: cost accounting question icwai inter

Cash received on account of the contract up to 31st December was Rs. 480,000 being 80% of the work certified.The value of materials in hand was Rs. 20,000. The plant has undergone 20% depreciation.Prepare contract account.Solution : In the books of Obama and Osama Ltd.

Contract AccountRs. Rs.

To Materials 240,000 By Work certified (480,000/0.80) 600,000To Wages 328,000 By Plant 32,000To Plant 40,000 By Materials in hand 20,000To Overheads 17,200To Notional profit 26,800

652,000 652,000To Profit and loss a/c(26,800 x 2/3 x 0.80) 14,293 By Notional profit 26,800To Balance 12,507

26,800 26,800

os gh yksx [kq'k jg ldrs gS tks fd okdbZ vanj ls vPNs gSA

Page 133: cost accounting question icwai inter

Total12

A firm of contractors furnished the following information for the year ended 31.12.000 (Rs. In lakhs)

Page 134: cost accounting question icwai inter

You should compute the contract escalation, because contract account will remain incomplete without this.

Working Notes: 1 Computation of escalation: Rs. In lacs

Page 135: cost accounting question icwai inter

Working Notes: 1 Computation of escalation: Rs. In lacs : For Wages increase

Page 136: cost accounting question icwai inter

3. You should revise your knowledge of Accounting Standards (AS) -- 7, Construction Contracts.

Page 137: cost accounting question icwai inter

The profit transferred is given as Rs. 60,000 and cash certified is 0.75 x work certified. Using these

If some one points out your mistakes, be happy that at least someone is interested in what

Prabhu builders Ltd. commenced on 1st April,2007 on a contract of which the agree price was Rs. 5 lakhs. The

Materials costing Rs. 10,000 proved unsuitable and were sold for Rs. 11,500 and a part of plant was scrapped and sold for Rs. 1,700. Of the contract price, Rs. 240,000 representing 80% of work certified had been received by 31st March, 2008 and on that date the value of plant on the job was Rs. 8,000 and the value of materials was Rs. 3,000.

It was decided to (a) Estimate what further expenditure would be incurred in completing the contract, (b) Compute from the estimate and the expenditure already incurred, the total profit that would be made on the contract and (c) Ascertain the amount of profit to be taken which the value of work certified bears to the contract price. Details of the

Page 138: cost accounting question icwai inter

The notional profit, will be divided in profit transferred to profit and loss a/c and reserve to be taken to next year.

(d) that further Rs. 15,000 would have to be spent on plant and the residual value of the plant on 30th sept. 2008

Prepare contract account for the year ended 31st March, 2008 and show calculations of the sum to be credited to

Material costing Rs. 10,000 was sold for Rs. 11,500. The profit earned is Rs. 1,500. This profit is not related to contract and this entire transaction should be kept out of contract a/c. You can do this in two

1. Remove the cost of material sold from the contract a/c. Material consumed is Rs. 105,000 − Rs. 10,000

2. Credit contract a/c with the sale proceeds of material i.e. Rs. 11,500 and debit it with amount of profit

Page 139: cost accounting question icwai inter

Cash received on account of the contract up to 31st December was Rs. 480,000 being 80% of the work certified.

Page 140: cost accounting question icwai inter

Chapter 5: Process Costing cwa inter cwa final ca inter ca final8 0 12 0

Further data regarding normal waste, costs etc. are given below:Process X Process Y

Cost incurred Materials Rs. 10,000 5,000Labour Rs. 20,000 15,000Overheads Rs. 10,000 8,000

Normal waste as % of input 8% 5%Realisable value of waste per unit in Rs. 5 8Required: (a) Process accounts (b) Normal loss accounts(c) Abnormal loss / gain accounts(d) Selling price per unit of the finished product, if the management wants 25% profit on sales.Solution: Tutorial notes:

Working notes:1.Computation of abnormal loss/gain : (in units) Process X Proces YInput 10,000 9,000Normal loss 800 450normal output 9,200 8,550Actual output 9,000 8,600Abnormal loss/gain 200 loss 50 gain2. Valuation of abnormal loss/gain Process x Rs.Total cost of input 80,000 Remember : Life is a series of changing seasons.less sale value of normal loss 4,000Normal output 9,200Cost per unit of good units 8.26

Sachin and Amitabh CompanyProcess X Account Fig. in Rs.

Particulars Units Rate Amount Particulars Units Rateno. Rs./unit Rs. no. Rs./unit

nqfu;k ,d cgqr cM+s vkbZus dh rjg gSA ;g vkidks vkidk vlyh psgjk fn[kk nsrh gSA vxj vki izseiw.kZ gSa] nksLrkuk gS] ennxkj gSa rks nqfu;k Hkh vkids fy;s iszeiw.kZ] nksLrkuk vkSj ennxkj gks tk;sxhA nqfu;k ogh gS tks vki gSAProblem:1(cwa inter dec.05)

A product passes through two distinct processes X and Y before completion. During a certain period, 10,000 units of crude material were introduced in process X at a cost of Rs. 40,000. After processing in dept. X, 9,000 units of processed material were transferred to process Y for finishing. From process Y finally 8,600 units of the finished product were obtained and transferred to Finished goods store.

1. While preparing process accounts, first of all, accounts for the number of units should be prepared as working notes. This gives abnormal loss/gain arising during the process. All abnormal loss/gain should be valued as good units. The good units should be valued at normal loss figures irrespective of actual number of good units produced.

2.Students should make a particular note of treatment given to abnormal loss / gain. In preparing process accounts the abnormal loss / gain are treated as good units and valued accordingly at par with normal output. While preparing abnormal loss / gain account they are treated as waste and the diffrence between their value as good units and value as waste, is taken to profit and loss account.

Remember : We grow the most by our greatest sufferings.

Page 141: cost accounting question icwai inter

To input units 10,000 4.00 40,000 By pro. Y a/c 9,000 8.26To materials 10,000 By Nor. loss 800 5.00To Labour 20,000 By Abn. loss 200 8.26To Overheads 10,000

80,000 10,000Process Y Account

Particulars Units Rate Amount Particulars Units Rateno. Rs./unit Rs. no. Rs./unit

To input units 9,000 8.26 74,348 By Fin.goods 8,600 11.55from process x By Nor. loss 450 8.00To materials 5,000To Labour 15,000To Overheads 8,000To abnormal gain 50 11.55 578

9,050 102,926 9,050

Calculation of rate per unit of finished goods is as follows:Total cost of input tranferred from process x 74,348Materials (all figures in Rs.) 5,000Labour 15,000Overheads 8,000total cost of input 102,348Less value of normal loss (450 x 8) 3,600Cost of 8550 units of output 98,748Cost per unit 11.55

Normal loss account Particulars units rate amount Particulars units rate

no. Rs./unit Rs. no. Rs./unitTo process x a/c 800 5.00 4,000 By abn. gain 50 8.00To process y a/c 450 8.00 3,600 By Bank 1,400To abnormal a/c 200 5.00 1,000

1,450 8,600 1,450Abnormal loss account

Particulars units rate amount Particulars units rateTo process x a/c 200 8.26 1,652 By nor. loss 200 5.00

By P/L a/c1,652

Abnormal gain accountParticulars units rate amount Particulars units rateTo normal loss a/c 50 8 400 By pro. Y a/c 50 11.55To P/L a/c 178

578Part B: Computation of selling price if profit is 25% on sales Rs./unitCost of finished product 11.55Add: profit at 33.33% of cost (equal to 25% of selling price) 3.85Selling price 15.40

Feel good right now and smile.

vf/kdka'k yksx mrus lq[kh ugha gS ftrus lq[kh oks gks ldrs gSS] nq[k dh ckr ;g gS fd oks vius nq[kksa dk fuekZ.k [kqn dj ysrs gSA

Problem:2(cwa inter dec.04)

Page 142: cost accounting question icwai inter

Particulars Process 1 Process 2 Material consumed Rs. 33,472 27,483Direct labour Rs. 80,000 72,000Overheads Rs. 120,000 108,000Normal waste in process as % of input 3% 1%Sale value of waste Rs. / kg 2.0 3.0Actual output during the month (kg) 93,000 92,200Prepare the three process accounts relating to abnormal loss / gain, if any.

Working notes: Process 1: Process 2 Process 3Input kg. 96,000 93,000 92,200less loss 3% 2,880 1% 930 1% 922norma.l Output kg. 93,120 92,070 91,278Actual output kg. 93,000 92,200 91,500Abnormal loss/gain kg. 120 loss 130 gain 222Calculation of cost per kg of output of processes taking abnormal loss/gain as good units

Process 1: Process 2 Process 3Rs. Rs. Rs.

Input 480,000 706,800 912,780(96,000 x 5) (93,000 x 7.60) (92,200 x 9.90)

Material consumed 33,472 27,483 47,166Direct labour 80,000 72,000 56,000Overheads 120,000 108,000 84,000Total Rs. 713,472 914,283 10,99,946less value of scrap 5,760 2,790 4,610

(2880 x2) (930 x 3) (922 x 5)Net cost of output Rs. 707,712 911,493 10,95,336Normal output in kg kg. 93,120 92,070 91,278Cost per kg. Rs. 7.60 9.90 12.0Transferred to next process 93,000 kg. @ Rs.7.60/kg 92,200 kg @ Rs. 9.90/kg

Dhakadhak and Jhakaas Company Ltd.Process 1 Account

Particulars Qty, Rate Amount Particulars Qty, RateTo Kg Rs./unit Rs. By Kg Rs./unit

Input 96,000 5.00 480,000 93,000 7.60Material consumed 33,472 Normal loss 2,880 2.00Direct labour 80,000 Abnormal loss 120 7.60Overheads 120,000Total 96,000 713,472 96,000

Process 2 AccountParticulars Qty, Rate Amount Particulars Qty, RateTo Kg Rs./unit Rs. By Kg Rs./unit

A company manufactures its sole product by passing the raw material through three distinct processes in its factory. During the month of April 2004, the company purchased 96,000 kg of raw material at Rs. 5 per kg and introduced the same in process 1. Further particulars of manufacture for the month are given below.

Solution: (Please see tutorial notes of previous question)

Process 2 transfer

Page 143: cost accounting question icwai inter

Input (from process 1) 93,000 7.60 706,800 92,200 9.90Material consumed 27,483 Normal loss 930 3.00Direct labour 72,000 (1% of 93,000)Overheads 108,000Abnormal gain 130 9.90 1,287Total 93,130 915,570 13

Process 3 AccountParticulars Qty, Rate Amount Particulars Qty, RateTo Kg Rs. Rs. By Kg Rs.

Input (from process 2)92,200 9.90 912,780 91,500 12

Material consumed 47,166 Normal lossDirect labour 56,000 (1% of 92,200) 922 5Overheads 84,000Abnormal gain 222 12 2,664Total 92,422 11,02,610 92,422

Abnormal loss accountTo process 1 a/c 120 kg 912 By Cash @ Rs. 2/kg

pkykdh er djks] uqdlku mBkvksxsA By Profit and loss account 912

Abnormal gain accountQty. Rate Amount Qty. Rate

To process 2 a/c 130 kg Rs. 3 390 By proc. 2 a/c 130 kg Rs. 9.9

To process 3 a/c 222 kg Rs. 5 1,110 By proc. 3 a/c 222 kg Rs. 12.0

To profit and loss account 2,4513,951

(i) Opening stock of work in progress 500 units MaterialLabourOverhead

(ii) Cost incurred during April 2003: 14,000 units MaterialLabourOverhead

(iii) Process loss: Normal loss : 10% Value of scrapped units : Rs. 10 per unit.Actual loss during April 2003 : 1,500 unitsDegree of completion : Materials 100% Labour and overhead : 60%(iv) Closing work in progress : 1,000 unitsDegree of completion : Materials 100% Labour and overhead : 70%

Process 3 transfer

Finished goods

Problem:3(cwa inter june 03)

From the following information relating to Process I of a factory for the month of April 2003, Prepare the statement of equivalent production, statement of cost, statement of evaluation and process account, using the average cost method.

Page 144: cost accounting question icwai inter

(v) Processed units transferred to Process II: 12,000 units.

Solution: Tutorial notes: vHkh fdlh ds ckjs esa vPNk lksfp;sA

The solution goes as follows:(i) Statement of equivalent production : All figures in units

Input Particulars Output Materials % Labour %500 Opening wip ( not considered in average cost method)

14,000

Transferred to next pr 12,000 12,000 100% 12,000 100%Normal loss 1,450Abnormal loss 50 50 100% 30 60%Closing WIP 1,000 1,000 100% 700 70%

14,500 14,500 13,050 12,730This equivalent production will be used in determining the elementwise cost of completed units :(ii) Statement of cost : all figures in Rs. Particulars Materials Labour Overhead Opening WIP 27,000 8,000 12,500Cost incurred during period 574,750 119,300 178,450Total cost 601,750 127,300 190,950Less value of normal scrap ( 1,450 x 10) 14,500Net cost 587,250 127,300 190,950Equi. production in units (see above statement) 13,050 12,730 12,730Cost per unit in Rs. 45 10 15This cost per unit elementwise will be used in determining the cost of production and abnormal loss/gain(iii) Statement of Evaluation :Particulars Materials Labour Overhead Cost per unit in Rs. 45 10 15Output transferred to next process in units 12,000 12,000 12,000Output transferred to next process in Rs. 540,000 120,000 180,000

1. This question is tricky and important both. Statement of equivalent units calls for the determination of cost of material, labour and overhead in good units. The normal loss and not the actual loss should be taken to calculate the number of goods units coming out of a particular process.

2. When average cost method of inventory valuation is used in process accounting, the completion of opening work-in-progress is not converted into equivalent units. Worded differently, no distinction is made between units which are partially completed in previous period and the units which are introduced and completed during the current period.

3. Following two points should be kept in mind while following the average cost method inventory costing, (i) Units completed will not be divided in two categories i.e. opening inventory and units introduced and completed during the period as done in fifo method and (ii) Cost of opening work-in-process is added elementwise with cost incurred during the period.

4. In average cost method of inventory valuation the cost of opening inventory is added elementwise to cost incurred during the process and the agreegate of these two is divided by sum of (i) completed units during the period and (ii) equivalent completed units of closing inventory.

5. In short, according to this method opening inventory of work-in-progress and its costs are merged with the production and cost of the current period, respectively. An average cost per unit is determined by dividing the total cost by the total equivalent units, to ascertain the value of the units completed in process.

Introduced during the month

Page 145: cost accounting question icwai inter

Abnormal loss ( 50 units) 50 30 30Cost of abnormal loss ( 50 units) 2,250 300 450Closing WIP in units 1,000 700 700Cost of closing WIP 45,000 7,000 10,500Total 587,250 127,300 190,950

(iv) Process I AccountParticulars Units Amount Rs. unitsTo opening wip 500 47,500 By process II transferred 12,000To Materials 14,000 574,750 By normal loss 1,450To labour 119,300 By abnormal loss 50To overhead 178,450 By Closing wip 1,000

14,500 920,000 14,500Note: there is no balancing figure in the above accounts. The accuracy is thus verified.

Process % of normal waste related to input Realisable value in Rs./unitI 5 0.70II 7 0.80III 10 1.00

The details of cost data and output for a month are as follows:Processes Fig. in Rs.

I II IIIMaterial consumed 120,000 40,000 40,000Direct labour cost 80,000 60,000 60,000Production expenses 40,000 40,000 28,000Output units 38,000 34,600 32,000

Prepare process account for the month.Solution: Statement of abnormal gain/loss during the processes ( in units )Process Input waste % waste Output Actual output Ab.gain

I 40,000 5 2,000 38,000 38,000 0II 38,000 7 2,660 35,340 34,600III 34,600 10 3,460 31,140 32,000 860

Geoge Bush Manufacturing company Ltd.Process I Account

particulars Units Rate Amount particulars Units rate Rs.nos. Rs./unit Rs. nos. Rs./unit

To introduced units 40,000 320,000 By process II 38,000 14.70Material consumed 120,000 transferDirect labour cost 80,000 By normal loss 2,000 0.70Production expenses 40,000

560,000Rate of output to process II is calculated as follows:(560,000 – 1,400)/ 38,000 = Rs. 14.70 per unit.

Process II Account

Problem:4(cwa inter dec.03)

Production in a manufacturing company passes through three distinct processes I, II and III. The output of each process is transferred to finished goods stock. The normal wastage in each process and the realisable value of the same are given below:

Process I was fed with 40,000 units of input costing Rs. 320,000. There were no opening and closing work-in-progress

Page 146: cost accounting question icwai inter

Particulars units Rate Amount Particulars units RateTo Process I transfer 38,000 14.70 558,600 By process III 34,600 19.71Material consumed 40,000 transferredDirect labour cost 60,000 By normal loss 2,660 0.80Production expenses 40,000 By abn. loss 740 19.71

698,600Rate of output to process III is calculated as follows:(6,98,600 – 2660 x 0.80) / ( 38,000 – 2,660) = Rs. 19.71

Process III AccountParticulars units Rate Amount Particulars units Rate

nos. Rs./unit Rs. nos. Rs./unitTo process II transfer 34,600 19.71 681,888 By finished 32,000 25.90Material consumed 40,000 GoodsDirect labour cost 60,000 By normal loss 3,460 1.00Production expenses 28,000To abnormal gain 860 25.90 22,274

35,460 832,162 35,460

Rate of finished goods is calculated as follows:(809,888 – 3,460 x 1) / ( 34,600 – 3,460) = Rs. 25.90 / unit.

Particulars Process A Process B Process CUnits of input issued 9,000Cost per unit Rs. 150Sundry material Rs. 23,500 25,000 15,000Direct labour Rs. 80,000 207,200 26,110Direct expenses Rs. 2,250 7,200 8,100Selling price/ unit Rs. 200 280 600The actual outputs obtained vis-à-vis normal process losses from the three processes were:

Output units Loss %Process A 8,400 5

B 5,700 10C 3,660 3

Solution:

Problem:5(cwa inter june.02)

Product X in a manufacturing unit passes through thre processes–A, B and C. The expenses incurred in the three processed during the year 2001 were as under:

During the year, three-fourth of the output of process A and two-third of the out of process B were tranferred to the next process and the balances were sold outside.The entire amount of the process C however, was sold outside. The losses of the three processes were sold at Rs. 5 per unit for process A. Rs. 10 per unit for process B and Rs. 15 per unit for process C.

Prepare the three processes accounts and a statement of income considering a total selling and distribution expenses of Rs.. 45,000 which is not allocated to the processes.

Page 147: cost accounting question icwai inter

Statement of abnormal gain/loss during the processes ( in units )Process Input waste % waste Output Actual output Ab.gainA 9,000 5 450 8,550 8,400B 6300* 10 630 5,670 5,700 30C 3,800** 3 114 3,686 3,660*( 3/4 of A = 0.75 x 8,400)**( 2/3 of B = 2/3 x 5,700)

Process A AccountParticulars Units Rate Rs. Amount Particulars Units rate Rs.

nos. Rs./unit Rs. nos. Rs./unitTo introduced units 9,000 150 13,50,000 By process B 6,300 170Sundry material 23,500 By nor. loss 450 5Direct labour 80,000 By Abn. loss 150 170Direct expenses 2,250 By Bank 2,100 170

9,000 14,55,750 9,000Cost per unit of normal output is worked out as under:Cost per unit = (13,50,000 + 23,500 + 80,000 –2,250)/ ( 9,000 – 450) = Rs.170 per unit.

Process B AccountParticulars Units Rate Rs. Amount Particulars Units rate Rs.

nos. Rs./unit Rs. nos. Rs./unitTo Process A transfer 6,300 170 10,71,000 By process C 3,800 230Sundry material 25,000 By nor. loss 630 10Direct labour 207,200 By Bank 1,900 230Direct expenses 7,200Abnormal gain 30 230 6,900

6,330 13,17,300 470Cost per unit of normal output is worked out as under:Cost per unit = (10,71,000 + 25,000 +207,200 + 7,200 –6,300)/(6,300 – 630) = Rs. 230/unit

Process C AccountParticulars Units Amount Particulars Units rate Rs.

nos. Rs. nos. Rs./unitTo process B transfer 3,800 874,000 By finished 3660 250Sundry material 15,000 GoodsDirect labour 26,110 By nor. loss 114 15Direct expenses 8,100 By abn. loss 26 250

3,800 923,210 3,800Cost per unit of normal output is worked out as under:Cost per unit = (874,000 + 15,000 + 26,110 + 8,100 – 1,710) / (3,800 – 114) = Rs. 250/unit

Income Statement

Tutorial Notes: The speciality of this question is that some of the output of a process is transferred to the next process and some output is also sold in the market. The selling price of the output is also given e.g. for process A it is Rs. 200/unit, for process B it is Rs. 280/unit and for process C it is Rs. 600/unit. Student should particularly note that in process account the actual proceeds of unit sold is not relevant. All units are to be valued as good units of output with loss taken as normal loss. The diffrence should be taken to P and L account. Therefore in preparing process account of process A, although the 1/3 rd of 8,400 units i.e. 2,100 units are sold in the market at Rs. 200/unit fetching Rs. 420,000 as revenue, the process account will not be credited with this amount. It will be credited with the rate of output as obtained in the process account.

Page 148: cost accounting question icwai inter

Cost of sales Rs. By salesProcess A : 2100 units @ Rs. 170 357,000 Process A : 2100 units @ Rs. 200Process B :1900 units @ Rs. 230 437,000 Process B : 1900 units @ Rs. 280Process C:3660 units @ Rs. 250 915,000 Process C:3660 units @ Rs. 600Abnormal loss By abnormal gainProcess A : 150 units @ Rs. (170 – 5) 24,750 Process B : 30 units @ Rs. (230 – 10)Process C:26 units @ Rs. (250 – 15) 6,110Selling and distribution overhead 45,000Net profit 13,69,740

31,54,600

Data relating to work done in Process 'A' of a company during the month of April 2000 is given below:Opening work-in-progress 1000 units Rs.

Materials 40,000Labour 7,500

Overheads 22,500Materials introduced in process 'A' ( 19,000 units) 740,000Labour 179,500Overhead 538,500Units scrapped 1,500 Degree of completion material 100%, Labour and overhead 80%Closing work-in-progress 1000 units.Degree of completion Materials 100% Labour and overheads 80%Units finished and transferred to Process B 17,500 units.Normal loss 5% of total input including opening work-in-progress

Scrapped units fetch Rs. 20 per piece.Required : (a) Statement of equivalent production

(b) Statement of cost (c) Statement of distribution of cost(d) Process 'A' account and other accounts.

Solution:Tutorial notes:

(i) Statement of equivalent production :Input Particulars Output Materials % Labour %

1,000 Opening wip ( not considered in average cost method)19,000 Units introduced

17,500 17,500 100 17,500 100

Problem:6(cwa inter dec..00)

1. First of all let us see whether there is any abnormal loss or gain. Normal loss is given as 5% of input including work-in-progress.Normal loss is 5% of ( 1000 + 19,000) = 1000 units. The output should be { 19,000 ( not 20,000) – 1000} = 18,000 units but the output is given as 17,500 units which shows that there is an abnormal loss of 500 units.

2. The degree of completion of work-in-progress has not been given. This implies that average cost method should be followed in determining the equivalent production. Thus opening work-in-progress will be ignored while computing the equivalent production.

3. Note that the degree of completion of scrapped units has also been given. You should not be confused to apply this degree to normal loss units. Normal loss units should not be considered at all in computing the equivalent production. The degree of completion of scrapped units is relevant only for abnormal loss units.

Transferred to next process

Page 149: cost accounting question icwai inter

Normal loss 1,000 – – – –Abnormal loss 500 500 100 400 80Closing WIP 1,000 1,000 1,000 800 80

20,000 20,000 19,000 18,700(ii) Statement of cost :Particulars Material Labour Overhead

Rs. Rs. Rs.Opening WIP 40,000 7,500 22,500Cost incurred during period 740,000 179,500 538,500Total cost 780,000 187,000 561,000Less value of normal scrap ( Rs. 20 x 1000) 20,000Net cost 760,000 187,000 561,000Equivalent production in units (see above statement) 19,000 18,700 18,700Cost per unit in Rs. 40 10 30(iii) Statement of distribution of cost:Particulars Materials Labour Overhead Output transferred to next process in units 17,500 17,500 17,500Cost per unit in Rs. 40 10 30

Rs. Rs. Rs.Output transferred to next process in Rs. 700,000 175,000 525,000Abnormal loss ( 500 units) 500 400 400Cost of abnormal loss ( 500 units) 20,000 4,000 12,000Closing WIP in units 1,000 800 800Cost of closing WIP 40,000 8,000 24,000Total 760,000 187,000 561,000

(iv) Process I AccountParticulars Units Amount Particulars UnitsTo opening wip 1000 70,000 By process II transf. 17,500To Materials 19,000 740,000 By normal loss 1,000To labour 179,500 By abnormal loss 500To overhead 538,500 By Closing wip 1,000

20,000 15,28,000 20,000Normal Loss Account

Particulars Units Amount Particulars UnitsTo Process A a/c 1000 20,000 By cost ledger control a/c 1,000

Abnormal loss AccountTo Process A a/c 500 36,000 By cost ledger control a/c 500

By Costing p/l account 36,000

SBL Ltd. Furnishes you the following information relating to process-Business for the month of April 2006

(i) Opening work-in-progress : Nil (ii) Units introduced – 10,000 units @ Rs. 5 per unit(iii) Expenses related to process B : (a) Processing material Rs. 24,600, (b) Direct labour Rs. 10,400 (c) Overhead Rs. 5,000(iv) 8,000 units of finished output were transferred to the next process during the month.(v) Normal loss in the process : 10% of input (vi) Closing work-in-progress 800 units,

degree of completion Material 100% Labour and overhead 50%

Problem:7(cwa inter june..06)

Page 150: cost accounting question icwai inter

(vii) Degree of completion of abnormal loss : Material 100%, Labour and overhead 80%(viii) Scrap realisation Normal loss @ Rs. 2/unit Abnormal loss : @ Rs. 4/unitYou are required to prepare: (1) Statement of equivalent production

(2) Statement of cost of each element;(3) Statement of Evaluation ;(4) Process – B a/c(5) Abnormal loss a/c

Solution :This can be solved on the lines of previous question. The salient figures are shown

Material Labour Overhead Finished goods Element wise net cost Rs. 72,600 10,400 5,000Equivalent production Rs. 9,000 8,560 8,560Cost per unit Rs. 8.07 1.21 0.58

Statement of evaluation : Abnormal loss Closing unitsUnits cpu Rs. Cost Rs. units cpu Rs.

Material 200 8.07 1,614 800 8.07Labour 160 1.21 194 400 1.21Overhead 160 0.58 93 400 0.58

1,900

In April,03 units of production and direct material cost incurred are as follows:Particulars Units produced Extrusion Form TrimMaterial costs : Rs. Rs. Rs.Standard 10,500 126,000 42,000 0Deluxe 5,250 63,000 21,000 15,750Executive 3,500 42,000 14,000 10,500

19,250 231,000 77,000 26,250The total conversion costs for the month of April 2003Total conversion costs in Rs. 606,375 297,000 155,250Required:1. For each product produced by RST. Ltd. during April 03, determine the unit cost and total cost.

Solution : vkt gh ladYi ysa fd vki ,d cgqr [kq'k balku cusxsaA

Problem 8 (ca pe-ii may 03)

RST.Ltd manufactures plastic moulded chairs. Three models of moulded chairs, all variation of the same design are standard Deluxe and Executive. The company uses an operating costing system.

RST. Ltd has extrusion, form, trim and finish operation. During the extrusion operation, the plastic sheets are produced. During the forming operation, the plastic aremoulded into chair seats and the legs are added. The standard model is sold after this operation. During the trim operation, the arms are added to the Deluxe and Executive models, and the chair edges are smoothed.

Only the Executive model enters the finish operation, in which padding is added. All the units produced receive the same steps within each operation.

2.Now consider the following information for May. All unit costs in May are identical to the April unit costs calculated as above in (1). At the end of May, 1,500 units of the Deluxe model remain in work-in-progress.These units are 100% complete as to materials and 65% complete in the trim operation. Determine the cost of the Deluxe model wip inventory at the end of May.

First of all let us calculate the unit cost of all the four operations. The extrusion and form operations are conducted in all the three models. The trim operation is conducted on deluxe and executive models and the finish operation is conducted only on executive models.

Page 151: cost accounting question icwai inter

Statement showing material cost and conversion cost per unit of the operation.

Particulars Extrusion Form Trim FinishEquivalent units produced 19,250 19,250 8,750 3,500Material cost in Rs. 231,000 77,000 26,250 21,000Material cost per unit in Rs. 12 4 3 6Conversion cost of operation in Rs. 606,375 297,000 155,250 94,500Conver. cost of operation/unit in Rs. 31.50 15.43 17.74 27.00Statement showing unit cost and total cost model wise:Models Standard Deluxe Executive

Rs. Rs. Rs.Extrusion material 12.0 12.0 12.0Form material 4.0 4.0 4.0Trim material 0.0 3.0 3.0Finish material 0.0 0.0 6.0Extrusion conversion 31.5 31.5 31.5Form conversion 15.4 15.4 15.4Trim converstion 0.0 17.7 17.7Finish conversion 0.0 0.0 27.0Total cost in Rs. / unit 62.9 83.6 116.6Units produced 10,500 5,250 3,500Total cost in Rs. 660,450 438,900 408,100Statement showing cost of 1500 units of work-in-progress of deluxe model Note: Deluxe model does not undergo finish operation. The work-in-progress is 100% complete in respect of material and 65% complete in trim.

unit cost equivalent units Total cost Rs.Extrusion material 12.0 1,500 18,000Form material 4.0 1,500 6,000Trim material 3.0 1,500 4,500Extrusion conversion 31.5 1,500 47,250Form conversion 15.4 1,500 23,100Trim converstion (65%) 17.7 975 17,258Total cost of 1500 units of work-in-progress of deluxe model: 116,108

From the following information for the month of October 2003, prepare process III cost a/cOpening work-in-progress in process III 1,800 units at Rs. 27,000Transfer from process II 47,700 units at Rs. 536,625Transfer to ware house 43,200 unitsClosing work-in-progress of process III 4,500 unitsUnits scrapped 1,800 unitsDirect materials added in process III Rs. 177,840Direct wages Rs. 87,840Production overheads Rs. 43,920Degree of completion : Opening stock Closing stock ScrapMaterial 80% 70% 100%Labour 60% 50% 70%Overheads 60% 50% 70%The normal loss in the process was 5% of the production and the scrap was sold @ Rs.6.75/unit.Solution: Tutorial notes:

?kj esa iM+h csdkj oLrqvksa dks ;k rks O;ofLFkr :i ls tek nsuk pkfg;s ;k fQj fdlh dks ns nsuk pkfg;SA csrjrhc dckM+ udkjkRed mtkZ iSnk djrk gS] dyg djokrk gS rFkk vkus okys /ku dks jksdrk gSA phuh

Problem 9 (CA-PE-II Nov. 03)

Page 152: cost accounting question icwai inter

The solution goes as follows:No. of unit produced = Unit introduced + Opening wip – Closing wip.No. of unit produced = 47,700 + 1,800 – 4,500 = 45,000 units.Normal loss @ 5% of production = 5% of 45,000 units = 2,250 unitsActual loss is given as 1,800 units. Thus there is abnormal gain of 450 units ( 2,250 – 1,800)(i) Statement of equivalent production :Input Particulars Output Material I Material II Labour

1,800 Opening work-in-progress 1,800 0 360 72047,700 Units introduced

Completed (43200 - 1800) 41,400 41,400 41,400 41,400Normal loss 2,250Abnormal gain (450) (450) 450 450Closing WIP 4,500 4,500 3,150 2,250

49,500 49,500 45,450 44,460 43,920Tutorial notes on above statement:

(ii) Statement of cost :Particulars Material I Material II Labour Overhead

Rs. Rs. Rs. Rs.

·  Small problems small achievements. Big problems big achievements. What do you select ? (1) The normal loss has been given as 5% of production,but production is not known. It is hidden somewhere. Think where it is hidden. Only after knowing production, normal loss can be ascertained and then compared with the actual loss of 1800 units to determine the abnormal loss/gain.

(2) Whenever the degree of completion of cost elements is given in opening work-in-progress, the average cost method should not be used because it ignores opening work-in-progress in the evaluation of equivalent production. In such cases either fifo or lifo method should be used. In this solution fifo method is applied.

(3) According to FIFO method, the units first entering the process are completed first. Thus the units completed during the period would consist partly of the units which were incomplete at the beginning of the period and partly of the units introduced during the period.

The cost of the completed units is affected by the value of the opening inventory, which is based on the cost of period. The closing work-in-progress is valued at current cost.

(4) In the computation of equivalent production, special care should be taken in respect of opening work-in-progress. If degree of completion of any element is given, the balance should be taken for computation of equivalent units in fifo method. For illustration, let us say degree of completion of 400 unit of opening wip is given as 75%, then in fifo method the equivalent production in the process would be 25% of 400 units i.e 100 units.(5) Degree of completion of abnormal gain/loss, if any, has not been mentioned, these units can be assumed as good units as 100% complete.

(6) Degree of completion of normal loss or scrap should not be considered in computation of equivalent production.

Opening wip of 1800 units is 20% completed in material and 40% completed in labour and overhead in the process.Closing work-in-progress of 4,500 units is 70% complete in material, and 50% complete in labour and overhead.Material I units will bear the cost transferred to the process. Thus units representing opening stock and partly finished will not appear in this column since they are complete so far as the cost transferred to the process is concerned.

Material II units bear both the costs viz. cost transferred to the process and the cost relating to the process. These units will appear in both the columns viz. Material I and Material II.

Thus column Material II in statement of equivalent production will include: (i) units equal to work done on opening stock (ii) units introduced and completed and (iii) units equal to work done on closing stock.

Page 153: cost accounting question icwai inter

Cost transferred 536,625Cost incurred during period 177,840 87,840 43,920Less value of normal scrap (6.75 x 2,250) 15,188Net cost 521,438Equiv. production units (see above stment) 45,450 44,460 43,920 43,920Cost per unit in Rs. 11.47 4.00 2.00 1.00

Material I Mate. II Labour Overhead Opening stock of wip Rs. 27,000Completed work-in-progress units 360 720 720Cost / unit in Rs. 4.0 2.0 1.0Cost of completion of opening wip 1,440 1,440 720Units introduced and completed (Rs. 18.47 x 41,400)Total cost of finished goods units ( 1,800 + 41,400) i.e 43,200 unitsCost of closing stock of wip :units 4,500 3,150 2,250 2,250

Rate 11.47 4.0 2.0 1.0Cost 51,615 12,600 4,500 2,250

Abnormal gain of 450 units @ Rs. 18.47The reader should himself prepare the process III a/c and then compare it with the one given below:

Process III A/cTo Opening work-in-progress 1,800 27,000 By normal loss 2,250To Process II A/c 47,700 536,625 By Finished goods 43,200To Direct materials 177,840 By Closing wip 4,500To Direct labour 87,840To Production overhead 43,920To Abnormal gain 450 8,313

49,950 881,538 49,950

Operation Input Rejection Output Labour and units units units Overhead Rs.

1 21,600 5,400 16,2002 20,250 1,350 18,9003 18,900 1,350 17,5504 23,400 1,800 21,6005 17,280 2,880 14,400

You are required to:(i) Determine the input required in each operation for one unit of final output.

Solution: Tutorial notes:

(iii) Statement of apportionment of cost:

The difference in the two sides is due to approximation of decimal figures. Tne closing stock, abnormal gain and finished goods have been evaluated @ Rs. 18.47 instead of Rs. 18.4728, hence the difference.

Problem 10 (ca inter-II nov.96)

In a manufacturing company, a product passes through 5 operations. The output of 5th operation becomes the finished product. The input rejection, output and labour and overheads of each operation for a period are as under:

(ii) Calculate the labour and overhead cost at each operation for one unit of final product and the total labour and overhead cost of all operations for one unit of final product.

Page 154: cost accounting question icwai inter

Operation Input Rejection Output Rejection as %units units units

1 21,600 5,400 16,2002 20,250 1,350 18,9003 18,900 1,350 17,5504 23,400 1,800 21,6005 17,280 2,880 14,400

(i) Determination of the input required in each operation for one unit of final output.Operation Output Rejection as % Input

units of output units5 1.0 20.00% 1.24 1.2 8.33% 1.33 1.3 7.70% 1.42 1.4 7.10% 1.51 1.5 33.30% 2.0

(ii) Labour and overhead cost per unit of operation and final productOperation Input Lab.& overhead

units Total Per unit1 21,600 194,400 9 2.0 18.02 20,250 141,750 7 1.5 10.53 18,900 245,700 13 1.4 18.24 23,400 140,400 6 1.3 7.85 17,280 86,400 5 1.2 6.0

60.5Total labour and overhead cost of all operations for one unit of output is Rs. 60.50.

The following information is given in respect of Process no. 3 for the month of January 2001:Opening stock of 2000 units made up of (fig. in Rs.) Direct materials Direct materials II Direct labour Overheads

12,350 13,200 17,500 11,000Transferred from Process 2: 20,000 units @ Rs. 6.00 per unitTransferred to Process 4: 17,000 unitsExpenditure incurred in Process no. 3 ( in Rs. ) Direct materials

Direct labourOverheads

Scrap 1,000 units Material 100% Labour 60% Overhead 40%Normal loss : 10% of production. Scrap units realised @ Rs. 4 per unit.Closing stock 4,000 units : Degree of completionDirect materials 80% Direct labour 60% Overhead 40%

Prepare process 3 account using average price method, alongwith necessary supporting statements.

If you see the input units of each operation, you will find that the inputs of different operations are taken from different processes and not from one process. The results are independent of one another. The above table is useful to determine the % of rejection of each operation as shown below:

The question is to determine the number of units to be introduced in operation 1, to receive one unit of final product at the end of process 5. Reverse calculation as shown below will help:

Input units required /unit of final output

Labour and overhead cost per unit of final output

Problem 11 (ca inter-II may 01)

Solution : Tutorial notes of problem 3, solved previously should be properly understood.

Page 155: cost accounting question icwai inter

Working notes

1. Calculation of normal loss:

Production = Opening stock + introduced – Closing stock = 2000 + 20,000 – 4000 = 18,000 units

Normal loss @ 10% of production = 10% of 18,000 units = 1,800 units

Actual loss is given as 1,000 units. Thus there is abnormal gain of 800 units (1,800 – 1,000)

Statement of Cost of components:Direct materials Direct materials II Direct labour

Opening stock 12,350 13,200 17,500

Introduced 120,000 – –

Less scrap@ Rs. 4 7,200

In process II 30,000 60,000

125,150 43,200 77,500

Note : Actual scrap is 1,000 units but here normal scrap of 1,800 units is to be considered.

Statement of equivalent production

Normal loss Total units

Material I% 100% 100% 100%Units 17,000 (800) 4,000 20,200 125,150Material II% 100% 100% 80%Units 17,000 (800) 3,200 19,400 43,200Labour% 100% 100% 60%Units 17,000 (800) 2,400 18,600 77,500Overhead % 100% 100% 40%Units 17,000 (800) 1,600 17,800 71,000

Total units 17,000 800 4,000 1,800 22,000Statement of evaluation

Cost of 17,000 units of Finished goods 17,000 x Rs. 16.58Cost of 800 units of abnormal gain 800 x Rs. 16.58Cost of Work-in-progress (Closing stock) of 4,000 units :

Material I Material II Labour Overheadunits nos. 4,000 3,200 2,400 1,600Rate Rs./unit 6.20 2.23 4.17 3.99Total Rs. 24,782 7,126 10,000 6,382

Process III A/cunits Amount units Rate Rs.

To opening stock 2,000 54,050 By normal loss 1,800 4.00To Process II 20,000 120,000 By finished goods 17,000 16.58To Direct materials 30,000 By Closing stock 4,000

(1) The normal loss has been given as 10% of production,but production is not known. It is hidden somewhere. Think where it is hidden. Only after knowing production, normal loss can be ascertained and then compared with the actual loss of 1000 units to determine the abnormal loss/gain.

Introduced and completed

Abnormal Gain

Closing Stock

Total cost Rs.

Page 156: cost accounting question icwai inter

To Direct labour 60,000To overheads 60,000To Abnormal gain 800 13,262

22,800 337,312 22,800

Product J : 500 kg. Product K : 350 kg. Product L : 100 kg. Toxic waste : 50 kg.The toxic waste is disposed at a cost of Rs. 16.50 per kg and arises at the end of processing.

Process II which is used for further processing of product K into product K2, has the following cost structure:

Fixed cost : Rs. 2,64,000 per week. Variable cost : Rs. 16.50 per kg processed.The following actual data relate to the first week of the month:Process I OutputOpening work-in-progress Nil Product JMaterial input 40,000 kg costing Rs. 660,000 Product KLabour input Rs. 440,000 Product L Variable overheads Rs. 176,000 Toxic wasteFixed overheads Rs. 264,000 Closing wip Process IIOpening work-in-progress NILInput of K 14,400 kg.Output of K2 13,200 kg.

Required:

(ii) Prepare the taxic waste account and Process II account for the first week of the month.

(iii) Comment on the method used by the JKL ltd. to attribute the pre-separation costs to joint products.

(a) If product K could be sold at the point of separation at Rs. 47.30 per kg;(b) If the 60% of weekly fixed cost of process II were avoided by not processing product K further.Solution : Tutorial Notes:

Problem 12 (ca inter-II may 04)

JKL limited produces two products– J and K together with a by-product L from a single main process (process I). Product J is sold at the point of separation for Rs. 55 per kg, whereas product K is sold for Rs. 77 per kg after further processing into product K2. By-product L is sold without further processing for Rs. 19.25 per kg.

Process I is closely monitored by a team of chemists, who planned the output per 1,000 kg of input materials to be as follows:

Closing work-in-progress (50% converted and conversion costs were incurred in accordance with the planned cost structure.)

(i) Prepare Process I account for the first week of the month using the final sales value method of attribute the pre-separation costs to joint products.

(iv) Advise the management of JKL ltd. whether or not, on purly financial grounds it should continue to process K into K2.

1. This question is tricky and hard. You must think a lot before attempting to solve. You must try to solve this before seeing the solution. You can commence your line of thinking on the following points.

2. In all questions previously solved, the normal loss is in the form of scraped units which brings in some revenue or nil revenue and as such the input cost is either reduced to the extent of the realisation or remains the same. In this particular question, the waste is disposed off @ Rs. 16.50 per kg. Thus while calculating the input cost, the cost of disposal of waste is to be added.

Page 157: cost accounting question icwai inter

8. There is no abnormal gain/loss in Process II.

Working Notes: 1.Computation of normal output with actual input of 40,000 kg.

Particulars Output / 1000 kg.Abnormal gain/ loss

Product J 500 20,000 19,200 800Product K 350 14,000 14,400 400Product L 100 4,000 4,000Toxic waste 50 2,000 2,400There is an overall abnormal loss of 400 kgs.2. Evaluation of abnormal gain/loss

Opening work-in-progress (fig. in Rs.) NilMaterial input 40,000 kg costing 660,000Labour input 440,000Variable overheads 176,000Fixed overheads 264,000Total Cost 15,40,000

Add: Cost of disposal of normal waste of 2000 kg @ Rs. 16.50 33,000Less: Sale of by-product L, 4.000 kg @ Rs. 19.25 77,000

Net cost of input 14,96,000No. of good units or output 40,000 x 0.85 34,000 kg.Valuation of abnormal loss (14,30,000 / 34,000) 44.00 per kg.

3. Evaluation of pre-separation cost:Net cost of input as determined above 14,96,000Less: Evaluation of abnormal loss @ Rs. 44 for 400 kg. 17,600

Pre-separation cost to be apportioned 14,78,400

3. There are two joint products coming out of the process. One is a case of abnormal gain while the other is of abnormal loss. The evaluation of abnormal gain/loss should not be done outputwise but should be done on the total number of units. A view of the question reveals that there is overall abnormal loss of 400 units, thus evaluation of this abnormal loss should be carried out in normal way as a ratio of net cost of input / net normal out put.

4. Net cost of input will be [ Total cost of input – realisation from by-product plus cost of disposal of normal waste of 2,000 kg @ Rs. 16.50 ]. You must also note that this cost is not actual cost of input but notional cost of input for evaluation of abnormal loss. The actual cost of input can be determined only after the evaluation of abnormal loss, as usual.

5. Pre-separation cost is to be attributed to the joint products in the ratio of their sales values. While sales value of J and K can be determined, the pre-separation cost is to be computed. You must think how the pre-separation cost can be determined.

6. (a) and (b) of part (iv) of the question are easy. You should be able to solve both the parts without solving the main question.

7. From the question it is evident that the normal loss consisting of by-product and waste constitutes 15% of the input. Thus normal output as 85% of input should be taken while evaluating the abnormal loss.

9. The closing work-in-progress is given as 1,200 kg. Its evaluation needs some calculations to be done. It is 50% converted. The conversion cost is Rs. 16.50 /kg as variable cost and Rs. 264,000 as fixed cost. Please try to evaluate the fixed cost component of closing wip of process II right now.

Output for 40,000 kg of input

Actual output

It is evident that normal output is 85% of input because 1000 kg of input yields 100 kg of by-product and 50 kg of toxic waste, aggregating 15% of normal loss.

Page 158: cost accounting question icwai inter

4. Allocation of joint costs to Products J and K in the ratio of their final sales value:

Allocation of Joint Cost of Rs. 14,78,400Products Output Rate Rs. Sales value in Rs. Allocation of joint cost

J 19,200 55 1056000 Rs. 721,171K 14,400 77 1108800 Rs. 757,229

The solution goes as follows:Answer (i) Process I A/c

Units rate amount Units rateTo Opening wip Nil By product L sales 4,000 19.25To Material 40,000 660,000 By normal loss 2,000To Labour input 440,000 By abnormal loss 400 44.00To Variable overheads 176,000 By joint cost of J 19,200 37.56To Fixed overheads 264,000 By joint cost of K 14,400 52.59To Normal loss 16.5 33,000 By Closing wip 0

40,000 15,73,000 40,000Note: There is no balacing figure in the above account.Answer (ii) Toxic Waste A/c Units rate amount

Rs. kg. Rs./unit Rs.

By balance 33,000 By process I A/c 2000 kg 16.5 33,000

Answer (iii) Process II A/c

Before process a/c can be drawn, the evaluation of closing work-in-progress needs to be done.Evaluation of Closing work-in-progress

Fixed cost per unit is Rs. 264,000/13,800 = Rs. 19.13/kg.Evaluation of Closing work-in-progress : Please find out what is wrong in the following tableComponent Quantity Rate % completion Equivalent units

Units Rs./unit unitsMaterial 1,200 52.59 100% 1,200Variable cost 1,200 16.50 50% 600Fixed cost 1200 19.13 50% 600Total cost of work-in-progress

Total variable cost 16.50 x 14,400 Rs. 237,600Total fixed cost Rs. 264,000Total cost incurred Rs. 501,600Total no. of goods units (14,400 – 600) nos. 13,800Cost per unit of work-in-progress (501,600/13,800) Rs. 36.35Cost of 600 equivalent units of work-in-progress (600 x 36.35) Rs. 21,809Cost of material of 1,200 kg @ Rs. 52.59 Rs. 63,108

Total cost of work-in-progress for process II a/c Rs. 84,917Process II A/c can now be drawn:

Process II A/c

Fixed cost incurred is Rs. 264,000. The equivalent units completed in respect of fixed cost,is indirectly calculated as input minus incomplete units. The input is 14,400 kg and the incompleted quantity is 50% of 1,200 kg. Thus equivalent quantity for computation of fixed cost per unit for evaluation of wip is 14,400 – 600 i.e 13,800 kg.

This is where the students generally commit mistakes.The above table is incorrect because the variable cost per unit is taken as per input units whereas it should be taken as per output unit. The correct evaluation is as follows:

Page 159: cost accounting question icwai inter

Units rate amountnos Rs./unit Rs. Units amount

To process I A/c 14,400 52.59 757,296 By Product K2 nos. Rs.To Variable cost 16.50 237,600 (bal.figure) 13,200 1173979To Fixed cost 264,000 By closing wip 1,200 84,917

14,400 1258896 Err:522Answer (iv)

Other methods which can be used for this purpose include(i) Physical measure method(ii) Net realisable method and (iii) Constant gross profit margin method.

Answer (v) Advise to the management of JKL limitedIf further processed, total cost = sales value at split off + variable cost and fixed cost of processing

Rs. 47.30 x 14,400 + Rs. 16.50 x 14,400 + Rs. 2,64,000 Rs. Revenue to be realised = Rs. 77 x 14,400 Rs. As the revenue is less than the additional cost, it should not be processed any further.

Total cost = 14,400 x (47.30 + 16.50) + 60% of 2,64,000 = Rs. Revenue to be realised = Rs. 77 x 14,400 = Rs. Additional revenue due to further processing Rs. In this case, it should be processed further as the incremental revenue exceeds the incremental cost.

Alternatively :Incremental sales revenue per kg. from further processing (77–47.30)Less : Incremental variable cost per kg. of further processingIncremental contribution per kg. from further processingIncremental contribution of total output of 14,400 kg.Less : Avoidable fixed cost as 60% of Rs. 2,64,000Net incremental revenueBreak - even point = Avoidable Fixed cost / Incremental contribution per kg. (158,400/13.2)Break - even point in units 12,000

Process I Process IIDirect materials Rs. 60,000 40,000Direct labour Rs. 40,000 30,000Production overheads Rs. 39,000 40,250Normal loss Rs. 8% 5%Output kg. 18,000 17,400Less: Realisation of Rs. / unit 2.00 3.00

The company uses Final sales value method for the attribution of pre-separation costs to the joint products. This method is generally adopted by most of the companies in the industry, and is very popular.

(b) In this case only 60% of the fixed cost of Rs. 2,64,000 is to be considered for further processing because it can be avoided.

Hence further processing should take place if the output is exceeding 12,000 kg.per week, otherwise the output should be sold @ Rs. 47.30 per kg. at the split-off point.

Problem 13 (ca inter-II nov. 02)

A product passes through two processes. The output of process I becomes the input of Process II and the output of Process II is transferred to warehouse.The quantity of raw material introduced into Process I is 20,000 kg. at Rs. 10 per kg. The cost and output data for the month under review are as under:

The company's policy is to fix the Selling price of end product in such a way as to yield a profit of 20% on selling price.

Page 160: cost accounting question icwai inter

Solution: Stepwise solution is given below:First take the process I. See whether there is any abnormal gain/loss in the process.

Input Normal loss @ 8% Normal output Actual output Abnormal gain/loss20,000 1,600 18,400 18,000 400

Evaluate the abnormal loss: Rs.Input 20,000 kg @ Rs. 10 per kg. 200,000Direct materials 60,000Direct labour 40,000Production overheads 39,000Total cost of input 339,000Less: Realisation from normal loss @ Rs. 2 per kg for 1600 kg. 3,200Cost of input for evaluation of abnormal loss For 18,400 kg. 335,800Rate per kg. for evaluation of abnormal loss (335,800/18,400) Rs./kg 18.25Valuation of abnormal loss of 400 kg. Rs. 7,300Now Process I account can easily be prepared.

Process I a/cParticulars Kg. Amount Particulars Kg. Rate

Rs. Rs./kgTo input 20,000 200,000 By nor. loss 1,600 2To direct materials 60,000 By abn. loss 400 18.25To direct labour 40,000 By transfer toTo overheads. 39,000 process II 18,000 18.25

20,000 339,000 20,000Note : There is no balancing figure in the above account.Now take process II. See whether there is any abnormal gain/loss in the process.

Input Normal loss @ 5% Normal output Actual output Abnormal gain/loss

18,000 900 17,100 17,400 300Evaluate the abnormal gain: Rs.Input 18,000 kg @ Rs. 18.25 per kg. 328,500Direct materials 40,000Direct labour 30,000Production overheads 40,250Total cost of input 438,750Less: Realisation from normal loss @ Rs. 3 per kg for 900 kg. 2,700Net cost of input for evaluation of abnormal gain For 17,100 kg. 436,050Cost of abnormal gain per kg. (436,050/17,100) 25.50

Process II a/cKg. Rate Rs. Amount Kg. Rate

To input 18,000 18.25 328,500 By normal loss 900 3.00To direct materials 40,000To direct labour 30,000 By transfer to 17,400 25.50To overheads. 40,250 warehouseTo abnormal gain 300 25.50 7,650

18,300 446,400 18,300Note : There is no balancing figure in the above account.Determination of Selling price of the end product: Rate

Rs./unit

Page 161: cost accounting question icwai inter

Cost of end product 25.50Profit is 20% of the selling price (25% of the cost) 6.375Selling price of the end product 31.875

Opening WIP 200 units (25% complete) valued at Rs. 5,000800 units received from Process 1 valued at Rs. 8,600840 units were transferred to Process 3 and CWIP is 160 units ( 50% complete)The cost for the period were Rs. 33,160 and no units were scrapped.Required: Prepare Process account using average cost method.Solution: Solve yourself. Brief solution is given below.

units in units out Material LabourOverheads 1,000 Completed 840 840 840 840

Cwip 160 80 80 801,000 1,000 920 920 920

Average cost of completed units = (5,000+8,600+33,160)=46,760.Cost per completed unit = 46,760/920 = Rs. 50.82 therefore cost of 840 units would be Rs. 42,694.Cost of cwip of 160 units 50% complete would be 80 x 50.82 = Rs. 4,066

The following data relate to Process QUnits and degree of completion of opening and closing work-in-progress and scrap are as follows

Opening wip Closing wip ScrappedUnits 4,000 3,000 4,000Material 100% 100% 100%Labour 60% 50% 80%Overheads 60% 50% 80%Received during the month of April 1998 from process P. 40,000 units at Rs. 1,71,000Expenses during the month in process Q Rs.Material 79,000Labour 138,230Overheads 69,120Normal loss ; 5% of current input; Spoiled goods realised Rs. 1.50 each on sale.Completed units are transferred to warehouse.Required : You are required to prepare(i) Equivalent units statement (ii) Process Q account and (iii) Statement of cost per equivalent unit and total costs. (iv) any other account necessary.Solution: Brief solution is given below. Fifo method is used.

Statement of equivalent production Particulars Material Labour

Total cost Rs. 247,000 138,230Completed units nos. 38,000 37,700Cost per equivalent unit Rs./unit 6.50 3.67Process A/c: debit side total : Rs. 502,950 Credit side :Normal loss:3000,completed units:450,400,Cwip:27,750 and Ab.loss:21,800.

Following information is available regarding Process A for the month of February 1999Units in process as on 1.2.99(all material used,25% complete in labour and overheads)

Problem 14 (ca inter-II nov. 95)

Process 2 receives units from Process 1 and after carrying out work on the units transfer them to Process 3. For the accounting period the relevant data were as follows:

Problem 15 (ca inter-II may 98)

Problem 16 (ca inter-II may 99)

Page 162: cost accounting question icwai inter

Units introduced 16,000Units completed 14,000Units in process on 28.2.99 6,000

Work-in-progress as on 1.2.99 Cost during the monthRs. Rs.

Materials 6,000 25,600Labour 1,000 15,000Overheads 1,000 15,000Presuming the average method of inventory is used, Prepare:(i) Equivalent production units statement (ii) Process account (iii) Statement of cost per equivalent unit and total costs. Solution: Read tutorial notes of Problem 3 solved previously. Brief solution follows.Statement of equivalent production :

Input Output Material Labour20,000 Completed 14,000 Rs. 14,000 14,000

Closing work-in-progress 6,000 Rs. 6,000 2,00020,000 20,000 Rs. 20,000 16,000

Total cost Rs. Rs. 31,600 16,000Value of output transferred Rs. 50,120Value of closing work-in-progress Rs. 13,480

Direct material added in Process II (Opening WIP) 2000 units at Rs. 25,750Transfer from Process II 53,000 units at Rs. 4,11,500Transferred to Process IV 48,000 unitsUnits scrapped 2,000 unitsClosing stock of Process III 5,000 units

Rs. Direct material added in Process III 197,600Direct labour 97,600Production overheads 48,800

Degree of completion: Opening stock Closing stock ScrapMaterial 80% 70% 100%Labour 60% 50% 70%Overheads 60% 50% 70%The normal loss in the process was 5% of the production and the scrap was sold @ Rs. 3/unit.Solution:This can be solved on the lines of solution given with problem 9. Brief solution followsProduction = (2,000 + 53,000 – 5,000) = 50,000 units. Normal loss @ 5% = 2,500 units.Actual units scrapped = 2,000 units thus there must be abnormal gain of 500 units.

Statement of equivalent productionunits in units out Mater. A Mater. B Labour Overheads

2,000 Opening stock 2,000 0 400 800 80053,000 46,000 46,000 46,000 46,000 46,000

Problem 17 (ca inter-II nov.06)

From the following information for the month of October 2005, prepare Process Cost accounts for Process III. Use First-in-first-out method to value equivalent production.

Introduced and Completed

Page 163: cost accounting question icwai inter

Closing stock 5,000 5,000 3,500 2,500 2,500Abnormal gain 500 500 500 500 500Normal loss 2,500

55,000 55,000 50,500 49,400 48,800 48,800Statement of cost of each element

Mater. A Mater. B LabourCost Rs. 411,500 197,600 97,600Less: Scrap of normal loss 2500 units at Rs. 3 7500Total cost Rs. 404,000 197,600 97,600Equivalent production units 50,500 49,400 48,800Cost per unit in Rs. Rs. 8.0 4.0 2.0

Costt Sheet relating to Process III. Equivalent units for completion and cost.Mater. A Mater. B Labour Overheads

Equivalent units opening wip 0 400 800 800Cost for completion of Owip 0 1,600 1,600 800 4,000

Equivalent units closing wip 5,000 3,500 2,500 2,500Cost incurred 40,000 14,000 5,000 2,500Abnormal gain 500 units @ Rs. 15

Rs.Process III

Units Amount Units nos. Rs. nos.

To Owip 2,000 25,750 By normal loss 2,500To Process II a/c 53,000 411,500 By Process IV a/c 48,000To direct materials 197,600 (690,000 + 4,000 + 25,750)To direct labour 97,600 By Cwip 5,000To overheads 48,800To abnor.gain 500 7,500

55,500 788,750 55,500Please note that there is no balancing figure in the above account.

Direct materials : Rs. 15,000, Direct labour : Rs. 18,000 and Factory overheads: Rs.12,000

Packing material : Rs. 4,000, Direct wages : Rs. 3,500 and Factory overheads : Rs. 4,500Required :

Introduced and completed 46,000 units @ Rs. 15/ unit

Problem 18 (ca inter-II may.06)

A company produces a component, which passes through two processes. During the month of April 2006, material for 40,000 components were put into Process I of which 30,000 were completed and transferred to Process II Those not transferred were 100% complete as to material cost and 50% complete as to labour and overheads costs. The Process I costs incurred were as follows:

Of those transferred to Process II, 28,000 units were completed and transferred to finished goods stores. There was a normal loss with no salvage value of 200 units in Process II. There were 1,800 units, remained unfinished in the process with 100% complete as to material and 25% complete as regard to wages and overheads.

No further process material costs occur after introduction at the first process until the end of the second process, when protective packing is applied to the completed components. The process and packing costs incurred at the end of the process II were:

Page 164: cost accounting question icwai inter

Statement of equivalent production,cost per unit and process account for both the processes.Solution:

Statement of equivalent production for Process IUnits Material Labour Overheads

Units completed 30,000 30,000 30,000Closing work-in-progress 10,000% of completion of work-in-progress 100% 50% 50%Equivalent production of work-in-progress 10,000 5,000 5,000Total Equivalent production units 40,000 35,000 35,000Cost Incurred during the process Rs. 15,000 18,000 12,000Cost per unit elementwise Rs./unit 0.38 0.51 0.34Cost of closing work-in-progress Rs. 3,750 2,571 1714Cost of material transferred to process II (45,000 – 8,036 or 30,000 x 1.23)

Statement of equivalent production for Process IUnits Material Labour Overheads

Units completed 28,000 28,000 28,000Equivalent production of work-in-progress 1,800 450 450Total Equivalent production units 29,800 28,450 28,450Cost Incurred during the process Rs. 36,964 3,500 4,500Cost per unit elementwise Rs./unit 1.24 0.12 0.16Cost of closing work-in-progress Rs. 2,233 55 71Cost of material transferred to process II (44,964 – 2,369 or 28,000 x 1.52)Add: Cost of packing material Rs.Total Cost Rs.

Opening work-in-progress (2,000 units) Material LabourRs. 80,000 15,000

Material introduced in process 'A' 38,000 1480000 359,000

Units scraped 3,000 unitsDegree of completion Material 100% Labour and overheads Closing Work-in-progress : 2,000 units Degree of completion Material

Labour & overhead Units finished and transferred to Process 'B' 35,000 unitsNormal loss: 5% of total including opening work-in-progressScrapped units fetch Rs. 20 per piece.You are required to prepare statement of : (i) equivalent production (ii) cost (iii) distribution cost (iv) Process 'A' account and (v) Abnormal loss account.Solution : Statement of equivalent Production (in units)

Material Labour Overheads units 40,000 37,400 37,400

Cost Rs. 1520000 374,000 1122000Cost per unit Rs. 38 10 30Statement of distribution of cost: Rs. Completed and transferred to process 'B' 28,00,000Abnormal loss of 1000 units 72,000

Problem 19 (ca inter-II may.07)

Following details are related to the work done in Process 'A' of XYZ company during the month of March 07: (fig.in Rs.)

Page 165: cost accounting question icwai inter

Closing Work-in-progress of 2,000 units 144,000 Process A Account

Details Units Amount Details UnitsRs.'000

To opening work-in-progress 2,000 140 By Normal loss 2,000To Material introduced 38,000 1,480 By Abnormal loss 1,000To Direct labour 359 By Process 'B' A/c 35,000To Overheads 1,077 By Closing Wip 2,000

40,000 3,056 40,000Normal Loss Account

Units Rs.'000 UnitsTo Process 'A' 2,000 40 By Cost Ledger Control 2,000

Abnormal Loss AccountTo Process 'A' 1,000 72 By Cost Leder Control A/c 1,000

By Costing P and L A/c72

Don't blame others when you receive a setback.Problem 20 (CWA Inter Dec. 08)

Relevant information for costing purposes relating to period 5 are as follows:Direct material added: Process 1 Process 2Units 2,000 1,250Cost Rs. 8,100 1,900Direct labour Rs. 4,000 10,000Production Overhead (% of direct labour Rs.) Rs. 150% 120%Output of process 2/ Finished goods in units 1,750 2,800Actual production overhead Rs. 17,800Workout cost per unit of output and losses.Solution :Tutorial Notes :1. The output of process 2 is 2,800 units. How much must be input? Think.2. What is the use of actual production overhead of Rs. 17,800?Computation of output and gains/losses from processes :

UnitsProcess 1: Input 2,000 Process 2: Input (1750 from process 1+ 1250)

Normal loss @ 10% 200 Normal loss @ 10%Normal output 1,800 Normal outputActual output 1,750 Actual outputAbnormal loss 50 Abnormal gain

Computation of Cost per unit of output of process 1:Process 1: Rs. Process 2: Rs.Materials 8,100 Materials 1,900Labour 4,000 Process 1 (1750 x 10) 17,500Overhead @ 150% of labour 6,000 Labour 10,000

Overhead @ 120% of labour 12,00018,100 41,400

Less : Scrape (200 x Rs. 0.50) 100 Less: Scrap (300 x Rs. 3) 900

A factory has two production processes. Normal loss in each process is 10% and scrapped units sell for Rs. 0.50 each from process 1 and Rs. 3 each from process 2.

Page 166: cost accounting question icwai inter

18,000 40,500Cost per unit (18,000 / 1,800) 10 Cost / unit (40,500 / 2,700) 15

Actual production overhead of Rs. 17,800 is not relevant in process accounts. The overheads absorbed by the processes are Rs. 18,000 (12,000 + 6,000). There is overabsorption of overheads of Rs. 200.

·       Practice asking and listening. You impress more by the way you listen rather than the way you talk.

Page 167: cost accounting question icwai inter

Total20

Remember : Life is a series of changing seasons.

AmountRs.

nqfu;k ,d cgqr cM+s vkbZus dh rjg gSA ;g vkidks vkidk vlyh psgjk fn[kk nsrh gSA vxj vki izseiw.kZ gSa] nksLrkuk gS] ennxkj gSa rks nqfu;k Hkh vkids fy;s iszeiw.kZ] nksLrkuk vkSj ennxkj gks tk;sxhA nqfu;k ogh gS tks

A product passes through two distinct processes X and Y before completion. During a certain period, 10,000 units of crude material were introduced in process X at a cost of Rs. 40,000. After processing in dept. X, 9,000 units of processed material were transferred to process Y for finishing. From process Y finally 8,600

1. While preparing process accounts, first of all, accounts for the number of units should be prepared as working notes. This gives abnormal loss/gain arising during the process. All abnormal loss/gain should be valued as good units. The good units should be valued at normal loss figures irrespective of actual number

2.Students should make a particular note of treatment given to abnormal loss / gain. In preparing process accounts the abnormal loss / gain are treated as good units and valued accordingly at par with normal output. While preparing abnormal loss / gain account they are treated as waste and the diffrence between

Remember : We grow the most by our greatest

Page 168: cost accounting question icwai inter

74,3484,0001,652

80,000

AmountRs.

99,3263,600

102,926

amountRs. 400

8,200

8,600

amount1,000

6521,652

amount578

578

vf/kdka'k yksx mrus lq[kh ugha gS ftrus lq[kh oks gks ldrs gSS] nq[k dh ckr ;g gS fd oks vius nq[kksa dk fuekZ.k

Page 169: cost accounting question icwai inter

Process 347,16656,00084,000

1%5.0

91,500

gain

AmountRs.

706,8005,760

912

713,472

AmountRs.

A company manufactures its sole product by passing the raw material through three distinct processes in its factory. During the month of April 2004, the company purchased 96,000 kg of raw material at Rs. 5 per kg and introduced the same in process 1. Further particulars of manufacture for the month are given below.

Page 170: cost accounting question icwai inter

912,7802,790

915,570

AmountRs.

10,98,000

4,610

11,02,610

240

672912

Amount1,287

2,664

3,951

Rs. 27,000

8,00012,50047,500

574,750119,300178,450

From the following information relating to Process I of a factory for the month of April 2003, Prepare the statement of equivalent production, statement of cost, statement of evaluation and process account, using

Page 171: cost accounting question icwai inter

vHkh fdlh ds ckjs esa vPNk lksfp;sA

Overhead

12,000

30700

12,730

all figures in Rs. Total

47,500872,500920,000

14,500905,500

70This cost per unit elementwise will be used in determining the cost of production and abnormal loss/gain

Total70

Err:522

1. This question is tricky and important both. Statement of equivalent units calls for the determination of cost of material, labour and overhead in good units. The normal loss and not the actual loss should be taken to

2. When average cost method of inventory valuation is used in process accounting, the completion of opening work-in-progress is not converted into equivalent units. Worded differently, no distinction is made between units which are partially completed in previous period and the units which are introduced and

3. Following two points should be kept in mind while following the average cost method inventory costing, (i) Units completed will not be divided in two categories i.e. opening inventory and units introduced and completed during the period as done in fifo method and (ii) Cost of opening work-in-process is added

4. In average cost method of inventory valuation the cost of opening inventory is added elementwise to cost incurred during the process and the agreegate of these two is divided by sum of (i) completed units during

5. In short, according to this method opening inventory of work-in-progress and its costs are merged with the production and cost of the current period, respectively. An average cost per unit is determined by dividing the total cost by the total equivalent units, to ascertain the value of the units completed in process.

Page 172: cost accounting question icwai inter

Err:522

Err:522Err:522

Amt. Rs.840,000

14,5003,000

62,500920,000

Realisable value in Rs./unit

Ab.loss0

740

AmountRs.

558,600

1400

560,000

Production in a manufacturing company passes through three distinct processes I, II and III. The output of each process is transferred to finished goods stock. The normal wastage in each process and the realisable

Process I was fed with 40,000 units of input costing Rs. 320,000. There were no opening and closing work-

Page 173: cost accounting question icwai inter

Amount681,888

2,12814,584

698,600

AmountRs.

828,702

3,460

832,162

Product X in a manufacturing unit passes through thre processes–A, B and C. The expenses incurred in the

During the year, three-fourth of the output of process A and two-third of the out of process B were tranferred to the next process and the balances were sold outside.The entire amount of the process C however, was sold outside. The losses of the three processes were sold at Rs. 5 per unit for process A. Rs. 10 per unit for

Prepare the three processes accounts and a statement of income considering a total selling and distribution

Page 174: cost accounting question icwai inter

Ab.loss150

26

AmountRs.

10,71,0002,250

25,500357,000

14,55,750

AmountRs.

8,74,0006,300

4,37,000

13,17,300

AmountRs.

915,000

1,7106,500

923,210

The speciality of this question is that some of the output of a process is transferred to the next process and some output is also sold in the market. The selling price of the output is also given e.g. for process A it is Rs. 200/unit, for process B it is Rs. 280/unit and for process C it is Rs. 600/unit. Student should particularly note that in process account the actual proceeds of unit sold is not relevant. All units are to be valued as good units of output with loss taken as normal loss. The diffrence should be taken to P and L account. Therefore in preparing process account of process A, although the 1/3 rd of 8,400 units i.e. 2,100 units are sold in the market at Rs. 200/unit fetching Rs. 420,000 as revenue, the process account will not be credited with this amount. It will be credited with the rate of output as obtained in the process account.

Page 175: cost accounting question icwai inter

Rs.420,000532,000

21,96,000

6,600

31,54,600

Data relating to work done in Process 'A' of a company during the month of April 2000 is given below:

Overhead

17,500

1. First of all let us see whether there is any abnormal loss or gain. Normal loss is given as 5% of input including work-in-progress.Normal loss is 5% of ( 1000 + 19,000) = 1000 units. The output should be { 19,000 ( not 20,000) – 1000} = 18,000 units but the output is given as 17,500 units which shows that there

2. The degree of completion of work-in-progress has not been given. This implies that average cost method should be followed in determining the equivalent production. Thus opening work-in-progress will be ignored

3. Note that the degree of completion of scrapped units has also been given. You should not be confused to apply this degree to normal loss units. Normal loss units should not be considered at all in computing the equivalent production. The degree of completion of scrapped units is relevant only for abnormal loss units.

Page 176: cost accounting question icwai inter

–400800

18,700

TotalRs.

80

Total

80Rs.

14,00,000

36,000

72,00015,08,000

Amount Rs.14,00,000

20,00036,00072,000

15,28,000

Amount20,000

10,00026,00036,000

SBL Ltd. Furnishes you the following information relating to process-Business for the month of April 2006

Page 177: cost accounting question icwai inter

Finished goods 78,926

8,0009.87

Cost Rs.6,456

484232

7,172

FinishRs.

00

21,00021,000

94,500

RST.Ltd manufactures plastic moulded chairs. Three models of moulded chairs, all variation of the same design are standard Deluxe and Executive. The company uses an operating costing system.

RST. Ltd has extrusion, form, trim and finish operation. During the extrusion operation, the plastic sheets are produced. During the forming operation, the plastic aremoulded into chair seats and the legs are added. The standard model is sold after this operation. During the trim operation, the arms are added to the Deluxe and

Only the Executive model enters the finish operation, in which padding is added. All the units produced

2.Now consider the following information for May. All unit costs in May are identical to the April unit costs calculated as above in (1). At the end of May, 1,500 units of the Deluxe model remain in work-in-progress.These units are 100% complete as to materials and 65% complete in the trim operation. Determine

First of all let us calculate the unit cost of all the four operations. The extrusion and form operations are conducted in all the three models. The trim operation is conducted on deluxe and executive models and the

Page 178: cost accounting question icwai inter

Statement showing material cost and conversion cost per unit of the operation.

Total cost Rs.

?kj esa iM+h csdkj oLrqvksa dks ;k rks O;ofLFkr :i ls tek nsuk pkfg;s ;k fQj fdlh dks ns nsuk pkfg;SA csrjrhc dckM+ udkjkRed mtkZ iSnk djrk gS] dyg djokrk gS rFkk vkus okys /ku dks jksdrk gSA phuh

Page 179: cost accounting question icwai inter

The solution goes as follows:

Overhead 720

41,400

4502,250

43,920

Small problems small achievements. Big problems big achievements. What do you select ?(1) The normal loss has been given as 5% of production,but production is not known. It is hidden

somewhere. Think where it is hidden. Only after knowing production, normal loss can be ascertained and

(2) Whenever the degree of completion of cost elements is given in opening work-in-progress, the average cost method should not be used because it ignores opening work-in-progress in the evaluation of equivalent production. In such cases either fifo or lifo method should be used. In this solution fifo method is applied.

(3) According to FIFO method, the units first entering the process are completed first. Thus the units completed during the period would consist partly of the units which were incomplete at the beginning of the

The cost of the completed units is affected by the value of the opening inventory, which is based on the cost

(4) In the computation of equivalent production, special care should be taken in respect of opening work-in-progress. If degree of completion of any element is given, the balance should be taken for computation of equivalent units in fifo method. For illustration, let us say degree of completion of 400 unit of opening wip is given as 75%, then in fifo method the equivalent production in the process would be 25% of 400 units i.e 100

(5) Degree of completion of abnormal gain/loss, if any, has not been mentioned, these units can be assumed

(6) Degree of completion of normal loss or scrap should not be considered in computation of equivalent

Opening wip of 1800 units is 20% completed in material and 40% completed in labour and overhead in the

Closing work-in-progress of 4,500 units is 70% complete in material, and 50% complete in labour and Material I units will bear the cost transferred to the process. Thus units representing opening stock and partly finished will not appear in this column since they are complete so far as the cost transferred to the process is

Material II units bear both the costs viz. cost transferred to the process and the cost relating to the process.

Thus column Material II in statement of equivalent production will include: (i) units equal to work done on opening stock (ii) units introduced and completed and (iii) units equal to work done on closing stock.

Page 180: cost accounting question icwai inter

18.47

Total27,000

Err:522764,658Err:522

Err:5228,312

The reader should himself prepare the process III a/c and then compare it with the one given below:

15,188795,258

70,965

881,411

Labour and Overhead Rs.

194,400141,750245,700140,400

86,400

The difference in the two sides is due to approximation of decimal figures. Tne closing stock, abnormal gain and finished goods have been evaluated @ Rs. 18.47 instead of Rs. 18.4728, hence the difference.

In a manufacturing company, a product passes through 5 operations. The output of 5th operation becomes the finished product. The input rejection, output and labour and overheads of each operation for a period are

(ii) Calculate the labour and overhead cost at each operation for one unit of final product and the total labour

Page 181: cost accounting question icwai inter

Rejection as %of output

33.3%7.1%7.7%8.3%

20.0%

(i) Determination of the input required in each operation for one unit of final output.

Overheads

30,00060,00060,000

Prepare process 3 account using average price method, alongwith necessary supporting statements.

If you see the input units of each operation, you will find that the inputs of different operations are taken from different processes and not from one process. The results are independent of one another. The above table

The question is to determine the number of units to be introduced in operation 1, to receive one unit of final

Labour and overhead cost per unit of final output

Page 182: cost accounting question icwai inter

(in Rs. )Overheads

11,000

60,000

71,000

6.20

2.23

4.17

3.99

16.58

281,82213,262

48,290

Amount7,200

281,82248,290

(1) The normal loss has been given as 10% of production,but production is not known. It is hidden somewhere. Think where it is hidden. Only after knowing production, normal loss can be ascertained and

Cost per unit

Page 183: cost accounting question icwai inter

337,312

Process II which is used for further processing of product K into product K2, has the following cost structure:

in kg.

19,20014,400

4,0002,400

NIL

1,200 kg.

(iii) Comment on the method used by the JKL ltd. to attribute the pre-separation costs to joint products.

(b) If the 60% of weekly fixed cost of process II were avoided by not processing product K further.

JKL limited produces two products– J and K together with a by-product L from a single main process (process I). Product J is sold at the point of separation for Rs. 55 per kg, whereas product K is sold for Rs. 77 per kg after further processing into product K2. By-product L is sold without further processing for Rs.

Process I is closely monitored by a team of chemists, who planned the output per 1,000 kg of input materials

(i) Prepare Process I account for the first week of the month using the final sales value method of attribute

(iv) Advise the management of JKL ltd. whether or not, on purly financial grounds it should continue to

1. This question is tricky and hard. You must think a lot before attempting to solve. You must try to solve this before seeing the solution. You can commence your line of thinking on the following points.

2. In all questions previously solved, the normal loss is in the form of scraped units which brings in some revenue or nil revenue and as such the input cost is either reduced to the extent of the realisation or remains the same. In this particular question, the waste is disposed off @ Rs. 16.50 per kg. Thus while calculating

Page 184: cost accounting question icwai inter

8. There is no abnormal gain/loss in Process II.

Abnormal gain/ loss

kg. losskg. gain

3. There are two joint products coming out of the process. One is a case of abnormal gain while the other is of abnormal loss. The evaluation of abnormal gain/loss should not be done outputwise but should be done on the total number of units. A view of the question reveals that there is overall abnormal loss of 400 units, thus evaluation of this abnormal loss should be carried out in normal way as a ratio of net cost of input / net

4. Net cost of input will be [ Total cost of input – realisation from by-product plus cost of disposal of normal waste of 2,000 kg @ Rs. 16.50 ]. You must also note that this cost is not actual cost of input but notional cost of input for evaluation of abnormal loss. The actual cost of input can be determined only after the

5. Pre-separation cost is to be attributed to the joint products in the ratio of their sales values. While sales value of J and K can be determined, the pre-separation cost is to be computed. You must think how the pre-

6. (a) and (b) of part (iv) of the question are easy. You should be able to solve both the parts without solving

7. From the question it is evident that the normal loss consisting of by-product and waste constitutes 15% of the input. Thus normal output as 85% of input should be taken while evaluating the abnormal loss.

9. The closing work-in-progress is given as 1,200 kg. Its evaluation needs some calculations to be done. It is 50% converted. The conversion cost is Rs. 16.50 /kg as variable cost and Rs. 264,000 as fixed cost. Please

It is evident that normal output is 85% of input because 1000 kg of input yields 100 kg of by-product and 50

Page 185: cost accounting question icwai inter

amount

77,000

17,600721,171757,229

015,73,000

Fixed cost per unit is Rs. 264,000/13,800 = Rs. 19.13/kg.Please find out what is wrong in the following table

AmountRs.

63,1089,900

11,47884,486

Fixed cost incurred is Rs. 264,000. The equivalent units completed in respect of fixed cost,is indirectly calculated as input minus incomplete units. The input is 14,400 kg and the incompleted quantity is 50% of 1,200 kg. Thus equivalent quantity for computation of fixed cost per unit for evaluation of wip is 14,400 –

This is where the students generally commit mistakes.The above table is incorrect because the variable cost per unit is taken as per input units whereas it should be taken as per output unit. The correct evaluation is as

Page 186: cost accounting question icwai inter

If further processed, total cost = sales value at split off + variable cost and fixed cost of processing

11,82,72011,08,800

10,77,12011,08,800

31,680In this case, it should be processed further as the incremental revenue exceeds the incremental cost.

Rs. 29.716.513.2

190,080158,400

31,680(158,400/13.2)

kgs.

The company uses Final sales value method for the attribution of pre-separation costs to the joint products. This method is generally adopted by most of the companies in the industry, and is very popular.

(b) In this case only 60% of the fixed cost of Rs. 2,64,000 is to be considered for further processing because

Hence further processing should take place if the output is exceeding 12,000 kg.per week, otherwise the

A product passes through two processes. The output of process I becomes the input of Process II and the output of Process II is transferred to warehouse.The quantity of raw material introduced into Process I is 20,000 kg. at Rs. 10 per kg. The cost and output data for the month under review are as under:

The company's policy is to fix the Selling price of end product in such a way as to yield a profit of 20% on

Page 187: cost accounting question icwai inter

Abnormal gain/lossloss

AmountRs.

3,2007,300

328,500339,000

Abnormal gain/lossgain.

Amount

2,700

443,700

446,400

AmountRs.

Page 188: cost accounting question icwai inter

443,700110,925554,625

Cost per completed unit = 46,760/920 = Rs. 50.82 therefore cost of 840 units would be Rs. 42,694.

40,000 units at Rs. 1,71,000

Overheads

69,12037,700

1.83

Following information is available regarding Process A for the month of February 19994,000

Process 2 receives units from Process 1 and after carrying out work on the units transfer them to Process 3.

Page 189: cost accounting question icwai inter

Overheads 14,000

2,00016,00016,000

2000 units at Rs. 25,75053,000 units at Rs. 4,11,500

Overheads

From the following information for the month of October 2005, prepare Process Cost accounts for Process

Page 190: cost accounting question icwai inter

Overheads

48,800

48,800

48,8001.0

Costt Sheet relating to Process III. Equivalent units for completion and cost.

690,000

61,5007,500

759,000

AmountRs.

7,500

719,75061,500

788,750

A company produces a component, which passes through two processes. During the month of April 2006, material for 40,000 components were put into Process I of which 30,000 were completed and transferred to

were 100% complete as to material cost and 50% complete as to labour

Of those transferred to Process II, 28,000 units were completed and transferred to finished goods stores. There was a normal loss with no salvage value of 200 units in Process II. There were 1,800 units, remained unfinished in the process with 100% complete as to material and 25% complete as regard to wages and

No further process material costs occur after introduction at the first process until the end of the second process, when protective packing is applied to the completed components. The process and packing costs

Page 191: cost accounting question icwai inter

Overheads

45,0001.23

8,03636,964

Overheads

44,9641.52

2,35942,605

4,00046,605

Overheads45,000

1077000

80%100%

80%

Overheads

Following details are related to the work done in Process 'A' of XYZ company during the month of March 07:

Page 192: cost accounting question icwai inter

AmountRs.'000

4072

2,800144

3,056

Rs.'00040

205272

Units3,000

3002,7002,800

100

A factory has two production processes. Normal loss in each process is 10% and scrapped units sell for Rs.

Page 193: cost accounting question icwai inter

Actual production overhead of Rs. 17,800 is not relevant in process accounts. The overheads absorbed by the processes are Rs. 18,000 (12,000 + 6,000). There is overabsorption of overheads of Rs. 200.

Practice asking and listening. You impress more by the way you listen rather than the way you

Page 194: cost accounting question icwai inter

Chapter : 6 Joint Products and Bye Productscwa inter cwa final ca inter ca final Total

4 2 9 0 15

Problem: 1(cwa inter June 05)

All the three products are processed further after separation and sold as per details given below:

Particulars Product A By product B By product CSales Rs. 200,000 120,000Cost incurred after separation Rs. 20,000 15,000Profit as % of sales 25 20

Required:1. Prepare a statement showing the apportionment of joint costs to the products A, B and C

Solution: Tutorial Notes :

Modern Mills Ltd.Statement showing the apportionment of joint costs Particulars Product A By product B By product C Total

Rs. Rs. Rs. Rs.Sales 200,000 120,000 80,000 400,000Less: Profit 50,000 24,000 12,000 Err:522Cost of sales 150,000 96,000 68,000 314,000Less: Selling expenses @ 10% of total 31,400Apportionment of selling expenses to product A, B and C in the ratio of 20:40:40 as givenApportionment 6,280 12,560 12,560 31,400Total cost of production 143,720 83,440 55,440 282,600Less: Cost after split off 20,000 15,000 10,000 45,000Apportioned Joint costs 123,720* 68,440 45,440 237,600*(143,720 -- 20,000)

Part B:

gj vuqHko ,d ldkjkRed vuqHko gS vxj eSa mls rjDdh djus ;k vius vkidks lq/kkjus ds ekSds dh rjg ns[kwaA

Note : It is advised to depict the joint process by way of a leveled diagram. The diagram should contain quantity input, its rate and value in Rs., the split off point, and all details in respect of further processing. The diagrammatic presentation of entire joint process adds color to your answer.

Modern Mill Ltd. Manufactures certain grades of products known as A,B and C. During manufacture of product A, by product B and C emerge. The joint expenses of manufacture amount to Rs. 2,37,600.

Total fixed selling expenses are 10% of total cost of sales which are apportioned to the three products in the ratio 20:40:40.

2.If the product is not subject to further processing and is sold at the point of separation, for which there is a market at Rs.100,440 without incurring any selling expenses. Would you advise its disposal at this stage? Show the workings

You have to do certain thinking before solving the problem. Some hints are supplied here. Sales less profit is cost of sales. 10% of cost of sales is Selling expense which should be apportioned to three products in the given ratio. Cost of sales less selling expense is cost of production. Cost after split off is already given. Cost of production less cost after split off will be the apportioned joint cost. Please solve this on the above lines and compare your solution with the one given below:

Note that the total of apportioned joint costs tallies with the joint cost given in the question.

If the product B is sold at the split off point without incurring any selling expenses at Rs.100,440, the relevant cost for calculation of profit would be the apportioned cost viz. Rs. 68,440. Thus the profit would be 100,440 - 68,440 i.e. Rs. 32,000. This is more than the current profit of Rs. 24,000 by an amount of Rs. 8,000.

Page 195: cost accounting question icwai inter

Recommendation:

It is recommended that the product B should be sold at split off point and the product C should be processed further.

Joint processing cost: unit Quantity Rate (Rs.) Amount Rs.Raw material input kg 20,000 15 300,000Other materials 30,600Direct labour hours 4,000 20 80,000Production overheads 100,000Output, selling price and other particulars :Product Output Selling price Further processing Selling price

Without processing Cost After process.Kg. Rs./ kg Rs./ kg Rs./ kg

A 8,000 28 56,000 38B 6,000 30 60,000 42C 5,000 32 60,000 43X 500 6 1,500 8

Required:

(b) Which of the products should be further processed ? Show workings.

(c) What is the profit earned if all the main products are sold without further processing ? Give product wise details.

(d) If the further processing is done as suggested in (b), what is the total profit earned ? Give product wise details.

Solution:

Joint processing cost:Quantity Rate (Rs.) Amount Rs.

Raw material input 20,000 15 300,000Other materials 30,600Direct labour 4,000 20 80,000Production overheads 100,000Total Joint Cost 510,600Less: Sales value of by-product ( 500 kg @ Rs. 6) 3,000Net joint cost to be apportioned 507,600

(a) Statement showing the apportionment of joint costs Apportionment on the basis of sales value of three main products:Product Output Selling price Sales value Apportionment Cost per kg

kg Rs./kg Rs. Rs. Rs.A 8,000 28 224,000 201,600 25.20B 6,000 30 180,000 162,000 27.00C 5,000 32 160,000 144,000 28.80

19,000 564,000 507,600(b) Comparison of further processing cost with incremental sales valueProduct Output Incremental sale value Further processing cost

kg Rs./kg Total Rs.

Problem: 2 (cwa inter June 04)

A certain raw material on undergoing a chemical process yields three products A, B and C and by product X. The relevant particulars of the process for a month are given below:

(a) If the company apportions the joint cost after taking credit for the sale value of the by-product, in proportion to the sale value of the three main products at the point of separation, what is the cost per kg of each product at that stage.

Tutorial Notes: The fundamental question is ; What is the joint cost to be apportioned ? Think over it. Joint cost to be apportioned is to be computed first. Think how it can be computed. Rest of the question is easy.

Page 196: cost accounting question icwai inter

A 8,000 10 80,000 56,000B 6,000 12 72,000 60,000C 5,000 11 55,000 60,000X 500 2 1,000 1,500

In the light of above statement, the products A and B are advised for further processing.(c) Statement showing profit earned if all the products are sold without further processing:Product Output Selling Value Joint cost Profit

Kg. Rs./kg amount apportioned

A 8,000 28 224,000 201,600 22,400B 6,000 30 180,000 162,000 18,000C 5,000 32 160,000 144,000 16,000X 500 6 3,000 0 3,000

Total 567,000 507,600 59,400(d) Statement showing the profit earned with further processing Product Output Selling price Sales Joint cost Processing Total cost

After processing Revenue costKg. Rs./ kg Rs. Rs. Rs. Rs.

A 8,000 38 304,000 201,600 56,000 257,600B 6,000 42 252,000 162,000 60,000 222,000C 5,000 43 215,000 144,000 60,000 204,000X 500 8 4,000 0 1,500 1,500

Total 775,000 507,600 177,500

Details for a period are given below:Products Initial input Sales value Further processing Rejection rate

in units Rs./unit Cost Rs./unitA 5000 24 14B 8000 10 6C 10000 30 16X 44 5%Y 18 10%Z 48 8%Initial processing costMaterial Rs. 200,000Labour and overhead Rs. 200,000Required:

Solution:1. Apportionment of joint cost on the basis of sales revenue:Products Initial input Sales value Sales revenue Apportionment on the basis of

in units Rs./unit Rs. sales revenue 6:4:15 in Rs. A 5,000 24 120,000 96,000B 8,000 10 80,000 64,000C 10,000 30 300,000 240,000Total Sales revenue 500,000 400,000Total cost 400,000

Problem: 3 (cwa inter Dec.02)

Three products A, B and C are produced from a single process. Each product can be sold at the end of each process or can be further processed independently to produce superior products, which are marketed under different names X, Y and Z respectively.

1. Calculate the apportionment of costs to products A, B and C using sales value of production as the basis of apportionment

2. Explain whether the initial process should be undertaken and also state whether further processing is to be undertaken or not

Page 197: cost accounting question icwai inter

Total sales revenue exceeds the total cost; initial process should be undertaken.2. Statement showing cost and revenue if further processing is undertaken: All amount in Rs.

X Y Z

Initial output in units 5,000 8,000 10,000

Rejection 5% 10% 8%

Output in units 4,750 7,200 9,200

Rate of output in Rs./unit 44 18 48

Output revenue in Rs. 209,000 129,600 441,600

Original revenue in Rs. 120,000 80,000 300,000

Incremental revenue in Rs. 89,000 49,600 141,600

Incremental cost in Rs. (cost of further processing) 70,000 48,000 160,000

Incremental benefit (loss) 19,000 1,600 (18400)

Product Output Further processing cost (Rs.) Selling pr.at the sale pointAB 8,000 Kg. Rs./ kg 45BC 10,000 Kg. 80,000 Rs./ kg 60CD 12,000 Kg. 120,000 Rs./ kg 70DD 5,000 liters 60,000 Rs./liter 80Prepare :

1. Statement showing apportionment of joint cost on suitable basis and product wise profitability statement.

2. If the company finds a market for CD at Rs. 63/kg without further processing, will it be advisable to accept it ?

The solution goes as follows:First of all let us calculate the joint cost which is to be apportioned : Rs. In '000Cost of procurement of 40,000 liters @ Rs. 30 / liter :Rs. 30 x 40,000 1,200Cost of processing Rs. 300Total joint cost to be apportioned in Rs.'000 1,500Product AB BC CD DDOutput in units 8,000 10,000 12,000 5,000Selling price Rs./unit 45 60 70 80Sales revenue in '000 Rs. 360 600 840 400Further processing cost in '000 Rs. 0 80 120 60

In view of the above statement, it is evident that the product A and B should be further processed into product X and Y, while product C should not be processed any further.

Problem: 4 (cwa inter Dec.06)

In a chemical plant four different products viz. AB, BC,CD and DD emerge from the input of crude oil. Product AB can be sold immediately, but the remaining three products require further processing before they can be marketed.

In a month 40,000 liters of crude oil were procured at a cost of Rs. 30 per liter and processed at a cost of Rs. 3 lakhs. The details of output obtained, further processing cost, selling price per unit etc. are given below:

Solution: This is an easy question. Students should try to solve it before going through the solution

Tutorial Notes : 1.The apportionment of joint cost can not be done on basis of output of the process as the output have different units of measurement and their selling prices after further processing differ considerably. Thus the joint cost i.e. the cost of procurement, can be apportioned on the basis of net sales value i.e. sale proceeds less cost of further processing. This is NRV method.

2.The second part of the question is very easy. The additional revenue of product CD is Rs. 7/ kg i.e. Rs. 12,000 x 7 = Rs. 84,000 while the processing cost is Rs. 120,000. Thus it is advisable to sell the product CD without further processing.

Page 198: cost accounting question icwai inter

Net realisable value 360 520 720 340Apportionment of joint cost in 36:52:72:34 278 402 557 263Total cost 278 482 677 323Profit ( NRV -- Total cost) 82 38 43 17

Rs.'000Part B : Sales value if sold with further processing ( Rs. 70 x 12,000) 840

Sales value if sold without further processing ( Rs. 63 x 12,000) 756Additional revenue generated because of further processing 84Cost of further processing 120

X-- 186 tons @ Rs. 1,500 per ton; Y-- 527 tons @ Rs. 1,125 per ton and Z--736 tons @ Rs. 750 per ton.

X -- 180 tons, Y -- 60 tons and Z -- 25 tons and there was no opening and closing work-in-progress.

Required:

(a) Net realizable value NRV method of joint cost allocation.(b) Constant gross profit margin percentage NRV method of joint cost allocation.

2. Compare the gross-margin percentages for X,Y and Z using two methods in requirement 1 above.Solution:

As the cost of further processing is more than the additional revenue by Rs. 36,000, it is advisable to sell the product CD at the split off point without further processing.

Problem 5 (CA PE-II May 03)

ABC Ltd. operates a simple chemical process to convert a single material into three separate items, referred to here as X,Y and Z. All three end products are separated simultaneously at a single split-off point.

Product X and Y are ready for sale immediately upon split off without further processing or any other additional costs. Product Z, however, is processed further before being sold. There is no available market price for Z at split off point.

The selling prices quoted here are expected to remain the same in the coming year. During 2002--03 the selling prices of the items and the total amounts sold were:

The total manufacturing costs for the year were Rs. 6,25,000. An additional Rs. 3,10,000 was spent to finish product Z.

There were no opening inventory of X.Y or Z. At the end of the year, the following inventories of complete units were on hand:

1. Compute the cost of inventories of X,Y and Z for balance sheet purposes and cost of goods sold for income statement purpose as of March 31, 2003 using:

Revise the theory portion of allocation of joint cost by NRV method and by constant gross profit margin percentage method, before going through the solution.

(i)         Average unit cost method : - Under this method total process cost (upto the point of separation) is divided by total units of joint products produced. On division average cost per unit of production is obtained.

(ii)       Contribution margin method :- According to this method joint costs are segregated into two parts – variable and fixed. The variable cost are apportioned over the joint products on the basis of units produced (average method) or physical quantities and fixed costs are apportioned on the basis of contribution marginal.

(iii)      Market value of the point of separation : - This method is used for the apportionment of joint costs to joint products upto the split off point. It is difficult to apply this method if the market value of the products at the point of separation are not available. It is a useful method where further processing costs are incurred disproportionately.

(iv)      Market value after further processing : - Here the basis of apportionment of joint cost is the total sales value of finished products. The use of this method is unfair where further processing costs after the point of separation are disproportionate or when the joint products are not subjected to further processing.

Page 199: cost accounting question icwai inter

The solution goes as follows:

Statement showing the apportionment of joint costs on the basis of Net realizable value:Particulars X Y ZSales tons 186 527 736Closing stock tons 180 60 25Production tons 366 587 761Selling price Rs./ton 1,500 1,125 750Total realization value Rs. 549,000 660,375 570,750Less cost of further processing Rs.. 0 0 310,000Net realisable value Rs. 549,000 660,375 260,750Joint cost allocated* Rs. 233,399 280,748 110,854 625,000* on the ratio of 549,000 : 660,375 : 260,750. The joint cost is Rs. 625,000.Statement showing the cost of goods sold (by net realisable value method)Particulars X Y Z Total

Rs. Rs. Rs. Rs.Allocated joint cost 233,399 280,748 110,854 Err:522Additional cost 0 0 310,000 Err:522Cost of goods available for sale (CGAS) 233,399 280,748 420,854 Err:522Less cost of closing inventory* 114,786 28,697 13,826 Err:522Cost of goods sold 118,612 252,051 407,028 Err:522*Cost of goods sold: X = 233,399 x ( 180/366); Y = 280,748 x (60/587) and Z = 110,854 x (25/761)

Income Statement Statement showing gross margin and gross margin percentage

By using net realisation value methodParticulars X Y Z TotalSales in tons 186 527 736Selling price in Rs. /ton 1,500 1,125 750Sales revenue in Rs. 279,000 592,875 552,000Less: Cost of goods sold 118,612 252,051 407,028 Err:522Gross Margin Rs. 160,388 340,824 144,972 Err:522Gross Margin % 57.50% 57.49% 26.26%Calculation for gross profit margin percentage:Particulars X Y Z Total

Rs. Rs. Rs. Rs.Final sales value of production 549,000 660,375 570,750 17,80,125Less Joint cost and additional cost ( 625,000 + 310,000) 9,35,000Gross profit margin 8,45,125Gross profit margin as % 47.47%Statement of joint cost allocation of inventories of X,Y and Z for Balance sheet purposesUsing constant gross margin percentage net realizable methodParticulars X Y Z Total

Rs. Rs. Rs. Rs.Total realization value 549,000 660,375 570,750 17,80,125Gross profit margin % 47.47% 47.47% 47.47%Gross margin 260,610 313,480 270,935 8,45,025Cost of goods sold 288,390 346,895 299,815 9,35,100

(v)       Net realizable value method : - From the sales value of the joint products (at finished stage) following are deducted :

a)     estimated profit margins b)     selling and distribution expenses, if any, c)     post – split off costs.

The resultant figure so obtained is known as net realizable value of joint products. Joint costs are apportioned in the ratio of net realizable value.

Page 200: cost accounting question icwai inter

Less addition cost 0 0 310,000 3,10.000Joint cost allocated 288,390 346,895 (10185) 625,100Comment :The joint cost total comes to Rs. 625,100 in place of Rs. 625,000; this is due to approximation.The negative joint cost allocated to product Z, illustrates unusual feature of Nrv method.Statement showing cost of goods sold for income statement purposesBy using constant gross profit margin percentage net realisation method (in Rs.)Particulars X Y Z TotalAllocated joint cost 288,390 346,895 (10185) 625,100Additional cost 0 0 310,000 310,000Cost of goods available for sale 288,390 346,895 299,815 Err:522less Cost of closing stock * 141,831 35,458 9,849 Err:522Cost of goods sold 146,559 311,437 289,966*X =288,390 x(180 /366) ; Y = 346,895 x ( 60 / 587) and Z = 299,815 x (25 / 761)

Income Statement Statement showing gross margin and gross margin percentageBy using constant gross profit margin percentage net realisation method In Rs. Particulars X Y Z TotalSales revenue 279,000 592,875 552,000 14,23,875Less cost to goods sold 146,559 311,437 289,966 Err:522Gross margin 132,441 281,438 262,034 Err:522Gross margin percentage 47.47% 47.47% 47.47%Comparative statement of goods margin percentage of X,Y and ZMethod Gross margin percentages

X Y ZNet realisable value 57.50% 57.49% 26.26%Constant gross margin percentage of NRV. 47.47% 47.47% 47.47%

(i) Chocolate powder liquor base and (ii) Mix-chocolate liquor base.

Production and sales data for October, 2004 are:Cocoa beans processed 7,500 poundsCost of processing Cocoa beans to split off point Rs. 712,500

Production Sales Selling price Rs./poundChocolate powder 3,000 pounds 3,000 pounds 190.0Milk powder 5,100 pounds 5,100 pounds 237.5

Required:

Problem 6 (CA PE-II Nov.04)

Pokemon Chocolates manufactures and distributes chocolate products. It purchases cocoa beans and processes them into two intermediate products:

These two intermediate products become separately identifiable at a single split off point. Every 500 pounds of cocoa beans yields 20 gallons of chocolate-powder base and 30 gallons of milk-chocolate liquor base.

The chocolate powder liquor base is further processed into chocolate powder. Every 20 gallons of chocolate-powder liquor base yields 200 pounds of chocolate powder. The milk-chocolate liquor base is further processed into milk-chocolate. Every 30 gallons of milk-chocolate liquor base yields 340 pounds of milk chocolate.

The October,2004 separable costs of processing chocolate-powder liquor into chocolate powder are Rs. 302,812.50. The October 2004 costs of processing milk-powder liquor base into milk chocolate are Rs. 623,437.50.

Pokemom fully processed both of its intermediate products into chocolate powder or milk chocolate. There is an active market for these intermediate products. In October 2004, Pokemon could have sold the chocolate powder liquor base for Rs. 997.50 a gallon and the milk-chocolate liquor base for Rs. 1,235 a gallon.

Page 201: cost accounting question icwai inter

(a) Sales value method (b) Physical measure (c) Estimated net realisable value NRV(d) Constant gross margin percentage NRV

Solution: All figures are in Rs. Unless otherwise stated. Decimals in costs are omitted.

2. Revise your knowledge of allocation of joint costs before you try to solve this problem.

Statement showing the allocation of joint cost to CPLB and MCLB:

First of all let us calculate the quantity of CPLB and MCLB to be produced by 7,500 pounds of cocoa beans

Rate : 20 gallons of CPLB and 30 gallons of MCLB for every 500 pounds of cocoa beans.7,500 pounds will yield 300 gallons of CPLB and 450 gallons of MCLB. Allocation is as follows:Sales value method Qty Rate Rs. Sales value Allocation*CPLB 300 997.5 299,250 249,375MCLB 450 1235 555,750 463,125* Joint cost Rs. 712,500 in the ratio 299250:555750 between CPLB and MCLB.

Joint cost of Rs. 712,500 to be allocated to CPLB and MCLB in the ratio of 300:450 or 2:3Product Qty . Allocation of Rs. 712,500CPLB 300 285,000MCLB 450 427,500Net realisable value method: units Chocolate powder Milk chocolateQty produced in pounds Pounds 3,000 5,100

Rs. Rs.Selling price in Rs./pound 190 237.5Sales value of production 570,000 1211250Less: Separable cost 302,813 623,438Net realisable value at split off point 267,188 587,813Allocated joint cost in the ratio of NRV. 222,656 489,844

Total sales value of production =Rs. 5,70,000 + Rs. 12,11,250 17,81,250Total cost (joint + separable) = Rs. 712,500 + 302,813 + 623,838 16,38,750Gross margin ( Sales -- cost ) 142,500Gross margin % on sales (142,500 / 17,81,250) 8%Allocation of joint cost is as follows: Chocolate powder Milk chocola Total

Rs. Rs. Rs.Final sales value of production 570,000 1211250Less Gross margin @ 8% 45,600 96,900Cost of goods sold 524,400 1114350Less :Separable cost after split off 302,813 623,437Joint cost allocated 221,587 490,913 Err:522(ii) Gross margin % of CPLB and MCLB:

Chocolate Powder Liquid Base Fig. in Rs. Decimals omittedMethods Sv Physical NrvFinal sales value of production 570,000 570,000 570,000

1. Calculate how the joint cost of Rs. 7,12,500 would be allocated between the chocolate powder and milk-chocolate liquor bases under the following methods:

2. What is the gross-margin percentage of the chocolate powder and milk-chocolate liquor bases under each of the methods in requirement 1.

3. Could Pokmon have increased its operating income by a change in its decision to fully process both of its intermediate products? Show your computations.

Tutorial Notes : 1.This is very easy question. Although it takes time to go through it, Students are advised to note meaningful data while going through lengthy problem such as this.

Physical measure method: Joint cost to be allocated in the ratio of output.

Gross margin method: Let us first calculate the gross margin of the process.

This is simply rearrangement and tabulation of results obtained in the four methods of allocation of joint costs as narrated method. This is explained as given below:

Page 202: cost accounting question icwai inter

Less: Separable cost decimal omitted) 302,812 302,812 302,812Less; Joint cost allocated 249,375 285,000 222,656Gross margin ( Sales -- cost ) 17,813 (17812) 44,532Gross margin % Margin/sales 3.1% -3.1% 7.8%Milk powder liquid base:Final sales value of production 1211250 1211250 1211250Less: Separable cost decimal omitted) 623,438 623,438 623,438Less; Joint cost allocated 463,125 427,500 489,844Gross margin ( Sales -- cost ) 124,687 160,312 97,968Gross margin % Margin/sales 10.3% 13.2% 8.1%(iii) Analysis of further processing:

CPLB MCLBIncremental revenue ( 570,000 -- 997.5 x 300) 270,750 655,500

(12,11,250 -- 1235 x 450 )Incremental cost of processing 302,812 623,438Incremental operating income (32062) 32,062

A B XQuantity sold kgs. 17,000 5,000 44,000Sales value Rs. 1224000 250,000 792,000

Required(i) Prepare a statement showing the apportionment of joint costs to A and B.

(iii) Prepare a statement showing the product wise and total profit for the period.

(iv) State with supporting calculations as to whether any or all of the products should be further processed or not.

Solution: Question is easy. Try to solve it yourself.

Statement showing the apportionment of joint costs to A,B and X by Sales value methodProducts A B C TotalOutput in kgs 18,000 10,000 54,000Selling price Rs. 50 40 10Sales value Rs. 900,000 400,000 540,000 1840000Joint cost allocation ( In the ratio 90:40:54) 630,000 280,000 378,000 1288000

In view of the above observation, it is evident that CPLB should not be processed beyond split off point: it should be sold at split off point. MCLB should be processed further to generate incremental operating income of Rs. 32,062.

Problem 7 (ca inter.Nov.96)

A company processes a raw material in its Department 1 to produce three products A,B and X at the same split off stage. During a period 180,000 kgs of raw material was processed in Department 1 at a total cost of Rs. 12,88,000 and the resultant output of A,B and X were 18,000 kgs, 10,000 kgs and 54,000 kgs respectively. A and B were further processed in Department 2 at a cost of Rs. 180,000 and Rs. 150,000 respectively.

X was further processed in Department 3 at a cost of Rs.108,000. There is no waste in further processing. The details of sales effected during the period were as under:

There were no opening stocks. If these products were sold at split-off stage, the selling price of A, B and X would have been Rs. 50, Rs. 40 and Rs. 10 per kg. respectively.

(ii) Present a statement showing the cost per kg of each product indicating joint cost and further processing cost and total cost separately.

The apportionment of joint cost is suitably done on sales value method. It should be noted that actual sales value will not be used for apportionment purposes but the sales value of production would be used for this purpose. Thus the quantity actual sold of A,B and X of 17,000 kg, 5,000 kg and 44,000 kg will not be used for apportionment of joint cost but the actual quantity of production will be used.

Page 203: cost accounting question icwai inter

Joint cost per kg. in Rs. 35 28 7Cost of further processing Rs. 180,000 150,000 108,000Cost of further processing per kg. Rs. 10 15 2Total cost per kg. in Rs. 45 43 9

Products units A B COutput in kgs kg. 18,000 10,000 54,000Less: Quantity sold kg. 17,000 5,000 44,000Closing stock kg. 1,000 5,000 10,000Total cost per kg. in Rs. Rs. 45 43 9Valuation of closing stock in Rs. Rs. 45,000 215,000 90,000Sales value (given directly) Rs. 1224000 250,000 792,000Cost of production from previous statement Rs. 630,000 280,000 378,000Cost of further processing Rs. 180,000 150,000 108,000Total cost Rs. 810,000 430,000 486,000Profit : (sales + closing stock - cost) Rs. 459,000 35,000 396,000(iv) Justification for further processing:Sales value after further processing Rs. 1224000 250,000 792,000Sold quantity in kg. kg. 17,000 5,000 44,000Selling price in Rs.per kg. Rs./kg 72 50 18Selling price at the point of split-off Rs./kg 50 40 10Incremental sales revenue for further processing Rs./kg 22 10 8Less: Cost of further processing Rs./kg 10 15 2Incremental profit (loss) in Rs. Per kg. Rs./kg 12 (5) 6Product B should not be processed further while A and X should be processed further.

The April production and sales information are as follows:Production tons Sales tons Selling price per ton.

Caustic Soda 1,200 1,200 1,250Chlorine 800PVC 500 500 5,000

Required:

1. Sales value at split-off;2. Physical measure (ton);3. Estimated net realisable value.

(ii) What is gross margin percentage of Caustic Soda and PVC under these methods cited in (i) above ?

(iii) In this part, the product wise profit is to be computed. Note that entire quantity of production is not sold, thus there must be some closing stock left which is to be evaluated. The quantity of closing stock should be found out and then it should be multiplied with the total cost per kg. This will give you the valuation of closing stock. Do it yourself first.

Problem 8 (ca inter.may.2000)

Inorganic Chemical purchases salt and processes it into more-refined products such as caustic soda, chlorine and PVC. During the month of April, 2000, Inorganic Chemicals purchased salt for Rs. 10,00,000. Conversion cost of Rs. 15,00,000 were incurred upto the split-off point, at which time two saleable products were produced : Caustic soda and Chlorine. Chlorine can be further processed into PVC.

All 800 tons of chlorine were further processed, at an incremental cost of Rs. 500,000 to yield 500 tons of PVC. There were no byproducts or scrap from this further processing of chlorine. There were no beginning or ending inventories of caustic soda, chlorine or PVC in April.

There is an active market for chlorine. Inorganic chemicals could have sold all its April production of chlorine at Rs. 1,875/ton.

(i) Calculate, how the joint costs of Rs. 25,00,000 would be allocated between Caustic soda and Chlorine under each of the following methods;

Page 204: cost accounting question icwai inter

Solution: Easy question. Do it yourself.(i) 1. Joint Cost Allocation To Caustic Soda and Chlorine : Sales value at split-off method

Caustic SodaSales in tons 1,200Selling price in Rs. Per ton 1,250Sales value in Rs. In lacs 15.0Allocation of joint cost Rs. 25 lacs in the ratio 1:1 12.5(i) 2. Joint Cost Allocation To Caustic Soda and Chlorine : Physical measure methodSales in tons 1,200Allocation of joint cost Rs. 25 lacs in the ratio 12:8 15(i) 3. Joint Cost Allocation To Caustic Soda and Chlorine : Net realizable value methodSales in tons 1,200Selling price in Rs. Per ton 1,250Sales value in Rs. In lacs 15.0Less : Further processing cost in Rs. Lacs 0Net realisable value at split-off point 15Allocation of joint cost Rs. 25 lacs in the ratio 15:20 10.71(ii) Statement of Gross margin % of Caustic soda and PVC By The Three MethodsCaustic Soda Sales value at split-off Physical measure NRV.

Sale Rs. In lacs 15.00 15.00 15.00Less: Cost at split-off 12.50 15.00 10.71Gross margin Rs. 2.50 0.00 4.29Gross margin % 17% 0% 29%PVCSale Rs. In lacs 25.00 25.00 25.00Less: Cost at split-off 12.50 10.00 14.28Less: Further processing cost 5.00 5.00 5.00Gross margin Rs. 7.50 10.00 5.72Gross margin % 30% 40% 23%(iii) Justification of further processing:

Ascertain the joint cost per unit.Solution : Statement of ascertaining the joint cost per unit of P and Q.

P QRs./ unit

Selling price per unit Rs. 13.75 8.75Less: margin @ 25% on cost or 20% on sales 2.75 1.75Cost of sales 11.00 7.00Less: Post spilt off cost 5.00 4.00Pre-split off net joint cost per unit 6.00 3.00No. of units of output units 8,000 6,000

(iii) Life Time Swimming Pool Products offer to purchase 800 tons of Chlorine in May, 2000 at Rs. 1,875 a ton. This sale would mean that no PVC would be produced in May. How would accepting the offer affect May Operating Income ?

It is very clear that sales value at split-off is Rs. 15 lacs (Rs. 1,875 x 800) and that after processing (with a cost of Rs. 5 lacs) is Rs. 25 lacs. Thus further processing must be undertaken because it will yield an incremental operating income of Rs. 5 lacs.

Problem 9 (ca inter.may.98)

Two products P and Q are obtained in a crude form and require further processing at a cost of Rs. 5 for and Rs. 4 for Q per unit before sale. Assuming a net margin of 25% on cost, their sale prices are fixed at Rs. 13.75 and Rs. 8.75 per unit respectively. During the period, the joint cost was Rs. 88,000 and the outputs were: P : 8,000 tons and Q : 6,000 tons.

Page 205: cost accounting question icwai inter

Total cost of output Rs. Rs. 48,000 18,000The joint cost apportionment in the ratio of 48:18Apportionment of Rs. 88,000 in the ratio 48:18 Rs. 64,000 24,000Ascertained joint cost per unit Rs./unit 8.00 4.00

Products M N O POutput in Gallons 300,000 100,000 50,000 50,000

The joint cost of purchasing the crude vegetable oil and processing it were Rs. 40,00,000.

Further processing cost to make super products Sales Rs. Rs.

Super M 80,00,000 120,00,000Super N 32,00,000 40,00,000Super P 36,00,000 48,00,000

Product M Product N Product PRs. 20,00,000 12,00,000 28,00,000

You are required to answer :

Solution : Very easy. See solution to Problem 8. Use figures in lacs to save time.Allocation of joint cost M N O PSales value at split off method 10 6 10 14Physical measure method: 24 8 4 4NRV method 20 40 10 6Profit (loss) due to further processing 20 (4) (16)

You are required to prepare the following in respect of the coming year:

(i) Statement showing income forecast of the company assuming that none of its products are to be further processed.

(i) Statement showing income forecast of the company assuming that products A,B,C and E are to be further processed.

Problem 10 (ca inter.nov.99)

The Sunshine Oil Company purchases crude vegetable oil. It does refining of the same. The refining process results in four products M, N, O and P. In the most recent month (October 1999), the output at split off point was :

Sunshine has no beginning or ending inventories. Sales of product O in October was Rs.20,00,000. Total output of product M,N and P was further refined and then sold. Data related to October, 1999 are as follows:

Sunshine had the option of selling products M,N and P at the split-off point. This alternative would have yielded the following sales for the October 1999 production;

(i) How the joint cost of Rs. 40,00,000 would be allocated between each product under each of the following methods (a) Sales value at split off (b) Physical output (gallon) and (c) estimated net realisable value ?

(ii) Could Sunshine have increased its October 1999 operating profits by making different decisions about the further refining of the product M,N or P ? Show the effect of any change you recommend on operating profit ?

Problem 11 (ca inter.nov.97)

Sunmoon Ltd. produces 200,000; 30,000 ; 25,000 ; 20,000 and 75,000 units of its five products A,B,C,D and E respectively in a manufacturing process and sells them at Rs.17, Rs. 13, Rs. 8, Rs. 10 and Rs. 14 per unit. Except product D, remaining products can be further processed and then can be sold at Rs. 25, Rs. 17, Rs. 12 and Rs. 20 per unit in case of A,B,C and E respectively.

Raw material costs Rs. 35,90,000 and other manufacturing expenses cost Rs. 5,47,000 in the manufacturing process which are absorbed on the products on the basis of their Net realisable value. The further processing costs of A,B,C and E are Rs. 12,50,000; Rs. 1,50,000, Rs. 50,000 and Rs. 1,50,000 respectively. Fixed costs are Rs.4,73,000.

(iii) Can you suggest any other production plan whereby the company can maximize its profits. If yes, then submit a statement showing income forecast arising out of adoption of that plan.

Page 206: cost accounting question icwai inter

Solution: Note that fixed cost is not to be apportioned.Total joint cost to be apportioned = Rs. 35,90,000 + Rs. 5,47,000 = Rs. 41,37,000.

A B C D EProduction in units 200,000 30,000 25,000 20,000 75,000After processing selling price 25 17 12 10 20Final sales value of production 50.0 5.1 3.0 2.0 15.0Less: Further processing cost 12.5 1.5 0.5 0.0 1.5Net Realisable Value 37.5 3.6 2.5 2.0 13.5Apportionment of joint cost of Rs. 41.37 lacs in the ratio of 37.5 : 3.6 : 2.5 : 2.0 : 13.5Apportioned Joint Cost 26.25 2.52 1.75 1.40 9.45

Sunmoon Ltd.Statement showing income forecast when no product is processed further.

A B C D EProduction in units 200,000 30,000 25,000 20,000 75,000Selling price at split-off point Rs./ unit 17 13 8 10 14Sales revenue Rs. Lacs 34.00 3.90 2.00 2.00 10.50Apportioned joint cost 26.25 2.52 1.75 1.40 9.45Excess sales revenue 7.75 1.38 0.25 0.60 1.05Total excess sales revenue 11.03Less Fixed cost 4.73Profit Rs. Lacs 6.30Statement showing income forecast when products A,B,C and E are processed further.

A B C D ERs. Lacs Rs. Lacs Rs. Lacs Rs. Lacs Rs. Lacs

Final sales value of production 50.0 5.1 3.0 2.0 15.0Less: Further processing cost 12.5 1.5 0.5 0.0 1.5Net Realisable Value 37.50 3.60 2.50 2.00 13.50Less : joint cost apportioned 26.25 2.52 1.75 1.4 9.45Excess sales revenue over cost 11.25 1.08 0.75 0.60 4.05Total excess sales revenue 17.73Less: Fixed cost 4.73Profit 13.00Suggested Production plan for maximizing profitExcess sales revenue over cost without and with processing is given below

A B C D ERs. Lacs Rs. Lacs Rs. Lacs Rs. Lacs Rs. Lacs

Without further processing 7.75 1.38 0.25 0.6 1.05With Further processing 11.25 1.08 0.75 0.6 4.05

Production plan for maximization of profit when B and D are not processed furtherA B C D E

Production in units 200,000 30,000 25,000 20,000 75,000Selling price 25 13 12 10 20Sales revenue Rs. In lac. 50 3.9 3 2 15Less: further processing cost 12.50 0.00 0.50 0.00 1.50Less: Joint cost apportioned 26.25 2.52 1.75 1.40 9.45Excess sales revenue 11.25 1.38 0.75 0.60 4.05Less: Fixed cost ProfitBest production plan for maximization of profit:Products A,C and E should be sold after further processing and Products B and D should be sold at split-off point.

Above table shows that further processing reduces the sales revenue of product B by an amount of Rs. 30,000. Thus product B should not be processed further but should be sold at split-off point.

Problem 12 (ca inter.may 02)

Page 207: cost accounting question icwai inter

Cost of material introduced in department P : Rs. 12,68,800Direct wage department P Q R Amount in Rs. 384,000 96,000 64,000

Factory overheads of Rs. 464,000 are to be apportioned to the department on direct wage basis.

During the month under reference, the company sold all three products after processing them further as under:

Products A B COutput sold kg. 44,000 40,000 20,000Selling price per kg Rs. 32 24 16

Required:

(i) Prepare a statement showing the apportionment of joint cost of joint products.

(iii) What processing decision should have been taken to improve the profitability of the company.(iv) Calculate the product-wise and total profit arising from your recommendation as above.Solution:

Joint cost in department P = Cost in P + Factory overheads apportionmentJoint cost = Rs. 12,68,000 + Rs. 3,84,000 + Rs. 3,07,200 = Rs. 19,60,000.Answer is given below. Solve it yourself. A B CJoint cost apportioned Rs. 880,000 880,000 200,000Profit as per present policy Rs. 355,200 (35,200) 55,200Profit if B is not processed further Rs. 355,200 0 55,200

Item Direct materials Direct labour Variable overhead Fixed ohs.Cost in Rs. 90,000 120,000 100,000 100,000

Will you recommend further processing ?Solution:

Item Material Labour Voh Foh Total process costCost in Rs. 90,000 120,000 100,000 100,000 Err:522Add : Raw materials cost of 150,000 kg @ Rs. 12 1800000

In a chemical manufacturing company, three products A,B and C emerge at a single split off stage in department P. Product A is further processed in department C, Product B in department Q and product C in department R and department Service. There is no loss in further processing of any of the three products. The cost data for the month are as under:

There are no opening or closing stocks. If these three products were sold at the split off stage, that is, without further processing, the selling price would have been Rs. 20, Rs. 22 and Rs. 10 each per kg respectively for A,B and C.

(ii) Present a statement showing product-wise and total profit for the month under reference as per the company's current processing policy.

Tutorial Notes: The fundamental question is ; What is the joint cost to be apportioned ? Think over it. Joint cost to be apportioned is to be computed first. Think how it can be computed. Rest of the question is easy.

Problem 13 (ca inter.)

A company plants processes 150,000 kg of raw materials in a month to produce two products P and Q.T he cost of raw materials is Rs. 12 per kg. The process cost per month are :

The loss in the process is 5% of the input and the output ratio of P and Q which emerge simultaneously is 1:2. The selling price of the two products at the point of split off are P : Rs. 12/kg and Q Rs. 20 per kg. A proposal is available to process entire quantity of current output of P into S. The price per kg of S is Rs. 15 and each kg. of S will require one kg. of P. The cost of processing of P into S including other materials is Rs. 185,000 per month.

You are required to prepare a statement showing the monthly profitability based both on the existing manufacturing operations and on further processing.

Joint cost has not been given directly. It is to be computed first. It is total cost upto the split-off point. It is sum of raw materials cost and process cost.

Page 208: cost accounting question icwai inter

Total joint cost in Rs. 22,10,000

Total input quantity of raw materials in kg. 150,000Less: Loss in the process @ 5%. 7,500Total output ( to be distributed in 1:2 for obtaining out of P and Q) 142,500Quantity of output of P (1/3 rd of 142,500) 47,500 Sales value @ Rs. 12 570,000Quantity of output of Q (2/3 rd of 142,500) 95000 Sales value @ Rs. 20 1900000Apportionment of joint cost of Rs. 22.10 lacs in the ratio 57:190 between P and Q.Joint cost apportioned to P (3/13 of Rs. 22,10,000) 510,000Joint cost apportioned to Q (10/13 of Rs. 22,10,000) 1700000Statement showing profitability of existing operations ( No further processing)

P QOutput quantity in kg. 47,500 95,000Selling price in Rs. 12 20Sales value Rs. 570,000 1900000Less Joint cost 510,000 1700000Profit 60,000 200,000Evaluation of further processing of P into S:Incremental revenue of S is Rs. 3/kg (Rs. 15 -- Rs. 12) Rs.Total incremental revenue of 47,500 kg @ Rs. 3/kg. 142,500Total incremental cost of processing 47,500 kg 185,000Loss due to further processing in Rs. 42,500

The existing profit is Rs. 60,000 for product P. Further processing results in loss of Rs. 42,500 hence not recommended.

Products A B CSelling price at split-off point Rs./liter 100 120 150Selling price after further processing Rs./liter 200 200 250Post separation point cost Rs. 350,000 450,000 200,000Output liters 3,500 2,500 2,000

You are required to :

(ii) from the profit statement comment how profit could be maximized if one or more products are sold at split-off points.

Solution : Joint cost is Rs. 240,000, to be apportioned to A,B and C in 35:25:20.Profit statement at split off point (without further processing): Rs.'000

A B C TotalSelling price at split-off point Rs./liter 100 120 150Output liters 3,500 2,500 2,000Sales revenue at split off point 350 300 300Apportioned joint cost in 35:25:20 105 75 60 240Profit 245 225 240 710The overall profit without further processing is Rs. 710,000.

The next step is to look for the basis of allocation of joint cost to individual products. The obvious basis is weight of output but it is unsuitable because the selling prices of the products are quite different, i.e. Rs. 12/kg for P and Rs. 20/kg for Q. The next basis is sales value basis which is appropriate.

To calculate sales value of P and Q, the output quantities of both the products must be known. Let us find that out as follows:

Problem 14 (cwa final June 06)

A company produces three joint products in one common process. Each product can be separately processed further after split-off point. The estimated data for a particular month are as under :

Pre-separation point joint costs are estimated to be Rs. 240,000. As per current practice such costs are apportioned to the three products according to production quantity.

(i) Prepare a statement of estimated profit or loss for each product and in total for the month if all the three products are processed further, and

Page 209: cost accounting question icwai inter

Profit statement with further processing of all the three products

Products A B C Rs.'000Selling price after further processing Rs./liter 200 200 250Output liters 3,500 2,500 2,000Sales revenue 700 500 500 1,700Post separation point cost 350 450 200 1,000Apportioned joint cost in 35:25:20 105 75 60 240Profit / (Loss) 245 (25) 240 460

Whether to process further or not : figures in Rs.'000

A B C TotalProfit / (Loss) with further processing 245 (25) 240 460Profit without further processing 245 225 240 710Inference : Further processing is not advised. It results in the overall loss of Rs. 250,000.

The company used 10,000 kgs of petals last month.The costs involved in reducing the petals into 'Charm' and 'Flower oil' were as follows:Direct materials Rs. 150,000 Direct labour :Rs. 90,000 and Indirect cost Rs. 60,000 The costs of producing 'Wild Scent' and 'Personality' from the flower oil were :Direct materials : Rs. 5,000 ; Direct labour : Rs. 10,000 and Indirect costs : Rs. 5,000.Total production for the month, with no closing work-in-progress was:Charm : 250 liters; Wild Scent : 500 liters and Personality : 1250 liters.

Additional costs, entirely separate for each product of processing and selling are :Charm : Rs. 70,000 ; Wild Scent : Rs. 160,000 and Personality : Rs. 40,000.You are required :

(ii) to prepare a product-line income statement, assuming no opening or closing stocks.Solution: Joint costs to be allocated to the three products consist of

Direct materials Rs. 150,000Direct labour Rs. 90,000Indirect Costs Rs. 60,000Total 300,000

Charm 250 liters cost Rs. 70,000 Sales value Rs. 400,000

Costs Rs. 300,000Wild scent cost Rs. 160,000 SV Rs. 200,000

Flower oil cost Rs. 20,000

Personality 1250 litersCosts Rs. 40,000 SV Rs. 50,000

Relative sales value at separation point 1: Rs. Rs.Charm Rs. 400,000 -- Rs. 70,000 330,000Flower Oil Wild Scent Rs. 200,000 -- Rs. 160,000 40,000

Personality Rs. 50,000 -- Rs. 40,000 10,000

Problem 15 (cwa final June 07)

The Petal Perfume company processes a secret blend of flower-petals into three products. The process works in such a way that the petals are broken down into a high-grade perfume, 'Charm' and a low-grade flower oil. The flower oil is then processed into a low-grade perfume, 'Wild Scent' and a Cologne 'Personality'.

The sale price of Charm is Rs. 40 for 25 ml, that of Wild Scent is Rs. 10 for 25 ml and that of Personality is Rs. 1 per 25 ml.

(i) to allocate the joint costs of producing Charm, Wild Scent and Personality using appropriate relative sales value method;

S1

S2

S1 is first separation point.

Page 210: cost accounting question icwai inter

Less cost of flower oil 20,000 30,000

Joint cost to be apportioned to Charm Rs. 275,000Joint cost to be apportioned to Flower oil Rs. 25,000S2 is second separation point.

Joint cost apportioned to Wild Scent Rs. 36,000Joint cost apportioned to Personality Rs. 9,000Product wise Income Statement : fig. in Rs. Product Charm Wild Scent Personality TotalSales revenue 400,000 200,000 50,000 Err:522Apportioned Joint cost 275,000 36,000 9,000 Err:522Own cost 70,000 160,000 40,000 Err:522Total cost 345,000 196,000 49,000 Err:522Net Income 55,000 4,000 1,000 Err:522

When life gives you thousands reasons to cry, show the world that you have million reasons to smile.

Thus the joint cost of Rs. 300,000 to be apportioned to Charm and Flower oil in the ratio of 300,000 : 30,000.

At second separation point the total joint cost is Rs. 20,000 + Rs. 25,000 (apportioned from first separation point) = Rs. 45,000 which is to be apportioned to Wild scent and Personality in the ratio of net realisable values viz. 40,000:10,000.

Page 211: cost accounting question icwai inter

All the three products are processed further after separation and sold as per details given below:

By product C80,00010,000

15

gj vuqHko ,d ldkjkRed vuqHko gS vxj eSa mls rjDdh djus ;k vius vkidks

It is advised to depict the joint process by way of a leveled diagram. The diagram should contain quantity input, its rate and value in Rs., the split off point, and all details in respect of further processing. The diagrammatic

Modern Mill Ltd. Manufactures certain grades of products known as A,B and C. During manufacture of product A, by

2.If the product is not subject to further processing and is sold at the point of separation, for which there is a market at Rs.100,440 without incurring any selling expenses. Would you advise its disposal at this stage? Show the workings

You have to do certain thinking before solving the problem. Some hints are supplied here. Sales less profit is cost of sales. 10% of cost of sales is Selling expense which should be apportioned to three products in the given ratio. Cost of sales less selling expense is cost of production. Cost after split off is already given. Cost of production less cost after split off will be the apportioned joint cost. Please solve this on the above lines and compare your solution with the one

If the product B is sold at the split off point without incurring any selling expenses at Rs.100,440, the relevant cost for calculation of profit would be the apportioned cost viz. Rs. 68,440. Thus the profit would be 100,440 - 68,440 i.e. Rs.

Page 212: cost accounting question icwai inter

It is recommended that the product B should be sold at split off point and the product C should be processed further.

Selling priceAfter process.

(c) What is the profit earned if all the main products are sold without further processing ? Give product wise details.

(d) If the further processing is done as suggested in (b), what is the total profit earned ? Give product wise details.

Net gainor loss

A certain raw material on undergoing a chemical process yields three products A, B and C and by product X. The

The fundamental question is ; What is the joint cost to be apportioned ? Think over it. Joint cost to be

Page 213: cost accounting question icwai inter

24,00012,000-5,000

-500

Profit

Rs.46,40030,00011,000

2,50089,900

Rejection rate

Three products A, B and C are produced from a single process. Each product can be sold at the end of each process or can be further processed independently to produce superior products, which are marketed under different names X, Y

1. Calculate the apportionment of costs to products A, B and C using sales value of production as the basis of

2. Explain whether the initial process should be undertaken and also state whether further processing is to be

Page 214: cost accounting question icwai inter

Selling pr.at the sale point

2. If the company finds a market for CD at Rs. 63/kg without further processing, will it be advisable to accept it ?

In a chemical plant four different products viz. AB, BC,CD and DD emerge from the input of crude oil. Product AB can be sold immediately, but the remaining three products require further processing before they can be marketed.

In a month 40,000 liters of crude oil were procured at a cost of Rs. 30 per liter and processed at a cost of Rs. 3 lakhs. The details of output obtained, further processing cost, selling price per unit etc. are given below:

This is an easy question. Students should try to solve it before going through the solution

1.The apportionment of joint cost can not be done on basis of output of the process as the output have different units of measurement and their selling prices after further processing differ considerably. Thus the joint cost i.e. the cost of procurement, can be apportioned on the basis of net sales value i.e. sale proceeds less cost of further

Page 215: cost accounting question icwai inter

Profit ( SV -- Total cost) 82 118 163 77

X-- 186 tons @ Rs. 1,500 per ton; Y-- 527 tons @ Rs. 1,125 per ton and Z--736 tons @ Rs. 750 per ton.

X -- 180 tons, Y -- 60 tons and Z -- 25 tons and there was no opening and closing work-in-progress.

ABC Ltd. operates a simple chemical process to convert a single material into three separate items, referred to here as

Product X and Y are ready for sale immediately upon split off without further processing or any other additional costs. Product Z, however, is processed further before being sold. There is no available market price for Z at split off point.

Revise the theory portion of allocation of joint cost by NRV method and by constant gross profit margin percentage

(i)         Average unit cost method : - Under this method total process cost (upto the point of separation) is divided by total units of joint products produced. On division average cost per unit of production is obtained.

(ii)       Contribution margin method :- According to this method joint costs are segregated into two parts – variable and fixed. The variable cost are apportioned over the joint products on the basis of units produced (average method) or

(iii)      Market value of the point of separation : - This method is used for the apportionment of joint costs to joint products upto the split off point. It is difficult to apply this method if the market value of the products at the point of separation are not available. It is a useful method where further processing costs are incurred disproportionately.

(iv)      Market value after further processing : - Here the basis of apportionment of joint cost is the total sales value of finished products. The use of this method is unfair where further processing costs after the point of separation are

Page 216: cost accounting question icwai inter

The resultant figure so obtained is known as net realizable value of joint products. Joint costs are apportioned in the

Page 217: cost accounting question icwai inter

These two intermediate products become separately identifiable at a single split off point. Every 500 pounds of cocoa

The chocolate powder liquor base is further processed into chocolate powder. Every 20 gallons of chocolate-powder liquor base yields 200 pounds of chocolate powder. The milk-chocolate liquor base is further processed into milk-

Page 218: cost accounting question icwai inter

Decimals in costs are omitted.

First of all let us calculate the quantity of CPLB and MCLB to be produced by 7,500 pounds of cocoa beans

Milk chocolate

Fig. in Rs. Decimals omittedGm nrv

570,000

1. Calculate how the joint cost of Rs. 7,12,500 would be allocated between the chocolate powder and milk-chocolate

2. What is the gross-margin percentage of the chocolate powder and milk-chocolate liquor bases under each of the

3. Could Pokmon have increased its operating income by a change in its decision to fully process both of its

1.This is very easy question. Although it takes time to go through it, Students are advised to note

This is simply rearrangement and tabulation of results obtained in the four methods of allocation of joint costs as

Page 219: cost accounting question icwai inter

302,812221,587

45,6018.0%

1211250623,438490,913

96,8998.0%

(iv) State with supporting calculations as to whether any or all of the products should be further processed or not.

In view of the above observation, it is evident that CPLB should not be processed beyond split off point: it should be sold at split off point. MCLB should be processed further to generate incremental operating income of Rs. 32,062.

A company processes a raw material in its Department 1 to produce three products A,B and X at the same split off stage. During a period 180,000 kgs of raw material was processed in Department 1 at a total cost of Rs. 12,88,000 and the resultant output of A,B and X were 18,000 kgs, 10,000 kgs and 54,000 kgs respectively. A and B were further

X was further processed in Department 3 at a cost of Rs.108,000. There is no waste in further processing. The details of

(ii) Present a statement showing the cost per kg of each product indicating joint cost and further processing cost and

The apportionment of joint cost is suitably done on sales value method. It should be noted that actual sales value will not be used for apportionment purposes but the sales value of production would be used for this purpose. Thus the quantity actual sold of A,B and X of 17,000 kg, 5,000 kg and 44,000 kg will not be used for apportionment of joint cost but the

Page 220: cost accounting question icwai inter

(ii) What is gross margin percentage of Caustic Soda and PVC under these methods cited in (i) above ?

(iii) In this part, the product wise profit is to be computed. Note that entire quantity of production is not sold, thus there must be some closing stock left which is to be evaluated. The quantity of closing stock should be found out and then it should be multiplied with the total cost per kg. This will give you the valuation of closing stock. Do it yourself first.

Inorganic Chemical purchases salt and processes it into more-refined products such as caustic soda, chlorine and PVC. During the month of April, 2000, Inorganic Chemicals purchased salt for Rs. 10,00,000. Conversion cost of Rs. 15,00,000 were incurred upto the split-off point, at which time two saleable products were produced : Caustic soda and

All 800 tons of chlorine were further processed, at an incremental cost of Rs. 500,000 to yield 500 tons of PVC. There were no byproducts or scrap from this further processing of chlorine. There were no beginning or ending inventories of

There is an active market for chlorine. Inorganic chemicals could have sold all its April production of chlorine at Rs.

(i) Calculate, how the joint costs of Rs. 25,00,000 would be allocated between Caustic soda and Chlorine under each of

Page 221: cost accounting question icwai inter

Chlorine800

187515.012.5

80010

5005,000

25.05.020

14.29

(iii) Life Time Swimming Pool Products offer to purchase 800 tons of Chlorine in May, 2000 at Rs. 1,875 a ton. This sale would mean that no PVC would be produced in May. How would accepting the offer affect May Operating Income ?

It is very clear that sales value at split-off is Rs. 15 lacs (Rs. 1,875 x 800) and that after processing (with a cost of Rs. 5 lacs) is Rs. 25 lacs. Thus further processing must be undertaken because it will yield an incremental operating income of

Two products P and Q are obtained in a crude form and require further processing at a cost of Rs. 5 for and Rs. 4 for Q per unit before sale. Assuming a net margin of 25% on cost, their sale prices are fixed at Rs. 13.75 and Rs. 8.75 per unit respectively. During the period, the joint cost was Rs. 88,000 and the outputs were: P : 8,000 tons and Q : 6,000 tons.

Page 222: cost accounting question icwai inter

The joint cost of purchasing the crude vegetable oil and processing it were Rs. 40,00,000.

(i) Statement showing income forecast of the company assuming that none of its products are to be further processed.

(i) Statement showing income forecast of the company assuming that products A,B,C and E are to be further processed.

The Sunshine Oil Company purchases crude vegetable oil. It does refining of the same. The refining process results in four products M, N, O and P. In the most recent month (October 1999), the output at split off point was :

Sunshine has no beginning or ending inventories. Sales of product O in October was Rs.20,00,000. Total output of product M,N and P was further refined and then sold. Data related to October, 1999 are as follows:

Sunshine had the option of selling products M,N and P at the split-off point. This alternative would have yielded the

(i) How the joint cost of Rs. 40,00,000 would be allocated between each product under each of the following methods (a)

(ii) Could Sunshine have increased its October 1999 operating profits by making different decisions about the further refining of the product M,N or P ? Show the effect of any change you recommend on operating profit ?

Sunmoon Ltd. produces 200,000; 30,000 ; 25,000 ; 20,000 and 75,000 units of its five products A,B,C,D and E respectively in a manufacturing process and sells them at Rs.17, Rs. 13, Rs. 8, Rs. 10 and Rs. 14 per unit. Except product D, remaining products can be further processed and then can be sold at Rs. 25, Rs. 17, Rs. 12 and Rs. 20 per

Raw material costs Rs. 35,90,000 and other manufacturing expenses cost Rs. 5,47,000 in the manufacturing process which are absorbed on the products on the basis of their Net realisable value. The further processing costs of A,B,C and E are Rs. 12,50,000; Rs. 1,50,000, Rs. 50,000 and Rs. 1,50,000 respectively. Fixed costs are Rs.4,73,000.

(iii) Can you suggest any other production plan whereby the company can maximize its profits. If yes, then submit a

Page 223: cost accounting question icwai inter

TotalRs. Lacs

11.0317.73

Rs. Lacs

73.9014.5041.3718.03

4.7313.30

Above table shows that further processing reduces the sales revenue of product B by an amount of Rs. 30,000. Thus

Page 224: cost accounting question icwai inter

S 36,000

Factory overheads of Rs. 464,000 are to be apportioned to the department on direct wage basis.

During the month under reference, the company sold all three products after processing them further as under:

(i) Prepare a statement showing the apportionment of joint cost of joint products.

(iii) What processing decision should have been taken to improve the profitability of the company.

In a chemical manufacturing company, three products A,B and C emerge at a single split off stage in department P. Product A is further processed in department C, Product B in department Q and product C in department R and department Service. There is no loss in further processing of any of the three products. The cost data for the month are

There are no opening or closing stocks. If these three products were sold at the split off stage, that is, without further processing, the selling price would have been Rs. 20, Rs. 22 and Rs. 10 each per kg respectively for A,B and C.

(ii) Present a statement showing product-wise and total profit for the month under reference as per the company's

Tutorial Notes: The fundamental question is ; What is the joint cost to be apportioned ? Think over it. Joint cost to be

A company plants processes 150,000 kg of raw materials in a month to produce two products P and Q.T he cost of raw

The loss in the process is 5% of the input and the output ratio of P and Q which emerge simultaneously is 1:2. The selling price of the two products at the point of split off are P : Rs. 12/kg and Q Rs. 20 per kg. A proposal is available to process entire quantity of current output of P into S. The price per kg of S is Rs. 15 and each kg. of S will require one kg.

You are required to prepare a statement showing the monthly profitability based both on the existing manufacturing

Joint cost has not been given directly. It is to be computed first. It is total cost upto the split-off point. It is sum of raw

Page 225: cost accounting question icwai inter

The existing profit is Rs. 60,000 for product P. Further processing results in loss of Rs. 42,500 hence not recommended.

(ii) from the profit statement comment how profit could be maximized if one or more products are sold at split-off points.

The next step is to look for the basis of allocation of joint cost to individual products. The obvious basis is weight of output but it is unsuitable because the selling prices of the products are quite different, i.e. Rs. 12/kg for P and Rs. 20/kg

To calculate sales value of P and Q, the output quantities of both the products must be known. Let us find that out as

A company produces three joint products in one common process. Each product can be separately processed further

Pre-separation point joint costs are estimated to be Rs. 240,000. As per current practice such costs are apportioned to

(i) Prepare a statement of estimated profit or loss for each product and in total for the month if all the three products are

Page 226: cost accounting question icwai inter

Direct labour :Rs. 90,000 and Indirect cost Rs. 60,000

The Petal Perfume company processes a secret blend of flower-petals into three products. The process works in such a way that the petals are broken down into a high-grade perfume, 'Charm' and a low-grade flower oil. The flower oil is then

The sale price of Charm is Rs. 40 for 25 ml, that of Wild Scent is Rs. 10 for 25 ml and that of Personality is Rs. 1 per 25

(i) to allocate the joint costs of producing Charm, Wild Scent and Personality using appropriate relative sales value

Page 227: cost accounting question icwai inter

When life gives you thousands reasons to cry, show the world that you have million reasons to smile.

At second separation point the total joint cost is Rs. 20,000 + Rs. 25,000 (apportioned from first separation point) = Rs. 45,000 which is to be apportioned to Wild scent and Personality in the ratio of net realisable values viz. 40,000:10,000.

Page 228: cost accounting question icwai inter

Chapter 7 : Reconciliation of Financial and Cost Accounts :

Format of Cost SheetA. Consumption of Direct materials

Opening stock of materials Add : (Purchase + Carriage inwards) Less: Closing stock of materials

Material consumedB. Direct Wages : Paid xxx

Less : Opening outstanding xxxAdd : Closing outstanding xxx

C. Direct ExpensesI. Prime Cost :D. Factory Overheads Indirect materials + loose tools + factory rent and taxes + factory lighting + factory insurance+ Power and Fuel + Research and experiment + depreciation of plant + Works Manager's salaryII. Factory Cost or Works Cost : Prime Cost + Factory OverheadsE. Office and Administrative Overheads

Office rent and taxes + Office salaries + Office insurance + Mobiles, phones and postage+ Depreciation on office furniture and equipment + Legal expenses + Audit fees

III . Cost of Production : Factory cost or works cost + Office and Administrative OverheadsIV. Cost of Goods Sold : Cost of Production

+ Opening stock of finished goods − Closing stock of finished goods F. Selling and Distribution Overheads :Showroom rent and tax + Sales man's salaries and commission + Traveling + Advertisement + Bad debts + Debt collection expenses + Carriage outwards etc.V, Cost of Sales : Cost of goods sold + Selling and Distribution Overheads VI. Sales = Cost of Sales + Profit

Don't blame anybody when you receive a setback.Never count on luck for better things in life.Problem 1: (CA Inter May 2006)

Sales 2,500,000 Details of Work-in-progress :Opening stock 0 Materials 30,000Materials 1,000,000 Labour 20,000Wages 500,000 Factory Overheads 20,000Factory Overheads 450,000 Other General ItemsAdministrative Overheads 260,000 Goodwill written offSelling and distribution Overheads 180,000 Interest on Capital Finished goods (1,230 units) 150,000

Prepare a statement reconciling the profit as per cost records with the profit as per financial records.Solution : Tutorial Notes:

·       oks fnu vkidk fcYdqy csdkj x;k ( ftl fnu vki [kqydj gals ugha gksa-

A + B + C Direct Materials + Direct Wages + Direct Expenses

·       vki ifjfLFkfr Hkys gh u cny ik,a ij eufLFkfr rks gj gkyr esa cny gh ldrs gSa-

The following figures are available from the financial records of ABC Manufacturing Company Ltd. for the year 31.3.2006 (figures in Rs.)

In the costing records, factory overhead is charged at 100% wages, administrative overhead is charged 10% of factory cost and selling and distribution overhead at the rate of Rs. 10 per unit sold.

1. You can compute the profit as per financial records in the usual manner by preparing the Profit and loss a/c for the year. You should find the profit before you see any further.

Page 229: cost accounting question icwai inter

2. You must find profit for the year as Rs. 110,000. Check it.

4. You should find the profit as per cost sheet as Rs. 300,000. Check it.5. After finding both the profits you can go ahead with the preparation of reconciliation statement.

Cost Profit as per Cost Records : Figures in Rs. '000Opening stock 0Materials 1,000Wages 500Prime Cost 1,500Add : Factory overheads as 100% of wages 500Factory Cost for the year 2,000

Cost Sheet Rs.'000Opening stock 0Materials 1,000Labour 500Direct Expenses 0Prime cost 1,500Add : Factory Overheads at 100% of wages 500

2,000Less : Cost of Work-in-progress (given) 70Factory Cost of completed units ( 20,000 + 1,230) 1,930Add : Office and Administrative overheads as 10% 193Cost of 21,230 units 2,123Less : Cost of Closing stock of Finished goods ) 123[Cost of 1,230 units is (21,30,000 / 21,230) x 1,230]

3. You should revise your knowledge of cost sheet before you compute the profit as per cost records. Think how can you compute the profit as per cost sheet. You are required to find out the profit on 20,000 units.

6. It is meaningless if you straight away see the following solution. Please prepare your own solution and compare it with the one given below.

Note 1: Actual factory overheads of Rs. 450,000 is not known when cost sheet is prepared. It is charged at 100% of wages. Thus factory overheads charged in cost sheet would be 100% of Rs. 500,000 while the actual is Rs. 450,000. This will Reduce the profit by Rs. 50,000 as the actual factory overhead is less. This is first point of reconciliation.

Note 2: The factory cost of the year is Rs. 20 lac, which is inclusive of cost of work-in-progress and finished goods as well. You are required to find out the cost of 20,000 units sold during the year. You must deduct cost of work-in-progress and finished goods to find out the cost of 20,000 units.

Note 3: You must know that the office and administrative overheads are charged on the units completed during the year. Therefore it is essential that you must include the finished goods in the factory cost. Thus you must deduct the work-in-progress only (and not cost of finished goods) from the factory cost of the year which will give you factory cost of (20,000 + 1,230) units. Charge on this the administrative overheads at 10%, you will find the cost of 21,230 units. From this deduct the cost of finished goods ( Not the given figure of Rs. 150,000), you will find the production cost of 20,000 units.

Note 4 : To production cost of 20,000 units add selling and distribution overheads at Rs. 10 per unit. Cost of sales becomes known here. Sales of 20,000 units is given as Rs. 25 lacs. The difference is profit as per cost sheet.

Note 5 : Both the profits are known. Prepare the reconciliation statement.

( don't take the given figure of Rs. 150,000)

Page 230: cost accounting question icwai inter

Production cost of 20,000 units ( 2,123 − 123) 2,000Add : Selling and Distribution overheads as 10% 200Cost of Sales 2,200Sales 2,500Profit ( Sales − Cost of sales) 300

Profit and loss a/c for the year ended 31.3.06 (Rs.'000)To Opening stock 0 By Sales (20,000 units) 2,500To Materials 1,000 By Closing stockTo Wages 500 Work-in-progress 150To Factory Overheads 450 Finished goods (1,230 units) 70To Office and Administrative Overheads 260To Selling and Distribution Overheads 180To Goodwill written off 200To Interest on capital 20To Profit (balancing figure) 110

2,720 2,720Reconciliation Statement : Rs.'000Profit as per Cost records : 300Add : Factory Overhead As per cost sheet 500

Administrative Overheads As per cost sheet 193Selling Overheads As per cost sheet 200 893

Less : Factory Overhead As per financial records 450Administrative Overheads As per financial records 260Selling Overheads As per financial records 180 (890)

Less: Closing stock As per cost sheet (123)Add : Closing stock As per financial records 150Less : Goodwill written off (200)

Interest on capital (20) (220)Profit as per Financial Records 110

Problem 2 : (CWA Inter June 2007)

Direct Materials 5,000,000 Factory OverheadsDirect Wages 3,000,000 Administrative OverheadsBad debts 80,000 Selling and Distribution OverheadsLegal Charges 10,000 Preliminary expenses written offDividend received 100,000 Interest received on depositsSales (120,000 units) 12,000,000

Closing stock :Finished goods (4,000 units) 320,000Closing stock : Work-in-progress 240,000

Prepare a statement reconciling the profit as per cost records with the profit as per financial records.Solution : Tutorial Notes

Note : No reconciliation is necessary for work-in-progress as its value is same in both records.

·       ,slk ugha gS fd nqfu;k esa flQZ vkids lkeus gh eqf'dysa gS-

The following figures have been extracted from the financial accounts of a manufacturing firm for the first year of its operations : (figures in Rs.)

The cost accounts for the same period reveal that the direct material consumption was Rs. 56,00,000. Factory overhead is recovered at 20% of prime cost. Administrative overhead is recovered at Rs. 6 per unit of production. Selling and distribution overheads are recovered at Rs. 8 per unit sold.

Page 231: cost accounting question icwai inter

4. Prepare the reconciliation statement and compare your solution with the following.5. Figures are for the first year of operations. It signifies that opening stock is nil.6. It will not be of much value if you see the following solution without preparing your own.

Profit and loss a/c for the year ended….. (all figures in Rs.'000)

To Opening stock 0 By Sales (120,000 units) 12,000To Direct Materials 5,000 By Closing stock 240To Direct wages 3,000 Work-in-progress 320To Factory Overheads 1,600 Finished goods (4,000 units)To Gross Profit 2,960

12,560 12,560To Administrative Overheads 700 By Gross profit 2,960To Selling Overheads 960 By Dividend received 100To Bad debts 80 By Interest 20To Prel.expenses written off 40To Legal Charges 10To Net Profit 1,290

3,080 3,080Cost Sheet Rs.'000

Direct Materials 5,600Direct Wages 3,000Prime Cost 8,600Factory Overheads (20% of Prime cost) 1,720

10,320Less : Closing Work-in-progress (as given) 240Works cost of 124,000 units 10,080Add : Administrative Overheads ( 124,000 units @ Rs. 6) 744Cost of Production (124,000 units) 10,824Less : Finished goods (4,000 units x 10,824 / 124,000) 349Cost of Goods sold (120,000 units) 10,475Add : Selling and Distribution Overheads (120,000 x Rs. 8) 960Cost of Sales 11,435Sales Revenue 12,000Profit : (Sales − Cost of Sales) 565

Reconciliation Statement Profit as per cost records 565Add : Material Consumed as per cost records 5,600

Factory Overheads as per cost records 1,720Administrative Overheads as per cost records 744Dividend Income 100Interest Received 20 8,184

Less : Material consumed as per financial records 5,000Factory Overheads as per financial records 1,600

1. The figures are for the first year of operations. What does it signify? What will you derive from this information?

2. Find out the profit for the year by preparing Profit and loss a/c. The gross profit is Rs. 29.60 lacs and net profit is Rs. 12.90 lacs. Use figures in Rs.'000 for convenience.

3. Find the profit as per cost records. See the previous problem for assistance. Some key figures are : Works cost for 124,000 units : Rs. 100.80 lacs. Cost of sales : Rs. 114.35 lacs. Profit : Rs. 5.65 lacs.

Page 232: cost accounting question icwai inter

Administrative Overheads as per financial records 700Bad debts 80Preliminary expenses written off 40Legal Charges 10 (7,430)

Add : Closing stock as per Financial records 320Less : Closing stock as per Cost records (349)Profit as per financial records 1,290

Problem 3: (CWA Inter June 2006)

(i) Sales Rs. 10,00,000 (50,000 units) (ii) No opening and closing stock of finished goods.(iii) Direct materials and direct wages were Rs. 500,000 and Rs. 250,000 respectively.(iv) Actual factory expenses were Rs. 150,000 of which 60% are fixed.(v) Actual administrative expenses were Rs. 45,000 which are completely fixed.(vi) Actual selling and distribution overheads were Rs. 30,000 of which 40% are fixed.(vi) Interest and dividend received Rs. 15,000.You are required to : (i) Find out the financial profit for the year ended 31.3.06

(iii) Prepare a reconciliation statement of profit shown by financial and cost book.Solution : Tutorial Notes1.Find the financial profit by preparing the Profit and loss a/c. Use figures in Rs.'000.

Profit and loss a/c for the year ended 31.3.06 (Rs.'000)To Opening stock 0 By Sales (50,000 units) 1,000To Materials 500 By Interest and dividend 15To Wages 250To Factory Overheads 150To Office and Administrative Overheads 45To Selling and Distribution Overheads 30To Profit (balancing figure) 40

1,015 1,015

Factory Expenses Admin.Expenses Selling ExpensesVariable Fixed Variable Fixed Variable

Rs. 60,000 90,000 0 45,000 18,000Units 50,000 60,000 50,000 60,000 50,000Rate per unit 1.20 1.50 0.00 0.75 0.36

Cost Sheet for the year ended 31.3.06Direct Materials (Rs.'000) 500Direct Wages 250Prime Cost 750Add : Factory expensesVariable 60

·       le; cgqr lh leL;kvksa dks lqy>k nsrk gS- le; ml leL;k dks Hkh lqy>k ldrk gS tks vkt vkidks fpafrar dj jgh gS- blhfy;s Lekby djuk lh[k yksA

The following information is available from the financial books of a company having a normal production capacity of 60,000 units for the year ended 31.3.2006

(ii) Prepare the cost sheet and find the costing profit assuming that the indirect expenses are absorbed on the basis of normal production capacity.

2. In cost books, the fixed expenses will be absorbed on the normal capacity while the variable expenses will be charged on the number of units sold.

Page 233: cost accounting question icwai inter

Fixed @ Rs. 1.50/unit for 50,000 units 75 135Works Cost 885Add: Administrative ExpensesVariable 0Fixed @ Rs. 0.75/unit for 50,000 units 38 38Cost of Production 923Add : Selling and Distribution expensesVariable 18Fixed @ Rs. 0.20 for 50,000 units 10 28Cost of Sales 951Sales revenue 1,000Profit (Sales − cost of sales) 50Reconciliation Statement :Materials and wages being same in both records, will not find place in reconciliation statement.Profit as per financial records : 40Add : Factory Overheads 150

Administrative Overheads 45Selling Overheads 30 225

Less : Factory Overheads as per cost records 135Administrative Overheads as per cost records 38Selling Overheads as per cost records 28 (201)Interest and dividend (15)

Profit as per Cost records : 50

Problem 4: (CA Inter adapted)

(i) Factory Overheads under-recovered 6,000(ii) Administrative Overheads over recovered 4,000(iii) Depreciation as per financial books 120,000(iv) Depreciation recovered in costs 130,000(v) Interest on investments not included in cost books 20,000(vi) Income tax provided 120,000(vii) Transfer fees credited in financial books 2,000(viii) Stores adjustment credited in financial books 2,000Solution : Reconciliation Statement Loss as per cost books 328,000Add : Factory overheads under-recovered 6,000

Income tax not provided in costs 120,000Depreciation as per financial records 120,000 246,000

Less : Administrative overheads over-recovered 4,000Interest on investments not included in costs 20,000Depreciation as per cost records 130,000Transfer fees 2,000Stores adjustment 2,000 (158,000)

Profit as per financial records 416,000

·  [kqn ls iwNsa fd ftl ckr dh eSa bruh fpark dj jgk gwa og esjs thou esa lpeqp fdruk egRo j[krh gS-·     ladV] ifjorZu vkSj nqfo/kk gh lqvoljksa dk l`tu djrs gSa-

Profit of a company is Rs. 416,000 as per financial books while the costing books reveal a net loss of Rs. 328,000. Following information is available : figures in Rs.

Page 234: cost accounting question icwai inter

Problem 5: (CWA Inter Adapted)From the following particulars, prepare : (figures in Rs.)

a) A statement of cost of manufacture for the year ;b) A statement of profit as per cost accounts ;c) Profit and loss a/c as per financial books and ;d) Reconciliation statement of financial profit and costing profit.

Opening stock of raw materials 100,000 (all figures in Rs.)Closing stock of raw materials 150,000Opening stock of finished goods 200,000Closing stock of finished goods 50,000Purchase of raw materials 600,000Wages 250,000

Solution : Tutorial Notes :

3. Do you know the difference between the cost of production and cost of goods sold?

5. Read the question again and think how you can determine the cost of production.The solution goes as follows : All figures in Rs. '000Statement of Cost of Production (Manufacture) Rs.'000Opening stock of raw materials : 100Add : Purchase during the year 600Less : Closing stock of raw materials 150Material consumed during the year 550Wages 250Prime Cost 800Add : Factory expenses as 25% of prime cost 200Works Cost 1,000Add : Office overheads as 75% of factory overheads 150Cost of Production 1,150Statement of Cost of Goods sold, Profit and Sales :Cost of Production 1,150Add : Opening stock of finished goods 200Less : Closing stock of finished goods 50Cost of goods sold 1,300Add : Profit as 25% of cost 325

Charge factory overheads at 25% on prime cost. Office overheads will be levied at 75% on factory overheads. Actual work expenses amounted to Rs. 193,750 and actual office expenses amounted to Rs. 152,500. The selling price is fixed at 25% above cost price.

1. You don't know the selling price. Without this, you cannot prepare the profit and loss a/c and determine the profit as per financial accounts. Think how can you determine the selling price and the sales for the year.

2. The selling price is fixed at 25% above cost price. The cost price is to be determined to arrive at the selling price. The selling price cannot be determined because the number of units sold is not given. What will you do ?

4. Cost of production does not take into account the opening and closing stock of finished goods. When cost of production is adjusted for opening and closing stock of finished goods, it gives you the cost of goods sold. You can add 25% on cost of goods sold to determine the sales for the year. Your problem now is how to determine the cost of production. Cost of production does not take into account the selling and administrative overheads.

Page 235: cost accounting question icwai inter

Sales 1,625Profit and loss a/c for the year ended…..

To Opening stock of raw materials 100.00 By Sales 1,625.00To Opening stock of finished goods 200.00 By Closing stock To Purchases 600.00 Raw materials 150.00To Wages 250.00 Finished goods 50.00To Works expenditure (given) 193.75To Office expenses (given) 152.50To Profit (bal. figure) 328.75

1,825.00 1,825.00Reconciliation of financial profit with costing profit :Profit as per financial books 328.75Add : Factory overheads as per financial books 193.75

Office overheads as per financial books 152.50 346.25Less : Factory overheads as per costing books 200.00

Office overheads as per costing books 150.00 (350.00)Profit as per Costing books 325.00

Hkxoku viuh gj fxQV leL;kvksa esa yisVdj nsrk gS];s mldh vknr gSAProblem 6 : (CWA Inter June 08 adapted)The profit and loss a/c of ABC ltd. for the year ended 31.3.08 is as follows :(Rs.'000)

Profit and loss a/c for the year ended 31.3.08To Materials 480 By Sales 960To Wages 360 By Work-in-progressTo Direct expenses 240 Materials 30To Gross profit 120 Wages 18

Direct expenses 12 60By Closing stock 180

1,200 1,200To Administrative expenses 60 By Gross profit 120To Net profit 66 By Dividend received 6

126 126

Prepare a statement of Costing Profit and loss a/c and reconcile the costing profit with financial profit.Solution : Tutorial Notes:1. Where will you adjust the Work-in-progress figures in cost accounts?

As per the cost records, the direct expenses have been estimated at a cost of Rs. 30 per kg and administrative expenses at Rs. 15 per kg. During the year production was 6,000 kg and sales were 4,800 kg.

2. The production is for 6,000 kgs while the sales is for 4,800 kgs. On which of these two figures, you will charge the direct expenses and administrative expenses and why ?

3. As the production is 6,000 kg and sales is 4,800 kg., there must be closing stock of 1,200 kg. How will you find the value of this closing stock?

4. How will you determine the cost of goods sold? The sales is given as Rs. 960,000. Costing profit is the difference between Sales and COGS.

5. Financial profit is given as Rs. 66,000. Costing profit you have determined in the above steps. Reconciliation statement can be prepared.

Page 236: cost accounting question icwai inter

8. Solve the problem on the basis of previously solved problems and then see the following solution.Statement showing Cost of Goods Sold and Sales :Material consumed during the year 480Less : Material in WIP 30 450Wages 360Less : Wages in WIP 18 342Direct expenses ( 6,000 kg. x Rs.30) 180Administrative expenses ( 6,000 kg. x Rs. 15) 90Cost of Production of 6,000 kg 1,062Cost of production of 4,800 kg (1062 x 4,800 / 6,000) 850Cost of Closing stock of 1,200 kg (1,062 x 1,200 / 6,000) 212Sales revenue for 4,800 units 960Profit as per Cost records : (960 − 850) 110Reconciliation Statement : (see carefully)Profit as per Cost records : 110Add : Direct expenses as per cost records 180

Administrative expenses as per cost records 90 270Less : Direct expenses as per financial records 240

Administrative expenses as per financial records 60 (300)Add : Closing stock as per financial records 180Less: Closing stock as per cost records (212)Add : Direct expenses in WIP in Financial records 12Add : Dividend received in Financial records 6Profit as per financial records 66

6. Work-in-progress is not part of production, hence its value should be removed from the cost of production. Opening WIP should be added to the cost of production while closing WIP should be deducted.

7. The overheads will be charged on the units produced because you are first to determine the cost of production of 6,000 units. From this you will determine the cost of 4,800 units by simple arithmetic.

Note : If the selling and distribution overheads are to be charged, these will be charged on the units sold and not on units produced.

·       gj pht vklku gksus ls igys dfBu gksrh gS-

Page 237: cost accounting question icwai inter

+ Power and Fuel + Research and experiment + depreciation of plant + Works Manager's salary

Office rent and taxes + Office salaries + Office insurance + Mobiles, phones and postage

: Factory cost or works cost + Office and Administrative Overheads

+ Opening stock of finished goods − Closing stock of finished goods

70,000

200,00020,000

Prepare a statement reconciling the profit as per cost records with the profit as per financial records.

oks fnu vkidk fcYdqy csdkj x;k ( ftl fnu vki [kqydj gals ugha gksa-

vki ifjfLFkfr Hkys gh u cny ik,a ij eufLFkfr rks gj gkyr esa cny gh ldrs gSa-

The following figures are available from the financial records of ABC Manufacturing Company Ltd. for the

In the costing records, factory overhead is charged at 100% wages, administrative overhead is charged 10% of factory cost and selling and distribution overhead at the rate of Rs. 10 per unit sold.

1. You can compute the profit as per financial records in the usual manner by preparing the Profit and loss

Page 238: cost accounting question icwai inter

5. After finding both the profits you can go ahead with the preparation of reconciliation statement.

3. You should revise your knowledge of cost sheet before you compute the profit as per cost records. Think how can you compute the profit as per cost sheet. You are required to find out the profit on 20,000

6. It is meaningless if you straight away see the following solution. Please prepare your own solution and

Actual factory overheads of Rs. 450,000 is not known when cost sheet is prepared. It is charged at 100% of wages. Thus factory overheads charged in cost sheet would be 100% of Rs. 500,000 while the

the profit by Rs. 50,000 as the actual factory overhead is less. This

: The factory cost of the year is Rs. 20 lac, which is inclusive of cost of work-in-progress and finished goods as well. You are required to find out the cost of 20,000 units sold during the year. You must deduct cost of work-in-progress and finished goods to find out the cost of 20,000 units.

: You must know that the office and administrative overheads are charged on the units completed during the year. Therefore it is essential that you must include the finished goods in the factory cost. Thus you must deduct the work-in-progress only (and not cost of finished goods) from the factory cost of the year which will give you factory cost of (20,000 + 1,230) units. Charge on this the administrative overheads at 10%, you will find the cost of 21,230 units. From this deduct the cost of finished goods ( Not the given

: To production cost of 20,000 units add selling and distribution overheads at Rs. 10 per unit. Cost of sales becomes known here. Sales of 20,000 units is given as Rs. 25 lacs. The difference is profit as per

( don't take the given figure of Rs. 150,000)

Page 239: cost accounting question icwai inter

1,600,000700,000960,000

40,00020,000

Prepare a statement reconciling the profit as per cost records with the profit as per financial records.

,slk ugha gS fd nqfu;k esa flQZ vkids lkeus gh eqf'dysa gS-

The following figures have been extracted from the financial accounts of a manufacturing firm for the first

The cost accounts for the same period reveal that the direct material consumption was Rs. 56,00,000. Factory overhead is recovered at 20% of prime cost. Administrative overhead is recovered at Rs. 6 per unit of production. Selling and distribution overheads are recovered at Rs. 8 per unit sold.

Page 240: cost accounting question icwai inter

(all figures in Rs.'000)

1. The figures are for the first year of operations. What does it signify? What will you derive from this

2. Find out the profit for the year by preparing Profit and loss a/c. The gross profit is Rs. 29.60 lacs and net

3. Find the profit as per cost records. See the previous problem for assistance. Some key figures are : Works cost for 124,000 units : Rs. 100.80 lacs. Cost of sales : Rs. 114.35 lacs. Profit : Rs. 5.65 lacs.

Page 241: cost accounting question icwai inter

Selling ExpensesFixed

12,00060,000

0.20

le; cgqr lh leL;kvksa dks lqy>k nsrk gS- le; ml leL;k dks Hkh lqy>k ldrk gS tks vkt vkidks fpafrar dj jgh gS- blhfy;s Lekby djuk

The following information is available from the financial books of a company having a normal production

(ii) Prepare the cost sheet and find the costing profit assuming that the indirect expenses are absorbed on

2. In cost books, the fixed expenses will be absorbed on the normal capacity while the variable expenses

Page 242: cost accounting question icwai inter

[kqn ls iwNsa fd ftl ckr dh eSa bruh fpark dj jgk gwa og esjs thou esa lpeqp fdruk egRo j[krh gS-

Profit of a company is Rs. 416,000 as per financial books while the costing books reveal a net loss of Rs.

Page 243: cost accounting question icwai inter

Charge factory overheads at 25% on prime cost. Office overheads will be levied at 75% on factory overheads. Actual work expenses amounted to Rs. 193,750 and actual office expenses amounted to Rs.

1. You don't know the selling price. Without this, you cannot prepare the profit and loss a/c and determine the profit as per financial accounts. Think how can you determine the selling price and the sales for the

2. The selling price is fixed at 25% above cost price. The cost price is to be determined to arrive at the selling price. The selling price cannot be determined because the number of units sold is not given. What

4. Cost of production does not take into account the opening and closing stock of finished goods. When cost of production is adjusted for opening and closing stock of finished goods, it gives you the cost of goods sold. You can add 25% on cost of goods sold to determine the sales for the year. Your problem now is how to determine the cost of production. Cost of production does not take into account the selling and

Page 244: cost accounting question icwai inter

Hkxoku viuh gj fxQV leL;kvksa esa yisVdj nsrk gS];s mldh vknr gSA

Prepare a statement of Costing Profit and loss a/c and reconcile the costing profit with financial profit.

As per the cost records, the direct expenses have been estimated at a cost of Rs. 30 per kg and administrative expenses at Rs. 15 per kg. During the year production was 6,000 kg and sales were 4,800

2. The production is for 6,000 kgs while the sales is for 4,800 kgs. On which of these two figures, you will

3. As the production is 6,000 kg and sales is 4,800 kg., there must be closing stock of 1,200 kg. How will

4. How will you determine the cost of goods sold? The sales is given as Rs. 960,000. Costing profit is the

5. Financial profit is given as Rs. 66,000. Costing profit you have determined in the above steps.

Page 245: cost accounting question icwai inter

8. Solve the problem on the basis of previously solved problems and then see the following solution.

6. Work-in-progress is not part of production, hence its value should be removed from the cost of production. Opening WIP should be added to the cost of production while closing WIP should be deducted.

7. The overheads will be charged on the units produced because you are first to determine the cost of production of 6,000 units. From this you will determine the cost of 4,800 units by simple arithmetic.

Note : If the selling and distribution overheads are to be charged, these will be charged on the units sold and not on units produced.

Page 246: cost accounting question icwai inter

Chapter 8 : Operating, Service and Relevant Costingcwa inter cwa final ca inter ca final Total

3 7 0 7 17

Following cost data are available for consideration:1. Petrol @ Rs. 45 per liter. 2. Repairs and maintenance @ 50 paise per km.3. Insurance-- Rs. 4800 per year per car. 4. Taxes--Rs. 2400 per year per car.5. Tyres @ 40 paise per km. 6. Drivers wages and Bonus Rs. 30,000 per year per car.Which proposal is to be recommended by you?Solution: Tutorial Notes:

Statement showing comparative cost of operationFor an annual run of 20,000 kms.per annum All fig. in Rs.

Particulars Rate Hired CarRs. Rs. Rs.

Petrol Consumption Rs. 45/lit/12 km 75,000 75,000Repairs and maintenance Rs. 0.50/km 10,000 0Tyre Rs. 0.40/km 8,000 8,000Insurance Rs. 4800/car 4,800 4,800Taxes Rs. 2400/car 2,400 2,400Depreciation Rs. 40,000/car 40,000Driver's wages and bonus Rs. 30,000/car 30,000Hire charges 30,000Reimbursement of expenses 100,000Total Cost 170,200 104,800 115,400Total cost per km of run( Cost/20,000) 8.51 5.24 5.77

ukdkeh ls mcjus dk 'kfDr'kkyh rjhdk ;g lksp gS fd ;g ukdkeh vafre ;k Qk;uy ugh gSA dqN djus dh xqtkb'k vHkh ckdh gSA vkidh cM+h ;kstukvksa ds fygkt ls ;g ukdkeh cgqr NksVh gS vkSj blls dqN lh[kdj u;k iz;kl fd;k tk ldrk gSA

Problem: 1(cwa inter June 05)

Acme company is considering three proposals for conveyance facilities for its sales staff, who normally travels on an average 20,000 kilometers per annum locally. The proposals are as follows:

1. Purchase and maintain own fleet of cars. Average cost of a car is Rs. 2.50 lacs. Petrol consumption is @ Rs. 12 km per liter. Each has a resale value of Rs. 50,000 at the end of 5 years.

2.Allow the executives to use their own car and reimburse expenses @ Rs. 5 per km and insurance premia.

3. Hire cars from outside agency for Rs. 30,000 per year per car, the company shall also bear the cost of petrol, taxes, tyres etc. @ Rs. 3.75 per km.

If the company purchases and maintains its own fleet of 5 cars investing Rs. 12.5 lacs, the opportunity cost should also be considered.

Company's Car

Executive's Car

Recommendation: In view of the above statement, it is clear that the maintaining its own fleet of car is the costliest option while using executive's car is the most economical among the three options and is recommended.

·       mB cka/k dej D;ksa Mjrk gS] fQj ns[k [kqnk D;k djrk gS-

Page 247: cost accounting question icwai inter

Total fleet of vehicles : 600 single deck buses Average passengers occupying each trip : 50Variable expenses : Total for 36,000 km./bus p.a. in Rs. Administrative Expenses:Fuel and diesel oil 60,000 Establishment cost 87,50,000Tyres, tubes and batteries 80,000 Staff welfare cost 365,000Lubricants 15,000 Gratuity 375,000Spare parts and accessories 24,000 Rates and taxes 300,000Wages including bonus and fringe benefits 140,000 Traveling and conveyance 190,000Fixed Expenses : Postage and Telephone 130,000Road tax 6,000 Stationery 180,000Insurance 7,500 Audit fees 75,000Rent for garage/ night parking 15,000 Legal expenses 95,000Maintenance garage establishment cost 8,000 Electricity 360,000Original cost per bus 750,000 Interest and bank charges 830,000

Miscellaneous 170,000

Depreciation on buses is computed @ 20% on Straight line method

1st stage travel 2 km. 2nd stage travel 5 km3rd stage travel 10 km 4th stage travel 15 km.Assume that fare is changed in proportion to kilometer traveled.Solution:

Some working is necessary to find out the administrative cost per bus per km of run.

2. Administrative expenses per bus per km can be computed as follows:Administrative Expenses: Rs. lacs Variable expenses : Total for 36,000 km./bus p.a. in Rs.Establishment cost 87.50 Fuel and diesel oil 60,000Staff welfare cost 3.65 Tyres, tubes and batteries 80,000Gratuity 3.75 Lubricants 15,000Rates and taxes 3.00 Spare parts and accessories 24,000Traveling and conveyance 1.90 Wages including bonus and fringe benefits 140,000Postage and Telephone 1.30 Total variable expenses 319,000Stationery 1.80 Variable expenses per bus per km in Rs. 8.86Audit fees 0.75 Fixed Expenses :Legal expenses 0.95 Road tax 6,000Electricity 3.60 Insurance 7,500Interest and bank charges 8.30 Rent for garage/ night parking 15,000Miscellaneous 1.70 Maintenance garage establishment cost 8,000Total Admin. expenses 118.2 Depreciation (750,000 / 5 150,000

Problem: 2(cwa inter dec 02)

New India transport corporation has been experiencing financial and operational problems faced with spiraling rise in fuel prices, high cost of spares and also high wages. Reproduced below are the operational expenses worked out for the one year ended 30th Sept. 02.

The aggregate of administrative overhead is to be absorbed in the operating cost on the basis of available number of buses in the operation (assuming average breakdown @ 12.50%)

Based on the above information, you are required to advise management on the fare structure assuming cost plus 15% for the following stages of travel (rounded off to nearest 50 paise)

This question is very easy question. Student should try to solve it completely before going through the solution given below:

1. Total no. of buses 600, out of which break down is 12.50% i.e. 75. The number of buses in operation is thus 525.

Page 248: cost accounting question icwai inter

Total fixed expenses 186,500Fixed Expenses : per km (186,500 / 36,000) 5.18

Total administrative expenses/per bus (118.20 x 100,000 / 525) Rs. 22514Administrative Expenses per bus per km Rs. 0.625

total operational cost of per bus per km Rs.Variable expenses 8.86Fixed expenses 5.18Administrative expenses 0.62total operational cost of per bus per km 14.66Profit margin @ 15% 2.20Fare to be charged per bus 16.86Average passenger per bus 50Fare to be charged per bus per passenger 0.3372Fare structure advised to the management:First stage of travel 2 km. 0.3372 x 2 paise 67Second stage of travel 5 km 0.3372 x 5 paise 169Third stage of travel 10 km 0.3372 x 10 paise 337Fourth stage of travel 15 km 0.3372 x 15 paise 506

The minibus is 24 seater and is expected to run 6 two-way trips during the 25 days in a month.

Solution: Tutorial Notes:

3. On the lines given above, the student should try to solve this problem.

·       gekjs ckjs esa gekjs nq'euksa ds fopkj gels T;knk lgh gksrs gSa-Problem: 3 (cwa inter I June 01)

Carrywell enterprises has been permitted to run a minibus on a route covering 20 kms. The minibus has been purchased at a cost of Rs. 1 lakh, part of which was financed through bank loan and balance by loan from other sources.

The annual charges for the minibus are insurance Rs. 4,000, road tax Rs. 2,000 and garage rent Rs. 1,200. Cost of repairs and maintenance is estimated at Rs. 6,000 per annum while replacement of tyre and tube will cost Rs. 480 per month. Office expenses are estimated at Rs. 600 per month. Petrol and oil will cost @ 45 paise per km.

Two drivers and two conductors are engaged at a monthly salary of Rs. 500 and Rs. 350 respectively. In addition drivers and conductors are entitled to 5% of the sale of tickets.

The effective of vehicle is estimated at 5 years, at the end of which the vehicle will fetch scrap value of Rs. 10,000.

You are required to submit passenger fare structure for approval by the transport authority which allow 20% profit on net sales. Interest on loan is allowed as cost, if installments are paid regularly, assume the amount of interest to be Rs. 6,720 p.a.

1. The depreciation is given on time basis. It can be considered as fixed cost. Method of charging depreciation is not specified. For simplification of calculation, straight line method should be used. If depreciation is given on km. basis, it should be charged as variable cost.

2. The commission of driver and conductor is charged @ 5% on sales. The sales already includes 20% profit on it. The cost is not known. To solve this complexiity, the cost before charging commission should be computed first. Suppose this is X If sales is denoted as S, the equation becomes S = X + S/5 + S/20. The value of X is already known, S can be known. Worded differently, the cost is 75% of the sales and sales can be known by dividing the cost by 0.75.

Page 249: cost accounting question icwai inter

A. Standing Charges Rs. Per annum B. Operating charges p.a. Rs. / monthInsurance 4,000 Petrol and oil (see below) 32,400 2,700Road tax 2,000 Tyre and tube 5,760 480Garage rent 1,200 Total Cost 38,160 8,640Office exp. @ Rs. 600/month 7,200 Sales (Cost */ 0.75) (8640/0.75) 11,520Depreciation 18,000 Commission on 5% of sales 576Repairs and maintenance 6,000 Profit @ 20% of sales 2,304Driver's salary 12,000Conductor's salary 8,400Interest on loan 6,720 Total km. ( 25 x 6 x 2 x 20 ) 6,000Total charges p.a. in Rs. 65,520 Total passenger km (24 seats) 144,000Total charges per month 5,460 Sale/ passenger / km 0.08

The costs of operation of flights are :Fuel cost variable : 95,000 per flightFood served on board on non-chargeable basis 130 per passengerFixed costs : Aircraft lease 350,000 per flight

Crew, landing charges etc. 72,000 per flightCommission 5% of the fare

Required: 1.Calculate the net operating income per flight.

Solution: No. of passenger = 60% of 160 = 961. Computation of net operating income per flight: fig. in Rs.Fare revenue per flight @ Rs. 7,000 for 96 passengers 96 x 7000 672,000Variable cost

Fuel 95,000Food @ Rs. 130/passenger 12,480Commission @ 5% of revenue 33,600Total Variable cost 141,080Contribution per flight 530,920Fixed Costs Lease 350,000

Crew 72,000 422,000Net income per flight 108,920

2. Net operating income if fare is reduced to Rs. 6720 with occupancy 108 passengersFare revenue per flight @ Rs.6720 for 108 passengers 108 x 6,720 725,760

4. Depreciation is (100,000 -- 10,000) / 5 = Rs. 18,000 p.a. This should be shown by way of working note.

Petrol and oil is calculated as ( 25 days x 6 x 2 trips x 20 km x 0.45 Rs. )

·       fdlh ds Hkh ihNs va/ks tSls u pysa] u gh mls rksrs tSls nqgjk;sa] u gh mldh canj tSlh udy djsa-Problem: 4 (cwa final June 2000)

In order to develop tourism, Reliable Airline has been given permit to operate three flights to and fro in a week between station A and station B. The airline operates a single aircraft of 160 seats capacity. The normal occupancy is estimated at 60% throughout the year of 52 weeks. The one way fare is Rs. 7,000.

2. The Airline expects that its occupancy will increase to 108 passengers per flight if the fare is reduced to Rs. 6,720. Advise whether proposal should be implemented.

3. A travel agency firm proposes to charter the aircraft for one return trip (to and fro) in each month on payment of a fixed charge of Rs. 5 lacs per flight. The travel agency will meet the fuel and food costs. Should the Airline accept this proposal ?

Page 250: cost accounting question icwai inter

Variable cost Fuel 95,000Food @ Rs. 130/passenger 14,040Commission @ 5% of revenue 36,288Total Variable cost 145,328Contribution per flight 580,432Fixed Costs Lease 350,000

Crew 72,000 422,000Net operating income per flight 158,432

The proposal appears to be acceptable in view of increased contribution and operating income.

A truck carries 500 cases for both inbound and outbound shipments.

A 25% safety factor is desired since flow of the trucks throughout the month is not uniform.

Solution:Particulars Inbound OutboundTotal cases per annum in lacs 60 60% to be handled by trucks 70% 90%Cases to be handled by trucks in lacs 42 54Cases per worker-hour 200 175Total worker-hours required 21,000 30,857Add:25% as safety measure 5,250 7,714Total hours required 26,250 38,571Grand total of hours required for inbound and outbound trucks 64,821Total hours available per annum = 52 weeks x 5 days x 2 shifts x 8 hrs 4,160Truck entries required = (64,821/4,160) 15.6Note: Loading will be carried out in outbound trucks and unloading will be on inbound trucks.

Types of services A B CAnnual fee Rs./ Job 3,000 2,400 1,800Annual variable cost Rs./ Job 1350 800 810

3. The present contribution at 96 occupancy and with fare of Rs. 7000 per passenger comes to Rs. 530,960. The travel agency is offering Rs. 500,000 per flight. Any offer below the present contribution of Rs. 530,920 should not be accepted as it will incur loss to Reliable Airline.

·       [kqn lq[kh jgus ds fy;s nwljksa dks [kq'k djuk t#jh gS-Problem: 5 (cwa final June 06)

A warehouse handles 60,00,000 cases each of inbound and outbound shipments per year, operating two shifts, five days a week. Each shift is of 8 hours durations. 70% of inbound shipments and 90% of outbound shipments are by trucks.

The truck loading rate is 175 outbound cases per worker-hour while the unloading rate is 200 inbound cases per worker-hour.

The trucks stand at the entry to the warehouse while being loaded or unloaded. The warehouse manager wants to design a new warehouse with sufficient number of truck entries to the warehouse to handle the operations mentioned above.

Please calculate the number of truck-entries the warehouse should have from the data given above, assuming that the width of the entry permits only one truck to loaded or unloaded at a time.

Problem: 6 (cwa final June 06)

A firm in the profession of rendering software services provides three different kinds of services to its clients. The following are the data relating to these services;

Page 251: cost accounting question icwai inter

Annual fixed cost Rs./ Job 600 320 225

Find the annual revenue needed by the firm to break even.Solution:Types of services A B C Rs.Annual fee Rs./ Job 3,000 2,400 1,800Annual variable cost Rs./ Job 1350 800 810Contribution Rs./ Job 1,650 1,600 990Proportion of service 2 3 5Contribution per set of three services 3,300 4,800 4,950 13,050Annual fee per set of services 6,000 7,200 9,000 22,200No. of sets for break even : Fixed cost / Contribution = 574,200 / 13,050 = 44Break even sales = 44 x Rs. 22,200 = Rs. 976,800.

(i)

(ii)

P (Rs.) Q (Rs.)Original Cost 49,000 35,000Accumulated Depreciation 34,000 25,000Written down value 15,000 10,000Estimated value at end of useful life 7,000 1,000Age years 4 6Estimated remaining useful life years 1 2

Start of the year End of yearRs. Rs.

Machine P Replacement cost 56,000 63,000Resale value 10,000 7,000

Machine Q Replacement cost 42,000 46,000Resale value 6,000 4,000

The total annual fixed cost is budgeted at Rs. 574,200 and none of these costs are specific to any type of the services provided by the firm.

The firm has estimated the number of service contracts to be sold in the next year in the proportion of 20%, 30% and 50% respectively for the three types of services namely A,B and C.

Problem: 7 (cwa final June 06)

Soft toys Pioneer Ltd. wants to add one more item, Bhulu, to its range of products. According to an outside market research firm, in order to retain its specialty value, Bhulus should be marketed for just one year as other competitors will imitate is soon thereafter.

You are required to ascertain the cash flows, based on relevant costing concept, in respect of the following:

The research firm has submitted its bill of services to the company for Rs. 50,000 which is still to be paid.

Two of the existing machines P and Q, would be required to make Bhulus. You collect data about the machines as under.

Details are also available of cash values relating to the two machines at the start and at the end of the year during which Bhulus would be produced:

If machine P is not used for the manufacture of Bhulus then it would be used to manufacture existing products the sale of which would result in an estimated Rs. 70,000 net receipts.

Machine P is one of a number of identical machine types used regularly on various products by Soft toys Pioneer Ltd. Each of this type of machine is replaced as soon as it reaches the end of useful life.

Page 252: cost accounting question icwai inter

(ii) Use of machine P and Q :

(h) Epitomizing the above point on relevancy basis, we conclude the following:Rs.

Cost of using machine P : 49,000Cost of not using machine P : 56,000Cost of using machine Q : 2,000Net cost of not manufacturing Bhulus : 5,000

Machine Q is the only one of its type within the firm and if not used in the manufacture of Bhulus would be sold immediately.

Explain your working--taking differential approach i.e. showing cash flows if Bhulus are manufactured and if Bhulus are not manufactured.

Please restrict your answer to the above two information-- research cost and machine usage -- for a decision by the management on the project Bhulu.

Answer : The point wise answers are as follows.

(i) Research Cost : It is a liability of the firm to pay this bill whether or not it uses the results of research. The cash outflow of Rs. 50,000 relates to the project Bhulu but is not relevant for decision making on the fate of Bhulu. It is a past committed cost and as such irrelevant for the decision on the project.

(a) Machine P is one of several identical machines being used by Soft Toys. If it is not used it would result in an estimated net receipts of Rs. 70,000. Net receipts of Rs. 70,000 is not relevant because Machine P is already doing this or other machines available with Soft toys will do it.

(b) Whether Machine P is used or not in the manufacture of Bhulu, it is not going to be sold by Soft Toys because it still has one remaining year of useful life, hence its resale value of Rs. 10,000 at the beginning of the year is not relevant.

(c) If Bhulus are decided to be manufactured for a year, then the existing machine P will have to be used for this purpose. It implies that the machine being used for manufacture of Bhulu, will have to be replaced at the start of the year requiring Rs. 56,000 as the replacement cost. This machine will have resale value of Rs. 7,000 at the end of the year. Thus the net cost of using machine P for manufacture of Bhulu is Rs. 49,000 (ignoring, of course, the time value of money).

(d) If it is decided that Bhulus are not to be manufactured, the existing machine will have to be replaced at the end of the year which will cost Rs. 63,000 having resale value of Rs. 7,000. Thus the net cost of not using the machine P for the manufacture of Bhulus will be Rs. 56,000 (ignoring, of course, the time value of money).

(e) Whether Bhulus are manufactured or not, the machine P is not going to be replaced at the start of the year, hence its resale value at the start of the year is not relevant.

(f) Whether Bhulus are manufactured or not, the machine Q is not going to be replaced at the start of the year, hence its replacement value at the start of the year is not relevant.

(g) If machine Q is used for the manufacture of Bhulus, it resale value will diminish by Rs.2000 at the end of year. So, the cost of using machine Q for the manufacture of Bhulus is Rs. 2000 (ignoring again, the time value of money).

·       eu dk gks rks vPNk- eu dk u gks rks vkSj Hkh vPNk-Problem 8 (ca final may 98)

A limited operates a lodging house with a restaurant, shops and recreative facilities attached. Its manager has entrusted you with the planning of the coming year's operations, more particularly on the level of profits the company was likely to earn.

Page 253: cost accounting question icwai inter

Some other relevant data available to you is as under :(i) Variable cost to volume ratio:

Shops RestaurantCost of goods sold 40% 30%Supplies 5% 15%Others 5% 10%

(iii) Annual fixed costs for the entire complex are Rs. 19,50,000.From the above, you are required to prepare :(a) an income statement for the coming year and ;

Solution :(a) Statement of Expected Income of A Ltd. Company for the year……………(A) Revenue : Rs. '000Receipts from hotel rooms (100 rooms x 250 days x Rs. 150 x 75%) 2,812.50Shops ( 100 rooms x 2 persons x 250 days x Rs. 30 x 75%) 1,125.00Restaurant ( 100 rooms x 2 persons x 250 days x Rs. 60 x 75%) 2,250.00

6,187.50(B) Variable costs :Hotel room ( 100 rooms x 250 days x Rs. 25 x 75%) 468.75Shops (100 rooms x 2 persons x 250 days x Rs. 30 x 75%) x 50% 562.50Restaurant ( 100 rooms x 2 persons x 250 days x Rs. 60 x 75%) x 55% 1,237.50

2,268.75Contribution : 3,918.75Less Fixed cost : 1,950.00Expected profit : 1,968.75Income Statement based on Manager's Suggestion :(A) Revenue : Rs. '000Receipts from hotel rooms (100 rooms x 250 days x Rs. 125 x 90%) 2,812.50Shops ( 100 rooms x 2 persons x 250 days x Rs. 30 x 90%) 1,350.00Restaurant ( 100 rooms x 2 persons x 250 days x Rs. 60 x 90%) 2,700.00

6,862.50(B) Variable costs :Hotel room ( 100 rooms x 250 days x Rs. 25 x 90%) 562.50Shops (100 rooms x 2 persons x 250 days x Rs. 30 x 90%) x 50% 675.00Restaurant ( 100 rooms x 2 persons x 250 days x Rs. 60 x 90%) x 55% 1,485.00

2,722.50Contribution : 4,140.00Less Fixed cost : 1,950.00Expected profit : 2,190.00

The lodging house has 100 double-bed rooms, which are likely to be rented at Rs. 150 per day. The manager expects an occupancy ratio of 75% for a period of 250 days during the tourist season. It is also anticipated that both the beds in a room will be occupied during the period.

Each person staying in the lodging house is expected to spend, on the basis of past statistics, Rs. 30 per day in shops attached to the lodge and Rs. 60 per day in the restaurant. The recreational facilities are not charged to the customer.

·       Just because something is difficult does not mean that you should not try, it means that you should try harder.

(ii) For the lodging house, the variable costs are Rs. 25 per day per occupied room for cleaning, laundry etc.

(b) an analysis to indicate whether the manager's suggestion of reducing the room rent to Rs. 125 per day to enhance the occupancy ratio to 90% should be accepted.

Page 254: cost accounting question icwai inter

Required:

Solution : very easy question. Rs. LacsTuition fees paid by the students last year 12,000 students @ Rs. 3,600 432.00University Expenditure total (fees is 75% of university expenditure) 576.00Less Fixed cost : 300.00Variable cost 276.00Variable cost per student : (276 x 100,000 / 12,000) Rs. 2,300Endowment and Contribution (as 25% of university total expenditure) 144.00(i) Statement showing availability of funds next year:Tuition fees 11,200 students @ Rs. 4,200 470.40Endowment and Contribution (same as previous year) 144.00Grant 50.00Less: Variable cost (10% above previous year) (1.1 x 2300 x 11200) 283.36Less Fixed cost : (Rs. 300 lakhs + Rs. 30 lakhs increase) 330.00Balance Available for Capital improvements 51.04(ii) Computation of tuition fees for Break-even:Variable cost Rs. 2,300 x 1.10 x 12,000 students 303.60Add :Fixed cost : (Rs. 300 lakhs + Rs. 30 lakhs increase) 330.00Capital improvements 40.40Total cost 674.00Less : Endowment, contribution and grants 194.00Balance for Tuition fees 480.00Tuition fees to be charged to each student (480 x 100,000) / 12,000 Rs. 4,000

The expected profit is Rs. 19.68 lacs while profit if manager suggestion is adopted is Rs. 21.90 lacs, therefore, it is advised that the suggestion of the manager to reduce the room rent to Rs. 125 per day to enhance the occupancy ratio to 90% should be accepted.

·       lcls vPNk tokc gS dqN djds fn[kk nsuk-Problem 9 (ca final may 00)

A private university with a current enrolment of 12,000 students is reviewing cost and revenue data for the past academic year. Student tuition is Rs. 3,600 a year. Tuition normally covers 75% of university expenditures. The remaining 25% comes from endowments and contributions.

During the last year the fixed cost amounted to Rs. 300 lakhs. The rest of the costs varied with student enrolment. Cost have been rising more rapidly than tuition or contributions, and the university just broke even last year. A tuition increase is being contemplated. The budget committee thinks endowment revenues and contributions will remain constant at last year's level for the next several years.

The fixed costs are expected to increase by Rs. 30 lakhs and the variable costs are expected to increase by 10%. The president of the university tells the budget committee that he expects a new grant of Rs. 50 lakhs in addition to the normal contributions for each of the next 5 years from a large corporation owned by an alumnus of the university. The university has been postponing a number of major capital improvements and building projects.

(i) If the grant is received and tuition is raised to Rs. 4,200, how much money would the university have available in the first year for capital improvement and building with student enrolment of 11,200 and the expected cost increases?

(ii) If the grant is received and costs increases as predicted for the coming year, what tuition should the university charge to break-even with its current enrolment of 12,000 students after providing Rs. 40.40 lakhs for capital improvements?

·       ,glkuean gksuk vkSj mls tkfgj djuk mRd`"B yksxksa

Page 255: cost accounting question icwai inter

The following combinations of sales is expected:Complete sets of dress, cap and hand bag 68%Dress and Cap only 12%;Dress and handbag only 9%Dress only : 11%

The selling prices and the other costs to complete the three items, once they are cut, are as follows:

Dress Cap HandbagSelling price per unit Rs. 400 29 18Other costs per unit Rs. 48 6.5 3Other costs per unit excludes the cost of material and cutting:You are required to prepare a statement showing:

(i) Should the company go in for caps and handbags along with dresses? Substantiate your answer

Solution:

1. The combination of 3,000 dresses and relevant cost is as follows:Combination % No. Price Rs. Sales Rs. Complete sets of dress, cap and bag 68% 2040 447 911,880Dress and Cap only 12% 360 429 154,440Dress and Handbag only 9% 270 418 112,860Dress only 11% 330 400 132,000

13111802. Material consumption and its cost for various combinations is as follows:

·       ,glkuean gksuk vkSj mls tkfgj djuk mRd`"B yksxksa dk xq.k gS- ;g ?kfV;k yksxksa esa ugha ik;k tkrk-Problem 10 (ca final may 01)

Ze-Te Fashions is a high-fashion women's garments manufacturer. It is planning to introduce a new fashion garment in the market in the forthcoming Diwali season. Four meters of cloth (material) are required to layout the dress pattern. After cutting, some material remains that can be sold as a cut-piece. The left-over material can also be used to manufacture a matching cap and handbag.

Ze-Te expects to sell 2,500 dresses, if matching caps and handbags are not provided and 20% more, if matching caps and handbags are made available. The market research indicates that the cap and/or handbag cannot be sold independently, but only as accessories with the dress.

The material used in the dress costs Rs. 60 per meter. The cost of cutting the dress, if the cap and handbag are not manufactured, is estimated at Rs. 20 a dress and the resulting remnants can be sold for Rs. 5 for each dress cut out.

If the cap and handbag are to be manufactured, it requires a more delicate and skillful cutting and hence cutting cost will increase by Rs. 8 per dress.

(ii) What are the non-quantitative factors that could influence the company's decision to manufacture caps and handbags that match the dress?

After second reading you may find that your calculations belong to (1) two categories of sales i.e. 2500 dresses and 3000 dresses (2) Costs consists of three main elements (i) material (ii) cutting cost and (iii) other costs and (3) there are four combinations possible (i) dress only (ii) dress and cap only (iii) dress and handbag only and (iv) Complete set of dress, cap and handbag.

If you are a bit confused about to where to begin in framing solution, you can start with making various working notes relevant to the solution.

Page 256: cost accounting question icwai inter

For 2,500 dresses:Material ( 4 meters x Rs. 60/metre -- Rs. 5) = @ Rs. 235 587,500For 3,000 dresses: Material :( 4 meters x Rs. 60/metre -- Rs. 5) = @ Rs. 235 for 330 dresses 77,550( 4 meters x Rs. 60/metre) = @ Rs. 240 for balance 2,670 dresses 640,800

718,3503. Cutting cost for 2,500 dresses: (2500 x Rs. 20) 50,0004. Cutting costs for 3,000 dresses:

(2,670 dresses @ Rs. 28 + 330 dresses @ Rs. 20) 81,3605. Other costs for 2500 dresses (2500 x Rs. 48) 1200006. Other costs for 3,000 dresses:Complete set of dress, cap and handbag(2040 dresses x Rs. 57.50) 117,300Dress and cap only ( 360 dresses x Rs. 54.50) 19,620Dress and handbag only ( 270 dresses x Rs. 51) 13,770Dress only ( 330 dresses x Rs. 48) 15,840

166,530Statement showing Profitability of Ze-Te FashionsWithout caps and handbags With caps and hand bags

Expected sales in units 2,500 3,000Rs. Rs.

Sales revenue: 10,00,000 13,11,180Materials 587,500 718,350Cutting costs (i) 50,000 81,360Cutting costs (ii) 120,000 166,530

757,500 966,240Operating profit 242,500 344,940

Part ii : The non-quantitative factors to be considered:

2. Reliability of estimates of costs, sales, prices and demand.

4. Impact on production and sales of other products of ze-Te.

The company can make an additional profit of Rs. 102,440 if matching caps and handbags are provided with the dresses. The company is advised to go with this option.

1. The matching should be proper and appealing. The availability of matching material with adequate quantity be ensured

3. Production planning and distribution, display and supply of all the four combinations should be timed properly.

·       cqf)ekuksa ij ew[kksZa ds galus dh izFkk nqfu;k esa gj txg gS-Problem 11 (cwa final June 08)

A transport company operates a bus service on 25 km long route. Purchase price of the chassis was Rs. 8.5 lacs and it was furnished at a cost of Rs. 1.5 lacs. It will have a residual value of Rs. 2 lakh at the end of 5 years of normal working life.

Annual Road Tax amounts to Rs. 24,000, while insurance premium is valued at 2.5% of the total cost of the bus. Annual repairs is estimated at Rs. 30,000. Garage rent is payable @ Rs. 4,000 per month. Tyre and tube will cost 50 paisa per km.

Page 257: cost accounting question icwai inter

The bus will make 4 round trips per day for 25 days in a month, carrying 42 passengers in each trip.

Assuming 20% profit on takings, prepare(a) Operating Statement for a year, indicating Fixed and Variable cost, and(b) Cost per passenger-kilometer, with working notes.Solution:

You should begin with making working notes :Working Notes 1: Commission of driver and conductor:

Working Notes 1: Computation of Total Passenger Kilometers:Length of one trip 25 km.One round trip ( to and fro) 50 km.Four times a day 200 km.For 25 days in 12 months 60,000 km.In each trip, 42 passengers are carried.Total no. of passenger kilometers = 60,000 x 42 = 25.20 lac passenger kilometers.Working Notes 2: Computation of DepreciationCost to be depreciation in 5 years : Cost of Chassis + Cost of furnishing less residual cost

: (8.5 +1.5 -- 2.0) = Rs. 8 lacs @ Rs. 160,000 per year.

Working Notes 3: Segregation of Fixed cost and Variable cost Fixed costs : Rs. per passkmDepreciation 160,000Road Tax 24,000Insurance @ 2.5% on cost of bus 25,000Annual Repairs 30,000Garage Rent @ Rs. 4,000 per month 48,000Salary of driver Rs. 3,500 x 12 42,000Salary of conductor Rs. 2,500 x 12 30,000Administrative expenses 47,000Fixed cost per passenger km (406,000 / 25,20,000) 406,000 0.16Variable cost (i) Operating expenses :Diesel, Oil and Lubricants @ Rs. 4.50/ km for 60,000 km. 270,000Tyre and Tube @ 50 paise per km. 30,000

Salaries of one driver and one conductor will be Rs. 3,500 and Rs. 2,500 per month respectively. In addition, they will receive 10% of the collection as commission, which will be shared between the driver and conductor in the ratio 60:40.

Other administrative salaries and expenses will cost Rs. 47,000 per annum. Diesel, oil and lubricants will cost @ Rs. 4.50 per km.

Tutorial notes: The solution is very easy if you are not confused about 'total takings'. Total takings implies the total revenue that the project is likely to receive during the concerned period.

It is stated that the driver and conductor are entitled to commission of 10% of total takings. Total takings mean total revenue including profit of the project. Thus Total takings will consist of three parts namely profit 20%, commission 10% and 70% as cost. As the total takings is not known, you should find the cost first, then divided it by 0.70 to find total takings from which the profit and commission can be computed.

Working Notes 3: Insurance charges would be @ 2.5% on total Cost i.e. Rs. 10 lacs and NOT on Rs. 8 lacs which is being depreciated. Insurance premium Rs. 25,000.

Page 258: cost accounting question icwai inter

300,000 0.12Total cost of operation (70% of total takings) 706,000Total Takings (706000 / 0.70) 1008571 0.40Commission to driver and conductor 10% of total takings 100,857 0.04

Profit @ 20% of total takings 201,714 0.08

The sale price of product A and product B are as under :Price per unit Rs. Wholesale Retail

Product A 90 180Product B 99 198

Solution: Tutorial Notes:

Computation of demand and margin when B is produced: in lakh unitsCurrent market size 50Current market size of A (40% of market size) 20Current market size of B (45% of market size) 22.5Less : Switching because of higher price (30% of existing market of A of 20) 6.0

Total Demand for Product B 16.5Total margin (selling price -- cost of manufacture -- Sale and admin.cost) Rs. Lakh( Rs. 99 -- Rs. 67 -- Rs. 20) Margin Rs. 12 x 16.5 lakh) 198Computation of margin when A is continued to be produced:

Margin per unit Rs. 10 ( 90 -- 60 -- 20)Total margin @ Rs. 10 per unit for market size of 20 lakh unit 200Additional margin if A is continued as such 2

Recommendation : The company is advised to continue with A as such.

Problem 12 (cwa final dec.06)

A company is currently producing a product called product A and the unit cost of manufacturing is Rs. 60. Some improvement is proposed which will increase the unit cost of manufacture of the improved product called product B to Rs. 67. Additionally the sale and administration cost will be Rs. 20 per unit for both products.

The improvement in product A will increase the total market of this type of product by 5% from the current market size which is for 50 lakh pcs. The current market share of product A is 40% which would remain as such, if the manufacture continues to sell product A. However, it has been feared that with increased selling price of the proposed product B, 30% of the existing customer of product A will switch over to other products marketed by competitors at a cheaper price although such products will not have improved features. Both product A and product B would require to be updated after every year in a rapidly changing market.

You are required to give your opinion with relevant calculations as regards which product the company should produce.

1. Two selling prices of each product have been given. Which should be used for computation ? Since it is to be decided as to which products should be produced, it would be prudent to take the lesser of the two in consideration. Thus wholesale price will be considered and the retail price being higher, would be ignored.

2. The production of B would reduce the sales volume because of switching of old customers. The demand should be computed in working notes.

Problem 13: (cwa final June 03)

Page 259: cost accounting question icwai inter

The budget for calendar year 2003 was prepared in Dec.2003

The plan to buy the office would be implemented but on 1st October 2003.You are required to present:(a) the original budgeted Profit and loss statement for the entire year 2003.(b) the actual profit and loss statement for the first half year.(c) the revised budgeted profit and loss statement for the second half year.

Solution: Tutorial Notes:

Nomad Travels is a travel agent and tour operator in India, engaged in two activities viz. domestic ticketing and inbound tours. In the inbound operation, foreign tourists visit India on package tours Nomad Travels arranges for their travel as well as boarding and lodging and other facilities required for a comfortable stay.

Nomad Travels was expecting an inbound revenue of $ 1 million for the year 2002. Ticketing commission in the domestic industry were expected to be Rs. 50 lakhs. Dollar rate for this purpose was assumed at Rs. 50.

On the expenses side, Nomad Travel's operating expenses were budgeted at Rs. 10 lakhs per annum. On the inbound tour packages, Nomad Travels would typically spend 70% of the revenue on various costs including hotel, vehicle hire, domestic travel and miscellaneous items. Hotel would comprise half of this cost, vehicle 20%, domestic travel 20% and miscellaneous items 10%.

The first half year was expected to generate 25% of the revenue and the second half the balance (on inbound tour activity). Domestic ticketing was expected to be uniform through the year.

Nomad Travels pays income tax in four equal installments at 30% of their profit after considering export benefits. For the purpose of the tax working, profits would be considered as per latest actual data for each year and the latest budget available at that time.

Nomad Travels was considering buying an office for Rs. 45 lakhs of which Rs. 30 lakhs would be funded through a bank loan. This was to be effected on 1st July 2003. This loan would attract interest at 10% per annum and is payable quarterly. Repayment of this principal would be required to be effected every quarter end over a 5 year period.

Nomad Travels' regular banking limits are Rs. 100 lakhs, carrying interest at 12% per annum debited at the end of each quarter. Limits are generally utilized fully.

During the course of the year, various adverse factor affected the industry including the Iraq war and the SARS virus.

In the first half of the year, inbound traffic was down 20% and dollar rate was Rs. 48. Hotels reduced their rates by 10% due to low customer occupancy and Nomad Travels benefited to some extent. Other tour costs remained at budgeted levels. Operating costs, in fact, increased by 2% due to under budgeting in December 2002. Domestic air traffic, activity remained 3% above budget.

For the second half year, Nomad Travels is conservative about its projections now. It believes that revenue achievement on inbound traffic will be 80% of the original budget and domestic traffic will remain 3% above budget. Dollar rate would rise to Rs. 49.

Operating costs would increase by 3% over the original budget. Hotels would offer discounts of 10% of their earlier quoted levels, while other inbound costs would remain at budgeted levels.

· Look people in the eyes.

1. This is a very long and narrative question. In first reading you may not be able to understand properly. Generally lengthy questions are simple in solution and this problem is no exception.

Page 260: cost accounting question icwai inter

9. Next Para has nothing to do with solution. Skip it.

12. It is advisable to use figures in lacs to reduce labour and save time.

Solution goes as follows: Nomad TravelsProfit and Loss Statements fig. in lakhs

Original Original Original Actual RevisedBudget Budget budget budget

Full year 1st half 2nd half 1st half 2nd half

2. Read the question twice. During the second reading you must skip first Para as it has nothing to do with solution. As you read along, start preparing working notes on every point you come across.

3. In third Para, It must be year 2003 although the year 2002 is printed. It is expected of you to make a note on this. You also get two figures Rs. 500 lakhs budgeted revenue for 2003 on inbound tours and domestic ticketing commission is Rs. 50 lakhs. You should know one million = 10 lakhs.

4. Next Para says that the operating cost is Rs.10 lakhs per month. Thus total operating cost per annum would be Rs. 120 lakhs. Next you find that inbound tours has costs @ of 70% of revenue. Thus cost would be Rs. 350 lakhs (70% of Rs. 500 lakhs). This cost should be distributed to hotels (50%), vehicle (20%), domestic travel 20% and misc. items (10%). With this you can compute the gross profit of inbound tour envisaged in the original budget. You can calculate the gross profit at this stage as Rs.80 lakhs. (500 + 50 -- 120 -- 350 = 80 lakhs). Solution to part (a) is ready.

5. In next Para, you come across income tax payment @ 30%. Nothing can be done at this stage. Leave this Para for a while. When you have the total profit ready, you come to this Para again.

6. Next Para is related to capital expenditure. In first reading you must have gathered that profit and loss account is to be prepared. Capital expenditure is not related with profit and loss account neither any depreciation figures or methods have been mentioned, so depreciation is not to be computed.

7. The interest figures on loan will present some thinking because the question says that interest along with the principal is paid quarterly. Thus interest of both the quarters will not be same but will be different. You should calculate the interests for both the quarters at this stage. The principal of Rs. 30 lakhs is paid quarterly in 5 years time. Thus 20 installments are to be paid for the principal. The installment per quarter comes to Rs.1.50 lakhs. Thus interest for first quarter from July 1,03 to Sept.30,03 is on Rs.30 lakhs @ 10% per annum (or 2.5% per quarter) i.e. Rs. 75,000 (2.5% of 30) while interest on second quarter from Oct.1,03 to Dec.31,03 will be calculated on Rs.28.5 lakhs (30 -- 1.5) and comes out to be Rs.71,250 (2.5% of 28.50). Total interest from July 1,03 to Dec.31,03 is thus Rs.1,46,250. You can prepare a table showing all these calculations.

8. Limits of Rs. 100 lakhs @ 12% is utilised fully. This means Rs. 12 lakhs is paid per annum on account of interest. This figure should go to the gross profit of inbound tours prepared in step 4. You deduct this amount and get the figure of profit before tax. Apply tax @ 30% to arrive at profit after tax.

10. Next Para is about actual situation in first half of 2003. This is related to part (b) of the question. You have already prepared answer to part (a). You should apply the given % to get the actual for budgeted figures. Same operations to be done on second half for projections.

11. You should prepare original budget first. Then budgeted first and second half of year in the ratio of 25% and 75% followed by actual figures of both the halves. Domestic commission and operating costs should be divided equally in two halves in budgeted figures.

Page 261: cost accounting question icwai inter

In bound toursGross Revenue (lac dollars) 10 25 75 20 60Dollar rate (Rs.) 50 50 50 48 49Gross Revenue (Rs.. lacs) 500 125 375 96 294Costs (Rs. in lakhs) 25% 75%Hotels 175 43.75 131.25 31.50 94.5Vehicles 70 17.50 52.50 14.00 42Domestic travels 70 17.50 52.50 14.00 42Misc.Items 35 8.75 26.25 7.00 21Total costs 350 87.50 262.50 66.50 199.5Gross Profit 150 37.50 112.50 29.50 94.5Domestic ticketing commissi 50 25.00 25.00 25.75 25.75Total Gross earnings 200 62.50 137.50 55.25 120.25Less: Operating cost 120 60.00 60.00 61.20 61.8Operating profit 80 2.50 77.50 (5.95) 58.45Interests:Bank limits 12 6.00 6.00 6 6Loan for Office 1.47 0.00 1.47 0 0.75Total interest 13.47 6.00 7.47 6 6.75Profit before tax 66.53 (3.5) 70.03 (11.95) 51.7Tax @ 30% 19.96 9.98 9.98 9.98 1.94Profit after tax 46.57 49.75Working Note:1. Taxation figures are as follows: First half actual (11.95)

Second half projections 51.7Total Income 39.75Tax @ 30% 11.92Tax already paid in first half 9.98Balance to be paid 1.94

(iv) Share of General Building expenses: Rs. 35,000.

Problem 14: (ca final May 07)

A research project, to date, has cost a company Rs. 2,50,000 and is under review. It is anticipated that, should the project be allowed to proceed, it will be completed in about one year and can be sold for Rs. 4,00,000. The following information is available:

(i) Materials have just been received for Rs. 60,000. These are extremely toxic, and if not used in the project, have to be disposed of by special means at Rs. 15,000.

(ii) Labour: Rs. 75,000. The men are highly skilled. If they are released from the Research project, they may be transferred to the Works Department of the company and consequently the sales could increase by Rs. 1,50,000. The accountant estimates that the prime cost of those sales would be Rs. 1,00,000 and the overhead absorbed (all fixed) would amount to Rs.25,000.

(iii) Research Staff: Rs. 1,60,000. A decision has already been taken that this will be the last major piece of research undertaken and consequently, when work on the project ceases, the staff involved will be made redundant. Redundancy and severance pay have been estimated at Rs.25,000.

The managing director is not sure what is included in this amount, but the account staff charge similar amount each year to each department.

You are required to advise whether the project should be allowed to proceed and explain the reasons for the treatment of each of the amounts above in your analysis.

Page 262: cost accounting question icwai inter

Solution : Tutorial Notes:

1. The project cost till date is a sunk the cost, it is therefore not relevant for any decision making.

In short the following table can be presented :Particulars Relevancy ReasonProject cost till date Irrelevant Sunk costCost of material received Irrelevant Sunk costCost of labour Irrelevant Common costAbsorbed Fixed overheads Irrelevant Sunk costRedundancy and severance Irrelevant Common costShare of General building expenses Irrelevant Sunk cost

Rs. Sale price of the project 400,000Less: Contribution lost due to alternative use 125,000

Cost of Research staff 160,000Add: Cost of saving due to disposal of material 15,000Additional revenue if the project is continued 130,000Recommendation : The company is advised to continue with the project.

Material A 600 unitsMaterial B 1,000 unitsDirect labour 5,000 hours of unskilled

5,000 hours of skilled5,000 hours of highly skilled

Extra selling and delivery expenses Rs. 27,000Extra Advertising Rs. 18,000

2. The materials worth Rs. 60,000 is a sunk cost and is irrelevant. The disposal of unused material will cause an expense of Rs. 15,000 is relevant for decision making. If the project is continued, this cost would be saved. The project is abandoned, this cost would have to be incurred.

3. Cost of labour of Rs. 75,000 is a relevant cost. If the project is continued, the labour cost of Rs. 75,000 would have to be incurred and simultaneously a contribution of Rs.50,000 will have to be lost. Thus it will cost Rs. 125,000 to continue the project. If the research project is abandoned, this would give an extra contribution of Rs. 50,000 (Sales -- Prime cost).

4. The research staff cost of Rs. 160,000 is relevant. The severance cost of Rs.25,000 is not relevant because the staff has to be retrenched whether the project is continued or not.

5. Share of building expenses is not relevant, it is charged on the research project and if the project is terminated, this will be charged to some other account.

Problem 15: (ca final Nov. 2000)

B Ltd. is a company that has, in stock, materials of type XY that cost Rs. 75,000, but that are now obsolete and have a scrap value of only Rs. 21,000. Other than selling the material for scrap, there are only two alternative uses for them.

Alternative 1 : Converting the obsolete materials into a specialized product, which would require the following additional work and materials :

The conversion would produce 900 units of saleable product and these could be sold for Rs. 300 per unit.

Page 263: cost accounting question icwai inter

Material A Material BRs. Rs.

Acquiring cost at the time of purchase 100 per unit 10 per unitNet realisable value 85 per unit 18 per unitReplacement cost 90 per unit ---

Material C 1,000 unitsDirect labour 4,000 hours unskilled

1,000 hours semi-skilled4,000 hours of highly skilled

Rs.Direct labour, 6 hours of unskilled labour 18Raw materials 13Variable overhead : 6 hours at Re. 1 6Fixed overhead: 6 hours at Rs. 3 18

55The wage rate and overhead recovery rates for B Ltd. are :

Rs.. Per direct labour hr.Variable Overhead 1Fixed Overhead 2Unskilled labour 3Semi-skilled labour 4Highly skilled labour 5

Material A is already in stock and is widely used within the firm. Although present stocks together with orders already planned, will be sufficient to facilitate normal activity and extra material used by adopting this alternative will necessitate such materials being replaced immediately.

Material B is also in stock, but it is unlikely that any additional supplies can be obtained for some considerable time, because of an industrial dispute. At the present time material B is normally used in the production of product Z, which sells at Rs. 390 per unit and incurs total variable cost (excluding material B) of Rs. 210 per unit. Each unit of product Z uses four units of material B. The details of Material A and Material B are as follows:

Alternative 2 : Adopting the obsolete materials for use as a substitute for a sub-assembly that is regularly used within the firm. Details of the extra work and materials required are as follows :

1,200 units of the sub-assembly are regularly used per quarter at a cost of Rs. 900 per unit. The adaptation of Material XY would reduce the quantity of the sub-assembly purchased from outside the firm to 900 units for the next quarter only. However, since the volume purchased would be reduced, some discount would be lost and the price of those purchased from outside would increase to Rs. 1,050 per unit for that quarter.

Material C is not available externally though, 1,000 units required would be available from stocks, it would be produced as extra production. The standard cost per unit of Material C would be as follows:

Page 264: cost accounting question icwai inter

Solution :Tutorial Notes:

4. You should consider one alternative at a time. Let us first consider alternative 1.Alternative 1:

3. Direct labour cost of unskilled labour is indirectly given as Rs. 3/ hr. Try to find out where it is given.

The unskilled labour is employed on a casual basis and sufficient labour can be acquired to exactly meet the production requirements. Semi-skilled labour is part of the permanent labour force, but the company has temporary excess supply of this type of labour at the present time. Highly skilled labour is in short supply and cannot be increased significantly in the short-term, this labour is presently engaged in meeting the demand for the product L, which requires 4 hours of highly skilled labour. The contribution from the sale of one unit of product L is Rs. 24.

Given the above information, you are required to present cost information advising whether the stocks of Material XY should be sold, converted into a specialized product (alternative 1) or adopted for use as a substitute for a sub-assembly (alternative-2).

1. This is a very long and narrative question. In first reading you may not be able to understand properly. Generally lengthy questions are simple in solution and this problem is no exception. You should always keep time factor in mind while solving such lengthy question in the examination.

2. You should make notes along with reading the long questions. The purchase cost of XY is Rs. 75,000 which is sunk cost and therefore irrelevant for decision making.

3. XY can be sold at Rs. 21,000, whatever alternative is selected the revenue of Rs. 21,000 will have to be forgotten. This being the same for both the alternatives, is not relevant for decision making. However, for computing the cost and incremental revenue of the alternatives, the opportunity cost of Rs. 21,000 should be taken into account.

1. Material A is amply available but if it is used in specialized product it has to be replaced at a replacement cost of Rs. 90 per unit. Thus replacement cost of Rs. 54,000 (600 x Rs. 90) is relevant for decision making. The acquisition cost of Material A is a sunk cost and is not relevant.

2. 1,000 units of Material B will be used which can otherwise contribute to produce 250 units of Z ( 1,000 / 4). Each unit of Z contributes Rs. 180 (390 -- 210). Thus a contribution of Rs. 45,000 ( 250 x Rs. 180) will have to be sacrificed if alternative 1 is adopted. This is a relevant cost.

4. Semi skilled labour hours of 5,000 are required but the semi skilled labour is in excess supply, this makes the cost of semi skilled labour as irrelevant.

5. High skilled labour is in short supply. Alternative 1 requires 5,000 hours of High skilled labour which can otherwise produce 1,250 units of L ( 5,000 / 4). Each unit of product L gives contribution of Rs. 24. thus the contribution lost due to this alternative would be Rs. 30,000 ( 1,250 x Rs.24). Moreover, the wage rate of Rs. 5 per hour will have to be paid on account of alternative 1, hence wage payment of Rs. 25,000 is also relevant.

Page 265: cost accounting question icwai inter

7. The revenue generated under alternative 1 is Rs. 270,000 ( 900 units at Rs. 300 per unit).8. On the lines given above you can evaluate the alternative 1. Please do it yourself.Alternative 2:

3. Direct labour cost of unskilled labour is Rs. 3/ hr. Wage payment of Rs. 12,000 is relevant.

4. Semi skilled labour hours are not relevant as they are in excess supply.

7. Evaluate this alternative on the above lines. Evaluation of Alternative 1: Rs. Rs.Sales revenue 900 units @ Rs. 300/unit 270,000Less : Relevant cost Opportunity cost of Material XY 21,000Replacement cost of Material A 600 units @ Rs. 90/unit 54,000Contribution lost of Z 250 units @ Rs.180/unit 45,000Wage payment to unskilled labour 5,000 hrs.@ Rs. 3/hr. 15,000Wage payment to semi-skilled labour 0Wage payment to highly skilled labour 5,000 hrs @ Rs. 5/unit 25,000Contribution lost of highly skilled labour 1,250 units @ Rs. 24/unit 30,000Variable overhead 15,000 hrs @ Rs. 1/hr. 15,000Extra selling and delivery expenses 27,000Extra Advertising 18,000 250,000Excess of relevant revenue 20,000Evaluation of Alternative 2:Saving in purchase of sub-assembly :

Normal cost of purchase 1,200 units @ Rs.900 10,80,000Less: Revised cost of purchase 900 units @ Rs.1,050 9,45,000

Saving 135,000Less : Relevant CostOpportunity cost of Material XY 21,000Variable cost of Material C 1,000 units @ Rs. 37 37,000Unskilled labour 4,000 hrs @ Rs. 3 12,000

6. The recovery of fixed overhead is not at all relevant. The variable overhead of Rs. 1/ per direct labour hour is assumed to be related to alternative. Hence recovery of variable overhead of Rs. 15,000 is relevant. Semi-skilled labour hours are assumed to qualify for recovery.

1. It would require 1,000 units of Material C which requires a variable cost of Rs. 37/unit to produce. The fixed overhead of Rs. 18 is not relevant. The cost of production of C of Rs. 37,000 is relevant.

2. The cost being incurred for the purchase of 1,200 units of the sub-assembly is Rs. 10,80,000 (1,200 units at Rs. 900 per unit). The next quarter cost is Rs. 9,45,000 ( 900 units at Rs. 1,050 per unit). The saving on cost of purchase of Rs. 1,35,000 (10,80,000 -- 9,45,000) would be relevant.

5. 4,000 hours of high skilled labour would be required for this alternative. This can produce 1,000 units of L. The contribution lost due to this is Rs. 24,000. This is relevant. The wage payment of Rs. 20,000 ( 4,000 hours at Rs. 5 per hour) is relevant.

6. The recovery of fixed overhead is not at all relevant. The variable overhead of Rs. 1/ per direct labour hour is assumed to be related to alternative. Hence recovery of variable overhead of Rs. 15,000 is relevant. Semi-skilled labour hours are assumed to qualify for recovery.

Page 266: cost accounting question icwai inter

Semi-skilled labour 0Highly skilled labour 4,000 hrs @ Rs. 5 20,000Opportunity cost of Material L 24,000Variable overhead 9,000 hrs @ Rs. 1 9,000 123,000Excess of relevant revenue 12,000Recommendation : Rs.

Excess of relevant revenue Alternative 1 20,000Alternative 2 12,000

Net revenue of Alternative 1 8,000

Rs.Original cost estimate 175,200Costs incurred so far 148,500Costs to be incurred 29,700Progress payment received from original customer 100,000

Rs.Direct materials at cost 1,050Direct wages Deptt. A 15 man days

Deptt. B 25 man daysVariable overhead 25% of direct wages in each deptt.Delivery costs 1,350Fixed overhead will be absorbed at 50% of direct wages in each deptt.The following additional information is available :

4. Supervisory overtime payable for the modification is Rs. 1,050.

Because of higher revenue of alternative 1 to the extent of Rs. 8,000, it is recommended to convert material XY into a specialized labour.

Problem 16: (ca final May. 2001)

A company has nearly completed a job relating to construction of a specialized equipment, when it discovered that the customer had gone out of business. At this stage the position of the job was as under:

After searches, a new customer for the equipment has been found. He is interested to take the equipment, if certain modifications are carried out. The new customer wanted the equipment in its original condition, but without its control device and with certain other modifications. The costs of these conditions and modifications are estimated as under:

1. The direct materials required for the modification are in stock and if not used for modification of this order, they will be used in another job in place of material that will now cost Rs. 2,250.

2. Department A is working normally and hence any engagement of labour will have to be paid the direct wages rate of Rs. 120 per man day.

3. Department B is extremely busy. Its direct wages rate is Rs. 100 per day and it is currently yielding a contribution of Rs. 3.20 per rupee of direct wages.

5. The cost of control device that the new customer does not require is Rs. 13,500, if it is taken out, it can be used in another job in place of a different mechanism. The latter mechanism has otherwise to be bought for Rs. 10,500. The dismantling and removal of the control mechanism will take one man day in department A.

Page 267: cost accounting question icwai inter

Solution :Tutorial Notes:

10. You should prepare all the working notes first and use the figures in your final presentation.

Please frame your solution on the above lines and compare it with the one given below:The solution goes as follows:Working Notes: 1. Cost of Control Device : Rs.Cost saved if control device is used in another job 10,500

6. If the conversion is not carried out, some of the materials in the original equipment can be used in another contract in place of materials that would have cost Rs. 12,000. It would have taken 2 man days of work in department A to make them suitable for this purpose. The remaining materials will rallies Rs. 11,400 as scrap. The drawing, which are included as part of the job can be sold for Rs. 1,500.

You are required to calculate the minimum price, which the company can afford to quote for the new customer as stated above.

1. You should read the question attentively and look for the items and associated costs which are relevant for decision making. All costs which have already been incurred are irrelevant.

2. The original cost estimate, cost incurred so far have become irrelevant because they belong to past. The cost of Rs. 29,700 has to be incurred to complete the equipment, hence it is relevant. Progress payment made by the customer is also irrelevant.

3. The original cost of direct material is Rs. 1,050 which is not relevant. This material can be used in place of material which will now require Rs. 2,250 if purchased. Use of this material will require Rs. 2,250 to be spent on purchase of material hence it is relevant.

4. Direct labour of Dept. A will have to be paid Rs. 1,800 (15 man days @ Rs. 120). This is relevant cost.

5. The labour of Dept. B is yielding Rs. 3.20 as contribution for each rupee of labour cost. This contribution will have to be foregone if labors are used elsewhere. The cost of labour is Rs. 2,500 (25 man days @ Rs.100) which yield contribution of Rs. 8,000 (2,500 x 3.20). The contribution of Rs. 8,000 is relevant and the wage payment of Rs. 2,500 is also relevant because it is directly associated with the work.

6.The cost of control device is Rs. 13,500 which is irrecoverable hence it is irrelevant. But this can save Rs. 10.500 in another job. Dismantling will require Rs. 120 (one man day in dept. A). Don't forget to add variable cost @ 25% of labour cost in the dismantling cost. Thus net cost which can be saved is Rs. 10,350 (10,500 less Rs. 120 less Rs. 30), which is relevant.

7. The opportunity cost of Rs. 12,000 less associated expenses (2 man day in dept. A costing Rs. 240 and variable overhead @ 25% i.e. Rs. 60) is relevant. The net opportunity cost comes to Rs. 11,700.

8. Delivery cost of equipment of Rs. 1,350 is naturally relevant. Same is true for supervisory cost of Rs. 1,050.

9. You are likely to forget the variable overhead in computations. Variable overheads are to be recovered @ 25% of direct wages. All payments of direct wages should include the variable overhead.

Page 268: cost accounting question icwai inter

Less : Dismantling cost (one man day at Rs. 120) 120Variable overhead @ 25% 30 150Net cost saved 10,350

2. Opportunity cost of materialCost saving in another job 12,000

Less : Conversion cost (2 man days at Rs. 120) 240Variable overhead @ 25% 60 300Net Opportunity cost 11,700

Statement of Minimum price Rs.Cost to be incurred to bring the equipment in original condition 29,700Opportunity cost of the direct material 2,250Direct wages : Dept. A 15 man days @ Rs. 120 1,800

Dept. B 25 man days @ Rs. 100 2,500Contribution to be lost in dept.B @ Rs. 3.20 of direct wages (2,500 x Rs. 3.20) 8,000Variable overhead @ 25% of Rs. 4,300 ( 2,500 + 1,800) 1,075Delivery cost 1,350Supervisory cost 1,050Net opportunity cost of material 11,700Opportunity cost of material which can be sold as scrap 11,400Opportunity cost of drawing which can be sold 1,500

72,325Less : Cost saving due to use of control device 10,350

The minimum price which can be quoted 61,975

Resources per kg of XLabour

Grade 1 2 hoursGrade 2 6 hours

MaterialsA 2 unitsB 1 liter

The cost to A for each type of labour are:Grade 1 Rs.40 per hour, Grade 2 Rs. 20 per hour.

Problem 17: (ca final May. 2002)

A limited has been offered a contract that, if accepted, would significantly increase next year's activity level. The contract requires the production of 20,000 kgs of product X and specifies a contract price of Rs. 1,000 per kg. The resources required in the production of each kg. of X include the following:

Grade 1 labour is highly skilled and although currently under-utilized in the firm, it is A's policy to continue to pay Grade 1 labour in full. Acceptance of the contract would reduce the idle time of Grade 1 labour. Idle time payments are treated as non-production overheads. Grade 2 is unskilled with a high turnover, and may be considered a variable cost.

The materials required to fulfill the contract would be drawn from the materials already in stock. Material A is widely used within the firm and any usage for the contract will necessitate replacement. Material B was purchased to fulfill an expected order that was not received. If material B is not used for the contract, it will be sold.

Page 269: cost accounting question icwai inter

For accounting purposes FIFO is used. The various values and costs for A and B are as follows:

A BRs./unit Rs./unit

Book value 8 300Replacement cost 100 320Net realisable value 90 250

Variable production overheads are accurately estimated at Rs. 30 per productive labour hour.

Per unitSelling price in Rs. Rs. 700Labour Grade 2 4 hoursMaterials relevant variable costs Rs. 120

Required :

Solution : Tutorial Notes :

A single recovery rate for fixed factory overheads is used throughout the firm, even though some of these costs could be attributed to a particular product or department. The overhead is recovered by applying a pre-determined rate per productive labour hour. Initial estimates of next year's activity, which exclude the current contract, show fixed production overhead of Rs. 60,00,000 and production labour hours of 3,00,000. Acceptance of the contract would increase fixed production overheads by Rs. 22,80,000.

Acceptance of the contract would encroach on the resources used to produce and sale another product Y, which is also made by A limited. It is estimated that the sale of Y would then decrease by 5,000 units in the next year only. However, this reduction in sale of Y would enable attributable fixed overhead of Rs. 5,80,000 to be avoided. Information on Y is as follows:

Advise A limited on the desirability of the acceptance of the contract purely on economic consideration. Show your calculations.

·  Be pretty dressed. Maintain correct posture.

1. This is related to relevance of costs. Let us take each item and discuss its relevance to the contract under consideration.

2. Labour Grade 1. The labour is going to be paid by the firm whether it is engaged in the contract or not. Thus payment of labour Grade 1 is not relevant. Labour cost for this grade would remain same whether the contract is accepted or not.

3. It is mentioned that labour Grade 2 should be considered as variable cost. Payment on this account is relevant.

4. Material A will have to be replaced at Rs. 100/ unit. The replacement cost of material A to be used in the contract is relevant and should therefore be taken into account.

5. Material B will be sold at net realisable value if not used in the contract. Its realisable value is relevant for the contract.

6. Existing fixed overheads are not at all related to the contract hence they are irrelevant. However, the fixed overheads due to the contract of amount Rs. 22,80,000 are relevant.

7. Variable production overheads at Rs. 30/ labour hour are relevant. You should take the entire labour hours of both the grades for the computation of variable production overheads although the labour cost of grade 1 is not relevant to the contract but the variable cost due to this is relevant.

Page 270: cost accounting question icwai inter

8. Fixed overheads of Rs. 5,80,000 can be avoided due to the contract. This is naturally relevant.

After the above discussion, the relevant working notes can be prepared.Working notes :1. Relevant cost of labour :

Labour Grade 1: It is not relevant. No calculations are requiredLabour Grade 2: It is treated as variable cost. Rs. LacsLabour Grade 2 : 20,000 kg x 6 hr per kg. x Rs.20 per hour 24.0

2. Relevant cost of materials :Material A : It will have to replaced at Rs. 100 per unit.

Cost : 20,000 kg x 2 units x Rs. 100 / unit 40.0Material B : If not used, it will have to be sold at net realizable value.

Cost : 20,000 kg x 1 liter x Rs. 250 / liter 50.090.0

3. Fixed overheads to be avoided due to contract 22.84. Total labour hours required 20,000 x 8 = 160,000 hours

Variable cost due to labour hours 160,000 x Rs. 30 48.05. Fixed cost that can be avoided due to the contract 5.86. Contribution lost due to reduction of sale of Y by 5,000 units

Rs. / unitSelling price of Y 700Less : Labour Grade 2 of 4 hours @ Rs. 20/hour 80Less : Material cost 120Less : Variable production overheads 120 320Contribution lost due to acceptance of contract 380Total contribution lost in 5,000 units in Rs. Lacs 19.0Answer : Statement of desirability of acceptance of contractSales revenue ( 20,000 kg x Rs. 1,000) 200.0Add : Savings in fixed cost 5.8Less : Relevant cost of labour 24.0

Relevant cost of material 90.0Variable cost due to labour hours 48.0Incremental fixed cost 22.8Contribution lost on Y 19.0 203.8

Net revenue expected from the contract 2.0

9. The contribution lost due to reduction of sale of product Y by 5,000 units is due to the acceptance of the contract, hence it is relevant. You should not forget to take variable production overheads of Rs. 30/ labour hour in this analysis.

Note : Even the cost of labour of Grade 1 is not relevant, but variable overheads due to labour hours are relevant.

Conclusion : From the above statement it is evident that the acceptance of this contract will enhance operating income by Rs. 2.0 lacs, A Limited is advised to accept the contract.

Future is not something you await; It is something you create! Plan your future well in advance because that's where you will spend the rest of your life.

Page 271: cost accounting question icwai inter

mB cka/k dej D;ksa Mjrk gS] fQj ns[k [kqnk D;k djrk gS-

Page 272: cost accounting question icwai inter

2. Administrative expenses per bus per km can be computed as follows:

Page 273: cost accounting question icwai inter

gekjs ckjs esa gekjs nq'euksa ds fopkj gels T;knk lgh gksrs gSa-

Page 274: cost accounting question icwai inter

2. Net operating income if fare is reduced to Rs. 6720 with occupancy 108 passengers

Page 275: cost accounting question icwai inter

A truck carries 500 cases for both inbound and outbound shipments.

Page 276: cost accounting question icwai inter

2. Reliability of estimates of costs, sales, prices and demand.

Page 277: cost accounting question icwai inter

12. It is advisable to use figures in lacs to reduce labour and save time.

Page 278: cost accounting question icwai inter

(iv) Share of General Building expenses: Rs. 35,000.

Page 279: cost accounting question icwai inter

Chapter 9 : Marginal Costing cwa inter cwa final ca inter ca final Total31 9 1 6 47

Solution: Pv ratio is ratio of contribution/ sales. This is also ratio of Profit to Margin of safety.Thus we have,Pv Ratio- = 40% = Profit / Margin of safety = 30 / Mos Margin of safety = 30/ 0.40 = 75 lacs.

Solution: Rs.Profit before depreciation and interest 10% of sales 20 lakhsLess: depreciation 10 lakhsInterest 15 lakhs (25 lakhs)Loss 5 lakhsc/s ratio 40% i.e. Contribution = 40% of 200 lakhsContribution 80 lakhsFixed cost = Contribution + loss ( 80 + 5) 85 lakhs

Solution:

C/S ratio = Change in profit / Change in Sales = 16,000 / 40,000 = 40%.Problem 4 (cwa inter dec 05)

Total fixed cost of selling, distribution and administration is estimated to be Rs. 300,000 per month.

Further details relating to the products are given below:Product Demand Selling cost/unit Raw material Direct labour

units Rs./unit kg/unit hoursA 8,000 100 2.0 1.0B 6,000 120 2.5 1.2C 5,000 160 3.0 2.0D 3,000 220 4.0 3.0E 2,000 300 5.0 4.0Required:(a) Contribution analysis statement showing the relative profitability of the products under:

(i) normal condition without any constraint on resources

·       bZ";k] Mj] ruko] uQjr vkSj faprk vkids dksf'kdkvksa vkSj ra= dks detksj ;k u"V dj nsrh gSa ftlds dkj.k 'kkjhfjd vkSj ekufld chekfj;ka mRiUu gksrh gSaA fdlh ls u fp<+ss] fdlh ls uQjr u djsa] fdlh ij xqLlk u gksaaA 'kkafr ls lks;saA [kq'kh ls tkxsaAProblem 1: cwa Inter.June 05

A company maintains a margin of safety of 25% on its current sales and earns a profit of Rs. 30 lakhs per annum. If the company has a profit volume ratio of 40%, what is the current sales?

As given in the question, the margin of safety is 25% of the sales. Hence the current sales must be 75/ 0.25 = 300 lacs.

Problem 2: cwa Inter.dec. 05

A company has annual turnover of Rs. 200 lakhs and an average c/s ratio of 40%.It makes 10% of profit on sales before charging depreciation and interest which amount to Rs. 10 lakhs and Rs. 15 lakhs respectively. Find the annual fixed cost of the company.

Problem 3: cwa Inter.dec. 05

Sales for two consecutive months of a company are Rs. 380,000 and Rs. 420,000. The company's net profits for these two months amounted to Rs. 24,000 and Rs. 40,000 respectively. There is no change in C/S ratio or fixed costs. Find the c/s ratio of the company.

·       Small problems small achievements. Big problems big achievements. What do you select ?

A company has plans to manufacture five different types of product using a common raw material which is locally available according to requirements at Rs. 16 per kg. however, skilled labour required for manufacture is in short supply and current availability is only 30,000 hours per month @ Rs. 20 per hour.

Variable production overheads amount to Rs. 10 per labour hour and variable selling and distribution cost is 10% of sales value per unit.

Page 280: cost accounting question icwai inter

(ii) when skilled labour are in short supply;

(b) Production plans for optimum profit when available labour hours is only 30,000. what is the expected profit ?

Solution: Tutorial Notes:

(ii) if any constraints are given, the ranking should be done on the basis of contribution per unit of that constraint.

Contribution analysis Production A B C D ERaw material kg/unit 2.0 2.5 3.0 4.0 5.0Labour hr./ unit 1.0 1.2 2.0 3.0 4.0Variable cost Rs. per unit:Raw material @ Rs. 16/kg 32 40 48 64 80Labour Rs. 20/hour 20 24 40 60 80Prod. overhead Rs. 10/hour 10 12 20 30 40S & D. cost 10% of selling pric 10 12 16 22 30Total variable cost Rs.per unit 72 88 124 176 230Selling price 100 120 160 220 300Contribution per unit 28 32 36 44 70pv ratio 28.0% 26.7% 22.5% 20.0% 23.3%Profitability ranking I II IV V III(ii) Ranking when labour hours are in short supply:Contribution per unit 28 32 36 44 70Labour hr./ unit 1.0 1.2 2.0 3.0 4.0Contribution per unit per hr 28.0 26.7 18.0 14.7 17.5Ranking when labour is a const I II III V IV

(b) Production plans for optimum profit when available labour hours is only 30,000. what is the expected profit ?

Ranking Demand Total hrs Balance Producn.units hours units Rs. Rs.

A 8,000 1.0 8,000 22,000 8,000 28 224,000B 6,000 1.2 7,200 14,800 6,000 32 192,000C 5,000 2.0 10,000 4,800 5,000 36 180,000E 2,000 4.0 4,800 0 1,200 70 84,000D 3,000 3.0 0 44 0

Total contribution 680,000Less Fixed cost as given 300,000Profit 380,000Profitability statement when at least 10% of current demand is to be met:

Ranking Producn. Contribution / unit Contribution units Rs. Rs.

A 8,000 1.0 8,000 28 224,000B 7,200 1.2 6,000 32 192,000C 10,000 2.0 5,000 36 180,000

(c) if the company decides to produce and sell even relatively less profitable products to meet at least 10% of the current demand, what revised plan will you suggest? What is the anticipated profit?

·       Avoid newspaper in the morning.

(i) if no constraints are given, the ranking of the products should be done on the basis of contribution margin per unit or pv ratio. Contribution per unit is a good guide for ranking when some of the resources are scarce. If no constraints are there, pv ratio should be taken as guiding factor.

Rate of labour

Contribution / unit

Total contributio

n.

In this case the last ranked product D is to be produced in least quantity i.e.10% quantity will be produced which will consume 900 labour hours. The remaining 29,100 hours will be allotted to ranking already established above. The first three ranked products viz.A,B and C will consume 25,200 hours, and the balance 3900 hours will be consumed by product E.

Allotted hours

labour rate

Page 281: cost accounting question icwai inter

E 3,900 4.0 975 70 68,250D 900 3.0 300 44 13,200

Total contribution 677,450Less Fixed cost as given 300,000Profit 377,450

Particulars Plant A Plant B All fig. in Rs. CroresCapacity utilised 80% 60%Sales 4.80 2.40Variable cost 3.52 1.80Fixed cost 0.80 0.40You are required to work out:(i) Break even capacity of the merged plant; (ii) Profit earned at 75% of the merged plant(iii) Sales required to earn a profit of Rs. 1 crore.

Solution:(i) Break even capacity of the merged plant;

Particulars Plant A Plant B Merged plantCapacity utilised 100% 100% 100%Sales 6.00 4.00 10.00Less :Variable cost 4.40 3.00 7.40Contribution 1.60 1.00 2.60Less: Fixed cost 0.80 0.40 1.20Profit 0.80 0.60 1.40Pv ratio : Contribution / Sales 26.67% 25.00% 26.00%Break even sales : Fixed cost / Pv ratio 4.62(ii) Profit earned at 75% of the merged plant;To calculate the profit earned at 75%, all the above figures should be calculated at 75% capacity.

Merged plant Rs. Crores Capacity utilised 100% 75%Sales 10.00 7.50Less :Variable cost 7.40 5.55Contribution 2.60 1.95Less: Fixed cost 1.20 1.20Profit 1.40 0.75(iii) Sales required to earn a profit of Rs. 1 crore.This can directly be found out with the help of pv ratio:Contribution required = Fixed cost + Profit = 1.20 + 1.00 = 2.20 croresSales required = Contribution / Pv ratio = 2.20 / 0.26 = 8.46 crores

Solution:Break even sales : fixed cost / Pv ratio = 3 x 3 = Rs. 9 lakhs.

Margin of safety = Actual sales -- break even sales. Margin of safety is given as 10% of actual sales.

Actual sales should be Rs. 10 lakhs.

Solution: Hours Cost in Rs.12,000 170,000

Problem 5: (cwa inter I dec.04)

Two plants manufacturing the same product decide to merge. Particulars of operation of the two plants before the merger were as follows:

·       Help people but don't do what they should do for themselves.

For merger, the capacity of the plants must be same. Let us calculate various parameters of individual plants at 100% capacity. (figures in Rs. Crores)

Problem 6 :( cwa inter.june 04)

A company with a contribution / Sale ratio of 1 /3 and fixed cost of Rs. 3 lakhs per month should have a monthly sales of Rs. …………. Lakhs to maintain a margin of safety of 10%.

Problem 7 :( cwa inter.june 03)

The monthly cost of maintenance of machinery for 12,000 machine hour run is Rs. 170,000 and for 18,500 hours it is Rs. 202,500. The cost of maintenance for 14,000 hours is Rs……

Page 282: cost accounting question icwai inter

18,500 202,500Differential cost 6,500 32,500Variable cost per hour 5.00Variable cost for 12,000 hours 60,000Total cost 170,000Fixed cost 110,000Cost of 14,000 hour run = 110,000 + 14,000 x 5 = 180,000 Rs.

Solution:Margin of safety = Actual sales -- break even sales. Margin of safety is given as 25%.Break even sales can be found out by using the formula pv ratio = Contribution / Sales.Break even sales = 120 / 0.4 = Rs. 300 lakhs. This is 75% of sales as the margin of safety is 25%.Actual sales = 300 / 0.75 = Rs. 400 lakhs.

Required:(i) Profit-volume ratio, Break-even point and margin of safety for the first half of the year;(ii) Sales volume of the second half;(iii) Break even point and margin of safety for the whole year.

Solution:(i) The first part should be solved mentally; it is very simple.

The break even point is contribution / sales = 7 / 14 = 50%The break even sales = Fixed cost / pv ratio = 5 / 0.50 = Rs. 10 lakhs.The margin of safety = Sales -- Break even sales = 14 -- 10 = Rs. 4 lakhs.

Contribution = fixed cost -- loss = 5,00,000 -- 1,00,000 = Rs. 4,00,000.Pv ratio = 50%; Sales = Contribution / pv ratio = 4,00,000 / 0.50 = Rs. 8,00,000.Sales volume for the second half : Rs. 8,00,000.(iii) Computation of Break even point and Margin of safety for the whole year;

All fig. in lakhs First half 2nd half Total

Fixed cost 5 5 10Break even point 0.50 0.50 0.50Break even sales 20Actual sales 14 8 22Margin of safety (Actual sales -- BE sales ) 2

Products X YSelling price Rs./unit 80 100Direct material @ Rs. 20/kg 20 30

Problem 8 :( cwa inter.june 03)

A company's fixed cost amounts to Rs. 120 lakhs per annum and its overall pv ratio is 0.4. The annual sales of the co should be …… lakhs to have a margin of safety of 25%.

Problem 9: ( cwa inter June 03)

A company earned a profit of Rs. 200,000 on a sale of Rs. 14,00,000 during the first half of the year, the fixed cost being Rs. 5,00,000. however, during the second half of the year it incurred a loss of Rs. 1,00,000 although unit variable cost, selling price and fixed cost remained the same.

·       Watch a sunrise, at least once in a month.

For first half of the year : The contribution = profit + fixed cost = 2 + 5 = 7 lakhs. The sales is 14 lakhs.

(ii) During the second half of the year, the fixed cost remained the same but the company incurred a loss of Rs. One lakh. The fixed cost for the first half is given as Rs. 5 lakhs. Thus the fixed cost for the second half would be Rs. 5 lakhs. The loss is Rs. one lakh thus the contribution must be Rs. 4 ( 5 -- 1) lakhs. With pv ratio as 50% this would give sales to be 4 / 0.50 = Rs. 8 lakhs.

This is still simpler. The fixed cost for the whole year is Rs. 10 lakhs and the pv ratio is 50%. Thus the break even sales is Rs. 10 lakhs / 0.50 = Rs. 20.00 lakhs. The actual sales for the whole year is Rs. (14 + 8) = Rs. 22 lakhs. Thus the margin of safety being equal to actual sales less break even sales would be Rs. 2 lakhs ( 22 -- 2 ).

Problem 10 :( cwa inter.dec 03)

A company manufactures and sells two standard products X and Y using the same raw material, labour and identical machines. Further particulars are given below :

Page 283: cost accounting question icwai inter

Direct labour @ Rs.15/hr 15 15Variable overhead Rs. 15 15Machine hours required 0.50 0.75Fixed overhead allocated Rs. 12 18Maximum demand per annum units 18,000 15,000Current production per annum units 15,000 12,000

Required:1. Current contribution analysis. 2. Profit currently earned by the company.3. Alternative production plan, if any, more profitable to the company.4. Profit expected to be earned under the suggested plan.Solution: 1. Current Contribution AnalysisProducts X YSelling price Rs./unit 80 100Less ; Variable costDirect material @ Rs. 20/kg 20 30Direct labour @ Rs.15/hr 15 15Variable overhead Rs. 15 50 15 60Contribution per unit 30 40Pv ratio ( Contribution / Sales ) 37.50% 40%Machine hours required 0.50 0.75Contribution per machine hour 60 53.332. Profitability Statement for current year: Product X Y Total Rs.Current production per annum units 15,000 12,000Contribution in Rs.per unit 30 40Total Contribution 450,000 480,000 930,000Fixed cosx = 12 x Y = 18 x 12,000 180,000 216,000 396,000Profit earned per annum currently 630,000 696,000 13,26,0003. Alternative production plan, if any, more profitable to the company.

To optimise the machine hours product x should be produced upto maximum demand.Production of X for 18,000 units: Machine hours consumed = 18,000 x 0.50 = 9,000 hoursMachine hours available : 15,000 x 0.50 + 12,000 x 0.75 = 16,500 hoursThus balance ( 16,500 -- 9,000 ) = 7,500 hours should be utilised for production of Y.The production plan can be shown as below:Particulars X YMaximum demand per annum units 18,000 15,000Machine hour required for X 9,000Production of Y for 7,500 hours @ 0.75 hr/unit 10,0004. Profit expected to be earned under the suggested plan.Products X Y TotalProduction in units units 18,000 15,000Contribution per unit Rs. 30 40Total Contribution Rs. 540,000 600,000 11,40,000Less : Fixed cost Rs. 396,000Profit expected to earned under the suggested plan Rs. 744,000

Selling price per unit (Rs.) 60 Variable cost per unit (Rs.) 36

Labour and material are available according to requirements, but machine capacity cannot be increased immediately and the available capacity has been fully utilised by the current production plan.

The contribution per unit of limiting factor is the guiding factor when some resource is scarce. Here the machine hour is limiting factor, hence contribution per machine hour of both the products is to be compared and the more profitable is to be selected.

The contribution per machine hour of X and Y is Rs. 60 and Rs. 53.33 respectively. Thus the machine hours available should be utilised for product X first, and the remaining hours should be allotted to Y.

Problem 11 (cwa inter dec.02)

The following cost data are available from the records of M/s ABC ltd. with regard to their product 'Millennium'

Page 284: cost accounting question icwai inter

Fixed cost per unit (Rs.) 12 Normal output in units 100,000 unitsOther additional data available for four consecutive periods are as under:

Period I Period II Period II Period IV Total unitsOpening stock 0 0 30,000 20,000 50,000Production 100,000 120,000 110,000 90,000 420,000Sales 100,000 90,000 120,000 110,000 420,000Closing stock 0 30,000 20,000 0 50,000

Solution: Tutorial notes:Read the question twice. Following points should appear to you:1. The cost of goods sold has to be calculated for every period.

Particulars fig. in '0000 Period I Period II Period III Period iv TotalSales in units 10 9 12 11 42Sales in Rs. @ Rs. 60/unit 600 540 720 660 2,520Variable cost @ Rs. 36/unit 360 324 432 396 1,512Contribution 240 216 288 264 1,008Less Fixed cost as given 120 120 120 120 480Profit 120 96 168 144 528Under Absorption costing method

Particulars fig.'0000 Period I Period II Period III Period iv TotalSales in units 10 9 12 11 42Sales in Rs. @ Rs. 60/unit 600 540 720 660 2,520opening stock @ Rs. 48 0 0 144 96 240Cost of production @ Rs. 48 480 576 528 432 2,016sub total 480 576 672 528 2,256Less closing stock @ Rs. 48 0 144 96 0 240Cost of goods sold 480 432 576 528 2,016Less:overabsorption of overhead 24 12sub total 480 408 564 528 1,980Add;underabsorption of overhead 12Profit 120 132 156 120 528Profit by marginal costing 120 96 168 144 528

You are required to prepare a statement showing profit for different periods, under both marginal and absorption costing methods, showing under/over absorption of overheads, if any, and also giving your comments.

·  Smile and relax.

2.The overhead charges are not mentioned separately therefore absorption of overhead is irrelevant when marginal costing method is followed. The absorption becomes relevant when absorption method is followed.

3. The under or over absorption of fixed overhead can be directly found from the figures of opening stock and closing stock. The overhead absorption is Rs. 12/unit. In period II, the closing stock will be added to sales to compute gross profit. Thus profit under absorption method will be more by an amount equal to Rs. 360,000 ( = 12 x 30,000)

4. Similarly in period IV, the closing stock of period III becomes opening stock of this period. This closing stock is already loaded with Rs. 360,000. Profit for the period IV will be less by Rs. 360,000. however, this overloading will be obviated by loading of Rs. 240,000 ( = 12 x 20,000) on the closing stock of 20,000 units. Thus over all the profit will be less by Rs. 120,000 than the profit by marginal costing method.

5. It should be pretty evident in the minds of the reader that whatever method is followed for calculation of profit, the ultimate profit will remain the same although intermediate profits may be different.

Statement showing the profit, contribution and Sales under marginal costing method

Under Absorption costing method, the fixed cost is not treated separately but forms the cost of production. This necessitates the inclusion of opening as well as closing stock.

Under this method the cost per unit is computed taking fixed cost and variable cost jointly. Thus in this case the variable cost per unit is Rs. 36 and the fixed cost per unit is Rs. 12, the total cost per unit is Rs. 48. Thus whenever the sales is more than 100,000 (normal production), there will be over absorption of overheads and vice versa.

Page 285: cost accounting question icwai inter

Comments:1.Total profit remains same in the both the methods.

If margin of safety is 40% of sales and profit is Rs. 20,000 find out fixed cost.Solution: It is a very simple question. Margin of safety = Profit / Sales = 40%Profit is given as Rs. 20,000. Thus sales would be Rs. 20,000 / 0.40 = Rs. 50,000

A company's report of operations for the two years shows the following results:2000 2001

Sales Rs. 312,500 375,000Profit (loss) Rs. (7500) 10,000

1. At what level of sales does the company break-even assuming no change in its cost-volume-profit relationship ?2. Consider the following changes for the next year :(i) Selling price to decreased by 10% (ii) Variable cost to be decreased by 25%.(iii) Fixed cost to be increased by Rs. 32,500.What sales would be needed to generate a profit equal to 10% of sales ?Solution:First part: Computation of Fixed cost:Pv ratio = Change in profit / Change in sales = (10,000 + 7,500 ) / (375,000 -- 312,500) = 0.28 = 28%Variable cost is 72% of sales. Contribution = Pv ratio x Sales

2000 2001Rs. Rs.

Sales 312,500 375,000Contribution :@ 28% 87,500 105,000Less profit (add loss) 7,500 (10,000)Fixed cost 95,000 95,000Break-even sales (Fixed cost/pv ratio) 339,286Second part:

Let us assume that sales price for the year 2000 is Rs. 100. The contribution is 28% because pv ratio is 28%.

This year Next yearRs. Rs.

Selling price 100 90 10% decreaseVariable cost 72 54 25% decreaseContribution 28 36Pv ratio 28% 40% (36/90)Fixed cost 95,000 127,500 Rs. 32,500 increaseNext year the profit is to be 10% of sales.

0.40 S = ( Fixed cost + profit) = 127,000 + 0.10 S Which gives S = 425,000 Rs.

(i) Increase in selling price required for maintaining the existing profit-volume ratio.

2. Profit in period II is more in absorption costing method by Rs. 360,000 which is equal to loading of Rs. 12/unit of overhead on closing stock of 30,000 units ( Rs. 12 x 30,000)

3. Same logic as in point 2 above, goes to explain the less profit in period III and period IV under absorption costing method.

Problem 12 (cwa inter dec.02)

Problem 13 (cwa inter dec.02)

If S is the total sales and the profit is 0.10 S and contribution is 40% of sales. Following equation emerges:

Problem 14 (cwa inter dec.01)

The management of your organization is considering a wage increase of 20% effective from the beginning of next year. By virtue of various cost control drives already implemented, it is expected that there would be no further increase in any other cost. The management seeks your feedback on the following:

(ii) Assuming no increase in selling price is possible under the prevailing competitive environment, the extent to which the reduction in overall profit (following wage increase) may be recovered by utilizing the available capacity.

(iii) The overall effect on profitability if the present capacity is increased by 25% accompanied by an increase in fixed overhead by Rs. 100,000, selling price remaining at the present level. Assume full capacity utilisation.

Page 286: cost accounting question icwai inter

The following data is readily available: Rs.Present selling price Rs. per unit 80Variable cost Rs. per unit Material 30

Labour 20Variable overhead 10

Total capacity Units 60,000Sales Units 50,000Fixed overhead Rs. 200,000Solution:(i) Calculation for revised selling price to maintain the same profit-volume ratio:Particulars Present RevisedPresent selling price Rs./ unit 80 SVariable cost Rs. per unitMaterial 30 30Labour 20 24 20% increaseVariable overhead 10 60 10 64Contribution per unit in Rs. 20 (S -- 64)Profit-volume ratio = Contributi 25% 25%Profit volume ratio = 25% = ( S -- 64 ) / S Which gives S = Rs. 85.33(ii) Recoverable profit if no increase in selling profit is possible:Total capacity Units 60,000Sales Units 50,000Additional capacity available : 10,000Contribution per unit with increased labour Rs./unit 16Profit recoverable ( 10,000 x Rs. 16) Rs. 160,000(iii) Profit if present capacity is increased by 25% With increase PresentPresent capacity units 60,000 60,000Increase 25% units 15,000Total units 75,000 60,000Contribution per unit with increased labour Rs. 16 16Total contribution (75,000 x Rs. 16) Rs. 12,00,000 9,60,000Less fixed cost ( 200,000 + 100,000) Rs. 3,00,000 2,00,000Profit Rs. 9,00,000 7,60,000Increase in profit Rs. 1,40,000

A company gives the following data on cost and sales of the three products manufactured by it.Products

X Y ZSales mix (P.C. of sales value) 40 30 30Selling price per unit (Rs.) 400 250 200Variable cost per unit (Rs.) 200 100 120Fixed cost per unit (Rs.) 100 50 40Total sales : Rs. 100 lacs. The current level of production absorbs the entire fixed cost of the company.The management of the company wants to discontinue Z and introduce W improve profitability. The revised data on production and sales is as follows:

ProductsX Y W

Sales mix (P.C. of sales value) 50 30 20Selling price per unit (Rs.) 400 250 320Variable cost per unit (Rs.) 200 100 180Total sales : Rs. 100 lacs. You are required to determine: (a) Fixed cost of the company per annum.(b) Profit currently earned and profit likely to be earned after the introduction of W.(c) Current break-even sales and break-even sales after introduction of W.

Problem 15 (cwa inter June 00)

Page 287: cost accounting question icwai inter

Solution:Note : P.C. stands for Percent.(a) Statement of profit currently earned and total fixed cost:

Products TotalTotal sales : Rs. 100 lacs. X Y ZSales mix (Percent of sales value) 40 30 30Sales value in Rs. Lacs 40 30 30 100Selling price per unit (Rs.) 400 250 200Sales in units 10,000 12,000 15,000Variable cost per unit (Rs.) 200 100 120Contribution per unit Rs. 200 150 80Total contribution in Rs. Lacs 20 18 12 50Fixed cost in Rs. Per unit 100 50 40Total fixed cost in Rs. Lacs 10 6 6 22Profit currently earned in Rs. Lacs 28(b) Statement of profit after introduction of Product 'W'

Products TotalTotal sales : Rs. 100 lacs. X Y WhichSales mix (P.C. of sales value) 50 30 20Sales value in Rs. Lacs 50 30 20 100Selling price per unit (Rs.) 400 250 320Sales in units 12,500 12,000 6,250Variable cost per unit (Rs.) 200 100 180Contribution per unit Rs. 200 150 140Pv ratio (contribution / sales) 50% 60% 43.75%Total contribution in Rs. Lacs 25 18 8.75 51.75Total fixed cost in Rs. Lacs 22.00Total profit earned Rs. Lacs 29.75(c) Break-even sales Current and After the introduction of W.Particulars Current After introduction of W Total contribution in Rs. Lacs 50 51.75Total sales : Rs. lacs. 100 100Pv ratio 0.50 0.5175Fixed cost Rs. In lacs 22 22Breakeven sales ( fixed cost/ pv ratio) in la 44 42.51

(d) The product among X,Y and W having highest pv ratio should be selected for increased production.Particulars X Y ZSelling price per unit (Rs.) 400 250 320Variable cost per unit (Rs.) 200 100 120Contribution per unit Rs. 200 150 140Pv ratio (contribution / sales) 50% 60% 43.75%Y should be picked up for enhanced production by 20%.

Sales value of product Y =(12,000 + 20%) x 250 = Rs. 36 lacsSales value of product X =12,500 x 400 = Rs.50 lacsBalance sales value remaining for product W = (100 -- 50 -- 36) = Rs. 14 lacs.No. of units of W to be sold @ Rs. 320/unit ( 14,00,000 / 320) = 4375 unitsThe statement showing contribution and profit for increased production of Y is as below:

Products TotalParticulars X Y ZSales in units units 12,500 14,400 4,375Selling price per unit (Rs.) Rs. 400 250 320Variable cost per unit (Rs.) Rs. 200 100 180

(d) If it is possible to increase sales of any one of the products X,Y or W by 20% by keeping the total sales unchanged, what alternate mix would you suggest for higher profit.

It should be noted that total sales remains constant at Rs. 100 lacs despite the increased production of Y. Thus the product Z having least pv ratio should receive the balance available after product X and Y.

Page 288: cost accounting question icwai inter

Contribution per unit Rs. Rs. 200 150 140Pv ratio (contribution / sales) 50% 60% 43.75%Total contribution in Rs. Lacs Rs. Lacs 25 21.60 6.125 52.73Total fixed cost in Rs. Lacs Rs. Lacs 22.00Total profit earned in Rs. Lacs Rs. Lacs 30.73

Capacity utilisation fig. in Rs.60% 80%

Direct material 32,50,000 43,00,000Direct labour 12,00,000 16,00,000Electric power 120,000 160,000Consumables 240,000 320,000Repairs and maintenance 210,000 230,000Indirect labour 450,000 600,000Administrative expenses 300,000 300,000Salaries 400,000 400,000Selling and distribution expens 220,000 260,000Depreciation 300,000 300,000Miscellaneous expenses 110,000 130,000

Total 68,00,000 86,00,000Required :(a) Current sales and profit (b) Current breakeven sales

(c) Price to be quoted for the new order, if contribution / sales ratio is to be maintained at current level

Solution: Tutorial Notes:

The solution goes as follows:

Particularfig. in Rs.'000 60% 80% Diffe.for 20% Fixed costRepairs and maintenance 210 230 20 150Selling and distribution exp. 220 260 40 100Miscellaneous expenses 110 130 20 50Particulars Capacity utilisation Fixed costVariable cost 60% 80%

Rs.'000 Rs.'000 Rs.'000Direct material 3,250 4,300Direct labour 1,200 1,600Electric power 120 160Consumables 240 320

Problem 16 (cwa inter II June 00)

Atlas engineering company Ltd is currently working at 60% of its installed capacity, maintaining a contribution / sales ratio of 31.25%. For the coming year it is awaiting an additional order which will raise the capacity utilisation to 80%. Accordingly it has prepared flexible budgets at 60% and 80% for next year vide details given below:

(d) If the raw materials used by the company is in short supply, what minimum price should the company quote for the new order?

·       Avoid lifts. Use stairs wherever you can.1. Read the question carefully. All figures are in the tune of lakhs. Student should reduce their labour and gain time by using the figure in lakhs or omitting the '000. This is very helpful in examination hall.

2. Identify the items as variable cost, semi-variable cost and fixed cost. Semi-variable costs are to be separated into variable and fixed segments. This will yield variable cost and fixed cost separately, the sales less variable cost is contribution. Then by applying the given ratio of contribution/sales, the sales and corresponding profit can be known.

3. For the last portion, the raw materials is in short supply, then the contribution / raw materials needs to calculated. The contribution sacrificed for the use of raw materials required for the new order should be added to the variable cost to determine the minimum price to be quoted for the new order.

First of all let us identify the items having semi-variable costs and separate them in fixed and variable components. Such items are as follows:

Page 289: cost accounting question icwai inter

Repairs and maintenance 60 80 150Indirect labour 450 600Administrative expenses 300Salaries 400Selling and distribution expens 120 160 100Depreciation 300Miscellaneous expenses 60 80 50

Total 5,500 7,300 1,300

0.3125 = ( S -- 55) / S which gives Sales as Rs. 80 lakhs.At 60% capacity utilisation Sales Rs. 80 lakhsCurrent profit = Sales -- Variable cost -- Fixed cost = ( 80 -- 55 -- 13 ) = Rs. 12 lakhs.(b) Break even sales = fixed cost / pv ratio = 13/ 0.3125 = Rs. 41.60 lakhs.(c) Variable cost of new order :Variable cost at 80% -- Variable cost at 60% =(73--55) lakhsVariable cost of new order = Rs. 18 lakhs. The pv ratio remains same after new order.If Z be sales having pv ratio of 0.3125 and variable cost of Rs. 18 lakhs.Using pv = contribution / sales we have ( Z -- 18) / Z= 0.3125 Sales = Rs. 26.18 lakhs.Alternatively pv ratio = 0.3125 thus variable cost = ( 1 -- 0.3125) x Z = 18 giving Z = 26.18 lakhs.( d) The contribution at current level = Rs. 25 lakhs ( Sales 80 lakhs -- variable cost 55 lakhs)Contribution 25.0 Fig. in Rs. LakhRaw materials 32.5Rate of contribution / raw mater(25 / 32.5 ) 0.77Raw materials required for new 10.5Contribution sacrificed for new (10.5 x 0.77) 8.08Variable cost of new order 18Minimum Price to be quoted Rs. 26.08 lakhs.

Solution:Pv ratio = Profit / Margin of safety; Margin of safety = Profit / Pv ratio = 4 / 0.4 = Rs. 10 lakhs.

Margin of safety is 20% of sales as given.Sales = Margin of safety / 0.20 = Rs. 50 lakhs.Contribution = 40% of Sales = 40% of Rs. 50 lakhs = Rs. = 20 lakhs.

Solution: Pv ratio = Profit / Margin of safety = 10 / 40 = 25%

Break even sales = fixed cost / pv ratio = 20 / 0.25 = Rs. 80 lakhs.

Total sales = Break even sales + margin of safety = Rs. 120 lacs.

Particulars Plant A Plant BCapacity utilisation 80% 60%

Rs. Lacs Rs. LacsSales 400 240Variable cost 320 180Fixed cost 50 40After merger, the management wants information on the following:

(a) The contribution is sales minus variable cost. The variable cost at current level of 60% utilisation is calculated above as Rs. 55 lakhs. The contribution / sales ratio is 31.25% or 0.3125. Let S be the current sales. The equation below is evident.

Problem 17 (cwa inter II June 00)

A company has a margin of safety at 20% and earns a profit of Rs. 4 lakhs. If its contribution / sales ratio is 0.4, Calculate its current sales and fixed cost.

·       Don't wait until the conditions are perfect. They never will be.

Problem 18 (cwa inter II Dec 06)

A company has a margin of safety of Rs. 40 lakhs and earns a profit of Rs. 10 lakhs. It has a fixed cost of Rs. 20 lakhs, calculate the annual sales

Problem 19 (cwa inter II Dec 06)

Two cement plants decide to merge to earn higher profits. The working results of the two plants for last year were as follows:

Page 290: cost accounting question icwai inter

1. Capacity at which the combined plant will break even.2. Profit likely to be made if the combined plant works at 90% capacity.3. Sales required to earn a profit of Rs. 60 lakhs.4. Sales required to earn a profit of Rs. 60 lakhs, if the fixed cost is reduced by Rs. 10 lakhs.Solution:This is similar to problem 5 solved earlier. The answers are as follows.1. Break even capacity Rs. 405 lakhs. 2. Profit at 90% capacity Rs. 90 lakhs.3. Rs. 675 lakhs. 4. Rs. 630 lakhs.

Particular X Y ZDirect materials/unit in Rs. 65 80 90Direct labour Rate/hour Rs.Operation A 12 5 hours 10 hours 15 hoursOperation B 10 16 hours 8 hours 12 hoursOperation C 8 -- 5 hours 4 hoursCurrent selling price Rs./unit 400 560 650Production quantity budgeted in units 2,000 3,200 1,800Maximum sales feasible in units 2,500 4,000 2,400During the discussion, the following points were clarified:

Required:

1. Prepare a statement showing the budgeted contribution as per the above information.

Product X and Y : Reduction by 5% and Product Z : reduction by 4%Solution:

Particulars units X Y ZDirect materials/unit in Rs. 65 80 90Direct labour hoursOperation A hours 5 10 15Cost of operation A @ Rs. 12/hr. Rs. 60 120 180

Operation B hours 16 8 12Cost of operation B @ Rs. 10/hr Rs. 160 80 120

Operation C hours -- 5 4Cost of operation C @ Rs. 8/hr Rs. 0 40 32Total Labour cost Rs. 220 240 332Total variable cost Rs. 285 320 422Selling price / unit Rs. 400 560 650Contribution / unit Rs. 115 240 228Budgeted production units 2,000 3,200 1,800Budgeted contribution Rs. 230,000 768,000 410,400 14,08,400Part 2. Contribution per unit of limiting factor

Particulars X Y ZContribution / unit Rs. 115 240 228Operation A hours hours 5 10 15Contribution / hour of operationRs. 23.0 24.0 15.2Ranking II I III

Problem 20 (cwa inter I Dec 06)

Following information are available from the detailed budget submitted for approval of management for the year 2001-02 of a manufacture concern:

1. Operation A involves use of highly skilled labour which is in short supply and the availability of the same is just adequate to meet the budgeted output.

2. As confirmed by marketing manager, the entire quantity of production budgeted will be sold out. As such, there is no closing stock envisaged.

The managing director desires the working group to take a re-look into the entire budget exercise for increasing the contribution through the optimum use of available skilled labour.

2. Prepare a statement showing improvement over budgeted 3. Show the effect on contribution computed as per (2) above if the management decides to reduce the prices of production in the following manner to ward off competition.

Answer to 1. Statement showing budgeted contribution

Page 291: cost accounting question icwai inter

Maximum sales feasible in units 2,500 4,000 2,400

Total hours available for operation A: (2,000 x 5 + 3,200 x 10 + 1,800 x 15) = 69,000 hours

Contribution analys units hours contribution per hour RsTotal contribution in Rs.Product X 2,500 12,500 23.0 287,500Product Y 4,000 40,000 24.0 960,000Product Z 1,100 16,500 15.2 250,800Total 69,000 14,98,300Part 3. X Y ZCurrent selling price Rs./unit 400 560 650Reduction in selling price proposed 5% 5% 4%Reduced selling price Rs./unit 380 532 624Total variable cost as above Rs./'unit 285 320 422Contribution per unit 95 212 202Skilled labour hours required for operation 5 10 15Contribution per hour in Rs. 19.00 21.20 13.47Units to be produced 2,500 4,000 1,100Total Contribution in Rs. 237,500 848,000 222,200 13,07,000

Particulars A B CSales (% of total sales) 50 30 20Pv ratio % 42 45 50Raw materials (% of sales) 40 45 40Apportioned Fixed cost (% of sales) 30 25 20

Required:(a) Summary of working results for 2001 ( in rupees) and the profit earned by the firm.(b) The optimal product mix for 2002 in terms of sales.(c) Profit likely to be earned by the company as per plan (b) above.

Solution: Tutorial notes:

(a) Results for current year working performance: Rs. in lakhsParticulars A B CSales 100 60 40Pv ratio % 42 45 50Contribution 42 27 20Apportioned Fixed cost 30 15 8

The sequence would be like this: First 4,000 units of B will be produced in available hours; followed by A and if available still then followed by C.

Product Y will consume 4,000 x 10 = 40,000 hours leaving balance of 29,000 hours for X and then for Z.Product X will consume 2,500 x 5 = 12,500 hours leaving balance of 16,500 hours in which 1,100 units (16,500 / 15) of Z. could be produced.

· Associate yourself with people of good quality. It is better to be alone than to be in bad company.

Problem 21 (cwa inter II June 02)

KITCOS Ltd. Had a total sales of Rs. 200 lakhs during 2001. An analysis of the accounts revealed the following details for the three product lines A, B and C.

The company's production depends on the availability of imported raw materials. It can increase sales upto a maximum of 50% if 10% of additional sales amount is spent on advertisement.

For the year 2002, The company has been able to obtain an import license for import of raw materials costing Rs. 122.10 lakhs ( 10% above 2001 prices) and wants to plan its production/sales to obtain maximum profit keeping selling price unchanged.

·       Drink a lot of water daily.

1.The cost of raw materials goes up by 10% in 2002 over the price in 2001. Raw material is an element of variable cost, other elements of variable cost will not be subjected to price rise. Segregation of raw materials with other elements of variable cost is essential for determination of profit for 2002. Think how it can be done.

2. The fixed cost is apportioned as % of sales for 2001. The sales mix will undergo change in 2002. The fixed cost should remain the same in amount in both the years as change in fixed cost has not been mentioned in question.

Page 292: cost accounting question icwai inter

Profit 12 12 12(b) Optimal Product Mix for 2002 in terms of sales:

Rs. in lakhsParticulars A B CSales Rs.lakhs 100 60 40Contribution Rs.lakhs 42 27 20Raw materials % of sales 40 45 40Raw materials Rs.lakhs 40 27 16Contribution : Raw materials 1.05 1.00 1.125Ranking 2 3 1Availability of raw materials at 2001 prices 122.10 x (100/110) = Rs. 111.00 lakhsMaximum sales possible 150.0 90.0 60.0Raw materials required for maximum sales 60.0 40.5 24.0Allocation of raw materials as per ranking 60.0 27.0 24.0 111.00Pv ratio % 42 45 50Sales 150 60 60Variable cost 87 33 30Less: cost of raw materials 60 27 24Variable cost other than raw materials 27 6 6Raw materials at 2002 prices (10% above 66.0 29.7 26.4 122.10

(c) Profit likely to be earned by the company as per plan (b) above.Particulars A B C Total

Rs. Lacs Rs. Lacs Rs. Lacs Rs. LacsSales 150.0 60.0 60.0 270.0Cost of raw materials 66.0 29.7 26.4 122.1Variable cost other than raw materials 27.0 6.0 6.0 39.0Total Variable cost 93.0 35.7 32.4 161.1Contribution 57.0 24.3 27.6 108.9Apportioned Fixed cost same as previous y 30.0 15.0 8.0 53.0Advertisement @ 10% of additional sales 5.0 1.0 1.0 7.0Profit 22.0 8.3 18.6 48.9

The company has two different production / sales options as under:Option 1 A mix of 2 units of P for every 3 units of QOption 2 A mix of 1 unit of P for every 2 units of Q

Find out the combined Break -even point under each option and the optimal mix that the company should adopt.

Solution:Statement showing the contribution and Sales under both options :

Product P Product Q TotalBudgeted Selling price Rs./unit 3,600 4,320Less: Variable cost Rs./unit 1,800 3,600Contribution Rs./unit 1,800 720Option 1 : 2 units of P and 3 units of QNo. of units 2 3Total contribution Rs. 3,600 2,160 5,760Selling price Rs./unit 3,600 4,320Sales revenue Rs. 7,200 12,960 20,160Annual Fixed cost Rs. 176,000Pv ratio combined ( Combined contribution / Combined Sales ) 28.57%Break-even sales in Rs. (Fixed cost / 0.2857) Rs. 616,000No. of integrated units for break even sales ( 616,000 / 20,160) 30.56No. of individual units of P and Q 30.56 x 2 30.56 x 3

61 91

Problem 22 (cwa inter II dec 05)

ZED Limited produces two products, P and Q. The budgeted selling price per unit for P and Q are Rs. 3600 and Rs. 4320 respectively. Variable cost of production and sales of P and Q are Rs. 1800 and Rs. 3600 respectively. Annual fixed cost of the company amounts to Rs. 176,000.

Page 293: cost accounting question icwai inter

Thus for break-even sales 61 units of P and 91 units of Q are required to be sold.Option 2 : 1 unit of P and 2 units of QNo. of units 1 2Total contribution Rs. 1,800 1,440 3,240Selling price Rs./unit 3,600 4,320Sales revenue Rs. 3,600 8,640 12,240Annual Fixed cost Rs. 176,000Pv ratio combined ( Combined contribution / Combined Sales ) 26.47%Break-even sales in Rs. (Fixed cost / 0.2647) 664,889No. of integrated units for break even sales ( 664,889/12,240 ) 54.32No. of individual units of P and Q 54.32x 1 54.32x 2

54 108Thus for break-even sales 54 units of P and 108 units of Q are required to be sold.Comments: Comparative Statement of the two options is given below:

Pv ratio Contribution per combined unit Break-even sales in Rs. Option 1 28.57% 5,760 616,000Option 2 26.47% 3,240 664,889Option 1 results in lower amount of Break-even sales, it is recommended for adoption.

A Ltd. has been offered to buy a machine between M1 and M2. The following data is given:M1 M2

Annual output in units 10,000 10,000Fixed cost in Rs. 60,000 32,000Profit at above level in Rs. 60,000 48,000The market price of the product is expected to be Rs. 20 per unit. You are required to compute(i) Break-even point of each machine(ii) The level of sales at which both the machine earn equal profit;(iii) The range of sales at which one is more profitable from the other.Solution: (i) Break-even point of both the machines

M1 M2Annual output in units 10,000 10,000Selling price Rs. / unit 20 20Sales revenue Rs. 200,000 200,000Contribution ( fixed cost + profit) 120,000 80,000Pv ratio ( contribution / sales ) 60% 40%Break-even point (fixed cost / pv ratio) 100,000 80,000Break-even point in units 5,000 4,000Contribution per unit Rs. 12 8Variable cost per unit Rs. 8 12(ii) Level at which both earn equal profit:Let us say at the level of n units, both earn equal profitProfit of M1 ( Sales -- variable cost -- fixed 20 x n -- 8 x n -- 60,000 = 12n --60,000Profit of M2 20 x n -- 12 x n -- 32,000 = 8n --32,000Equating the above equations: 12n -- 60,000 = 8n -- 32,000 which gives n = 7,000 units.At a production of 7000 units, both the machines will earn same profit of Rs. 24,000.

ABC MNORs. Rs.

Sales 500,000 600,000

Problem 23 (cwa inter II June 06)

(iii) The break even point of M1 is 5000 units and that of M2 is 4000 units. Thus M2 begins to render profit above 4000 units, thus profit earning of M2 is more in the range 4000 to 7000 units. At a level of 7000 units, both give same profit. From above 7000 points M1 has more earning capacity because of its higher pv ratio. Higher pv ratio implies more contribution for same amount of sale.

Problem 24 (cwa inter I June 08)

ABC Ltd. and MNO Ltd. sell identical products in identical markets. Their budgeted income statement for the year 2007-08 are as follows:

·       Use good cheerful words in your language. Avoid negative words while speaking,

Page 294: cost accounting question icwai inter

Less Variable cost 400,000 180,000Contribution 100,000 420,000Less Fixed cost 20,000 270,000Budgeted Profit 80,000 150,000Calculate :(i) BEP for each company; (ii) Sales at which each company will earn a profit of Rs. 60,000(iii) Sales at which both companies will have same profits;(iv) Which company will earn more when (i) Heavy demand and (ii) Low demand.Solution : This question is similar to earlier questions. Answers are given below:(i) BEP for each company; ABC : Rs. 100,000 and MNO : Rs. 385,714(ii) ABC : Rs. 400,000 and MNO : Rs. 471,429 (iii) Sales Rs. 500,000

ComponentsVariable cost per unit

Rs./unitA 4 48 64B 5 60 75C 6 80 110Assembling Rs. 30/ unit of ZFixed costs per month amount to Rs. 50,000. Product Z is sold at Rs. 300 per unit.]

You are asked to find out the following : (a) Current demand and profit made by the company.

(c) Profit made by the company if suggested in 'b' is accepted.Solution : (a) Current Demand and profit :

Only 12,000 hours are available, which is just sufficient to meet the current demand of 800 units.Profit from current month's production : Rs./unit Total Rs.Sales price 300Less Variable cost 218Contribution 82 65,600Less Fixed cost 50,000Profit 15,600

(b) Component to be purchased from the market to meet 25% of additional demand from the next month onwards:

Components fig. in Rs. A B CMarket price 64 75 110In-house variable cost 48 60 80Additional cost 16 15 30Saving in hour per unit 4 5 6Saving per hour 4 3 5

·       Use good cheerful words in your language. Avoid negative words while speaking,

(iv) In case of high demand MNO ltd will earn more profit because higher pv ratio. In case of lower demand, ABC will earn more profit.

Problem 25 (cwa inter I June 08)

Sterling Industries Ltd. manufactures a product Z by making and assembling three components A, B and C. The components are made in a machine shop using three identical machines each of which can make any of the three components. However, the total capacity of the three machines is only 12,000 machine hours per month and is just sufficient to meet the current demand. Labour and assembling is available according to requirement. Further details are furnished below:

Machine hours required per unit

Market price at which the component can be purchased if required.

From the next month onwards the company expects the demand for Z to rise by 25%. As the machine capacity is limited, the company wants to meet the increase in demand by buying such numbers of A, B and C which is most profitable.

(b) Which component and how many units of the same should be brought from the market to meet the increase in demand.

Page 295: cost accounting question icwai inter

The allocation of machine hours would be in the order C, A and B.Hours used Balance

Produce C to full extent of 1,000 units 6,000 6,000Produce A to full extent of 1,000 units 4,000 2,000Produce B to 400 units 2,000 0Sterling should buy balance 600 units of B from the market.(c) Profit as per plan in (b) Rs.Sales value of 1,000 units of Z @ Rs. 300 per unit 300,000Variable cost

to make 1000 units of C @ Rs. 80 80,000to make 1000 units of A @ Rs. 48 48,000to make 400 units of B @ Rs. 60 24,000to buy 600 units of B @ Rs. 75 45,000to assemble 1,000 units of Z @ Rs. 30 30,000 227,000

Contribution 73,000Fixed cost 50,000Profit 23,000

19X1 19X2 19X3Production (units) 176,000 192,000 128,000Sales (units) 160,000 128,000 160,000

Rs. Rs. Rs.Cost of sales at standard 384,000 307,200 384,000Production cost variance 1,760 A 1,920 A 1,280 AVolume variance 8,000 F 16,000 F 16,000 ASales 480,000 384,000 480,000 S/admn. Costs fixed 48,000 48,000 48,000Closing stock 38,400 192,000 115,200Profit 54,240 42,880 30,720A means Adverse and F means Favorable.Required :1. Redraft the statement on marginal costing basis to show the closing stock, contribution and profit.2. Prepare a statement reconciling the profits arrived at by you with the profits given above.Solution:Factory fixed overhead rate 80,000/160,000 = Rs. 0.50 per unit.

19X1 19X2 19X3Standard cost of sales Rs. 384,000 307,200 384,000Sales quantity units 160,000 128,000 160,000Standard cost per unit Rs. 2.4 2 2.4Variance Rs. 1,760 1920 1280Production units 17,600 192,000 128,000Variance/unit Rs. 0.01 0 0.01Total Cost Rs. 2.41 2 2.41Fixed Costs Rs. 0.5 1 0.5Variable cost/unit Rs. 1.91 2 1.91

Statement of ProfitSales 480,000 384,000 480,000Production quantity 176,000 192,000 128,000Production Variable cost @ Rs. 1.91 336,160 366,720 244,480Closing stock in units 16,000 80,000 48,000

The component C will save the maximum amount per hour. The additional demand of 25% on existing demand of 800 units will make total demand 1,000 units.

Problem 26 (cwa final June 00)

A single product company recovers its fixed factory overheads of Rs. 80,000 on the basis of normal output of 160,000 units. The actual fixed overheads are same as budgeted fixed overheads. The management accountant presented the following statement of profit for 3 years on absorption basis:

Page 296: cost accounting question icwai inter

Variable cost of closing stock @ Rs.1.91 30,560 152,800 91,680Opening stock in units 0 16,000 80,000Variable cost of opening stock @Rs.1.91 0 30,560 152,800Contribution (Sales + Closing stock -- Production -- Opening Stock)Contribution 174,400 139520 174,400Less : Fixed Costs Productn. 80,000

S/A adm. 48,000 128,000 128,000 128,000Profit as per marginal costing 46,400 11,520 46,400Add : Closing stock overvalued 7,840 39,200 23,520Less: Opening stock overvalued 0 7,840 39,200Profit as per absorption costing 54,240 42,880 30,720Profit as per absorption costing is same as given in the question. Reconciled.

Component A B C Variable cost of production 3 4 7Fixed cost of production 1 2 4Outside purchase price 2.5 6 13Machine hours per unit 1 1 2Labour hours per unit 2 2 4You are required to decide which component should be bought out if :(i) the company is operating at full capacity:(ii) the production is limited to 4000 machine hours week(iii) the production is limited to 4000 labour hours per week.Solution:

If machine hours are limited to 4000 hoursComponent B C Variable cost of production 4 7Outside purchase price 6 13Extra cost per unit 2 6Machine hours per unit 1 2Extra cost per machine hour 4 3Labour hours per unit 2 4Extra cost per labour hour 1 1.5If machine hours are limited to 4000 hours: Component C should be bought outIf labour hours are limited to 4000 hours : Component B should be bought out.

Rs.Sales 40,000Profit 1,000Direct materials 4,000Direct labour 3200 hrs @ Rs. 5 16,000Fixed overheads 10,000

An enquiry is received from XYZ Ltd. for a gauge which would use Rs. 60 of Direct materials and 40 labour hours

(i) What is the minimum price to quote to XYZ Ltd ?

Problem 27 (cwa inter II June 07)

X ltd. manufactures three components used in the finished product. The component workshop is currently unable to meet the demand for components and the possibility of sub-contracting part of the requirement is being investigated on the basis of the following data:

It is obvious that component A should always be bought irrespective of limiting factor because its outside purchase price is lower than its variable cost.

Problem 28 (cwa inter II June 07)

X Ltd. manufactures special purpose gauges to customer specifications. The highly skilled labour force is always working to full capacity and the budget for the next year shows:

(ii) Would the minimum price be different if spare capacity were available but material were subject to a quota of Rs. 4,000 per year ?

Page 297: cost accounting question icwai inter

Solution:Contribution per unit of limiting factor (labour hour) = Rs. 20,000 / 3,200 = Rs. 6.25/hour

Rs. Minimum price to be quoted : Direct materials 60

Direct labour 40 hours @ Rs. 5 200Opportunity cost @ Rs. 6.25/hour 250

510If material is the limiting factor : Contribution per unit of material : Rs. 20,000 / 4,000 Rs. 5 per rupee of material.Minimum price to be quoted : Rs.

Direct materials 60Direct labour 40 hours @ Rs. 5 200Opportunity cost @ Rs.5 per rupee of material 300

560

Figs. – Rs. ’ 000 Sales 3,000

Direct materials 900 900Direct labour 600 600Overheads 900 900 2,400Profits 600 300

For the year 2008, the following are expected: (i) Reduction in the selling price by 10%. (ii) Increase in the quantity sold by 50%. (iii) Inflation of direct material cost by 8%. (iv) Price inflation in variable overhead by 6%. (v) Reduction of fixed overhead expenses by 25%. It is also known that :

(a) In 2006, overhead expenditure totaled to Rs. 8,00,000. (b) Total overhead cost inflation for 2007 has been 5% more than 2006. (c) Production and sales volumes have been 25% higher in 2007 than in 2006.

The high-low method is being used by the company to estimate overhead expenditure. You are required to:

(i) Prepare a statement showing the estimated trading results for 2008. (ii) Calculate the Break-even point for 2007 and 2008. (iii) Comment on the BEP and profits of the years 2007 and 2008.

Solution Brief answers are given below.(a) (i) Trading Results Figures Rs. ’ 000

2006 2007 2008Sales 3,000 4,050 Direct Material 900 1,458Direct labour 600 900Variable overhead 300 477

1,800 2,835Contribution 1,200 1,215Pv ratio 40% 30%Fixed cost 600 450Total Cost 2,400 3,285Profit 600 765

Minimum price to be quoted should accommodate opportunity cost in addition to direct materials and direct labour cost :

Problem 29 (ca inter may 07)

Problem 30 (cwa inter II June 07)

The Chief Cost Accountant of M/s XYZ Ltd. found to his surprise that the actual profit for the period ending 30th June 2006 was the same as budgeted in spite of realizing 10% more than the budgeted selling price.

Page 298: cost accounting question icwai inter

The following were the result:Particulars Budgeted Rs. Actual Rs. Sales 500,000 825,000Variable cost of sales 300,000 575,000Fixed cost 100,000 150,000Profit 100,000 100,000

Solution: Selling price is increased by 10%. Rs. Actual sales at budgeted price (100/110) x 825,000 750,000Variable cost @ 60% of sales as per budget 450,000Fixed cost of production as per budget 100,000Profit as per budget 200,000The actual profit goes down below the budgeted figure, it is because of the following:Increase in variable cost : ( 575,000 -- 450,000) 125,000Increase in fixed cost : ( 150,000 -- 100,000) 50,000Net increase in cost 175,000Net increase in revenue : ( 825,000 -- 750,000) 75,000Net decline in profit 100,000

Rs.

Sales 40,000Profit 1,000Direct materials 4,000Direct labour 3200 hrs @ Rs. 5 16,000Fixed overheads 10,000

An enquiry is received from XYZ Ltd. for a gauge which would use Rs. 60 of Direct materials and 40 labour hours

(i) What is the minimum price to quote to XYZ Ltd ?

Solution:Contribution per unit of limiting factor (labour hour) = Rs. 20,000 / 3,200 = Rs. 6.25/hour

Rs.Minimum price to be quoted : Direct materials 60

Direct labour 40 hours @ Rs. 5 200Opportunity cost @ Rs. 6.25/hour 250

510If material is the limiting factor : Contribution per unit of material : Rs. 20,000 / 4,000 Rs. 5 per rupee of material.Minimum price to be quoted : Rs.

Direct materials 60Direct labour 40 hours @ Rs. 5 200Opportunity cost @ Rs.5 per rupee of material 300

560

A company has sales of Rs. 100 lakhs of which 70% represents variable costs. Fixed costs are Rs. 20 lakhs.

If sales are now increased by 10%, what will be the improvement in profit ?

You are required to assist the Chief Cost Accountant in preparing the necessary explanations as to why the profit remained the same despite the increase in sales.

Problem 31 (cwa inter II June 07)

X Ltd. manufactures special purpose gauges to customer specifications. The highly skilled labour force is always working to full capacity and the budget for the next year shows:

(ii) Would the minimum price be different if spare capacity were available but material were subject to a quota of Rs. 4,000 per year ?

Minimum price to be quoted should accommodate opportunity cost in addition to direct materials and direct labour cost :

Problem 32 (cwa final June 06)

Page 299: cost accounting question icwai inter

On the other hand, if due to improved productivity, variable costs are decreased by 10%, what will be the profit ?

Compare the present profit with those under the cases as cited above.What is your recommendation ?Solution : Rs. In lacs

Present 10% increase 10% reduction inin sales variable cost

Sales 100 110 100Variable cost 70 77 63Contribution 30 33 37Less : Fixed cost 20 20 20Profit 10 13 17Recommendation :Profit is highest in the case of improved productivity. It is recommended

Calculate (I) P/v ratio; (II) Break-even point in volume and revenue and (III) Margin of safetySolution :Solve yourself. This question was asked in the paper of Project management (paper 13)The P/v ratio is 40%. Break-even at 50,000 units at Rs. 750,000.Margin of safety = Rs. 210,000

The following additional data is available:Availability per month Efficiency of utilisation

Equipment hours 500 85%Imported chemical in liters 1000 95%Skilled manpower hours 250 65%(i) What is the maximum possible production under the current conditions ?

Solution :Possible output with equipment hours condition :( 500 x 85% x 60 ) / 36 = 708 Components.Possible output with imported chemical : ( 1000 x 95% ) / 1.20 = 791 Components Possible output with skilled manpower ; ( 250 x 65% x 60 ) / 12 = 812 ComponentsThus minimum of the above three would be 708 components.

Problem 33 (cwa final June 06)

A new project has been set up by ABC Ltd. The company plans to achieve 80% capacity in the third year of operation, when the expected sales will be 64,000 units and the selling price Rs. 15/unit. The corresponding variable cost and fixed cost are estimated at Rs. 576,000 and Rs. 300,000 respectively.

Problem 34 (cwa final June 01)

A department of a company has to process a large number of components per month The process equipment time required is 36 minutes per component whether the requirement of an imported process chemical is 1.2 liter per component and skilled labour requirement 12 minutes for polishing and cleaning.

(ii) If skilled manpower availability is increased by overtime by 20%, will it result in increased production ?

(ii) If skilled manpower availability is increased by overtime, the production will not increase because the limiting factor is equipment hour availability.

Problem 35 (ca inter May.00)

The PTO Division of XYZ manufacturing company produces power take-off units for the farm equipment business. The PTO division headquarter in Mumbai has newly renovated, automated plant in Mumbai and an older, less automated plant in Pune. Both plants produce the same power take-off units for farm tractors that are sold to most domestic and foreign manufacturers.

The PTO Division expects to produce and sell 192,000 power take-off units during the coming year. The divisional production manager has the following data available regarding the unit costs, unit prices and production capacities for the two plants:

All fixed costs are based on a normal year of 240 working days. When the number of working days exceed 240 variable manufacturing costs increase by Rs.3 per unit for Mumbai and Rs. 8 per unit in Pune. Capacity for each plant is 300 working days.

Page 300: cost accounting question icwai inter

Mumbai Pune

Rs./unit Rs./unitSelling price 150.0 150.0Variable manufacturing costs 72.0 88.0Fixed Manufacturing costs 30 15Commission (5%) 7.5 7.5General and Administrative expenses 25.5 21.0Total unit cost 135.0 131.5Unit profit 15.0 18.5Production per day 400 units 320 unitsRequired :

1. Determine the annual break even units for each of the PTO's plant;

3. Determine the optimal production plant to produce the 192,000 units at Mumbai and Pune;4. Determine the resulting operating income of the PTO division.Solution:Tutorial Notes:

4. With the guidelines given above, you should first determine fixed cost at both the plants.

Working note 1: Determination of Fixed cost at both the plants ;Mumbai Pune

Production per day units units 400 320

XYZ manufacturing charges each of its plants as per unit fee for administrative services such as payroll, general accounting and purchasing, because management considers these services to a function of work performed at the plant. For each of the plants at Mumbai and Pune, the fee is Rs. 6.50 and represents the variable portion of general administrative expense.

Wishing to maximise the higher unit profit at Pune, the PTO's production manager has decided to manufacture 96,000 units at each plant. This production plan results in Pune's operating at capacity and Mumbai's operating at its normal volume. XYZ's Corporate Controller is not happy with this plan, because she does not believe it represents optimal usage of PTO's plants.

2. Determine the operating income that would result from the division production manager's plan to produce 96,000 units at each plant;

1. Problem is lengthy, you should read it twice, in second reading you can skip the first two paras because they do not contain any significant working material.

2. Fixed cost is given on per unit basis, just to test your knowledge. Fixed cost should always be calculated on total basis. Fixed cost per unit should be multiplied with the respective production to arrive at total fixed cost. This should be your working note no. 1.

3. The general and administrative expenses are also given in per unit basis, it is also given that the admn. expenses has Rs. 6.50 as fee of head office. This should be reduced from unit cost to arrive at the fixed cost per unit at each plant.

5. The normal capacity at Mumbai is 240 x 400 = 96,000 units and at Pune is 240 x 320 = 76,800 units. If production manager's plan is considered to produce 96,000 units at each plant, the variable manufacturing cost at Pune will increase by Rs. 8/unit. Note that this increase would be applicable only to additional capacity of (96,000 -- 76,800) 19,200 units.

6. Since the selling price is same for both the plants, the variable cost will decide which plant should be fully utilised. For optimum plan to produce 192,000 units, the plant having lower variable cost and thus having greater contribution per unit will be fully utilised and the balance capacity would be allotted to the remaining plant.

7. The variable cost at Mumbai is Rs. 86/ unit (72 + 7.50 + 6.50) while that at Pune is Rs. 102 (88 + 7.50 + 6.50). The contribution/unit at Mumbai is Rs. 64 and at Pune Rs.48. Thus for production of 192,000 units at PTO, Mumbai should operate at its full capacity of 120,000 ( 400 units for 300 days) and the balance of 72,000 units should be produced at Pune.

You should try to solve this problem on the above lines and then match your solution with the one given below:

Page 301: cost accounting question icwai inter

No. of days in a year no. 240 240 Total production per annum units 96,000 76,800 Fixed Manufacturing costs Rs. 30 15.0 Fixed General and administrative costs Rs. 26 21.0 Less: Charged for head office expenses Rs. (6.5) (6.5)Total Fixed cost per unit Rs. 49 29.5 Total Fixed cost per annum Rs.'000 4704 2,265.6 Working Notes 2: Determination of maximum capacity for both the plantsProduction per day units units 400 320 Maximum No. of days in a year no. 300 300 Total production per annum units 120,000 96,000 1. Determine the annual break even units for each of the PTO's plant; In Rs. Particulars Mumbai PuneSelling price Rs./unit 150 150.0 Variable cost Manufacturing cost 72 88.0

Commission (5% of Selling price) 8 7.5 General and Administrative expense 7 6.5 Total variable cost in Rs. /unit 86 102.0

Contribution Rs./unit 64 48.0 Fixed cost in Rs. 000 4,704 2,265.6 Break even point Units 73,500 47,200 Break even point Rs.'000 11,025 7,080

Mumbai PuneUnits produced 96,000 96,000.0Selling price Rs./unit 150 150.0Variable cost Manufacturing cost Rs./unit 72 88.0

Commission Rs./unit 8 7.5Company General expenses Rs./unit 7 6.5Total Variable cost per unit Rs./unit 86 102.0Variable cost in Rs. 000 Rs.'000 8,256 9,792.0Additional variable cost (19,200 x Rs. 8) Rs.'000 153.6Total Variable cost in Rs. '000 Rs.'000 8,256 9,945.6Sales revenue in Rs. '000 Rs.'000 14,400 14,400.0Contribution Rs.'000 6,144 4,454.4Less : fixed cost Rs.'000 4,704 2,265.6Profit Rs.'000 1,440 2,188.8Total operating income at both Rs.'000 3,628.8

3. Determine the optimal production plant to produce the 192,000 units at Mumbai and Pune;

Particulars units Mumbai Pune Selling price Rs. 150.0 150.0 No. of units no. 120,000 72,000.0 Sales revenue in Rs.'000 Rs.'000 18,000 10,800.0 Variable cost Rs./unit Rs./unit 86 102.0 Variable cost Rs.'000 Rs.'000 10,320 7,344.0 Additional Variable cost Rs.'000 (24,000 x Rs.'000 72.0 - Total Variable cost Rs.'000 Err:522 7,344.0 Contribution Rs.'000 Rs.'000 7,608 3,456.0 Less Fixed cost Rs.'000 4,704 2,265.6

Rs.'000 2,904 1,190.4

2. Determine the operating income that would result from the division production manager's plan to produce 96,000 units at each plant;

As the contribution per unit of Mumbai branch is Rs. 64 as against Rs. 48 for Pune Branch, the Mumbai branch would be fully utile upto 120,000 units and balance 72,000 units will be produced at Pune. The additional units above normal capacity of 96,000 units will call for additional variable cost Rs.3 per units for 24,000 units.

Page 302: cost accounting question icwai inter

Prime cost : Rs. 10, Variable overheads : Rs. 2.40 and Fixed overheads : Rs. 4 Total Rs. 16.40

Find out the point of the shut down for the quarter in units of product.Solution:The contribution per unit is Rs. 8. The avoidable fixed cost= total fixed cost -- (unavoidable fixed cost + additional shutdown cost)Avoidable fixed cost = Rs. 4 x 50,000 -- 74,000 -- 14,000 = Rs. 112,000No. of units = 112,000 / 8 = 14,000 units

Downstream units1 2 3

Capacity (kgs.) 60,000 40,000 20000Intermediate products required (kg) 66,000 20,000 14000Variable cost Rs./kg. 14 8 9Fixed cost Rs./kg. 4 6 3Profit Rs./kg. 2 1 4Total price Rs./kg. 20 15 16

You are required to :(a) work out the most profitable mix of level of production for the downstream units.

(b) compute the loss suffered for shortage of availability of intermediate from main plant, and

(c) work out the revised price of the production of downstream units to offset the loss suffered as above.Solution : Ranking of units

1 2 3Capacity (kgs.) 60,000 40000 20,000 120,000Intermediate products required (kg) 66,000 20000 14,000 100,000Intermediate products required per kg (kg) 1.10 1 0.70Contribution Rs. /kg 6 7 7Contribution Rs. /kg of inter product 5.45 14 10.00Ranking iii i iiRequirement of Intermediate Product (in kg.)Availability of Inter.product : 75,000 + 6,000 -- 1,000 = 80,000 kgMinimum requirement that must be met (70% capacity)Capacity at 70% 42,000 28000 14,000Intermediate product required 46,200 14000 9,800 70,000

Problem 36 (cwa final Dec.05)

The total cost of manufacturing a component is as under at a capacity of 50,000 units of production per quarter:

The selling price is Rs. 21 per unit. The variable selling and administrative expenses is 60 paise per component extra. During the next quarter only 10,000 units can be produced and sold. Management plans to shut down the plant estimating that the fixed manufacturing cost can be reduced to Rs.74,000 per quarter. When the plant is operating, the fixed overheads are incurred at a uniform rate throughout the year. Additional costs of plant shut down for the quarter are estimated at Rs.14,000.

Problem 37 (cwa final June 03)

A company has one main plant, which produces intermediate product for three downstream plants, which produces three different varieties of end product from the same intermediate. The three downstream plants are entirely dependent on the main plant for the intermediate product used as input in those plants. The following particulars are available regarding the three downstream units:

It has been observed that the downstream unit must operate at a minimum level of 70% capacity utilisation for economic reason.

The main plant can produce only 75,000 kg of output of the intermediate product during the coming year, There is an opening stock of 6,000 kgs of the intermediate product of which 1,000 kgs must be always retained. The main plant incurs variable cost of Rs.8 per kg in producing the intermediate product and recovers @ Rs.10 per kg from the downstream units.

Page 303: cost accounting question icwai inter

Extra production possible 18,000 12000 6,000Inter.product required kg 19,800 6000 4,200ranking iii i ii

Downstream Plant 1 2 3Intermediate product required kg. 46,200 20000 13,800 80,000Production of main product 42,000 40000 19,714(b) compute the loss suffered for shortage of availability of intermediate from main plant, Production possible 60,000 40000 20,000Production available 42,000 40000 19,714Production suffered 18,000 0 286Contribution suffered per unit 6 7 7Total contribution suffered 108,000 0 2,002 110,002Production Shortfal: Requireme 100,000

: Produced 75,00025,000

Loss of contribution @ Rs. 2 (Rs. 10 -- Rs. 8) 50,000Total Loss 160,002

Price revision required to offset Rs. 160,002:Loss per kg : 160,002 / 80,000 = Rs. 2Downstream units Existing price Increase required Revised price Rs.

1 20 Rs. 2 x 1.10 = Rs. 2.30 22.32 15 Rs. 2 x 0.50 = Rs. 1.00 16.03 16 Rs. 2 x 0.70 = Rs. 1.40 17.4

Product :- Strong Normal MilkAnnual sales (no. of packs 250 gm.) 6,000 5,000 1,000Selling price (Rs./pack) 50 40 30Unit cost (Rs./pack) 42 36 21Processing time/ pack (hours) 1.5 2 2The total processing hours available to the company is 16,000 hours which is fully utilised.

Fixed manufacturing overheads are fully absorbed in unit cost at a rate of 200% of variable cost.For the coming year the demand for the three products has been estimated as under:Strong : 6,000 packs ; Normal : 6,000 packs ; Mild : 2,000 packs.

Solution :Tutorial Notes :

Product :- Strong Normal MilkAnnual sales (no. of packs 250 gm.) 6,000 5000 1,000Processing time/ pack (hours) 1.5 2 2Processing time required 9,000 10000 2,000 21,000

Out of 80,000 kgs of available material, 70,000 is required to meet the minimum requirement of running at 70% level. Balance 10,000 hours should be allocated as per ranking.

The balance 10,000 will be allocated to Plant 2 : 6,000 kg then Plant 3: 4,000 kg. The production programme under this allocation would be as follows:

Problem 38 (cwa final June 03)

The sales, cost, selling price and processing time of three different herbal drinks produced by a company for the year just concluded are given below:

Considering that the selling prices are fixed and the processing time can be switched from one product line to another, calculate the best production programme for next operating year indicating the increase in net profit that will result.

1. Read the question carefully. Do you notice any discrepancy in it? There is some disagreement in the figures in the question. Please try to find that out.

2.The total processing time available is 16,000 hours. Processing time of product and sales do not match with available hours as follows:

Page 304: cost accounting question icwai inter

Product :- Strong Normal MilkAnnual sales (no. of packs 250 gm.) 6,000 5000 1,000Selling price (Rs./pack) 50 40 30Unit cost (Rs./pack) 42 36 21Processing time/ pack (hours) 1.5 1 2Fixed cost are absorbed at 200% of variable cost thus fixed cost :variable cost = 2 : 1Product :- Strong Normal MilkAnnual sales (no. of packs 250 gm.) 6,000 5000 1,000Selling price (Rs./pack) 50 40 30Variable cost 14 12 7Contribution /pack 36 28 23Total Contribution : 216,000 140000 23,000 379,000Processing time/ pack (hours) 1.5 1 2Contribution per unit of processing time 144,000 140000 11,500Ranking I II IIISuggested Production Programme for next year.Fixed cost are absorbed at 200% of variable cost thus fixed cost :variable cost = 2 : 1Product :- Strong Normal MilkAnnual Demand 6,000 6000 2,000Processing time/ pack (hours) 1.5 1 2Hours required 9,000 6000 4,000 19,000Hours allotted 9,000 6000 1,000 16,000Unit to be produced 6,000 6000 500Contribution /pack Rs. 36 28 23Total contribution Rs. 216,000 168,000 11,500 395,500Contribution in previous period 379,000Increase in profit as per suggested production programme 16,500

Solution :

Solution:

(i) At what level of sales, the company would have break-even ?(ii) Determine profit / loss on a forecasted sales volume or Rs. 8,00,00,000.

Thus either the processing time/pack is wrong or total time available given is wrong. Looking to both the possibilities, it is clear that if processing time/pack is 1 hour in place of 2 hours for product normal, it matches with available time. Let us assume that the processing time/pack for product normal is 1 hour in place of 2 hours. We can have the following table under this assumption:

Problem 39 (cwa final June 04)

B Ltd. has earned net profit of Rs. 1 lakh, and its overall P/V ratio and margin of safety are 25% and 50% respectively, What is the total fixed cost of the company ?

Margin of safety = Profit / Pv ratio = 1 / 0.25 = Rs. 4 lakhs. As the margin of safety is 50%, Break even sales and margin of safety would be equal. BES = Rs. 4 lakhs, the Pv ratio is 25%, the fixed cost would be equal to 25% of BE sales = 25% of Rs. 4 lakhs i.e. Rs. 1 lakh.

Problem 40 (cwa final June 04)

A company determines its selling price by marking up variable cost 60%. In addition, the company uses frequent selling price mark downs to stimulate sales. If the mark downs average 10%, what is the company's contribution margin ratio?

If variable cost is Rs. 100 the selling price would be Rs. 160. If selling price is reduced by 10%, new selling price would be Rs. 144. The contribution would be 44/144 i.e. 30.6%.

Problem 41 (cs final dec 08)

A company has annual fixed cost of Rs. 1,40,00,000. In the year 2007--08, sales amounted to Rs. 6,00,00,000 as compared with Rs. 4,50,00,000 in the preceding year 2006 --07. Profit in 2007 -- 08 is Rs. 42,00,000 more than in 2006-- 07. On the basis of the information, answer the following :

(iii) It there is a reduction in selling price by 10% in the financial year 2008 -- 09 and company desires to earn same amount of profit as in 2007-08, what would be the required sales volume?

Page 305: cost accounting question icwai inter

Solution : Solve yourself. Key answers are given below.Pv ratio = change in profit / change in sales = 28%.Break even sales = Rs. 5 crores.Profit at sales volume of Rs. 8 crores is Rs. 2.24 crores.Required sales volume = Rs. 8.40 crores.

Problem 42 (ca final nov 2001)

A BProfit Rs. 150,000 30,000Selling price in Rs. / unit 200 120P/V ratio in % 40% 50%

(iii) Simultaneous introduction of both the options, viz (i) and (ii) above.Solution : A BTutorial Notes :

Contribution / unit (pv ratio x selling price) Rs. 80 60Variable cost / unit Rs. 120 60If no. of units sold is Z for A and for B. Using Contribution -- Fixed cost = Profit

80 Z + 60 Z -- 15,00,000 = 1,50,000 + 30,000 giving Z = 12,000 units.Option 1: Variable cost increases by 7.50% and Fixed cost reduces by 15%

Rs. Rs. Rs. LacsPresent variable cost 120.00 60.00Increased variable cost 129.00 64.50Selling price / unit 200.00 120.00Contribution / unit 71.00 55.50Total contribution for 12,000 units in Rs. Lacs 8.52 6.66 15.18Less : Revised Fixed cost (15% less of Rs. 15.0 lacs) 12.75Profit 2.43Less : Existing profit 1.80Increase in profit 0.63(ii) Option 2:As the price elasticity of demand is one, therefore the revenue in respect of product will remain same.Revenue at present price 12,000 x Rs. 20 Rs. 24 lacsDemand at price of Rs. 240/unit 10,000 unitsRevised contribution per unit is Rs. 240 -- Rs. 120 = Rs. 120

A company manufactures two types of herbal products A and B. Its budget shows profit figures after apportioning the fixed joint cost of Rs. 15 lacs in the proportion of the number of units sold. The budget for 2002 indicates :

You are required to advise on the best option among the following, if the company expects that the number of units to be sold would be equal:

(i) Due to change in a manufacturing process, the joint fixed cost would be reduced by 15% and the variable cost would be increased by 7.5%;

(ii) Price to A could be increased by 20% as it is expected that the price elasticity of demand would be unity over the range of price.

1. You should know the meaning of demand elasticity to solve second part. Demand elasticity means demand varies inversely with the price. Worded differently, the product of price x demand will remain constant. In the present case the demand is 12,000 units at a price of Rs. 200/ unit. The product demand x price is Rs. 24 lacs. If the price increases by 20% it will become Rs. 240. Thus the demand at this price will be 24 lacs / 240 = 10,000 units. You should show this calculation by way of working note.

2. In part (iii) of the question simultaneous introduction of both the options is made. The price elasticity of product A is unity. Its demand will correspondingly reduce with increase in its price by 20%. No such change in demand of product B is anticipated, and it will remain at 12,000 units. You must be careful to calculate the contribution of A and B which will have different demands; A having 10,000 units and B having a demand of 12,000 units.

Page 306: cost accounting question icwai inter

Rs. LacsRevised contribution for 10,000 units = 10,000 x Rs. 120 12.00Contribution from Product B 6.66Total Contribution 18.66Less : Fixed cost 15.00Profit 3.66Less: Existing profit 1.80Increase in profit 1.86

(iii) Option 3: Simultaneous introduction of both the above options;A B

Rs. Rs.Selling price 240 120.0Less : Variable cost 129 64.5Contribution 111 55.5

Rs. LacsContribution for 10,000 units for A 11.10Contribution for 12,000 units for B 6.66Total Contribution 17.76Less : Fixed cost 12.75Profit 5.01Less : Existing profit 1.80Increase in profit 3.21

Recommendation :Options 1 2 3Increase in profit in Rs. Lacs 0.63 1.86 3.21

Problem 43 (ca final may 2000)

Components A B C D EMachine hours 20 28 24 -- --Labour hours -- -- -- 4 2

Rs. Rs. Rs. Rs. Rs.Variable cost 64 108 116 24 8Fixed cost as apportioned 36 52 64 26 22Assembly costs (all variable) Rs. 50 per batchSelling price Rs. 800 per batch.

Estimated increase in demand next year is 50% and fixed cost in general may increase by Rs. 10,000.

ssimistic Probability Most likely Probability Optimistic Probabilityview Rs. view Rs. view Rs.

A 120 0.25 110 0.50 80 0.25B 200 0.25 130 0.50 140 0.25C 160 0.25 140 0.50 120 0.25

You are required to :

In view of the above, the option 3 promising highest increase in profit of Rs. 3.21 lacs is recommended for adoption.

GG Ltd. manufactures and sells an equipment called water purifier. The cost data for each batch of ten numbers is as follows:

Maximum available machine capacity for making components A, B and C is 10,800 hours and it cannot be increased further. Labour is available for making components D and E and for assembling the product.

In order to increase production capacity to meet the increased market demand, the company decided to purchase one of the machine made components.

Quote Ltd. is the only supplier of components A, B and C. Because of incomplete records, it is unable to quote single figure prices. Its quotation is as follows:

Components

It is agreed between the companies that the price of each of the components will be determined on an overall basis based on information found in the quotation.

Page 307: cost accounting question icwai inter

Solution :Tutorial Notes :1. Find how many batches can be produced in available machine hours of 10,800.

4. Expected value of purchase cost of each of the components A, B and C can be found out.

The solution goes as follows :Working notes : Computation of no. of batches with 10,800 machine hoursComponents A B CMachine hours 20 28 24 72No. of batches possible with 10,800 machine hours avai(10,800/72) 150No. of batches with 50% increase ( 1.50 x 150) 225

Computation of Fixed cost and Future Fixed costFixed cost apportioned per batch (36 + 52 + 64 + 26 + 22) Rs. 200Fixed cost apportioned for 150 batches Rs. 30,000Add expected increase for additional capacity of 50% Rs. 10,000

Future Fixed cost Rs. 40,000Computation of expected purchase cost of components

A Expected value ( 120 x 0.25 + 110 x 0.50 + 80 x 0.25) Rs. 105B Expected value ( 200 x 0.25 + 130 x 0.50 + 140 x 0.25) Rs. 150C Expected value ( 160 x 0.25 + 140 x 0.50 + 120 x 0.25) Rs. 140

Computation of present contribution per batchSelling price Rs. 800Less : Variable cost Rs. 320

Assembly cost Rs. 50 Rs. 370Contribution Rs. 430Contribution for 150 batches Rs. 64,500

Answer (i) Maximum no. of batches if Component A is bought from outside :Available machine hours 10,800

Machine hours requir A not requiredB 28C 24

Total machine hours required 52No. of batches possible (10,800 / 52) 207.7

Maximum no. of batches if Component B is bought from outside :Available machine hours 10,800

(i) Indicate, in the context of key factor, the maximum number of batches that could be produced, if each of the three alternatives namely buying A, B or C is considered.

(ii) Analyse, the financial implication of purchase and advise which component is to be bought keeping in view the fact that production capacity will be limited to a 50% increase.

(iii) Prepare a Profit Statement for the period assuming that the component chosen by you is bought out and extra production is made and sold.

2. Each batch requires 72 machine hours (20 + 28 + 24). Thus batches possible under present capacity would be 150 (10,800 / 72). If capacity is increased by 50%, Batches produced would be 1.50 x 150 = 225. This is your working note.

3. Once the number of batches becomes known, the fixed cost can be determined. Fixed cost increases by Rs. 10,000 if capacity is increased by 50%, hence future fixed cost is also known.

5. Variable cost per batch is Rs. 320 (64 + 108 + 116 + 24 + 8) plus assembly cost per batch of Rs. 50. Selling price is Rs. 800. The contribution per batch is Rs. 430 which when multiplied by number of batches of 150 will give you the total contribution. Total contribution less fixed cost will give you profit.

6. You should also determine the number of batches possible with available 10,800 hours under three conditions namely (i) A is bought out (ii) B is bought out and (iii) C is bought out.

Page 308: cost accounting question icwai inter

Machine hours requir A 20B not requiredC 24

Total machine hours required 44No. of batches possible (10,800 / 52) 245.5No. of batches possible will be restricted to 225 only.

Maximum no. of batches if Component B is bought from outside :Available machine hours 10,800

Machine hours requir A 20B 28C not required

Total machine hours required 48No. of batches possible (10,800 / 52) 225.0

Answer (ii)Statement of financial implication when components are bought from outside

Components bought A B CRs. Rs. Rs.

Total variable cost/batch 64 108 116Expected purchase cost 105 150 140Increase in variable cost 41 42 24Present contribution 430 430 430Revised contribution 389 388 406No. of batches 207 225 225Revised contribution 80,523 87,300 91,350Less : Revised fixed cost 40,000

Profit 51,350Maximum contribution is given when component C is bought from outside.

Answer (iii)Profit statement when C is bought outsideParticulars Per batch For 225 batches

Rs. Rs.Sales revenue 800 180,000Less : Variable cost :Production cost of A 64Production cost of B 108Production cost of C 140Production cost of D 24Production cost of E 8 344 77,400Assembly cost 50Total variable cost 406 91,350Contribution 394 88,650

Problem 44 (ca final may 2000)

The cost structure is as under:Rs. Per unit

Direct materials 30Direct labour 20Variable overhead 10Profit 20

Unique products manufactures and sells in a year 20,000 units of a particular product to definite customers at a price of Rs. 100 per unit. The concern has a capacity to produce 25,000 units of the product per annum. To produce beyond 25,000 units per annum, the concern will have to install a new equipment at a cost of Rs. 15 lakhs. The equipment will have a life span of 10 years and will have no residual value. There is an offer from a client to purchase 10,000 units of the product regularly at a price of Rs. 90 per unit. The order, if accepted, will have to be over and above the existing level of production of 20,000 units.

Page 309: cost accounting question icwai inter

Solution :Tutorial Notes :

4. Frame your solution and compare with the one given below.The solution goes as follows :Analysis of proposal 1: Continue with present sales of 20,000 with increased charges

Rs./ unit Amount Rs.in lacsSales 100 20.00Less : Variable cost Direct materials (30 + 10% increase) 33Direct labour (20 + 25% increase) 25Variable overhead No increase 10 68 13.60Contribution 32 6.40Less : Fixed cost ( Rs. 20 x 20,000) + 10% increase 4.40Profit 2.00 Analysis of proposal 2: Sales of 25,000 with sales promotion expenses

Rs./ unit Amount Rs.in lacsSales 100 25.00Less : Variable cost Direct materials (30 + 10% increase) 33Direct labour (20 + 25% increase) 25Variable overhead No increase 10 68 17.00Contribution 32 8.00Less : Fixed cost ( Rs. 20 x 20,000) + 10% increase 4.40

Sales promotion expenses 0.50 4.90Profit 3.10Analysis of proposal 3: Sales of 30,000 with additional investment of Rs. 15.0 lacsSales 20,000 units at Rs. 100/ unit 100 20.00

10,000 units at Rs. 90/unit 90 9.00 29.00Less : Variable cost Direct materials (30 + 10% increase) 33Direct labour (20 + 25% increase) 25Variable overhead No increase 10 68 20.40Contribution for 20,000 units @ Rs. 32/ unit 6.40Contribution for 10,000 units @ Rs. 22/unit 2.20Total contribution 8.60

During the coming year, it has been estimated that the cost of direct materials, as compared to current year will increase by 10%. Because of certain wage agreement direct labour cost will increase by 25%. Fixed overheads will increase by 10%. If the new order is accepted, fixed overheads will increase further by Rs. 60,000 due to increased administrative charges.

You are required to analyse whether the concern should accept the order or instead of that try to secure order for the balance unused capacity, as available now, through some sales promotion expenses which will be Rs. 50,000 per annum. Ignore financial charges for the new investment.

1. You are to analyse three proposals. First to continue with present sales of 20,000 units with increased material, labour and Fixed charges. The fixed overhead has been indirectly given in the question. Find out where the fixed overheads are given.

2. The balance capacity available now is 5,000 units. The second proposal is to secure the market for this balance capacity by incurring sales promotion expenses to the tune of Rs. 50,000 per annum. Contribution and profit analysis for these two proposal is easy and you should attempt to do it.

3. The third proposal is to accept the order of 10,000 units at Rs. 90/ unit. This will require investment of Rs. 15 lacs. The depreciation on equipment would be Rs. 1.50 lac per annum. Additional fixed overheads would be Rs. 60,000. You are likely to commit mistake in computation of sales revenue for this proposal. Please try to analyse it with computation of contribution and profit.

Page 310: cost accounting question icwai inter

Less : Fixed cost ( Rs. 20 x 20,000) + 10% increase 4.40Depreciation on equipment 1.50Additional administrative charges 0.60 6.50

Profit 2.10Proposal 1 Proposal 2 Proposal 3

Rs. Lacs Rs. Lacs Rs. LacsSales 20.0 25.0 29.0Variable cost 13.6 17.0 20.4Contribution 6.4 8.0 8.6Fixed cost 4.4 4.9 6.5Profit 2.0 3.1 2.1Recommendation:

Problem 45 (ca final may 2000)A firm furnishes the following information :Capacity in units Unit cost Rs. Unit price Rs.

2,000 40 1003,000 35 954,000 34 945,000 32 ---6,000 31 ---

Solution :units Unit cost Rs. Incremental cost

2,000 40 80,000 for 1,000 units Rs.3,000 35 105,000 25,0004,000 34 136,000 31,0005,000 32 160,000 24,0006,000 31 186,000 26,000

Problem 46 (ca final Nov. 2000)

Rs.Regular selling price per unit 90Cost per unit Raw materials 25

Direct labour 0.50 hour/ Rs. 60 30Overhead 0.25 machine hour @ Rs. 40 10Total cost 65

In view of the above table, it is evident that profit under proposal 2 is highest and is equal to Rs. 3.10 lacs. The management of M/s Unique Products is advised to produce and sell 25,000 units @ Rs. 100/ unit and utilize its full capacity.

At present the firm is operating at 4,000 units capacity and has received an order for 2,000 units from an export market at Rs. 28 per unit. Should the order be accepted.

Company is operating at 4,000 units. Average incremental cost from 4000 units to 6000 units is Rs. 25/ unit. As the order is for Rs. 28/unit which is more than Rs. 25/ unit, the order is recommended to be accepted.

A Co. Ltd. manufactures several different styles of jeweler cases. Management estimates that during the third quarter, the company will be operating at 80% of the normal capacity. Because the company desires a higher utilisation of plant capacity, the co will consider a special order.

The company has received special order inquiries from two companies. The first order is from JCP Ltd., which would like to market a jeweler case similar to one of A Ltd's jewellery cases. JCP jewellery case would be marketed under JCP's own label. JCP Ltd. has offered A Ltd. Rs. 57.50 per jewellery case for 20,000 cases to be shipped by the last date of the quarter. The cost data for A Ltd. jewellary case that would be similar to the specification of JCP special order are as follows :

According to the specification provided by JCP Ltd. the special order case requires less expensive raw materials. Consequently the raw materials will only cost Rs. 22.50 per case. Management has estimated that the remaining costs, labour time and machine time will be the same for A Ltd. jewellery case.

Page 311: cost accounting question icwai inter

Rs.Raw materials 32.5Direct labour 0.50 hour @ Rs. 60 30.0Overhead 0.50 machine hour @ Rs. 40 20.0Total costs 82.5

Required :Should A Ltd. accept either special order ? Justify your answer and show the calculations.Solution :Tutorial Notes :This is long but easy question. Read it twice and frame your solution.

1. Can you find out the machine hours available, utilised and unutilized in the third quarter ?

6. Fixed overheads are not relevant in the analysis of special order.

The second order was submitted by K Ltd for 7,500 jewellery cases at Rs. 75 per case. These jewellery cases, like the JCP cases, would be marketed under K label and have to be shipped by the last date of the quarter. However, the K jewellery case is different from any jewellery case in the A Ltd. line. The estimated per unit cost of this case are as follows:

In addition, A Ltd. will incur Rs. 15,000 in additional setup costs and will have to purchase a Rs. 25,000 special device to manufacture these cases. This device will be discarded once the special order is completed.

The A Ltd.'s manufacturing capabilities to the total machine hours available. The plant capacity under normal operations is 90,000 machine hours per year or 7,500 machine hours per month. The budgeted fixed overhead for the current year amounts to Rs. 21,60,000. All manufacturing overhead costs are applied to production on the basis of machine hours at Rs. 40 per hour.

A Ltd. will have the entire quarter to work on the special orders. Management does not expect any repeat sales to be generated from the special order. Company practice precludes from subcontracting any portion of an order, when special orders are not expected to generate repeat sales.

You should take one order at a time and analyse. You should not take both the orders at the same time because you may be confused over the various figures.

2. Machine hours available per month are 7,500 hours. A Ltd. is operating at 80% capacity. It has 20% capacity as balance or 1,500 hours / month. A quarter consists of three months, hence machine hours available for special order are 4,500 hours. One unit for JCP Ltd. requires 0.25 hr, hence in balance 4,500 hours, only 18,000 (4,500/0.25) units can be produced. The order is for 20,000 units. You can assume that order can be partly completed.

3. You must also notice a special point in cost data. Total cost is given as Rs. 65/ unit. Total cost implies that the fixed overhead as well as variable overhead are included in the overhead cost per unit of Rs. 10. Note again that overhead is recovered at Rs. 40/ hr. You must now think how can you separate the fixed component of overhead from the total overhead charge of Rs. 40/ hour.

4. Fixed overhead is recovered on normal capacity. The budgeted fixed overhead is given as Rs. 21.60 lacs and budgeted capacity is 90,000 machine hours per annum. Thus fixed overhead must be charged at the rate of Rs. 24 (21.60 / 0.90). Thus in the cost data the overhead charge of Rs. 40/ hr contains Rs. 24/ hr as fixed component and Rs. 16/hr as variable component.

5. You can now determine the variable cost of special order of JCP ltd. as Rs. 56.50 (22.50 + 30.0 + 4.0). As the offered price is Rs. 57.50, this will give contribution of Rs. 1.0/ unit. The order is for 20,000 units but the available machine hours are 4,500 hrs only which can accommodate 18,000 units. You can assume that order can be partly completed. Thus accepting order of JCP Ltd. will give a contribution of Rs. 18,000 and should therefore be accepted. First analysis is over.

7. On similar lines as narrated above you can also analyse special order of K Ltd. The variable cost is Rs. 70.50 (32.50 + 30.0 + 8.0) per unit giving a contribution of Rs. 4.50/ unit (75.0 -- 70.5).

Page 312: cost accounting question icwai inter

The solution goes as follows :Working Notes: 1. Balance hours available per month 20% of 7,500 hrs 1,500 hours

Balance hours available per quart( 3 x 1,500) 4,500 hours2. Units which can be produced in balance 4,500 hours.

For JCP Ltd. @ 0.25 hr / unit (4,500 / 0.25) 18,000 unitsFor K Ltd. @ 0.50 hr / unit (4,500 / 0.50) 9,000 units

3. Computation of fixed and variable component in total overhead charge of Rs. 40.Budgeted hours 90,000Budgeted fixed overheads 21.60 Rs.lacsFixed overhead rate in Rs. ( 21.60 / 0.90) 24.00Variable overhead ( 40 -- 24) 16.00

Answer : Analysis of Special order of JCP Ltd.Balance hours available per quarter 4,500 hoursProduction @ 0.25 hr per unit 18,000 units

Rs. / unitSelling price 57.5Less : Variable cost Direct materials 22.5Direct labour 30.0Variable overhead @ 0.25 hr/ unit 4.0 56.5Contribution 1.0Contribution for 18,000 units Rs. 18,000

Balance hours available per quarter 4,500 hoursProduction @ 0.50 hr per unit 9,000 unitsProduction required 7,500 unitsAnalysis of Special order of K Ltd.

Rs. / unit Rs.Selling price 75.0Less : Variable cost Direct materials 32.5Direct labour 30.0Variable overhead @ 0.50 hr/ unit 8.0 70.5Contribution 4.5Contribution for 7,500 units 33,750Additional set up required Additional set up 15,000Special device 25,000 40,000Net loss due to the order 6,250

Decision :

If you want peace of mind, don't complain about others, rather change yourself.Problem 31: (CWA Inter June 09)

8. In the available 4,500 machine hours, 9,000 units @ 0.50 hr/ unit can be produced for K Ltd. The order is for 7,500 units only. Total contribution would be Rs. 33,750 (7,500 x 4.50). The additional setup required for this order is for Rs. 40,000 (Rs. 15,000 + Rs. 25,000). Thus accepting this order would imply a loss of Rs. 6,250 (40,000 -- 33,750). This order should not be accepted. Second analysis is over.

9. Please frame your solution neatly on the lines supplied above. Please prepare all the relevant working notes and compare your solution with the one given below.

1. The special order of JCP Ltd. for 20,000 units can be partly completed to the tune of 18,000 units of jewellery cases which will supply additional contribution of Rs. 18,000 to the firm. The balance of 2,000 cannot be completed due to capacity restraints.

2. The acceptance of special order of K Ltd. will cause a loss of Rs. 6,250 to A Ltd. hence it is not recommended.

Page 313: cost accounting question icwai inter

Product A B CRs. / unit Rs. / unit Rs. / unit

Sale price per unit 60 55 50Direct material 20 18 15Direct Labour 15 14 12Variable overhead 13 13 17Fixed cost is Rs. 100,000 per year.Estimated sales in units 2,000 2,000 2,000Machine hours per unit 6 2 1

You are required to : (a) What will be most profitable product mix that should be produced, and (b) the total contribution from the revised mix.

Solution :Product A B C

Rs. / unit Rs. / unit Rs. / unitSale price per unit 60 55 50Direct material 20 18 15Direct Labour 15 14 12Variable overhead 13 13 17Total variable cost 48 45 44Contribution/unit 12 10 6Machine hours 6 2 1Contribution per machine hour 2 5 6Ranking III II I

Profitability Statement :Product A B C

Contribution Rs./unit 12 10 6Units produced Units 1,000 2,000 2,000Total contribution Rs. 12,000 20,000 12,000 44,000Less Fixed cost Rs. 100,000Loss Rs. 56,000Problem 47: (CWA Inter June 08)

You are required to advise whether the company should make or buy the component B-15.(Assume that machine hours is the limiting factor.)Solution :

Decision : B-15 should be purchased from supplier as it will save Rs. 1 per component to the Novelty Ltd.Problem 48: CWA Inter June 06

(i) Pv ratio (ii) Break even point in terms of sales volume, sales revenue and % capacity and (iii) MOS,

New India Engineering Company Ltd., produces three components A, B and C. The following particulars are provided:

Due to break down of one of the machines, the capacity is limited to 12,000 machine hours only and this is not sufficient to meet the total sales demand.

First 2000 units of C will be produced which will consume 2000 machine hours. Machine hrs. now available will be 10,000 hrs. Product B, being second in rank, will require 2000 x 2 = 4,000 hrs. leaving 6,000 hrs. for production of A. In 6,000 hours, 1,000 units of A can be produced @ 6 hrs./ unit.

Novelty Ltd. produces a variety of products each having a number of component parts. Product P takes 5 hours to produce on M/c no. 20, working at full capacity. The selling price and marginal cost of Product-P are Rs. 100 and Rs. 60 respectively. A component part-B 15 could be made in the same machine in 2 hours for a marginal cost of Rs. 10 per unit. The supplier's price is Rs. 25 per unit.

The contribution is Rs. 8 per hour. If B-15 is produced, contribution for 2 hrs amounting to Rs. 16 will be lost. It will also require Rs. 10 to produce. The total cost of B-15 would be Rs. 26 while supplier is ready to supply it at Rs. 25.

A new project is being set up by ABC Ltd. The company plans to achieve 80% capacity in the third year of operation, when the expected sales will be 64,000 units and selling price will be Rs. 15 per unit. The corresponding variable cost and fixed cost would be Rs. 576,000 and Rs.300,000 respectively. Calculate the following from the given data

Page 314: cost accounting question icwai inter

Solution : Working notes:1. Variable cost : Rs. 576000 / 64,000 = Rs. 9 per unit.2. Contribution per unit = 15 − 9 = Rs. 6 per unit.3. Break even point : Fixed cost Rs. 300,000 / Contribution per unit = 50,000 units at sales of Rs. 750,000.4. Pv ratio : Contribution / Sales = 6/15 = 40%.5. MOS = Sales − BE sales = 960,000 (64,000 units at Rs. 15) − 750,000 = Rs. 210,000.

This is Life

Tell yourself .....This is life.When things don't go right, when there is no ray of light,

When it is too hard to survive, when all you have is to fight...Tell yourself…..This is life.

When there is competition to face, when you are lagging behind the race,When you have lost faith in God, when you are betrayed by a fraud…

Tell yourself…..This is life.When others don't respect you, when you are not among the admirable few,

When for a question, you can't find a solution, when all you are sure about is confusion…Tell yourself…..This is life.

When your destination is miles apart, when you don't know where to start,When all you see around is pain, when your hard work is in vain…

Tell yourself…..This is life.Even though all this happens, just have faith in yourself

And face the life with smile, things will surely change one day, because ………This is life.

When there is no friend, when life is on dead end,When world is not a paradise, when your confidence dies….

Page 315: cost accounting question icwai inter

When for a question, you can't find a solution, when all you are sure about is confusion…

Page 316: cost accounting question icwai inter

Chapter 10 Cost Volume Profit Analysis cwa inter cwa fin. ca inter ca final Total15 11 1 9 36

Solution: This is an easy question. You must do some thinking before going through the solution.Working notes: Suppose a is the cost of sales for the existing year.

Existing year Next yearMaterial 0.40 a 1.10 x 0.40 a = 0.44 a 10% increaseLabour 0.30 a 1.15 x 0.30 a = 0.345 a 15% increaseOverhead 0.30 a 0.30 a no increase

35,000 − 1.085a = 0.80 x (35,000 − a) which gives a = Rs. 24561.

Suppose a and b represent cost of sales and profit for the present year.

Increase in cost of sales = (0.44a + 0.345a + 0.30a) − (0.40a + 0.30a + 0.300.085aThe increase in cost of sales is equal to 20% of the existing profit i.e. 0.20bThe first relationship between cost of sales and profit is 0.085a = 0.20b or 8.5a = 20bThe second relationship is given by Sales = Cost of sales + profit i.e 35,000 = a + bSolving these two we get a i.e. Cost of sales = Rs. 24,561 and b i.e. Profit = Rs. 10,439Statement showing cost of sales and profit for present and next yearComponent Present year Next year Rs.Material 0.40 x 24,561 9,824 1.1 x 9,824 10,806Labour 0.30 x 24,561 7,368 1.15 x 7,368 8,473Overheads 0.30 x 24,561 7,368 7,368Cost of Sales 24,561 26,648Selling Price 35,000 35,000Profit at current selling price 10,439 8,352Check : Increase in cost of sales = (26,648 − 24,561) = 2,087

20% of existing profit = 0.20 x 10,439 2,088(ii) Selling price giving same percentage of profit as before on increased cost of sales:% of existing profit on cost of sales = 10,439 / 24,561 = 42.50%Increased cost of sales for the next year Rs. 26,648

·       viuh leL;k ds lq[kn var ;k lek/kku dh dYiuk djsaA ,slk yxkrkj djrs jgsa vkSj izlUurk eglwl djsaA ,d fnu ;gh lek/kku vkids lkeus vk tk;sxkAProblem 1: (cwa inter II dec.06)

A company presently sells an equipment for Rs. 35,000. Increase in prices of labour and material cost are anticipated to the extent of 15% and 10% respectively in the coming year. At present material cost represents 40% of cost of sales and labour cost 30% of the cost of sales. The remaining relate to overheads. If the existing selling price is retained despite the increase in labour and material prices, the company would face 20% decrease in the existing amount of profit on the equipment.

You are required to find out the selling price so as to give the same percentage of profit on increased cost of sales, as before. Prepare a statement of profit/loss per unit, showing the new selling price and cost per unit in support of your answer.

Existing profit = 35,000 − ( 0.40a + 0.30a + 0.30a) or (35,000 − a)The existing profit goes down by 20% in next year while the selling price remains stable at Rs. 35,000.

Profit in the next year : 35,000 − ( 0.44a + 0.345a + 0.30a) i.e. 35,000 − 1.085a. This is 80% of previous year profit. Thus we have the equation :

This question is also solved in suggested answer supplied by the institute in the following manner which is more difficult to understand.

Page 317: cost accounting question icwai inter

Add: Profit @ 42.50% of cost of sales Rs. 11,325New Selling price Rs. 37,973Statement showing profit under the present and revised selling price :(in Rs.)Component Present year Rs. Next year Rs. Material 9,824 10,806Labour 7,368 8,473Overheads 7,368 7,368Cost of Sales 24,561 26,648Selling Price 35,000 37,973Profit 10,439 11,325

Required:

Solution: Tutorial Notes:

Let the cost of sales be Rs. x, then the profit would be (42,000 - x)Cost structure would be as followsComponent This year anticipated increase Next year

Rs. Rs.Selling price 42,000 42,000Material 0.40 x 10% 0.44 xLabour 0.30 x 20% 0.36 xOverheads 0.30 x nil 0.30 xTotal cost 1.00 x 1.10 xProfit (42,000 − x) (42,000 − 1.1x)

The cost profit equation becomes :( 42,000 - 1.1 x) = 0.75 ( 42,000 - x) which gives x = 30,000.Thus cost of sales is Rs. 30,000 and profit is 42,000 - 30,000 i.e. Rs. 12,000.The percentage of profit with cost of sales is 12,000 / 30,000 = 40%This percentage of profit is to be maintained in next year with increased cost of sales.Calculation of new selling price of equipment:Componhis year anticipated increase Next year

Rs. Rs. Rs. Rs. Rs.Material 0.40 x 12,000 10% 1200 0.44 x 13,200Labour 0.30 x 9,000 20% 1800 0.36 x 10,800

Problem 2: (cwa inter I June 05)

MPC Ltd. Of Mumbai presently sells an equipment for Rs. 42,000. Increase in prices of material and labour cost are anticipated to the extent of 10% and 20% respectively in the coming year. Material cost represent 40% of cost of sales and labor cost 30% pf cost of sales. The remaining relates to overheads.

If the existing selling price is retained, despite the increase in labour and material prices, the company would face a 25% decrease in the existing amount of profit on the equipment.

1. Calculate a selling price so as to give the same percentage of profit on increased cost of sales as before;

2. Prepare a statement to profit/loss per unit showing the new selling price and cost per unit in support of your answer.

Neither the profit nor the cost of sales has been given. One of them needs to be assumed. The other can be found out by using the simple relationship Sales - Cost of sales = Profit.

It is given that if the existing selling price is retained, there would be 25% reduction in profit. In other words, profit for the next year would be 75% of the profit of present year.

Page 318: cost accounting question icwai inter

Overheads 0.30 x 9,000 nil 0 0.30 x 9,000Total 1.00 x 30,000 1.10 x 33,000Profit as 40% of cost of sales 12,000 13,200Selling price 42,000 46,200

Paramount Industries manufactures and sells two products A and B, details of which are given below:Product A B Flexible Budget (Cost in Rs.)Sales (units) 10,000 8,000 Labour hours 40,000 80,000Selling price Rs. 120 225 Indirect labour 80,000 160,000Costs Power 220,000 340,000Material 40 75 Supervision 200,000 200,000Labour @ Rs. 15 30 60 Depreciation 320,000 320,000

Misc. Expenses 48,000 88,000868,000 1108,000

Variable overheads are absorbed at rates per labour hour as per flexible given on the right.

1. Introduce a new product C. Details are as follows:Selling price/ unit Rs. 240 Initial demand 3,000 unitsMaterial Rs. 90Wages Rs. 75Variable overhead at the same rate as A and B.2. Accept an export order for 6,000 units of B at a reduced price of Rs. 180/unit.3. Lease out idle facilities at Rs. 10,000 p.m.You are required to: (a) Calculate the variable overhead and fixed overhead.(b) Profit at current level of operations.(c) Evaluate each of the options given above and comment.Solution:(a) Calculation the variable overhead and fixed overhead. Fig. in Rs.Flexible Budget (Cost in Rs.) Differ. Variable cost* Total fixed costLabour hours 40,000 80,000 40,000Indirect labour 80,000 160,000 80,000 (80,000 / 40,000) 2.0 0Power 220,000 340,000 120,000 (120,000 / 40,000) 3.0 100,000Supervision 200,000 200,000 0 200,000Depreciation 320,000 320,000 0 320,000Misc. Expenses 48,000 88,000 40,000 1.0 8,000* variable component = change in component / change in level 1.0 628,000(b) Profit at current level of operations.Particulars A B TotalCapacity 52% 52% 52%Sales in units 10,000 8,000 18,000Labour hours required per units 2 4

Rs./unit Rs./unitSelling price per unit 120 225Cost in Rs. Per unitMaterial 40 75Labour @ Rs. 15/hr. 30 60variable overhead Rs. 6 per labour hr. 12 24Total variable cost 82 159

Problem 3: (cwa inter June 02)

The current level of production utilizes only 52% of the installed capacity. The firm wants to increase capacity utilisation and is examining the following options:

Page 319: cost accounting question icwai inter

Contribution Rs./unit 38 66Total contribution Rs. 380,000 528,000 Err:522Less fixed cost (calculated in 'a' above) 628,000Profit Err:522(c) Evaluation of each of the options given above and comment.Particulars Options fig. in Rs.

Introducing Export Lease outproduct C product B

Demand in units 3,000 6,000Selling price per unit 240 180Cost per unitMaterials 90 75Labour 75 60Variable overhead 30 24Total variable cost 195 159Contribution 45 21Total contribution 135,000 126,000Less fixed cost Rental Receipt Rs. 10,000/unit 120,000Comment:In the light of above observation, introducing product C seems to be the best option.

Sales in units 10,000Loss Rs. 10,000Fixed cost (for the year Rs.120,000) Rs. 30,000Variable cost per unit Rs. 8

Solution:Selling price per unit for the first quarter: Rs. Rs.Variable cost for 10,000 units @ Rs. 8/unit 80,000Add: Fixed cost for the quarter 30,000 110,000Less Loss 10,000

It should however, be noted that it is marginally better than export option. Export of product B opens new dimensions for the company as such due weightage be given.Problem 3: (cwa inter I dec.00, repeat final dec.00, inter II June 06,)

A review, made by the top management of ABC Ltd. Which makes only one product, of the result of the firm for the first quarter of the year revealed the following:

The finance manager who feels perturbed suggests that the company should at least break-even in the second quarter with a drive for increased sales. Towards this, the company should introduce a better packing which will increase the cost by 50 paisa per unit

The sales manager has an alternative proposal. For the second quarter additional sales promotion expenses can be increased to the extent of Rs. 5,000 and a profit of Rs. 5,000 can be aimed at during the period with increased sales.

The production manager feels otherwise. To improve the demand, the selling price per unit has to be reduced by 3%. As a result the sales volume can be increased to attain a level of Rs. 4,000 for the quarter.

The managing director asks you as a cost accountant to evaluate the three proposals and calculate the additional sales volume that be required in each case, in order to help him to take decision.

Page 320: cost accounting question icwai inter

Sales for 10,000 units 100,000Selling price in Rs. / unit 10Alternatively Rs.Fixed cost 30,000Less loss 10,000Contribution 20,000Contribution/unit 2Add: variable cost per unit 8Selling price in Rs. / unit 10Evaluation of finance manager's proposal: Rs.Revised variable cost per unit (increase of Rs. 0.50/unit) 8.5Revised contribution in Rs. per unit ( 10 − 8.5) 1.5No. of units to be sold to cover fixed cost of Rs. 30, (30,000 / 1.5) 20,000Break even sales @ Rs. 10/unit ( 20,000 x Rs. 10) 200,000Break even point (Contribution / sales) = 1.5 / 10 = 15%Thus additional 10,000 units need to be sold to break-even.Evaluation of Sales manager's proposal: Rs.Present fixed cost 30,000Add: Additional sales promotion expenses 5,000Revised fixed cost 35,000Add: Desired profit 5,000Revised contribution 40,000Revised sales volume in units with contribution @ Rs. 2/unit 20,000 unitsBreak even point (Contribution / sales) = 2.0 / 10 = 20%Revised sales @ Rs. 10 unit ( 20,000 x 10) 200,000Thus additional 10,000 units need to be sold to achieve desired profit or Rs. 5,000Evaluation of Production manager's proposal: Rs.Revised selling price with reduction of 3% on existing selling pri 9.7Less : Variable cost 8.0Revised contribution in Rs. per unit ( 9.7 − 8) 1.7Existing fixed cost 30,000Add: Desired profit 4,000 34,000Revised sales volume in units with contribution @ Rs. 1.7/unit (3 20,000Pv ratio ( 9.70 / 1.70) 17.50%Thus additional 10,000 units need to be sold to achieve desired profit or Rs. 4,000

Summary of the proposalsFinance manager Sales manager Production manager

Units to be sold 20,000 20,000 20,000Pv ratio 15% 20% 17.50%Net profit nil Rs. 5,000 Rs. 4,000Recommendation:

From the summary shown above, the proposal of sales manager has maximum profit and highest pv ratio, is recommended to be accepted and implemented. All the three proposals require same number i.e. 20,000 units to be sold, thus it becomes irrelevant for the decision-making.

Problem 4: (cwa inter June 06)

Novina industries ltd. has received an export order for its only product that would require the use of half of the factory's present capacity of 400,000 units per annum. The factory is currently operating at 60% level to meet the demand of its domestic market.

Page 321: cost accounting question icwai inter

Direct materials in Rs. / unit 2.50 Direct expenses in Rs. / unit 0.50Direct labour in Rs. / unit 1.00 Fixed Overhead in Rs. / unit 1.00

A : Accept the order and keep domestic sales unfulfilled to the excess demand for the same.

D : Reject the order and remain with the domestic market only.Solution:Fixed cost (Rs. 1/ unit for a production of 60% of 400,000 units) Rs. 240,000Proposal one Export order of 200,000 units accepted. Domestic sales 200,000 units only.Particulars S.P/unit V.C./unit Contribution Units Contribution Rs.' 000Export 4.5 4.0 0.5 200,000 100Domestic 6.0 4.0 2.0 200,000 400Total Contribution 500Less : Fixed cost 240Profit 160Proposal twoParticulars S.P/unit V.C./unit Contribution Units Contribution Rs.' 000Export 4.5 4.0 0.5 200,000 100Domestic 6.0 4.0 2.0 240,000 480Total Contribution 580Fixed cost ( 240,000 + Overtime 20,000 + fixed overhead 40,00 300Profit 280Proposal three

Particulars S.P/unit V.C./unit Contribution Units Contribution Rs. ' 000Export 4.50 4.00 0.50 200,000 100Domestic 6.00 4.00 2.00 200,000 400Outsourcing 6.00 4.25 1.75 40,000 70Total contribution 570Less Fixed cost ( 240,000 + 3,000 x 12 ) 276Profit 294Proposal fourParticulars S.P/unit V.C./unit Contribution Units Contribution Rs. '000Domestic 6.0 4.0 2.0 240,000 480Less : Fixed cost 240Profit 240Summary of results

As against current price of Rs. 6.00 per unit, the export order offers @ Rs. 4.50 per unit, which is less than the cost of production, the details of which are given below:

The condition of the export is that it has either to be accepted in full or totally rejected. The following alternative proposals are available for decision;

B : Increase factory capacity by installing a few balancing machinery and equipments and also by working extra time to meet the balance of the required capacity. This will increase fixed overhead by Rs. 20,000 annually and the additional cost of overtime will work out to Rs. 40,000 per annum.

C : Out-source the production of additional requirement by supplying direct materials and paying conversion charges of Rs. 1.75 per unit to a small converter, and engaging one supervisor at a cost of Rs. 3,000 per month to look after quality, packing and dispatch.

In this case the additional 40,000 units are outsourced @ Rs. 1.75/ unit with direct materials supplied @ Rs. 2.50 / unit. The variable cost for these 40,000 units is Rs. 4.25 / unit.

Page 322: cost accounting question icwai inter

Strategy A B C DContribution Rs.'000 500 580 570 480Fixed cost Rs.'000 240 300 275 240Profit Rs.'000 160 280 294 240Recommendation: Proposal C is recommended to be accepted.

Factories fig. in Rs. LacsA' bad Bombay Nagpur Total

Sales 300 1,200 600 2,100Direct materials 75 350 145 570Direct labour 75 280 140 495Factory overheads Variable 20 110 55 185

Fixed 40 120 60 220Administrative overheads 20 90 40 150Selling and distribution ove Variable 23 70 40 133

Fixed 15 50 30 95Head office expenses allocated 12 50 30 92Total cost of sales 280 1,120 540 1,940Profit 20 80 60 160

(a) Renew the lease and bear the impact.

Additional information:

1. The sale proceeds of Ahemedabad unit will be sufficient to liquidate all the liabilities there.

Solution: Tutorial Notes:

Factories fig. in Rs. LacsA'bad Bombay Nagpur Total

Sales 300 1,200 600 2,100Direct materials 75 350 145 570

Problem 5: (cwa inter II dec.00)

A large manufacturing company has three factories at Ahemedabad, Bombay and Nagpur. All the three factories produce the same product which is sold at Rs. 375 per unit. Budgets for the year 2001 the three factories are given below:

When the budget was at the final stage, the legal department pointed out that the lease of the Ahemedabad factory was due to expire on 31st Dec.2000 and could be renewed by enhancing the annual lease amount by Rs. 12 lakhs. The management accountant was asked to examine the following options:

(b) Close down the factory at Ahemedabad and simultaneously increase the production of the Bombay unit to meet the demand hitherto met by the Ahemedabad unit;

(c) Close down the factory at Ahemedabad and simultaneously increase the production of the Nagpur unit to meet the demand hitherto met by Ahemedabad unit.

2. If the production at Bombay factory is increased, the variable cost for the additional output will increase by Rs. 25 per unit to meet additional freight and other costs. There will also be an increase of Rs. 50 lakhs in the annual fixed cost inclusive of allocated head office costs.

3. If the production at Nagpur factory is increased, the variable cost for the additional output there will increase by Rs. 35 per unit. The annual fixed cost of the factory will increase by Rs. 40 lakhs, inclusive of allocated head office costs.

1. In option 2 and 3, the variable cost increases by Rs. 25 and Rs. 35 per unit respectively. Question nowhere mentions the number of units. It is hidden somewhere; Think where it is hidden.

Page 323: cost accounting question icwai inter

Direct labour 75 280 140 495Factory overheads Variable 20 110 55 185Selling and distribution ove Variable 23 70 40 133Total variable cost 193 810 380 1,383Contribution (Sales − Variable cost) 107 390 220 717Less : Fixed cost Factory overheads 40 120 60 220Administrative overheads 20 90 40 150Selling and distribution overheads 15 50 30 95Head office expenses allocated 12 50 30 92Total Fixed cost 87 310 160 557Profit: 20 80 60 160Option 1: (a) Renew the lease and bear the impact.

Option 2. (b) Close down the Ahemedabad factory.If A'bad factory is closed down and there is simultaneous increase in production of Bombay factory:

Revised budget figures for Bombay factory, would be as below: All figures in Rs. LacsParticulars Current Addition RevisedSales 1,200.0 300.00 1,500.00Less: Variable cost 810.00 202.50 1,012.50Less: Additional cost of Rs. 0.00 20.00 20.00Total Variable cost 810.00 222.50 1,032.50Contribution : 390.00 77.50 467.50Less : Fixed cost 310.00 50.00 360.00Profit 80.00 27.50 107.50Option 2. (c) Close down the Ahemedabad factory.

Revised budget figures for Nagpur factory, would be as below:Particulars Current Addition RevisedSales 600 300 900Less: Variable cost 380 190 570Less: Additional cost of Rs. 0 28 28Total Variable cost 380 218 598Contribution : 220 82 302Less : Fixed cost 160 40 200Profit 60 42 102

Problem 6: (cwa inter II dec.00)

If Ahemedabad factory is continued, It will cause an additional fixed cost of Rs. 12 lacs as lease rent every year. Thus the profit of the A'bad factory will come down to Rs. 8 lakhs from the current profit of Rs. 20 lakhs. The overall profit will be reduced by Rs. 12 lakhs and will stand at Rs. 148 lakhs.

The unit selling price is Rs. 375. In A'bad the number of units produced is ( Rs. 300 lakhs / Rs. 375 ) = 80,000 units.

Recommendation: In view of the above analysis; The best option is to close down the factory at Ahemedabad and increase production at Nagpur factory.

Last year a company earned 20% profit pre-tax profit on a sales turnover of Rs. 100 lakhs. To improve its profitability and competitiveness, the management has decided to reduce selling price by 10% and increase its output by 20%. Cuts are proposed to be effected on variable and fixed costs at 5% and 20% respectively. What effect will these have on the company's profit this year ? The company was having a fixed cost of Rs. 25 lakhs per annum last year.

Page 324: cost accounting question icwai inter

Solution: Rs..lakhs.Particulars Last year Calculation for this year. This yearSales 100 (100 x 0.90 x 1.20 ) 108.0Profit 20 25.3Contribution : 80 Fixed cost + Variable cost 82.7Fixed cost 25 Reduced by 20%) 20.0Variable cost 55 ( 55 x 0.95 x 1.20 ) 62.7Profit will go up from Rs. 20 lakhs to Rs. 25.3 i.e. an increase of 26.50%.

Product Production / Sales Cost Rs. Production / Sales Cost Rs.P 20,000 units 620,000 25,000 units 700,000Q 30,000 units 550,000 40,000 units 650,000

For P For QUnits Selling price/unit Rs. Units Selling price/unit Rs.

10,000 40 20,000 2415,000 38 30,000 2318,000 36 36,000 2220,000 33 40,000 20Required:

Solution: This question is very easy. Try to solve it before you see the solution which goes as below:

Product Production / Sales Cost Product Production / Sales Costunits Rs. units Rs.

P 20,000 620,000 Q 30,000 550,00025,000 700,000 40,000 650,000

Differ. 5,000 80,000 10,000 100,000Variable cost / unit (120,000Rs. 16 Variable cost/unit (100,000 /10,000) 10 For product P Fixed cost : (620,000 − 16 x 20,000) Rs. 300,000For product Q Fixed cost :(650,000 − 10 x 40,000) Rs. 250,000Option 1: When there is no constraints of labour hours:

For Product P :

Problem 7: (cwa inter II dec.02)

Modern Electronics produces two standard components P and Q widely used by electronics industry. Prices of this product is not steady and competition is acute.

P requires two hours of skilled labour per unit while Q requires only one hour of the same labour. The total availability of skilled labour is currently limited to 66,000 hours per annum and the company is trying to augment this capacity.

The cost department of the firm has prepared the following budget estimates for two possible levels of production / sales of these products for the year 2002.

The marketing department has made the following forecast of probable sale of the two products for 2002.

1. Assuming that the constraint on labour availability can be overcome, how many of the two products should be manufactured and sold for maximum profit ? What is the profit likely to be earned by the firm ?

2. If the availability of skilled labour cannot be augmented, what will be your recommendation and the corresponding profit earned by the firm ?

Costs of the two products at different levels do not vary linearly suggesting that there is fixed element in the total cost figures.

Page 325: cost accounting question icwai inter

Units 10,000 15,000 18,000 20,000Price Rs./unit 40 38 36 33V.Cost Rs. /unit 16 16 16 16Contribution 24 22 20 17Total contributio 240,000 330,000 360,000 340,000Less : Fixed cost Rs. 300,000Profit Rs. 60,000

For product QUnits 20,000 30,000 36,000 40,000Price Rs./unit 24 23 22 20V.Cost Rs. /unit 10 10 10 10Contribution 14 13 12 10Total contributio 280,000 390,000 432,000 400,000Less : Fixed cost Rs. 250,000

Rs. 182,000

Option 2: When labour hour is a limiting factor:

Rs. '000Contribution of 36,000 units of Q @ Rs. 12/unit 432Add :Contribution of 15,000 units of P @ Rs. 22 /unit 330Total contribution 762Less : Fixed cos( 300,000 + 250,000 ) 550Profit 212

Direct materials Rs. / unit 20.0 Total fixed cost p.a. Rs. 15 lakhs.Direct labour Rs. / unit 8.0Variable overhead Rs. / unit 4.0The company is considering the following options to increase profitability by Rs. 5 lakhs:1. Reduce selling price by 5% and

Solution:

Thus 18,000 units of P and 36,000 units of Q give maximum contribution of Rs. 792,000 and profit earned by the firm at this level will be Rs. 242,000.

The profit of the firm is maximum when 18,000 units of P at a selling price of Rs. 36/ unit and 36,000 units of Q at a selling price of Rs. 22/unit is produced. The total labour required for this combination is 18,000 x 2 + 36,000 x 1 = 72,000 hours whereas the labour hours available is only 66,000 hours.

The contribution per hour of P is Rs. 20 /10 = Rs. 10 while that of Q is Rs. 12 / 1 = Rs. 12. As the contribution per labour hour of Q is more, it should be produced first for full production of 36,000 units consuming 36,000 labour hours and the balance of 30,000 hours should be allotted to P for production of 15,000 units of P @ 2 hours per unit.

The most important point is that the selling price for 15,000 units production of P would be Rs. 38/unit and not Rs. 36 which is for the production of 18,000 units. Consequently the contribution per unit would be Rs. 22 per unit.

Problem 8: (cwa inter II dec.02)

A newly formed company has an installed capacity to manufacture 500,000 units of a certain product per annum. Current capacity utilisation is only 40% and the selling price of the product is Rs. 40 per unit. The product has the following cost profile.

2. Spend Rs. 5 lakhs on sales promotion keeping selling price unaltered. How many units have to be sold in either case ?

Page 326: cost accounting question icwai inter

Current producti40% of 500,000 units 200,000 unitsRs. Lacs

Current contribution @ Rs. 8/unit 16Fixed cost per annum 15Current profit 1Desired profit ( increase by Rs. 5 lakhs) 6

1.With revised selling price Rs. 38/unit (95% of 40), The contribution is (38 − 32 ) i.e. Rs. 6/unit.No. of units to be sold = (fixed cost + profit ) / Contribution per unit

= ( 15 + 6 ) 6 = 3.50 lakh or 350,000 units.

Thus no. of units required to be sold = 26 / 8 = 3.25 lakh unit or 325,000 units.

The following set of information is available ( Rs. / unit )Product Selling price Direct materials Direct wages Rs./unit

Rs. Rs. Dep.A Dep.B Dep.CP 300 60 20 15 10Q 275 30 20 20 10R 305 70 12 10 20The absorption rates of overhead on the direct wages are given below:

Dep.A Dep.B Dep.CVariable overhead 150% 120% 200%Fixed overhead 200% 240% 150%You are required to find out − (i) Profitability of the three products; and

(ii) P/v ratio of the three products.Also give your short comments on the decision of the management.Solution : Try to solve it by yourself. The solution follows:Statement showing the comparative profitability of the three products:Particulars P Q R

Rs./unit Rs./unit Rs./unit(A) Selling price 300 275 305Direct materials 60 30 70Direct labour 45 50 42Variable overhead Department A 150% of wages. 30 30 18Department B 120% of wages. 18 24 12Department C 200% of wages. 20 20 40( B) Total variable cost 173 154 182(C) Contribution (A) − (B) 127 121 123(D) Fixed Overhead Department A 200% of wages. 40 40 24Department B 240% of wages. 48 48 24Department C 150% of wages. 15 15 30Total Fixed Overhead 103 103 78Profit (C) − (D) 24 18 45Pv ratio (Contribution / Sales ) 42.3% 44.0% 40.3%

2. If additional Rs. 5 lakhs is spent The contribution required would be ( 15+6+5) i.e. Rs. 26 lakhs @ contribution Rs. (40 − 32) i.e. Rs. 8/ unit.

Problem 9: (cwa inter II june.03)

XYZ ltd. produces and sells three types of products - P, Q and R. The management has decided to discontinue the production of Q since there is not much profit in it.

Page 327: cost accounting question icwai inter

Comments:(i) Direct materials cost of the product C is lowest of the three.(ii) Pv ratio of product Q is the highest.

Results for 2002 − 03 were : amount Rs. Item Total company Kalidas TulsidasSales revenue 700,000 350,000 350,000Cost of merchandise sold 450,000 225,000 225,000Gross margin 250,000 125,000 125,000Operating expenses:Salaries and wages 63,000 30,000 33,000Supplies 45,000 22,500 22,500Rent and utilities 60,000 40,000 20,000Depreciation 15,000 7,000 8,000Allocated staff cost 60,000 30,000 30,000Total operating expenses 243,000 129,500 113,500Operating income ( loss ) 7,000 (4500) 11,500

Required:

2. Evaluate the performance of each book store. 3. Evaluate the performance of each manager.Solution: Performance report of Stores and Store-managers of Pupil's Book Company.

Item Total company Kalidas TulsidasRs. Rs. Rs.

Sales revenue 700,000 350,000 350,000Cost of merchandise sold 450,000 225,000 225,000Salaries and wages @ 8% of goods sol 36,000 18,000 18,000Supplies 45,000 22,500 22,500Total Variable cost 531,000 265,500 265,500Contribution margin 169,000 84,500 84,500Less : Fixed cost Controllable by store-managersSalaries and wages (Gross −Variable) 27,000 12,000 15,000

(iii) The direct labour cost of Q is the highest while the selling price is the lowest. The management is advised to look into this.

Problem 10: (cwa inter II dec. 04)

The Pupil's book Company has two book selling outlets: Kalidas book house and Tulsidas book house. Each store has a manager who has a great deal of decision authority over the individual stores. A central office however, handles advertising, marketing research, acquisition of books, legal services, and other staff functions. The Pupil book co. current accounting system allocates all costs to the stores.

Each bookstore manager makes decisions that affect salaries and wages, supplies and depreciation. In contrast, rent and utilities are beyond the manager's control because the manager did not choose the location or size of the store.

Supplies are variable costs. Variable salaries and wages equal to 8% of the cost of merchandise sold ; the remainder of salaries and wages are fixed cost. Rent, utilities and depreciation are also fixed costs. Allocated staff costs are unaffected by any events at the bookstores, but they are allocated as proportion to sales revenue.

1. Using the contribution approach, prepare a Performance report that distinguishes the performance of each bookstore from that of the bookstore manager.

Page 328: cost accounting question icwai inter

Depreciation 15,000 7,000 8,000Fixed cost controllable by store-manage 42,000 19,000 23,000Contribution controllable by store-mana 127,000 65,500 61,500Less: Fixed cost not controllable by managerRent and utilities 60,000 40,000 20,000Contribution by book−store 67,000 25,500 41,500Less : Allocated staff cost 60,000 30,000 30,000Operating income ( loss ) 7,000 (4,500) 11,500

3.Evaluation of performance of stores managers:

Following data are in respect of a firm manufacturing a single product for a particular period:Sales (20,000 units) Rs. 200,000Cost of production (20,000 units) Rs. 120,000Selling and distribution expenses Rs. 30,000Maximum capacity units 25,000

What minimum price may be quoted for the foreign offer ?2) What should be the minimum price had the offer size been 8,000 units instead of 4,000 units ?Solution: Tutorial Notes:

Solution goes as follows: Total Rs. Per unitsSales 20,000 units 200,000 10.0Variable CostsCost of production (120,000 − 40,000) 80,000 4.0

2.Evaluation of performance of book-stores:

The financial performance of book stores can be seen at contribution by book-stores above. The contribution by Kalidas is Rs. 25,500 and that by Tulsidas is Rs. 41,500. At a glance, the performance of Tulsidas looks far better than that of Kalidas, but actually it is not so. The rent and utilities which are beyond the control of the stores and the managers as well, eat away major portion of contribution of Kalidas. It must be investigated and if possible rectified, that why Kalidas is paying just double for rent and utilities than Tulsidas.

The contribution controllable by the managers are Rs. 65,000 by Kalidas manager and Rs. 61,500 for Tulsidas manager. The manager of Kalidas book stores incurs Rs. 4,000 less on account of fixed cost. In the absence of Master budget and flexible budget, the performance comparison is just approximate.

Problem 11: (cwa inter II June. 05)

Fixed costs included in cost of production are Rs. 40,000 and only variable cost included in selling and distribution expenses are commission @ 10% on sales and packing expenses @ 20 p.per unit.

1) An order for purchase of 4000 units is received from outside India. No sales commissions is payable on such foreign order but packing costs will be 80 p.per unit.

1. The maximum capacity is 25,000 units and present production is 20,000 units. New order is for 4,000 units. This will be within the capacity. Contribution from present units need not be sacrificed for new units.

2. If the order size is for 8,000 units, The total production required would be 28,000 units which is beyond the present capacity. Thus contribution from 3,000 units will have to be sacrificed if this order is to be accepted.

3. The costs are to be segregated into variable and fixed components. Note that no fixed cost need be incurred for the new order. Only variable costs are relevant for decision making regarding the new order.

Page 329: cost accounting question icwai inter

Selling and distribution expenses (10% o 20,000 1.0Packing expenses @ 20 paise/unit 4,000 104,000 0.2 5.2Contribution 96,000 4.8Less ; Fixed costsCost of production 40,000Selling and distribution (30,000 −20,000 6,000 46,000Profit 50,000Price Rs./unit to be quoted for new order of 4,000 unitsVariable cost for production Rs.4.0/ unit + packing charges @ R Rs. 4.80/unitPrice Rs./unit to be quoted for new order of 8,000 units Rs.Contribution from 3000 units @ Rs. 4.8/ units 14,400Variable cost @ Rs. 4.8/ unit for 8,000 units 38,400Total cost of new order of 8,000 units 52,800Minimum price per unit that can be quoted (52,000 / 8,000) Rs. 6.60 / unit

How much selling price should be increased to maintain the present rate of profit ?Solution: Solve on the lines of problem 12 above. The answer isAnswer : Current profit : Rs. 392.45 and future profit at current selling price = Rs. 235.47; Revised selling price : Rs. 1,808

First quarter Second quarterSales (50,000 @ Rs. 24) 1200000 (70,000 @ Rs. 24) 1680000Cost of goods sold 700,000 880,000Gross margin 500,000 800,000Selling and admin. 650,000 690,000Net income before tax (150000) 110,000

Required:

Problem 13: (ca inter may.01)

A company manufacturing radios which are sold at Rs. 1,600 per unit. The total cost is composed of 30% of direct materials,40% for direct wages and 30% of overheads. An increase in material price by 30% and 10% in direct wages is expected in the forthcoming year; as a result of which the profit at current selling price may decrease by 40% of the present profit per unit. You are required to prepare a statement showing current and future profit at present selling price.

Problem 14: (cwa final June 97)

A food processing firm recently introduced a new product of processed and formed potato chips. The chips are sold in packed cases and a case is considered the basic sales unit. Although management made detailed estimates of cost and volume before it undertook this venture, new projections based on actual cost experience are now needed.

Income statement for the two quarters are each thought to be representative of the costs and productive efficiency that can be expected in the next few quarters. There were virtually no inventories on hand at the end of either quarter. These income statements reveal the following :

The earnings and losses incurred by this product line are, so far, negligible in relation to the firm's overall earnings.

The firm's overall marginal and average income tax rate is 40% and the same is to be used to estimate the tax liability arising from the potato chips operation.

a. Management would like to know the breakeven point in terms of quarterly case sales for the chips.

Page 330: cost accounting question icwai inter

Solution:

Let us compute the variable and fixed cost element:First quarte Second quarter Difference Diff./unit Fixed cost

Units 50,000 70,000 20,000Rs. Rs. Rs. Rs. Rs.

Cost of goods s 700,000 880,000 180,000 9 250,000Selling and adm 650,000 690,000 40,000 2 550,000

Break-even point (Fixed cost Rs. 800,000) = Rs. 800,000/ Rs. 1 61,538 units.

After tax return required is 20% of Rs. 30 lakhs i.e. Rs. 6 lakhs. Tax rate is 40%The pre-tax return required would be Rs. 6 lakhs/ 0.60 = Rs. 10 lakhsPre-tax return per quarter = Rs. 10 lakhs/ 4 = Rs. 250,000Total contribution required = Fixed cost Rs. 800,000 + Return Rs. 250,000 = Rs. 1050,000Contribution per unit is Rs. 13. Break even sales in units = 1800 80,769Sales revenue at break-even = Rs. 24 x 80,769 Rs. 19,38,462Answer c. Proposed Selling price in Rs. Per unit ( Rs. 24 − Rs. 1.50) 22.50Sales volume (20% of second quarter sales i.e. = 1.20 x 70,000) 84,000 units

Rs.Contribution @ Rs. 11.50. (Variable cost Rs. 11 per unit) 966,000Less Fixed cost ( Rs. 800,000 + advertising expenses Rs. 150,000) 950,000Profit before tax 16,000

Manufacturing variable cost : Rs. 30; Manufacturing Fixed cost : Rs. 5Marketing variable cost : Rs. 15 ; Marketing Fixed cost : Rs. 9 and Selling price : Rs. 60.

b. Management estimates that an investment of Rs. 30 lakhs is made in this product line. What quarterly case sales and total revenue are required each quarter to earn an after tax return of 20% on investment.

c. The firm's marketing people predict that if the selling price is reduced by Rs. 1.50 per case and a Rs. 150,000 advertising campaign is mounted to call customers attention to the new reduced price, sales will increase by 20% over second quarter sales. Should the plan be implemented ?

Answer a.

A look at the figures of cost of goods sold and selling and admin expenses reveals that these are semi-variable in nature having some fixed cost element in them.

We thus have variable cost of the units sold as Rs. 11/unit and Rs. 800,000 as fixed cost. At selling price of Rs. 24/unit, the contribution per unit is Rs. 13.

Answer b.

The proposed change does not appear to be acceptable as it lowers the net profit before tax from Rs. 110,000 to Rs. 16,000.

The effect of advertising may not be apparent in the short run of one quarter. If management expects to explore new areas of customer exposure with the advertisement, the loss on profit may be ignored as the sales volume is likely to enhance further in coming quarters because of new exposure.

The marketing people may be supplied with this analysis to make and take suitable approach and action.

Problem 15: (cwa final June 2000)

The cost sheet per unit of a standard product manufactured by Excelco at a production level of 2,400 units per month is as under :-

Page 331: cost accounting question icwai inter

Manufacturing variable cost : Rs. 50; Manufacturing Fixed cost : Rs. 10Marketing variable cost : Rs. 20 ; Marketing Fixed cost : Rs. 6 and Selling price : Rs. 90.

Require

Solution :

Note that the costs are given in per month basis while the profit is given as Rs. 144,000 per annum.

Manufacturing fixed cost for standard : 2400 x 5 = Rs. 12,000Manufacturing fixed cost for Deluxe @ Rs. 10/ unitNo. of units of Deluxe model that can be manufactured = Rs. 12,000 / 10 = 1,200Computation of Total Fixed cost :

Rs.Manufacturing fixed cost for the standard product (2400 x 5) 12,000Marketing fixed cost of standard product ( 2400 x 9) 21,600Marketing fixed cost of Deluxe (1200 x 6) 7,200Total Fixed cost per month 40,800Add : Total profit required per month (144,000 / 12) 12,000Total contribution required from both the products 52,800Contribution given by Deluxe model @ Rs. 20/unit for 1,200 units 24,000Contribution to be given by the 2,400 units of standard product 28,800Contribution per unit of standard Rs. 12. Offer price payable is Rs. 60 − Rs. 12 i.e. Rs. 48/unit.

A company has prepared the following budget for the year :60% 80%Rs. Rs.

Raw materials 30,00,000 40,00,000Direct labour 18,00,000 24,00,000Factory Overheads 32,00,000 36,00,000Total 80,00,000 100,00,000The policy of the company is to charge 25% on the variable cost to cover profit.

The company has to execute a job as per details given below:

Excelco proposes to sub-contract the manufacture of the standard product to its Associate Company and use the plant capacity for the manufacture of Deluxe product whose cost price data are as under :

The manufacturing fixed overheads will remain unchanged in total at the original level envisaged for the standard product. The fixed marketing cost of the Deluxe product given above will be incurred over and above the fixed marketing costs as envisaged for the standard product. The company desires a minimum operating profit of Rs. 144,000 on the two products together per annum.

What maximum purchase price per unit of standard product should be quoted for sub-contracting the production of the Associate Company ?

The manufacturing fixed cost for standard product is Rs. 12,000. This will be fixed cost for Deluxe model if standard is sub-contracted. The marketing fixed cost for standard is Rs. 21,600 ( 2400 x 9). This will continue to be incurred even if the product is sub-contracted. In addition to this the marketing fixed cost for Deluxe will also be incurred. Thus total fixed cost would be aggregate of the following :

Problem 16: (cwa final June 2000)

Raw materials is in short supply and the company wants to utilize its available supply of raw materials in an optimum manner. Planned operating capacity is 80%.

Raw materials : Rs. 40,000 and Direct labour : Rs. 30,000. You are required to quote the price of the job in accordance with the policy of the company.

Page 332: cost accounting question icwai inter

Solution :

At 80% activity the overheads are Rs. 36 lacs and at 60% activity the overheads are Rs. 32lacs.Level Overheads Rs. In lacs80% 36 For 80% the variable cost is Rs. 16 lacs.60% 32 For 60% the variable cost is Rs. 12 lacs.

Differen20% 4Thus the fixed factory overheads is Rs. 20 lacs.

At 80% level of operation : Rs. In lacsDirect materials 40Direct labour 24Variable factory overheads (2/3rd of direct labour) 16Total variable cost 80Add : Profit @ 25% of variable cost 20Add : Fixed factory overheads 20Selling price 120Contribution 40Contribution as percentage of raw materials 100%

Rs. Product pricing : Direct materials 40,000

Direct labour 30,000Variable overheads (2/3rd of direct labour) 20,000Contribution foregone 40,000Selling price 130,000

Volume in unit p.m. Selling price/ unit (Rs.)(i) up to 50,000 1.8(ii) 50,000 to 80,000 1.7(iii) 80,000 to 90,000 1.6(iv) Above 90,000 1.5

Variable cost is Rs. 0.30/ unit and Average salary of a worker is Rs. 2000 p.m. Find out the production volume at which profit is the highest and calculate the maximum profit.Solution :

The figures of factory overheads indicate that the variation is not linear between the two levels of operation as is the case with raw materials and direct labour. This implies that there is some fixed cost element in factory overheads. Let us first calculate the fixed overheads of factory overheads.

The variable factory overheads may be assumed to be moving linearly with direct labour. It is 2/3rd of Direct labour.

The policy of the company is to charge 25% over variable cost. Let us find out the variable cost at 80% capacity which is planned level of operation.

Since the raw material is in short supply, contribution per unit of raw materials should be guiding factor for quotation of the price.

For the job in question, the raw materials required is Rs. 40,000. The contribution foregone for this quantity of raw materials is Rs. 40,000 as shown above

Problem 17: (cwa final dec. 2000)

Excellent Products Ltd. has a manufacturing capacity of 100,000 units of product A per month. The selling price varies with volume as follows

Fixed cost work out to Rs. 80,000 per month. But, for achieving a production beyond 60,000 units p.m. three additional workers and beyond 80,000 units p.m. five additional workers would be required.

Page 333: cost accounting question icwai inter

Production p.m. 50,000 60,000 70,000 80,000 90,000 100,000Rs. Rs. Rs. Rs. Rs. Rs.

Selling price Rs. 1.80 1.70 1.70 1.70 1.60 1.50Sales revenue 90,000 102,000 119,000 136,000 144,000 150,000V. cost @ Rs. 0. 15,000 18,000 21,000 24,000 27,000 30,000Contribution 75,000 84,000 98,000 112,000 117,000 120,000Fixed cost 80,000 80,000 80,000 80,000 80,000 80,000Additional fixed cost* 6,000 6,000 10,000 10,000Total Fixed cost 80,000 80,000 86,000 86,000 90,000 90,000Profit (loss) (5,000) 4,000 12,000 26,000 27,000 30,000

The following are the present cost and output of a manufacturer :Product Price/unit Rs. Variable cost / unit Rs. % of salesA 360 240 50%B 600 360 30%C 800 480 20%

Total Fixed cost per year: Rs. 150,000Volume of sales last year: Rs. 500,000

Product Price/unit Rs. Variable cost / unit Rs. % of salesA 360 240 50%B 600 360 20%D 850 450 30%

In the change worth taking ?Solution: Present ProposalProduct Price/unit Rs. Variable cost / unit Rs. % sales Sales Rs. Contribution

Rs. Rs. Rs. Rs.A 360 240 50% 250,000 83,333B 600 360 30% 150,000 60,000C 800 480 20% 100,000 40,000Total contribution 183,333

Proposed proposalProduct Price/unit Rs. Variable cost / unit Rs. % sales Sales Rs. Contribution A 360 240 50% 250,000 83,333B 600 360 20% 100,000 40,000D 850 450 30% 150,000 70,588Total contribution 193,922Increase in total contribution 10,588

Note : Additional fixed cost is incurred on account of 3 additional workers @ Rs. 2000/month beyond 6000 units and 5 additional workers beyond 8,000 units @ Rs. 2000/month.

Result ; The profit is maximum at Rs. 30,000 per month at a production and sales level of 100,000 units.Problem 18: (cwa final dec.02)

The manufacturer is considering whether to drop product C and replace it with product D. If this drop-add decision is taken, the cost and output would be as follows :

The drop-add decision would enhance the contribution by Rs. 10,588. It is recommended to go ahead with the proposal.

Fixed cost remaining the same under both the options, has not been considered in the decision making process.

Problem 19 (cwa final Dec.02)

Page 334: cost accounting question icwai inter

Asha Peripheral Limited ( Rs. In lakhs )Material 2,800 Sales 4,800Labour 1,260 Loss 800Variable overhead 1,260Fixed overhead 280Total 5,600 5,600

Solution:Let us compute the various cost elements of Sales; fig. in Rs. LakhsUnits sold in lakhs = Sales / Selling price per unit = 4,800 / 30=160 lakhs.Computation of Cost elements of sales (Rs. In lakhs) Standard Actual(i) Material cost @ Rs. 15/ unit (160 x 15) 2,400 2,800(ii) Labour cost @ Rs. 7/unit (160 x 7) 1,120 1,260(iii) Overhead cost @ Rs. 4/unit (160 x 4) 640 1,260

4,160 5,320Excess and extra use of resources than the standard requirement: 1,160

Rs. Lacs Rs. LacsMaterial 2,400 Sales 4,800Labour 1,120Variable overhead 640Fixed overhead 280Profit 360Total 4,800 4,800

A company produces two products AXE and BXE. The selling price and cost data are as under :AXE BXE

Per unit Rs. Rs.Selling price 1,200 1,800Direct materials 250 300Direct wages @ Rs. 5/hour No. of

employeesDepartments 1 200 300 300

2 100 200 1603 300 0 1804 0 400 240

Variable overheads 50 150Fixed overheads per annum Rs. 12,50,000The company operates 250 days of work per annum of 8 hours per day. The employees can not be increased nor can they be transferred from one department to another.

Asha Peripherals Ltd. is engaged in the business of manufacturing a Computer peripheral, which sells @ Rs. 30/unit. The material and labour costs are Rs. 15 and Rs. 7 per unit respectively. The variable overhead charges per unit are Rs. 4. The production account of Asha Peripherals for the year 2001 is as follows:

Assuming that there is no opening stock and closing stock of finished goods, work-in-progress, or unutilized material, find out whether the resources of material, labour and overhead are optimally utilised. Also draw, the revised Production account of Asha Peripheral Ltd.

It is evident from the above that management is not ensuring optimum utilisation of resources and incurring excess resources consumption to the extent of Rs. 1.160 lacs. The revised production account should be as follows:

Problem 20 (cwa final Dec.97)

Page 335: cost accounting question icwai inter

Required :

Solution :Tutorial Notes:

1. Computation of availability of labour hoursDepartments 1 2 3 4Number of employees 300 160 180 240Available labour hours @ 2000 hrs/emp 600,000 320,000 360,000 480,0002. Contribution analysis fig. in Rs.Per unit Selling price Direct materials Direct labou Voh. Contribution

Rs. Rs. Rs. Rs. Rs.AXE 1200 250 600 50 300BXE 1800 300 900 150 4503. Computation of hours needed for each product in every department :

Dept. AXE Available. Possiblewages hours production

Rs. in units1 200 40 300 7.5 600,000 15,0002 100 20 300 15.0 320,000 16,0003 300 60 300 5.0 360,000 6,0004 0 0 300 480,000 0

Dept. BXE Available. Possible

wages hours production

units1 300 60 450 7.5 600,000 10,0002 200 40 450 11.3 320,000 8,0003 0 0 450 0.0 360,000 04 400 80 450 5.6 480,000 6,000

Dept. Available. Hours reqd. for 6000 units of AXE Hr/unit for BXE Possiblehours hr/unit total Balance product.

1 600,000 40 240,000 360,000 60 6,0002 320,000 20 120,000 200,000 40 5,0003 360,000 60 360,000 0 0 0

1. Calculate the number of units of each product that should be manufactured to yield maximum profit and compute the amount of such maximum profit.

2. If the supply of direct materials is restricted to Rs. 15 lakhs per annum which product would you produce and in what quantity ?

1. It is evident from the question that the labour hours may be the limiting factor. Compute the labour hours available in every department. The contribution of both the products can be ascertained. The contribution analysis will show product offering highest contribution.

2. Compute the labour hours required by each product in respective departments. This will tell you the production capacity of every department, this is second limiting factor.

Contribution in Rs.

Contribution/hrhours

required

Contribution / hrhours

requiredContributio

n in Rs.

Contribution is highest in case of product AXE in department 2. It should be produced to the fullest possible extent. The available labour hours in every department will decide the possible production.

Department 3 will allow 6,000 units of AXE to be produced. The balance hours left after production of 6,000 units of AXE will be utilised for production of BXE.

Page 336: cost accounting question icwai inter

4 480,000 0 0 480,000 80 6,000It is recommended to produce 6,000 units of AXE and 5,000 units of BXE.Contribution Analysis of scheme of production as suggested above :Product AXE Rs. BXE Rs. TotalSelling price/unit 1,200 1,800Contribution/unit 300 450units produced 6,000 5,000Contribution Rs. Lacs 18.0 22.5 40.5Less Fixed cost Rs. Lacs 12.5Profit Rs. Lacs 28.0Contribution analysis with Direct materials as limiting factor (Rs. 15.0 lacs.)Product AXE Rs. BXE Rs.Contribution/unit 300 450Direct materials 250 300Contribution/unit of direct materials 1.2 1.5

You are required to compute the ratio of the mix of the raw materials 'AXE' and 'CXE'.Solution: Computation of Selling price of product 'A':

For one Kg. of product A: Raw materials weight kg. Rs./kg Amount Rs. AXE 0.625 150 93.75BXE 0.625 90 56.25Total Cost of material : 150

Add: Production expenses @ 40% of material cost 60Total cost of product A 210

Add: Profit @ 20% of total cost 42Selling price Rs. 252

Computation of proportions of materials AXE and CXE in 'A':Let material CXE required in product A is a kg.Then for 1 kg. of A, material AXE is required as (1.25 − a) kg.

Thus a kg. of CXE @ Rs. 75/kg and (1.25 − a) kg. of AXE @ Rs. 150/kg.should equate to Rs. 150.

The equation becomes : a x Rs. 75 + (1.25 − a) x Rs. 150 = 150 which gives a = 0.50 kg.

In this case product BXE should be produced first to the fullest extent possible. From the above table it is seen that dept. 2 will allow 5,000 units of B to be produced. The raw materials limitation will allow (Rs. 15 lac/ Rs. 300) 5,000 units to be produced.

Problem 21 (ca inter may.07)

Raw materials 'AXE' costing Rs. 150/kg. and 'BXE' costing Rs. 90/kg are mixed in equal proportions for making product 'A'. The loss of material in processing works out to 25% of the product. The production expenses are allocated at 40% of direct materials cost. The end product is priced with a margin of 20% over the total cost.

Material 'BXE' is not easily available and substitute raw material 'CXE' has been found for 'BXE' costing Rs. 75/kg. It is required to keep the proportion of this substitute material in the mixture as low as possible and at the same time maintain the selling price of the end product at existing level and ensure the same quantum of profit as at present.

For one kg. of product A, Input of 1.25 kg of materials is required. The raw materials 'AXE' and 'BXE' are mixed in equal proportions.

The selling price is to remain at Rs. 252, and the profit is to remain at Rs. 42 per kg. Thus the total cost of materials AXE and CXE should remain at Rs. 150 as worked out earlier.

Page 337: cost accounting question icwai inter

The requirement of AXE would be (1.25 − 0.50) = 0.75 kg.AXE and CXE are to be mixed in the proportion of 0.75:0.50 or 3:2.

Data from tentative budget for 1998 :Products P Q R S Production/Sales units 1,000 1,200 1,600 800Selling price per unit Rs. 100 130 120 150Variable cost per unit Rs. 60 80 50 70Labour hour per unit 3 4 2 5Material usage per unit kg. 2 3 4 5

(a) Determine the optimal sales mix for the company.

Comparison of Opportunity Costs with the Contribution of productsProducts P Q R S Production/Sales units 1,000 1,200 1,600 800Selling price per unit Rs. 100 130 120 150Variable cost per unit Rs. 60 80 50 70Contribution per unit Rs. 40 50 70 80Opportunity cost :Labour @ Rs. 2.50/hr/unit 7.5 10.0 5.0 12.5Material @ Rs. 16.25/kg/unit 32.5 48.75 65 81.25Total Opportunity cost 40 58.8 70.0 93.75Net Contribution 0 (8.75) 0 (13.75)

In view of the negative contribution of Q and S, Only P and R are advised for production.Computation of available labour hoursProducts P Q R S TotalProduction/Sales units 1,000 1,200 1,600 800Labour hour per unit 3 4 2 5Material usage per unit kg. 2 3 4 5Labour hours avahrs 3,000 4,800 3,200 4,000 15,000Material availablkg. 2,000 3,600 6,400 4,000 Err:522Computation of Quantity to be produced of P and Q :

From the available of labour hours 3P + 2R = 15,000From the available material 2P + 4R = 16,000Which give P = 3,500 units and Q = 2,250 units.

Problem 22 (cwa final Dec.97)

P,Q,R and S are the four types of products that appear in the price-list of a company with a note that a particular item or items may not be available on demand. The demand for the product is more than what the company can supply and non-supply of any of them will have no effect on the demand for the rest.

For the calendar year 1998, the company has made the following tentative budget that will use up all the available supplies of material and labour in that year.

A linear programming was made by the accountant of the company who stated that the opportunity costs or the shadow prices came to Rs. 2.50 per labour hour and Rs. 16.25 per kg. of material. He also suggested the product-mix which has since been forgotten. The accountant has left the company. The company now asks you as their Management Consultant to give your opinion about the budgeted program.

(b) What difference the sales mix in (a) will make from that in the tentative budget in respect of contribution ?Solution : You should try to solve this before you go through the solution.

Page 338: cost accounting question icwai inter

Contribution of proposed scheme = 3,500 x Rs. 40 + 2,250 x Rs. 70 297,500Products P Q R S Production/Sales units units 1,000 1,200 1,600 800Contribution per unit Rs. Rs. 40 50 70 80Total Contribution Rs. 40,000 60,000 112,000 64,000Total contribution of all the four products 276,000Contribution of proposed scheme as computed earlier 297,500Extra Contribution due to linear progra Rs. 21,500

Some selected financial figures for the last three years are given for a company as under:Year 1 Year 2 Year 3

G.P. Ratio 40% 30% 20%Stock Turnover 20 25 20Opening stock Rs. 90,000 110,000 130,000Closing stock Rs. 110,000 130,000 170,000Administrative Expenses every year amounted to 10% of Sales. Income tax rate is 40%

Statement of Profitability unit Year 1 Year 2 Year 3

Opening stock Rs. Rs. 0.90 1.10 1.30Closing stock Rs. Rs. 1.10 1.30 1.70Average stock Rs. Rs. 1.00 1.20 1.50Stock Turnover times 20 25 20Cost of goods sold Rs. LacsRs. Lacs 20 30 30G.P. Ratio % 40% 30% 20%Sales Rs. Lacs (20/0.6) 33.3 (30/0.7) 42.9 37.5G.P. Rs.lacs Rs. Lacs 13.3 12.9 7.5Less:Admin. Exp.@ 10% of Rs. Lacs 3.3 4.3 3.8Profit Before Tax Rs. Lacs 10.0 8.6 3.8Tax @ 40% Rs. Lacs 4.0 3.4 1.5Profit after tax Rs. Lacs 6.0 5.1 2.3Profit after tax as % of sales% 18% 12% 6%

Solution :

Problem 23 (cwa final Dec.01)

You are required to prepare a comparative statement of profit for the three years and give your comments for variations in profitability.

Solution : You should try to solve this before you go through the solution.

Tutorial Notes : The stock turnover should be taken on cost of goods sold and not on sales.Without carrying out any calculations whatsoever, It is obvious from the question that the performance of the company is deteriorating gradually as shown by the reduction of G.P. ratio from year 1 to year 3. Declining G.P. ratio indicates the upward trend of cost of goods sold. A thorough study in respect of various elements of cost of goods sold needs to be undertaken by the management.

Problem 24 (cwa final June 04)

B Ltd. which manufactures components for VCD, has a capacity to produce 4 lakh units. The market demand is sensitive to the sale price and the company could sell 1 lakh units at a price of Rs. 50 each. The demand thereafter would double for each Rs. 5 per unit fall in the selling price. The company expects a minimum margin of 25%. What would be the target cost of the company to sell at full capacity utilisation?

Page 339: cost accounting question icwai inter

Brilliant Company has provided you with the following information for two ye fig. in croresSales 2002 2003Cost of sales Material 444.44 480

Variable expenses 280 318.85Fixed expenses 100 120

50 61.15430 500

Profit (loss) 14.44 (20)

Sale price − by 20%, Material price − 15% and Expenses − by 20%

Solution :Working Notes:1.Material used in 2002 280.0

Less : 10% decrease in volume 28.0252.0

Actual material used 2003 318.85Less: 15% increase in prices (318.85 x 15) / 115) 41.59Actual material used in 2003 at 2002 prices 277.26Material usage variance 25.26 Adverse

2. Expenses in 2002 100Fall in sales volume : 10% 10

90Actual expenses 120Less : 20% increase (120 x 20/120) 20

100Variable expenses efficiency variance (90 − 100) 10 Adverse

3. Fixed expenses for 2003 61.15Less : 20% ( 61.15 x 20/120) 10.19

50.96Fixed expenses for 2002 50Volume variance 0.96 adverse

4. Sales volume variance / contribution: Profit and fixed cost : 64 64.44The drop is 10%, hence loss of contribution Rs. 6.44 6.44 Adverse5. Sales in 2003 480

Sales in 2003 at 2002 prices (480 x 20/120) 400Sales price variance 80 fav.

Brilliant Limited Company.Summary of variances and reconciliation of profit / (in Rs. Crores)

As the demand doubles for every Rs. 5 fall in selling price, beginning with Rs. 50 selling price at 1 lakh capacity, the selling price for 4 lakh units will be Rs. 40. The margin is 25%, thus the target cost should be 75% of the selling price i.e. Rs. 30.

Problem 25 (cwa final June 04)

It has been also observed that during the year 2003 the following increase in average prices were recorded:

You are required to analyze the reasons for change in profitability from 2002 to 2003 in a tabular form showing variances arising due to sales price variance, volume of sales variance (Contribution), material price variance, material use variance, variable expenses expenditure variance, variable expenses efficiency variance, fixed expenses expenditure variance and fixed expenses volume variance.

Page 340: cost accounting question icwai inter

Components Variance Adverse Favorable AmountProfit 2002 14.44Sales price variance 80Sales volume variance 6.44Materials price variance 41.59Materials usage variance 25.26Variable expenses expenditure variance 20Variable expenses efficiency variance 10Fixed expenses: Expenditure variance 10.19Fixed expenses : Volume variance 0.96

114.44 34.44Loss in 2003 20.00

Solutio Investment with Inventory turnover 12 : 18 / 12 = Rs. 2 lakhsInvestment with Inventory turnover 9 : 18 / 9 = Rs. 1.5 lakhsSaving in investment Rs. 0.50 lakhsSaving in interest @ 10% 10% of Rs. 50,000 Rs. 5,000

Solution: Demand 20,000 40,000 80,000Selling price Rs. 100 90 80

The following information of a company is available for the year 2006:Rs.

Sales 40,000Raw materials 20,000Direct wages 6,000Variable and Fixed overhead 10,000Profit 4,000Units sold 200 Nos.

Solution: Tutorial Notes :

(ii) The selling price will remain the same in both the years. The sales will naturally be different.

A company has forecast sales and cost of sales for the coming year as Rs. 25 lakhs and Rs. 18 lakhs respectively.

The inventory turnover has been taken as 9 times per year. In case the inventory turnover increases to 12 times and the short term interest rate on working capital is taken as 10%, what will be saving in cost?

Problem 26 (cwa final June 03)

A company has the capacity of production of 80,000 units and presently sells 20,000 units at Rs. 100 each. The demand is sensitive to selling price and it has been observed that with every reduction of Rs. 10 in selling price the demand is doubled. What should be the target cost at full capacity if profit margin on sale is taken as 25%

The margin is 25% on selling price. The target cost for a sale of 80,000 units should be 75% of selling price i.e.75% of Rs. 80 = Rs. 60

Problem 27 (ca final may 07)

In the year 2007, wages rate will increase by 50% and fixed cost will decrease by Rs. 600. if 300 units are sold in 2007, the total fixed and variable overhead will be Rs. 11,400. How many units should be sold in 2007, so that the same amount of profit per unit as in year 2006 may be earned?

(i) You must know how to separate the variable and fixed component from mixed figure. The fixed and variable overhead for two levels have been given. Variable and fixed components should be separated by way of working Note.

Page 341: cost accounting question icwai inter

2006 2007Variable and Fixed overhead 10,000 11400 + 600 12,000Difference for 100 units (Variable component) 2,000Variable component Rs. 4,000 6,000Fixed component Rs. 6,000 6,000Computation of Profit for the current year 2006:

2006 2007Rs. Rs. Rs.

Sales 40,000 200 200Raw materials 20,000 100 100Direct wages 6,000 30 45 (with increase of 50%)Variable cost 4,000 20 20Contribution 10,000 50 35Less: Fixed cost 6,000 30 Profit 4,000 20 20 remains same

The question says that profit per unit remains the same in both the years. Rs./ unit

Contribution 35Less: profit per unit 20Net contribution/unit 15Fixed Cost (6,000 − 600) Rs. 5400No. of units to be sold (5400 / 15) 360 units.

Required:

(ii) At what minimum sales will the company be better off by locking up business in 19x2 ?Solution: Tutorial Notes :

(iv) As the no. of units is to be computed, the cost per unit of current year sale must be determined first. You must note that the question is about same amount of profit per unit and not the total profit is to remain same.

(ii) This question is a simple one. You should try to solve it before you see the solution which goes as follows:

Problem 28 (ca final may 01)

In 19x1, The turnover of a company, which operated at a margin of safety of 25% amounted to Rs. 900,000 and its profit volume ratio was 33.33%. During 19x2, the company estimated that although the same volume of sales as in 19x1 would be maintained, the sales value would go down due to decrease in selling price. There will be no change in variable costs. The company proposes to reduce its fixed costs through an intensive cost reduction programme. These changes will alter the profit volume ratio and margin of safety to 30% and 40% respectively in 19x2.

Even if the company closed down its operations in 19x2, it would incur a minimum fixed cost of Rs. 50,000.

(i) Present a comparative statement indicating the sales, variable costs, fixed costs and profit for 19x1 and 19x2.

1. The pv ratio is 1/3 hence the variable cost must be 2/3 rd of sales. The sales is given as Rs. 9.0 lacs, the variable cost must be Rs. 6.0 lacs. Margin of safety is given as 25% which implies that the break sales is 75% of Rs. 9.0 lacs i.e. Rs. 6.75 lacs. You can very well compute the fixed costs.

Page 342: cost accounting question icwai inter

Working notes :

Rs.Break even sales (75% of Rs. 900,000) 675,000Fixed cost ( 1/3 of Break even sales) 225,000

2. Profit at sales of Rs. 900,000Sales 90,000Less : Variable cost at 2/3 rd of sales 600,000Contribution 300,000Less : Fixed cost 225,000Profit 75,000

4. Computation of Fixed cost in 19x2 Rs.Sales (variable cost / 0.70) (600,000 / 0.70) 857,143Less : Margin of safety at 40% of sales 342,857Break even sales 514,286Less : Variable cost at 70% of break even sales 360,000Contribution at break even sales or Fixed cost 154,286

5. Computation of profit in 19x2

Rs. Sales (variable cost / 0.70) (600,000 / 0.70) 857,143Less : Variable cost 600,000Contribution 257,143Less : Fixed cost 154,286Profit 102,857

Answer (Comparative statement of sales, variable cost, fixed cost and profit19x1 19x2

Rs. Rs.Sales 900,000 857,143Less : Variable cost 600,000 600,000Contribution 300,000 257,143Less : Fixed cost 225,000 154,286Profit 75,000 102,857

Answer (Minimum sales in 19x2 to close down :Fixed cost to be recovered ( 154,286 − 50,000) 104,286Sales required at pv ratio of 30% to cover fixed cost 347,620( 104,286 / 0.30 = 347,620)

2. You must compute the various figures separately for each year and then present them in comparative statement as required.

1. Margin of safety is 25%. The break even sales must be 75% of sales of 900,000. The pv ratio is 1/3. The fixed cost must be 1/3 of break even sales.

3. In the year 19x2, the variable cost remains same as in 19x1. The pv ratio in 19x2 is 30% which means variable costs constitute 70% of sales. Worded differently Sales = Variable cost / 0.70

Recommendation : If the sales goes below Rs. 347,620, the company is advised to close down its operations in 19x2.

Problem 29 (ca final nov 01)

Page 343: cost accounting question icwai inter

The projections for four products for 2002 are :Rs. Crores

A B C DSales 72.0 54.0 84.0 60.0CostsMaterial 48.0 30.0 54.0 36.0Labour 18.0 12.0 30.0 30.0Allocated overheadsManufacturing 6.0 4.8 7.2 4.8Admin. and Selling 2.4 1.2 3.6 2.4Total costs 74.4 48.0 94.8 73.2Profit (loss) (2.4) 6.0 (10.8) (13.2)The projected volume and sub-contracting charges are :

A B C DVolume ('000 nos) 2,000 1,500 3,000 2,000Sub-contracting charges in Rs./unit 80 70 90 130

Assume that the costs are incurred and revenues earned evenly in each of the calendar months.

Solution: Tutorial Notes :

X Ltd. has incurred losses during the past five years. Its projections for the year 2002 are not very encouraging. The management is seriously considering the closure of the only manufacturing unit. However, it is quite open to getting the products on a sub-contracting basis and to continue its administrative and marketing functions. Currently, four products are being manufactured and sold by catering to different markets. The management is also willing to sacrifice any of these products to ensure survival.

Manufacturing, Administrative and Selling overheads consist of staff salaries, rent, essential maintenance and tax payable to the local authorities.

In case the management decides to discontinue the manufacturing operations a minimum notice period of 3 months will be required to be given to the staff as well as to the landlords of manufacturing units and offices. You may assume that both the manufacturing as well as the administrative and selling overheads are fixed in nature, and that in the notice period as mentioned above, these expenses would continue to be incurred.

Assume that labour costs are related to the volume of operations and do not involve any notice period for discontinuance.

Based on the above, you are required to advise the management on the best option out of the options under its consideration viz. :

(i) Issue notices to the staff, the landlords of manufacturing unit and offices on the first day of the year and discontinue all the operations on that very day.

(ii) Issue notices as above on the first day of the year and continue the operations till the end of the notice period (only profitable products need to be continued)

(iii) Issue notices to the staff and the landlord, only in manufacturing unit, resort to sub-contracting and continue the administrative and marketing functions. (sub-contracting is to be done on profitable products only).

1. From the data given with the question, you may find that only B is yielding profit but it is misleading. Profitability is measured by positive contribution and not by profit. A clear look over the data can reveal that A and B are yielding positive contribution and may be considered for continuance if required.

Page 344: cost accounting question icwai inter

7. On the similar line profitability of other products viz B, C and D can be computed.

Solution goes as follows :Option 1: The company discontinues all operations and gives notices to all

Rs. CroresA B C D

Sales per annum 0.0 0.0 0.0 0.0Sales for three months 0.0 0.0 0.0 0.0CostsMaterial 0.0 0.0 0.0 0.0Labour 0.0 0.0 0.0 0.0Allocated overheads for three months Manufacturing 1.5 1.2 1.8 1.2Admin. and Selling 0.6 0.3 0.9 0.6Total costs 2.1 1.5 2.7 1.8Profit (loss) (2.1) (1.5) (2.7) (1.8) (8.1)Conclusion : This option would result in a loss of Rs. 8.1 crores to X Ltd.Option 2: Continue with profitable products only for three Rs. In crores

A B C DSales 72.0 54.0 84.0 60.0

2. You must note that overheads supplied are for the whole year while the period under consideration is 3 months. All the overheads are stated to be evenly occurring throughout the year, hence one-fourth of the overheads (and sales and costs as well) must be considered for computing profitability of the products.

3. This question is very simple, although you may be a bit confused in the beginning. Read it twice and consider the various options one by one. Always keep time factor in mind while solving lengthy questions like this in the examination.

4. On option 1, the discontinuance on the first day of next year. The firm has to pay all overheads during notice period of three months and except that no sales or costs take place. You may compute the total loss for this option very easily. Note that figures given are for the whole year and you should correspondingly reduce them for a period of three months (one-fourth to be exact).

5. As regards Option 2, you can easily identify that the profitable products are A and B only. In this option, the operations are continued for the period of three months for profitable products only. The contribution of A and B can be computed and after charging the overheads, any profit (loss) can be computed. This can be compared with the loss incurred under option 1.

6. As regard Option 3, you must note that sub-contracting is for the labour component only and the material will continue to be incurred by the company. The sub-contracting charges can be computed by multiplying sub-contracting charges / unit and the volume of products. For example, for product A the total sub-contracting charges would be 2,000,000 x Rs. 80 = Rs. 16 crores. This will save Rs. 2.0 crores on labor cost giving a total contribution of Rs. 8.0 crores (72 − 48 − 16 = 8). Sub-contracting is for the year and not for three months. It is already stated in the question that the labour discontinuance does not require any notice period. You can figure out the profitability for the quarter as well.

8. Question nowhere says that the sub-contracting is for the labour component only. Sub-contracting charges for A come to be Rs. 16 crores. The material for product A would be required for Rs.48 crores.. It would be rational to assume that the sub-contracting is for the labour component only.

Page 345: cost accounting question icwai inter

CostsMaterial 48.0 30.0 54.0 36.0Labour 18.0 12.0 30.0 30.0

66.0 42.0 84.0 66.0Contribution per annum 6.0 12.0 0.0 (6.0)Contribution per quarter 1.5 3.0 0.0 (1.5)Products A and B are yielding positive contribution and should be considered for continuance

A B TotalContribution per quarter 1.5 3.0 4.5Less : Allocated overheads for the quarter as calculated earlier 8.1

(3.6)Conclusion : This option would result in total loss of Rs. 3.60 crores to X Ltd.Option 3: Manufacturing unit is closed and sub-contracting for labour componentThe projected volume and sub-contracting charges are :

A B C DVolume ('000 nos) 2,000 1,500 3,000 2,000Sub-contracting charges in Rs./unit 80 70 90 130Total charges in Rs. Crores 16.0 10.5 27.0 26.0Add : Material 48.0 30.0 54.0 36.0Total Variable cost per annum 64.0 40.5 81.0 62.0Sales per annum 72.0 54.0 84.0 60.0Contribution per annum 8.0 13.5 3.0 (2.0)Contribution per quarter 2.0 3.4 0.8 (0.5)Conclusion ; Products A, B and C are yielding positive contributions and qualify for sub-contracting.

Statement of profit (loss) if A, B and C are sub-contractedA B C D Total

Allocated overheadsManufac. overheads for thr 1.5 1.2 1.8 1.2 5.7Admin. and Selling for thre 0.8 0.4 0.9 0.6 2.7

8.4Contribution per quarter 2.0 3.4 0.8 0 6.1Profit (loss) (2.3)

After the expiry of notice period, the manufacturing overheads will not be incurred and company would earn a profit of Rs. 3.4 crores (5.7 − 2.3) from the next quarter onwards.

Decision : Option 3 is most viable as it is going to earn a profit of Rs. 3.4 crores from the next quarter onwards although it will run in loss for the first quarter because of presence of manufacturing overheads.

Problem 30 (ca final nov 01)

P Ltd. manufactures plastic cans of a standard size. The variable cost per can is Rs. 4 and the selling price is Rs. 10 each. The factory of the company has eight machines of identical size. Any individual machine can produce 30 cans per hour. The factory works on 300 days per annum basic and the actual available hour per machine per day is 7.5. The company has an order of 420,000 cans from oil company, to supply. the yearly fixed cost of the company is Rs.20 lacs. P Ltd. has received an order from another firm for supplying 60,000 nos. of plastic moulded toys. The price of the toys is Rs.60 each and the variable cost is Rs. 50 each. While this order would be acceptable for supplying for total quantities only, on acceptance, a special mould costing Rs.2,25,000 would be required to acquired to manufacture toys. The firm study exercise has revealed that 15 nos. of toys can be produced per hour by any of the machines:

Page 346: cost accounting question icwai inter

Advise the company, with reasons in the following conditions:

Solution : Tutorial Notes :

(i) Whether to accept, the order of manufacturing moulded toys, in addition to supplying 4,20,000 nos. of cans or not;

(ii) Whether to accept the order of manufacturing moulded toys, if the order of cans increases to 5,40,000 nos. or not;

(iii) While a sub-contractor is willing to supply the toys, either whole or part of the required quantities at an all inclusive rate of Rs. 57.50 each, what would be the minimum excess capacity needed to justify the manufacturing of any portion of the toys order, instead of sub-contracting:

(iv) The company had an understanding that the orders of the cans will be increased during the year on negotiation, and planned and manufactured 4,50,000 cans during the year. For utilizing the excess capacity, they also accepted the toys order and sub-contracted only 15,000 nos. of toys. At the year's end, however, it was revealed that the order of the cans could be for 4,80,000 nos, if it was properly negotiated. How much loss has been suffered by the company due to improper prediction of demand and negotiation ?

1. You can solve this option wise. Consider option 1 and gather all the facts related to it and decide the feasibility and profitability of the option. Then take other options one by one and do the same.

2. Consider Option (i). Whether the order of manufacture of toy can be accepted in addition to supply 420,000 nos. of cans or not. Computer total machine hours as 8 machines x 7.5 hr per day x 300 days per annum = 18,000 hrs. One machine can produce 30 cans per hour hence production possible is 540,000 cans. The contribution per can is Rs.6, thus production of 4.2 lacs of cans will supply Rs.25.2 lacs of contribution. The production of 4.2 lacs will consume 14,000 hours @ 30 cans per hour leaving 4,000 hours balance for manufacture of toys. In 4,000 hours 60,000 toys can be produced @ 15 toys per hour. Each toy provides contribution as Rs.10 per toy. Thus 60,000 toys will provide Rs. 6.0 lacs contribution. The fixed cost for cans is Rs. 20 lacs and that for toys is Rs. 2.25 lacs. Total fixed costs for cans and toys are Rs. 22.25 lacs and Total contribution is Rs.31.2 lacs (25.2 + 6.0). Total profit for option 1 would be Rs. (31.2 − 22.25) i.e.8.95 lacs. Analysis of option 1 is over.

3. Now consider option 2. Entire machine hours are utilised for production of 540,000 cans giving total contribution of Rs. 32.4 lacs at Rs. 6/can and profit of Rs. 12.4 lacs (32.4 less fixed cost of Rs. 20 lacs). This is more than profit of Rs. 8.95 lacs of option 1. Analysis of option 2 is over.

4. Now consider Option 3. Sub-contracting will provide contribution of Rs. 2.5/ toy (60 − 57.5) while manufacturing will provide a contribution of Rs. 10/ toy. The fixed costs required for the toy is Rs. 225,000. The difference of contribution is Rs. 7.5 per toy. Thus upto 30,000 toys (225,000 / 7.5), sub-contracting is advisable and beyond 30,000 toys manufacturing in-house is advisable. The rate of manufacture of toys is 15 toys per hour, therefore, a minimum of 2,000 machine hours (30,000 / 15) are required to justify manufacturing instead of sub-contracting.

5. Consider condition given in point (iv). The company has produced 450,000 toys consuming 15,000 machine hours out of 18,000 total hours available. The company produced 45,000 toys in balance 3,000 hours @ 15 toys/hour and sub-contracted the balance of 15,000 toys receiving contribution per toy at Rs.2.5/toy. Total contribution is Rs. 31.875 lacs consisting of Rs. 27 lacs from cans + Rs. 4.5 lacs from toys + Rs. 0.375 lacs from subcontracting.

Page 347: cost accounting question icwai inter

The solution goes as follows :

Total machine hours available ; (8 machines x 300 days x 7.5 hr per day) 18,000Hours consumed by 420,000 cans @ 30 cans per hour 14,000Balance hours available for toys 4,000Production of toys in balance hours @ 15 toys / hour 6,000

Rs.lacsContribution from cans @ Rs. 6 per can (420,000 x Rs. 6) 25.20Contribution from toys @ Rs. 10 per toy (60,000 x 10) 6.00Total contribution 31.20Total Fixed cost (Rs. 20.0 lacs for cans + Rs. 2.25 lacs for toys) 22.25Profit 8.95

Option 2: The company accepts 540,000 nos of cansProduction possible in 18,000 hours available @ 30 cans / hr. 540,000

Rs. LacsContribution @ Rs. 6 / can 32.40Less : Fixed cost 20.00Profit 12.40

(iii) Comparison between sub-contracting and manufacturing Rs./unitContribution of manufacture of toys (60 - 50) 10.0Contribution of sub-contracting of toys (60 - 57.5) 2.5Additional contribution of manufacture 7.5Additional fixed cost for manufacture Rs. 225,000Volume needed to cover the fixed cost (225,000 / 7.5) units 30,000Time required @ 15 toys / hr. (30,000 / 15) hours 2,000Conclusion : The company needs 2,000 machine hours in excess to justify sub-contracting. (iv) Computation of cost of prediction error :Total machine hours available ; (8 machines x 300 days x 7.5 hr per day) 18,000Time required @ 30 cans / hour for 450,000 cans 15,000Balance hours available for toys 3,000Production of toys in balance hours @ 15 toys / hour 45,000Total nos of toys to be supplied 60,000Toys to sub-contracted 15,000Profit under existing plan : Rs. LacsContribution from 450,000 of cans @ Rs. 6/can 27.00Contribution from 45,000 toys @ Rs. 10 / toy 4.50Total contribution 31.50

6. It was revealed that order could have been for 480,000 cans. In that case, production plan would have been as follows 480,000 cans consuming 16,000 hours + 30,000 toys from balance 2,000 hrs + 30,000 toys from sub-contracting. Total contribution in Rs.28.80 lacs from cans + Rs. 3.0 lacs from toys + Rs. 0.75 lac from subcontracting Rs. 32.55 lacs.

7. The company has lost a contribution of Rs. (32.55 less 31.875) 67,500 due to improper prediction.

Option 1: The company accepts the order of moulded toys along with 420,000 nos of cans

Conclusion : It is advisable that the company should utilize the additional capacity of 4,000 hours for manufacture of toys because this will yield additional profit of Rs. 3.75 lacs (6.00 - 2.25) for the company.

Conclusion : Option 2 is better than Option 1 because it yields additional profit of Rs. 3.45 lacs (12.40 - 8.95) over option 1, for the company.

Page 348: cost accounting question icwai inter

Total Fixed cost (Rs. 20.0 lacs for cans + Rs. 2.25 lacs for toys) 22.25Profit 9.25Profit from sub-contracting ( 15,000 x Rs. 2.50) 0.375Total profit 9.625Profit under properly negotiated plan :Total machine hours available 18,000Time required @ 30 cans / hour for 480,000 cans 16,000Balance hours available for toys 2,000Production of toys in balance hours @ 15 toys / hour 30,000Total nos of toys to be supplied 60,000Toys to sub-contracted 30,000Profit under existing plan : Rs. LacsContribution from 480,000 of cans @ Rs. 6/can 28.80Contribution from 30,000 toys @ Rs. 10 / toy 3.00Total contribution 31.80Total Fixed cost (Rs. 20.0 lacs for cans + Rs. 2.25 lacs for toys) 22.25Profit 9.55Profit from sub-contracting ( 30,000 x Rs. 2.50) 0.75Total profit 10.30Loss of profit due to improper prediction (10.30 − 9.625) 0.675

At Sambalpur At BilaspurAnnual output M.T. 12,000 15,000Capacity Utilisation (%) 80 60Other variables (Rs. Lacs) 156 192Fixed cost (Rs. Lacs) 108 120You are required to determine:(i) The cost break-up of each unit per M.T. of output;

(iii) Cost savings, if any, as per the revised schedule of production.Solution Tutorial Notes :

Problem 31 (ca final nov 01)

A company has two plants − one at Sambalpur and the other at Bilaspur, where production of goods takes place.

The basic raw material requirement is 80% of the finished product, by weight. Such materials are available locally, but are limited to 6,000 MT at Rs. 1,800 M.T. at Sambalpur and 16,000 M.T. at Rs.2,000 at Bilaspur. Any extra requirements, will have to be procured from Jamshedpur at Rs. 2,500 per M.T. Other details are as under :

(ii) The quantity of production at each unit from the availability of local supplies of basic raw materials only, by keeping the same total production of the company, as a whole

1.Cost break up is required to determine the total cost with cost of every component. The cost relating to other variables and fixed cost are already supplied with the question. You are to determine the cost of raw materials for both the units. Think how can you determine the cost of raw materials for the annual output given.

2. Let us take Sambalpur first. The annual output is given as 12,000 tonnes. The raw materials must be 80% or 9,600 tonnes. The raw materials available locally is 6,000 tonnes therefore the balance raw materials of 3,600 will have to be procured from Jamshedpur at Rs.2,500 / tonne. Cost break up at Sambalpur is over. On similar lines you can make computations for Bilaspur.

Page 349: cost accounting question icwai inter

(i) Statement of Cost Break - up :At Sambalpur At Bilaspur

Annual output M.T. 12,000 15,000Raw materials required in tonnes @ 80% 9,600 12,000Raw materials available locally 6,000 16,000Raw materials to be procured from Jamshedput 3,600 0Cost of raw materials in Rs. Lacs Rs. Lacs Rs. LacsAt Sambalpur 6,000 x Rs. 1,800 108.0

3,600 x Rs. 2,500 90.0At Bilaspur 12,000 x Rs. 2,000 240.0Cost Break up : Total cost and Cost per tonne

Annual output at Sambalpur : 12,000 tonnes and at Bilaspur : 15,000 tonnes

At Sambalpur At BilaspurTotal Per tonne Total Per tonne

Rs.lacs Rs. Rs. lacs Rs.Raw materials cost 198.0 1,650 240.0 1,600Other variables 156.0 1,300 192.0 1,280Fixed cost 108.0 900 120.0 800

462.0 3,850 552.0 3,680(ii) Computation of total cost of output from local supplies only

At Spur At BpurRaw materials available in tonnes 6,000 16,000Output possible with raw materials 7,500 20,000Cost of raw materials in Rs. Lacs 108 320Cost of raw materials per tonne of output 1,440 1,600Other variable at Rs./ tonne 1,300 1,280Total cost 2,740 2,880Ranking I II

(iii) Cost Structure with revised production plan :At Sambalpur Rs. LacsRaw materials cost in Rs. Lacs 108.00Variable cost [(156 /12,000) x 7,500] 97.50Total Variable cost 205.50Add : Fixed cost 108.00Total Cost 313.50At BilaspurRaw materials required in tonnes for 19,500 tonnes of output 15,600Raw materials cost in Rs. L (15,600 x 2,000) 312.00Variable cost [(192 /15,000) x 19,500] 249.60Total Variable cost 561.60Add : Fixed cost 120.00Total Cost 681.60

At Spur At Bpur TotalRs. Lacs Rs. Lacs

Present Cost 462.00 552.00 1,014.00

3. The raw materials available at Spur is 6,000 tonnes which can produce 7,500 tonnes of output. The variable cost per tonne is Rs. 1,300 / tonne (156 lacs/12,000).

Thus production plan should to produce 7,500 tonnes at Spur and balance of 19,500 tonnes from Bpur for keeping the total output same as at present (present output 12,000 + 15,000).

Page 350: cost accounting question icwai inter

Cost as per revised schedule 313.50 681.60 995.10Saving 148.50 (129.60) 18.90

Rs. Rs.

Selling price per unit 900

Costs per unit

Direct materials 200

Variable machine operating costs (Rs. 100/ hr) 150

Manufacturing overheads costs 180

Marketing and Administrative costs 200 730

Operating Income per unit of 'XY 100' 170

XYZ limited can sell additional 3,000 units of 'XY 100', if it can outsource those additional units.

Rs. Rs.Selling price per unit 600Costs per unitDirect materials 200Variable machine operating costs (Rs. 100/ hr) 50Manufacturing overheads costs 60Marketing and Administrative costs 110 420Operating Income per unit of 'XY 100' 180Other information pertaining to the operation of XYZ limited is as follows:

(b) Variable marketing and administrative costs per unit for various products are as follows:Manufactured XY 100' Rs. 80Purchased XY 100' Rs. 40Manufactured XY 200 Rs. 60

Show your calculations.Solution :Solve yourself. The key answers are supplied hereunder:

XY 100 XY 200Fixed manufacturing overhead per unit Rs. 60 Rs. 25

Problem 32 (ca final may 02)

XYZ Limited is currently manufacturing 5,000 units of product 'XY 100' annually, making full use of its machine capacity. The selling price and total costs per unit associated with 'XY 100' are as follows:

ABC limited, a supplier of quality products, has agreed to supply upto 6,000 units of 'XY 100' per year at a price of Rs. 650 per unit delivered at XYZ's factory.

XYZ limited can use its facility to produce an alternative product 'XY 200'. It can sell upto 12,000 units of 'XY 200' annually. Estimated selling price and total costs per unit to manufacture ans sell 12,000 units of 'XY 200' are as follows

(a) XYZ limited use machine hours as the basis for assigning fixed manufacturing overhead. The fixed manufacturing overhead for the current year is Rs. 3,00,000. These costs will not be affected by product-mix decision.

Fixed marketing and administrative costs for the current year is Rs. 6,00,000. These costs will not be affected by product-mix decision.

Calculate the quantity of each product that XYZ Limited should manufacture and / or purchase to maximize the operating income.

Page 351: cost accounting question icwai inter

Variable manufacturing overhead per un Rs. 120 Rs. 35Variable cost of production per unit Rs. 470 Rs. 285Total machine hours available 7,500 hoursContribution per unit Rs. 350 Rs. 255Contribution per hour Rs. 233 Rs. 610Manufacture 1,000 units 12,000 unitsPurchase 6,000 units

Rs.Selling price 130.0Direct materials 30.0Component 'EH' 9.4Direct wages @ Rs. 7 / hr 28.0Factory overhead (50% fixed) 24.0Selling and distribution overheads (75% variable) 16.0Administrative overheads (fixed) 5.0The factory overheads are applied on the basis of labour hours.

Rs. Direct materials 30,000Direct labour 52,500Variable overheads 25,500Fixed overheads 33,000Total 141,000

The component 'EH' However, is available for purchase at the market at Rs. 7.90 per unit.

(a) Rent out the released capacity at Re.1 per hour;

Rs.Direct materials 42,000Direct labour 31,500Factory variable overheads 13,500Other variable overheads 25,500

Problem 33 (ca final may 04)

Panchwati Cement Ltd. produces '43 grade' cement for which the company has an assured market. The output for 2004 has been budgeted at 180,000 units at 90% capacity utilisation. The cost sheet based on output (per unit) is as follows:

To utilize the idle capacity and improve the profitability of the company, the following proposals were put up before the Board of Directors for consideration.

(i) An order has been received from abroad for 500 units of product '53 Grade' cement per month at Rs. 175 per unit. The cost data are :

Direct materials : Rs. 56 per unit, direct labour 10 hours per unit, selling and distribution overhead applicable to this product order is Rs. 14 per unit and variable factory overheads are chargeable on the basis of direct labour hours.

(ii) The company at present manufactures component 'EH', one unit of which is required for each unit of product '43 Grade'. The cost details of 15,000 units of component 'EH' are as follows:

(iii) In the event of company deciding to purchase the component 'EH' from market, the company has two alternatives for the use of capacity so released, which are as under:

(b) Manufacture component 'GYP' which can be sold at Rs. 8 per unit. The cost data of this component for 15,000 units are:

Page 352: cost accounting question icwai inter

Total 112,500Required :(i) Prepare a statement showing profitability of the company envisaged in the budget.(ii) Evaluate the export order and state whether it is acceptable or not.

Solution : Tutorial Notes : You can do some jottings along the reading of the question as follows:1. Direct labour : Rs. 28 for 7 hours hence Direct labour per hour ; Rs. 4.02. Variable factory overheads : Rs. 12 for 4 hours. Rate of variable factory overheads : Rs. 3/hr3. Time in hours for 180,000 units at 4 hrs / unit : 720,000 hrs.

Working notes :1. Variable cost of Component EH for 15,000 units

Rs. Rs./unitDirect materials 30,000 2.00Direct labour 52,500 3.50Variable overheads 25,500 1.70

7.20Market price 7.90

Profitability Statement of Panchwati Limited

Rs./unit Rs. LacsSelling price 130.0Direct materials 30.00Component 'EH' 7.20 Direct wages @ Rs. 7/hr 28.00Variable Factory overhead (50% x Rs. 24) 12.00Variable Selling and distribution overheads (75% x 12.00 89.20

Contribution 40.80Total contribution for 180,000 units in Rs. Lacs 73.44

Less : Fixed costsFactory Overheads (Rs. 12 x 180,000) 21.60Selling and distribution Overheads (Rs. 4 x 180,000) 7.20Administrative overheads (Rs. 5 x 180,000) 9.00Component EH ( Rs. 2.2 x 180,000) 3.96 41.76

(iii) Make an appraisal of proposal to manufacture component 'EH' and state whether the component 'EH' should be manufactured in the factory or purchased from the market. Assume that no alternative use of spare capacity is available.

(iv) Evaluate the alternative use of the spare capacity and state whether to manufacture or the component 'EH' and if your decision is to buy the component 'EH', which of the two alternative use of spare capacity will you prefer?

4. The labour cost per unit of EH is Rs. 3.50 (52,500 / 15,000). Labour rate is Rs. 7 per hour. Thus time required for EH per unit is 0.50 hrs. One unit of EH is used in one unit of '43 Grade'. Thus time required for 180,000 units of EH would be 90,000 hrs.

5. Time required for working at 90% capacity is 810,000 ( 720,000 + 90,000). Total labour hours available at 100% capacity is 900,000 hrs.

6. Export order would require 60,000 hrs ( 500 units/ month x 12 months x 10 hr/unit). The balance hours available at 100% capacity are 90,000 hours. Thus export order can be accommodated as far as time available is concerned.

It is advisable to make the component EH. It will save Rs. 126,000 per annum (Rs. 0.70 x 180,000) to the company.

Page 353: cost accounting question icwai inter

Profit 31.68Appraisal of Export Order

Rs./unitDirect materials 56Direct labour (10 hrs x Rs. 7/hr) 70Variable factory overhead (Rs. 3 x 10) 30Selling and distribution overheads 14Total variable cost 170Selling price offered 175Contribution per unit 5Contribution per annum (500 x 12 x Rs. 5)) Rs. 30,000

Appraisal of GYP:Rs. Rs./unit

Direct materials 42,000 2.80Direct labour 31,500 2.10Factory variable overheads 13,500 0.90Other variable overheads 25,500 1.70Total 112,500 7.50Selling price 8.00Contribution per unit 0.50Time required per unit @ Rs. 7/ ( 2.1 / 7.0) 0.30 hrs.

The working result of a company for two corresponding years are shown below : Year 1 Year 2

Rs. In lacs Rs. In lacsSales 1,200 1,540Direct materials 600 648Direct wages and variable overhead 360 412Fixed Overheads 160 300

1,120 1,360Profit 80 180

Export order requires 60,000 hrs of labour. The capacity is available to the extent of 90,000 hrs hence the order can be easily accommodated.

In view of the contribution of Rs. 30,000 per annum and utilisation of the spare capacity available, the export order is recommended for acceptance.

Production of EH requires 90,000 hours. If purchased from market the company will have to pay Rs. 126,000 more. If the capacity of 90,000 so released is rented out at Re.1/hr fetching Rs. 90,000 as revenue. Renting out will result in a loss of Rs. 36,000 hence not advisable. Rent above the rate of Rs. 1.40/ hr (126,000 / 90,000) should be acceptable.

The contribution per unit of GYP is Rs. 0.50. As each unit requires 0.30 hr, the contribution per hour for GYP would be Rs. 0.50/0.30 i.e. Rs. 1.67/ hr. For 90,000 hrs the total contribution will be Rs. 150,000.

Conclusion : As the contribution from GYP Rs. 150,000 is greater than the loss in purchasing EH from the market of Rs. 126,000, Company is advised to manufacture GYP and purchase EH from the market.

Problem 34 (ca final may 2000)

In year 2, there has been an increase in the selling price by 10%. Following are the details of material consumption and utilisation of direct labour hours during the two years:

Page 354: cost accounting question icwai inter

Year1 Year 2Direct materials consumption in m/ t 500,000 540,000Direct labour hours 75,00,000 80,00,000

(ii) Find out the break even point for both years.

Solution : Tutorial Notes :

Year 1 Year 2 Year 3Rs. Lacs Rs. Lacs Rs. Lacs

Base Budgeted ActualSales 1200 1,400 1,540Direct materials 600 700 648Direct wages and variable 360 420 412Total variable cost 960 1,120 1,060Contribution 240 280 480Fixed Overheads 160 160 300Profit 80 120 180Pv ratio (contribution / sales 20% 20% 31%Break even sales (fixed cost 800 800 962.5Break even sales in % 57% 63%Various variances can be computed :Material varianc (420 − 412) 8 favLabour variance(700 − 648) 52 favContribution var (480 − 280) 200 favFixed cost varia (300 − 160) 140 adv.Net variance 60 Fav.If margin of safety is 45%, the break even sales must be 55% of actual sales.Present margin of safety is 37% and the break even sales is 63% and present contribution is 31%.

Rs. LacsRevised contribution required : 1540 x (31/55) x 63% = Rs. 545.5 la 545.5Present contribution is 480.0Increase in sales required 65.5% increase in sales required ( 65.5/1540) 4.3%

(a) A Ltd. Makes and sells a single product. The company’s trading results for the year are:For the year 2007 Rs. 000

Sales 3,000Direct materials 900

(i) Keeping year 1 as base year, analyze the results of year 2 and work out the amount which each factor has contributed to change in profit.

(iii) Calculate the percentage increase in selling price that would be needed over the sale value of year 2 to earn a margin of safety of 45%.

1. Year 1 is taken as base year. The figures of year 2 must be altered in accordance with the base year. There has been increase of 10% in the selling price in year 2. The sales must be Rs. 1,400 lacs without increase (1540 / 1.10). The material is 50% of sales in year 1 so material in year 2 must be Rs. 700 lacs (50% of Rs. 1400 lacs).

2. The direct wages and variable overhead is 30% of sales in year 1, thus direct wages and variable overhead in year 2 must be 30% of Rs. 1400 lacs i.e. Rs. 420 lacs.

3. Compare the figures in three columns. First base year, second figures of year 2 with respect to year 1 as base and third the actual figures.

Problem 35 (ca final may 08)

Page 355: cost accounting question icwai inter

Direct labour 600Overheads 900 2,400

Profit 600For the year 2008, the following are expected:(i) Reduction in the selling price by 10%. (ii) Increase in the quantity sold by 50%.(iii) Inflation of direct material cost by 8%. (iv) Price inflation in variable overhead by 6%.(v) Reduction of fixed overhead expenses by 25%.It is also known that :(a) In 2006, overhead expenditure totaled to Rs. 8,00,000.(b) Total overhead cost inflation for 2007 has been 5% more than 2006.(c) Production and sales volumes have been 25% higher in 2007 than in 2006.The high-low method is being used by the company to estimate overhead expenditure.You are required to: (i) Prepare a statement showing the estimated trading results for 2008.(ii) Calculate the Break-even point for 2007 and 2008.(iii) Comment on the BEP and profits of the years 2007 and 2008.Solution :

2007 2008Sales in year 3,000 4,050Direct Material 900 1,458Direct Labour 600 900Variable overheads 300 477Total Variable cost 1,800 2,835Contribution 1,200 1,215Less : Fixed Overheads 600 450Profit 600 765Pv ratio ( contribution / sales) 40% 30%Break even sales 1,500 1,500

Increase in price = 5%Overhead cost for same production 800 x 5% + 800 = 840.Overhead increase due to quantity = 900 – 840 = Rs. 60

Both Fixed Cost and P/V ratio have declined by 25% equally. So BEP sales remain the same.The contribution is only Rs. 1,215 in 2008 though quantity is increased by 50%.This is due to increase in production cost and decrease in selling price. This ismore than made up by decrease in fixed cost so that overall profit has increased by 27.5%.

Giving up does not always mean that you are a looser, sometimes your are strong enough to let it go.

Sales value Rs. 760,000 Fixed overheads Rs. 190,000Direct material Rs. 140,000 Admin. and Selling Overheads Rs. 120,000Direct labour Rs. 190,000

(Note 1) Sales in 2008 : 3,000 x 1.5 x 0.9(Note 2) Overhead Cost in 2006 = Rs. 800 lacs.

Rs. 60 represents increase in variable Overhead in 2007 due to increase in quantity by 25%. Hence variable overhead in 2007 will be 5 times Rs. 60 = Rs. 300 lacs. Total overhead for 2007 is Rs. 900 lacs. This gives the fixed overheads for 2007 as Rs. 600 lacs.

Problem 36 (CWA Inter Dec.08)

A company produces 30,000 units of product A and 20,000 units of product B per annum. The sales value and costs of the two products are as follows:

50% of factory overheads are variable and 50% of administrative and selling overheads are fixed. The selling price of A is Rs. 12 per unit and Rs. 20 per unit for B.

Page 356: cost accounting question icwai inter

Assuming 60% capacity is used for manufacture of A and B, Calculate:(i) Present cost and profit (ii) Cost and profit after diversification;(iii) Give your recommendations as to whether to diversify or not.Solution : Tutorial notes :1. Selling price is 400% of direct labour. Thus direct labour would be 25% of selling price.1. Statement showing present cost and profitParticulars A B TotalProduction and sales units 30,000 20,000 50,000Selling price Rs./unit 12 20Sales value Rs. 360,000 400,000 760,000Variable costs Rs. Rs. Rs.Direct Labour (25% of sales) 90,000 100,000 190,000Direct material : Direct labour 2:3 4:5Direct material 60,000 80,000 140,000Factory overheads 45,000 50,000 95000Administrative and selling overheads 30,000 30,000 60,000Total variable cost 225,000 260,000 485,000Contribution 135,000 140,000 275,000Less : Fixed cost 155,000Profit 120,0002. Statement showing cost and profit after diversificationParticulars Units A B C TotalCapacity % 60% 60% 40%Production and sales Units 18,000 12,000 30,000 60,000Selling price Rs./unit 12 20 14Sales value Rs. 216,000 240,000 420,000 876,000Variable costs Rs. Rs. Rs. Rs.Direct material 36,000 48,000 75,000 159,000Direct Labour 54,000 60,000 90,000 204,000Factory overheads 27,000 30,000 45,000 102,000Administrative and selling overheads 18,000 18,000 30,000 66,000Total variable cost 135,000 156,000 240,000 531,000Contribution 81,000 84,000 180,000 345,000Less : Fixed cost 155,000Profit 190,0003. Recommendation:

The direct material and labour ratio for product A is 2:3 and for B 4:5. For both the products, the selling price is 400% of direct labour. The factory overheads are charged in the ratio of direct labour and administrative and selling overheads are recovered at a flat rate of Rs. 2 per unit for A and Rs. 3 per unit for B.

Due to fall in demand of the above products, the company has a plan to diversify and make product C using 40% capacity. It has been estimated that for C direct material and direct labour will be Rs. 2.50 and Rs. 3 per unit respectively. Other variable costs will be same applicable to the product A. The selling price of product C is Rs. 14 per unit and production will be 30,000 units.

The company should go ahead with proposed diversification as it enhances the profit from Rs. 120,000 to Rs. 190,000.

·       Start the day right. Read a good book on positive mental attitude before you read anything.

Page 357: cost accounting question icwai inter

Page 358: cost accounting question icwai inter

remains the same.

Page 359: cost accounting question icwai inter

Chapter 11 : Budgets and Budgetary Controlscwa inter cwa final ca inter ca final

18 3 1 0

The annual flexible budget of TBA Ltd. is as followsProduction Capacity 40% 60% 80% 100%Costs Rs. Rs. Rs. Rs.Direct wages 20,000 30,000 40,000 50,000Direct material 16,000 24,000 32,000 40,000Production overheads (fixed and variable) 11,400 12,600 13,800 15,000Admin. Overheads (fixed and variable) 5,800 6,200 6,600 7,000Selling and Distr.overheads (fixed and variable) 6,200 6,800 7,400 8,000Total 59,400 79,600 99,800 120,000

1.The present fixed cost will reduce by 60%;2.There will be a cost of Rs. 10,000 towards closing down operations;3.To maintain a skeleton maintenance service for which Rs.24,000 to be incurred;4. An initial cost of re-opening of Rs. 20,000to be incurred.The other option is to keep the factory operational for one year and wait for better time next year.You are required to work out the profitability under the two options and give your comment.Solution: Tutorial Notes:

Working notes: 40% 60% Difference Diff. for 401.Separation of variable and fixed components 1 2 3 4Production overheads (fixed and variable) 11,400 12,600 1,200 2,400Admin. Overheads (fixed and variable) 5,800 6,200 400 800Selling and Distr.overheads (fixed and variable) 6,200 6,800 600 1,200Total fixed costLess 60% if closing down is optedFixed cost for closure for one yearSolution:The costs associated with closing down operation are enumerated hereunder:

Rs.

mEehn dk fu;e Law of Expectation ;g dgrk gS fd ftl pht ;k ckr dh vki vkRefo'okl ds lkFk mEehn djrs gSa oks vkids thou esa lkdkj gks tkrh gSA vxj vki dke;kc] lsgrean vkSj le`) gksus dh mEehn djrs gSa rks vki osls gks gh tk;saxsA

Problem:1 cwa Inter.I June 05

The company is presently passing through a period of very lean market demand and operating at 50% capacity and have also selling its product at a discounted price generating a total sales revenue of Rs. 60.000 at that level.

It is expected that the market scenario will improve in the next year and, on a conservative estimate, the company is likely to operate at 70% capacity level with increased sales revenue of Rs. 120,000.

As an option, the management is considering to close down the operation for one year and restart operation after one year when the market conditions are likely to improve. If closed down for the year it estimated that:

1. Whenever total cost containing fixed and variable portion is given, it should be broken down to fixed and variable components by way of working notes.

2.There are two options which are to be considered and compared viz. to continue the current year operations at 50% level or to close down the operations for one year.

3. The question is silent about the profitability of next year operations estimated at 70% capacity, however, the students are advised to furnish the same.

Page 360: cost accounting question icwai inter

1.Closing down cost 10,0002. Fixed cost is 40% of current 7,6003.Maintenance cost 24,0004.Re-opening cost 20,0005.Total Cost 61,600Comparative Statement of profitability at 50% and 70% capacity levels

40% 50% 70%Direct wages 20,000 25,000 35,000Direct material 16,000 20,000 28,000Production overheads 2400 3,000 4,200Administrative overheads 800 1,000 1,400Selling and distribution overheads 1200 1,500 2,100Total variable cost 40,400 50,500 70,700Add : Fixed Cost 19,000 19,000

69,500 89,700Revenue (as given) 60,000 120,000Profit (loss) (9500) 30,300Comments:

Problem : 2 (cwa inter I dec.03)

Particulars Cost per unit in Rs.Direct material 12Direct labour 8Direct expenses 5Production overhead 10 (40% variable)Administrative overheads 5 (100% fixed)Selling and distribution overheads 6 (50% variable)

Solution:First of all fixed cost should be calculated:Production overhead is given as 60% fixed of unit price Rs. 10/ at 90% capacityProduction overhead = 6 x 90,000 = 540,000 Rs. Administrative overhead = 5 x 90,000 = Rs. 450,000Selling and distribution overhead = 3 x 90,000 = Rs. 270,000

Priyanka EnterprisesCapacity 60% 70% 80%Direct costs: Cost/unit in Total cost Total cost Total cost

Rs. Rs. lakhs Rs. lakhs Rs. lakhsDirect material 12.0 7.2 8.4 9.6Direct labour 8.0 4.8 5.6 6.4Direct expense 5.0 3.0 3.5 4.0Variable overhead:Production overhead 4.0 2.4 2.8 3.2Selling and distribution 3.0 1.8 2.1 2.4

If current year operations are continued, this will lead to a loss of Rs. 9500 as indicated above, this however is better than closing down which will lead to a loss of Rs. 61,600.

it is recommended that the company should continue to operate at current level and wait for the scenario change likely to take place in next year, which is estimated to fetch a profit of Rs. 30,300 at 70%capacity level.

A newly established manufacturing company has an installed capacity to produce 100,000 units of a consumer product annually. However, its practical capacity is only 90%. The annual capacity utilisation may be substantially lower, as the firm is new to the market and the demand is uncertain. The following budget has been prepared for 90% utilisation.

You are required to prepare budgets at 60%, 70% and 80% levels of capacity utilisation giving the unit variable cost, the fixed cost and the total costs under various heads at all the above levels.

Page 361: cost accounting question icwai inter

Total variable cost 32.0 19.2 22.4 25.6Fixed Overhead :Production overhead 5.4 5.4 5.4Administrative overhead 4.5 4.5 4.5Selling and distribution overhead 2.7 2.7 2.7Total fixed cost 12.6 12.6 12.6Problem : 3 (cwa inter.I june.02)

X YStandard cost per unit Rs. Rs.Materials ( 1 kg ) 12 (1.5 kg ) 18Labour 4 6Overhead (100% of labour) 4 6

Total cost 20 30Inventory of finished goods: Valued at standard costValue as on 1.07.02 Rs. 280,000 720,000

Rs. 200,000 360,000Standard profit margin (% of sales) 20% 25%Loss in processing (% of input) 8% 10%

You are required to prepare :-(a) Production budget for 2002--03.(b) Material consumption budget for 2002--03 ( in terms of units and value)(c) Wages budget for 2002--03(b) Material purchase budget for 2002--03 ( in terms of units and value)Solution:Tutorial Notes:Students should try to solve this problem mentally. The mental process can be described as follows:

For the production of 100,000 units, the material required would be Rs. 12.00 lakh (@ Rs. 12 per unit).The labour rate is Rs. 4 per unit thus labour budget would be Rs. 4 lakh.The solution goes as follows :Working notes:1. Calculation of estimation of units to be produced: Product X Product YMonthly sales Rs.lakh 2 4Estimated annual sales Rs. Lakh 24 48Standard cost per unit Rs. 20 30Profit margin on selling price 20% 25%Profit margin on cost 25% 33.33%Standard selling price Rs. 25 40Estimated annual sales in units (Sales / Selling price) 96,000 120,000

A company manufactures and sells two products X and Y using the same raw materials and the same type of labour. It is trying to prepare the annual budget for 2002--03. The marketing department has estimated the average monthly sales of X Rs. 2 lakh and that of Y as Rs. 4 lakh. Further details relating to the two products are given below:

Currently the raw material stock is equal to requirements for 1.5 month average production. However, the closing stock will be reduced to one month requirement.

Let us take product X first. The annual sales is Rs. 24 lakhs (monthly sale Rs. 2 lakh given) which has 20% of profit element. This gives cost of sales as Rs. 19.20 lakh. The standard cost per unit is Rs. 20, thus the no. of units to be sold is (19.20 / 20) = 96,000. The opening stock is (280,000 / 20 ) = 14,000 units and the closing stock is (200,000 / 20) = 10,000 units. Thus the units to be produced in the year must be (96,000 + 10,000 -- 14,000) = 92,000 units.

The process loss is given as 8%. It implies that for producing 92 units 100 units are to be introduced. Thus for producing 92,000 units of X, 100,000 units are to be budgeted and produced.

Page 362: cost accounting question icwai inter

2. Closing stock in Rs. 200,000 360,000Standard cost 20 30Closing stock in units 10,000 12,000

3. Opening stock in Rs. 280,000 720,000In units 14,000 36,000

………………CompanyProduction budget for the year 2000--03

Particulars Products TotalX (units) Y (units)

Estimated / Budgeted Sales (see w.n 1) 96,000 120,000 216,000Add: Closing stock ( w.n. 2) 10,000 12,000 22,000Less : Opening stock (w.n. 3) 14,000 24,000 38,000Units to be produced 92,000 108,000 200,000Loss in production 8% 12%Budgeted production 100,000 120,000 220,0002. Material consumption budget for the year 2002--03

ProductsParticulars X (units) Y (units)Production 100,000 120,000Requirement per unit in kg. 1 1.5Total requirement 100,000 kg. 180,000 kg.Total cost of material @ Rs. 12 per kg. 12 Rs. lakh 21.6 Rs. lakh3. Wages budget for the year 2002--03

ProductsParticulars X (units) Y (units)Production 100,000 120,000Wages per unit (Rs.) 4 6Total wages Rs. 400,000 720,0004. Material purchase budget for the year 2002--03

ProductsParticulars X (units) Y (units) TotalProduction 100,000 120,000Requirement of raw material in kg/unit 1 1.5Total requirement in kg 100,000 180,000 280,000Add: Closing stock as one month requirement ( 280,000 / 12) 23,333Less : Opening stock as 1.5 month requirement (280,000 x 1.5) /12 35,000Quantity of material to be purchased 268,333Total value @ Rs. 12/kg in lakhs 32.20Problem 4 (cwa inter I dec.01)

Variable overhead At 80% capacity levelRs.

Indirect material 144,000Indirect labour 64,000

Semi-variable overhead:Power (70% variable) 200,000Repairs and maintenance (60% fixed) 50,000

Fixed Overhead :Depreciation 36,000Insurance 16,000Others 108,000

Estimated direct labour hours 240,000Solution: This is a very easy question.

Work out a flexible budget for overhead expenses on the basis of following data available from the cash records and determine the overhead rates at 70%, 80% and 90% plant capacity levels:

Page 363: cost accounting question icwai inter

Separation of variable and fixed component of semi-variable costsSemi-variable overhead: 80% Fixed Variable

Power (70% variable) Rs. 200,000 60,000 140,000Repairs and maintenance (60% fixed) Rs. 50,000 30,000 20,000

Flexible budget for the period……………Particulars 70% 80% 90%1. Variable Overheads Rs. Rs. Rs.Indirect material 126,000 144,000 162,000Indirect labour 56,000 64,000 72,0002. Semi variable overheadsPower variable 122,500 140,000 157,500

fixed 60,000 60,000 60,000Repairs and maintenance variable 17,500 20,000 22,500

fixed 30,000 30,000 30,0003. Fixed OverheadsDepreciation 36,000 36,000 36,000Insurance 16,000 16,000 16,000Others 108,000 108,000 108,000Total Budgeted overheads 572,000 618,000 664,000Total Direct labour hours 210,000 240,000 270,000Overhead rate Rs./hour 2.72 2.58 2.46Problem 5 (cwa inter I June.00, similar cwa inter Ii dec.05)Galaxy Ltd. Is engaged in production of certain TV parts,100% capacity being 10,000 units.Given below are the information for January and February 1999.Months January FebruaryParts produced in units 6,000 9,000Elements of overhead costs Rs. Rs.Salaries 3,000 3,000Power 3,000 3,900Consumable stores 3,000 4,500Repairs 4,000 4,600Shop labour 1,500 2,250Depreciation 2,500 2,500Inspection 1,000 1,300

(b) Show the overhead absorption rate per unit at 100% capacity.Solution:First of all, semi variable costs are to be separated in fixed and variable segments:Semi-variable costs are shown below Variable costsParticulars 60% 90% Diff. Fixed 50% 80% 100%Production units 6,000 9,000 5,000 8,000 10,000Power 3,000 3,900 900 1,200 1,500 2,400 3,000Repairs 4,000 4,600 600 2,800 1,000 1,600 2,000Inspection 1,000 1,300 300 400 500 800 1,000

Production capacity in % 50% 80% 100%Production in units 5,000 8,000 10,000Production rate/hour in units 10 10 10Production hours 500 800 1,000Variable cost Rs. Rs. Rs.Direct material @ Rs. 2/unit 10,000 16,000 20,000

Rate of production per hour is 10 units. Direct material cost is Rs. 2 per unit and direct labour cost is Rs. 8 per hour. You are required to compute :

(a) Cost of production at 50%, 80% and 100% capacity respectively showing separately the fixed, semi variable and variable expenses in the flexible budget and

Page 364: cost accounting question icwai inter

Direct labour @ Rs. 8/hr. 4,000 6,400 8,000Prime costs 14,000 22,400 28,000Factory overhead variablePower 1,500 2,400 3,000Repairs 1,000 1,600 2,000Inspection 500 800 1,000Consumable stores @ Rs.500/1000 units 2,500 4,000 5,000Shop labour @ Rs. 250/1000 units 1,250 2,000 2,500Total Factory overhead variable 6,750 10,800 13,500Factory overhead fixedSalaries 3,000 3,000 3,000Depreciation 2,500 2,500 2,500Power 1,200 1,200 1,200Repairs 2,800 2,800 2,800Inspection 400 400 400Total Factory overhead fixed 9,900 9,900 9,900Total Factory overhead (fixed + variable) 16,650 20,700 23,400Add: Prime cost 14,000 22,400 28,000Total cost of production 30,650 43,100 51,400(b) Overhead absorption rate per unit at 100% capacity ( 23,400 / 10,000) Rs. 2.34Problem 6 (cwa inter I June.06)BMS Ltd. has prepared annual budget for the year ending 31.03.07 on the basis of 60% capacity:Particulars Amount (Rs. In lakhs)

Variable costs Semi - Variable ExpensesSales 150.00 Direct material 36.50 Repairs & Maintenance 5.30

Direct Labour 22.82 Indirect labour 7.70Direct Expenses 8.68 Supervision 6.00

Heating & lighting 3.00Fixed ExpensesSalaries : Managerial 9.50Rent, Rates and Taxes 6.60Depreciation 7.40Audit fees 6.50Total Cost of sales 120.00Budgeted profit 30.00Construct a flexible budget for 50%, 75% and 90% capacity utilisation, showing

(i) Fixed expenses remain constant at all levels

Solution: BMC Ltd.Flexible budget for the year ending 31.03.07 in Rs. Lakhs

Variable costActivity levels 60% 50% 75% 90%Direct material 36.5 30.42 45.63 54.75Direct labour 22.82 19.02 28.53 34.23Direct Expenses 8.68 7.23 10.85 13.02Direct cost 68.00 56.67 85.00 102.00Semi-variable costRepairs & Maintenance. 5.30 5.30 5.83 6.36Indirect labour 7.70 7.70 8.47 9.24Supervision 6.00 6.00 6.60 7.20

(a) Variable and semi-variable cost (business) Cost of sales and (c) profit - with the help of following assumptions:

(ii) Semi-variable expenses remain constant between 45% and 64% capacity, increasing by 10% between 65% to 80% and by 20% above 80% capacity.

Page 365: cost accounting question icwai inter

Heating & lighting 3.00 3.00 3.30 3.60Semi-variable cost 22.00 22.00 24.20 26.40Sub-Total 90.00 78.67 109.20 128.40Fixed expensesSalaries : Managerial 9.50 9.50 9.50 9.50Rent, Rates and Taxes 6.60 6.60 6.60 6.60Depreciation 7.40 7.40 7.40 7.40Audit fees 6.50 6.50 6.50 6.50Total fixed expenses 30.00 30.00 30.00 30.00Total Cost of sales 120.00 108.67 139.20 158.4Sales 150.00 125.00 187.50 225.00

(i) (ii) (iii) (iv) (v)Opening stock (units) 1,000 1,200 1,100 2,400 1,600Sales (units) 12,000 18,000 14,000 21,000 14,000Cost budgets ( in thousand of rupees )Direct materials 122 179 153Direct labour 73.2 113.1 91.8Production overheads (excluding depreciation ) 111 139.5 126.5Depreciation 45 45 45Administrative overheads 60 60 60Selling overheads 88 112 96Additional information:(i) Production above 16,000 units in a quarter will fetch bonus to workers in addition to regular wages.

Required: (A) Production budget for all the four quarters of 2001(B) Cost budget for the last quarter of 2001.

Solution :(A) Production Budget

(i) (ii) (iii) (iv)Sales in units 12,000 18,000 14,000 21,000Add: Closing stock 1,200 1,100 2,400 1,600Less: Opening stock 1,000 1,200 1,100 2,400Production : 12,200 17,900 15,300 20,200(B) Cost Budget: Production for Qtr.1 units 12,200

Direct materials for Qtr.1 Rs. 122,000Direct labour for Qtr.1 Rs. 73,200Material cost in Rs./ unit 10Direct labour cost in Rs./ unit 6

Calculation for Bonus : Qtr.2 and Qtr.4 qualify for bonus as the production is above 16,000 unitsFor Quarter 2: Rs.Direct labour payment For 17,900 units 113,100Direct labour @ Rs. 6/unit ( 17,900 x Rs. 6) 107,400Bonus for 1,900 units For (17,900 -- 16,000 ) i.e.1,900 units 5,700Bonus / unit ( 5,700 / 1,900) Rs.3 per unitFor Quarter 4: Rs.

Direct labour payment for 20,200 units @ Rs. 6/ unit 121,200Bonus for ( 20,200 -- 16,000) i.e. 4,200 units @ Rs. 3/unit 12,600

Problem 7 (cwa inter II June.02)

A Ltd. prepares its budget on a quarterly basis. Relevant extracts of the budget prepared by the firm for 2001 and 2002 are given below:

(ii) Variable overheads in production overheads vary with production and variable overheads in selling overheads vary with sales.

Page 366: cost accounting question icwai inter

Total wages for Quarter 4 133,800Calculation for fixed cost and variable cost of production overheads: Rs.

For 17,900 units Production overheads 139,500For 12,200 units Production overheads 111,000

Thus For 5,700 units Production overheads 28,500Variable cost per unit = Rs. 28,500 / 5,700 = Rs. 5.Fixed cost = Rs. 139,500 -- Rs. 5 x 17,900 = Rs. 50,000

Calculation for fixed cost and variable cost of selling overheads: Rs. For 18,000 units Selling overheads 112,000For 12,000 units Selling overheads 88,000

Thus For 6,000 units Selling overheads 24,000Variable selling cost per unit = Rs. 24,000 / 6,000 = Rs. 4Fixed cost = Rs. 112,000 -- Rs. 4 x 18,000 = Rs. 40,000

Cost budget for Quarter 4Particulars units Rs./unit Amount Rs. Direct materials 20,200 10 202,000Direct labour 20,200 6 121,200Production bonus 4,200 3 12,600Production overheads variable 20,200 5 101,000Selling overheads variable 21,000 4 84,000Total Variable cost 520,800Fixed cost Production overheads fixed 50,000Selling overheads fixed 40,000Depreciation 45,000Administrative Overheads 60,000Total Fixed cost 195,000

Total Cost 715,800

A company manufactures two products -- X and Y.A forecast of units to be sold in the first five months of the year is given below:Months Produc.X Product YJanuary 1,000 2,800February 1,200 2,800March 1,600 2,400April 2,000 2,000May 2,400 1,600Other information is as follows:Cost per unit Rs. Product X Product YDirect materials 12.5 19.0Direct labour 4.5 7.0Factory overhead 3.0 4.0There will be no opening and closing work-in-progress at the end of any month.

You are required to prepare: (i) Production budget for January to April and (ii) Summarized Production cash budget.

Solution:

2. Production = Sales + Closing stock -- Opening stock; is frequently used in preparation of production budget.

Problem 8 (cwa inter II June.03, dec. 04, dec. 06)

Finished goods in units, equal to half of the budgeted sales of the next month, should be in stock at the end of each month (including previous year December).

1. The closing stock of December is half of budgeted sales of January. Thus opening stock of January is closing stock of Dec. Thus opening stock of any month will be half of budgeted sales of that month.

Page 367: cost accounting question icwai inter

Production budget of product X for the period -- January to AprilProduct X

Months Budget sales Opening stock Closing stock Production required.January 1,000 500 600 1,100February 1,200 600 800 1,400March 1,600 800 1,000 1,800April 2,000 1,000 1,200 2,200Total 5,800 2,900 3,600 6,500

Product YJanuary 2,800 1,400 1,400 2,800February 2,800 1,400 1,200 2,600March 2,400 1,200 1,000 2,200April 2,000 1,000 800 1,800Total 10,000 5,000 4,400 9,400

Summarized Production Cash Budget:Quantity Material Labour Overhead Total

units Rs. Rs. Rs. Rs.Product X 6,500 12.5 4.5 3.0 20

81,250 29,250 19,500 130,000Product Y 9,400 19.0 7.0 4.0 30

178,600 65,800 37,600 282,000Total 259,850 95,050 57,100 412,000

Rs.Material cost 50,000 100% varyingLabour cost 30,000 100% varyingPower 3,000 80% varyingRepair and maintenance 3,500 80% varyingStores 2,000 100% varyingInspection 800 25% varyingDepreciation 10,000 100% varyingAdministrative overheads 3,600 25% varyingSelling overheads 4,500 50% varyingTotal 107,400Cost per unit 10.74

Solution:Flexible Budget at different capacity levels in units

Capacity in units 10,000 8,000 12,000Variable Cost: Rs. Rs. Rs.Material cost 100% Varying 50,000 40,000 60,000Labour cost 100% Varying 30,000 24,000 36,000Power 80% Varying 2,400 1,920 2,880Repair and maintenance 80% Varying 2,800 2,240 3,360Stores 100% Varying 2,000 1,600 2,400Inspection 25% Varying 200 160 240Depreciation 100% Varying 10,000 8,000 12,000Administrative overheads 25% Varying 900 720 1,080

Problem 9 (cwa inter II June.03)

The cost of an article at a capacity level of 10,000 units is given below. For a variation in capacity above or below this level, the individual expenses vary as given below:

Find out the unit cost of the product under each individual category of expenses at the production levels of 8,000 units and 12,000 units respectively.

Page 368: cost accounting question icwai inter

Selling overheads 50% Varying 2,250 1,800 2,700Total Variable cost 100,550 80,440 120,660Fixed costs:Power 20% Varying 600 600 600Repair and maintenance 20% Varying 700 700 700Inspection 75% Varying 600 600 600Administrative overheads 75% Varying 2,700 2,700 2,700Selling overheads 50% Varying 2,250 2,250 2,250Total fixed costs 6,850 6,850 6,850Total cost: 107,400 87,290 127,510Cost per unit 10.74 10.91 10.63

Cost in Rs. Per unitDirect materials 25Direct labour 10Factory overheads 20 (50% fixed )Selling and administrative overheads 15 ( one-third variable )Selling price 90

1. The capacity will be increased to 10 lakh units.

4. The average depreciation rate on the new investment is 10% based on straight line method.

After the expansion is put through, the company has two alternatives for operating expanded plant as under:

2. Sales can be increased to 10 lakh units subject to the following:(i) By an overall reduction of Rs. 10 per unit on all the units sold ,(ii) By increasing the variable selling and distribution expenses by 5%(iii) The direct materials costs would go down by 1% due to discount on bulk buying.

Required:1. Prepare a flexible budget at the level of 5 lakh, 8 lakh and 10 lakh units of production.2. Calculate the break-even point before and after expansion.3. Advise which level of output should be chosen for expansion.Solution:Working notes:

Factory overheads : Rs. LakhsFixed: 60.01. Selling and admn. Overheads Fixed : Rs.10 (two-third of Rs. 15) x 600,000 30.02. Material cost for 10 lakh units is 99% of Rs. 25 x 10 lakh: 247.53. Variable selling and dist. cost at 10 lakh units: 105% of Rs.5 x 10 lakh 52.54. Sales revenue at 10 lakh units : Rs. 80 x 10 lakh : 800.0

Flexible budget for SVT Ltd.Particulars

Problem 10 (cwa inter II June.04,similar June 07)

SVT Ltd., a manufacturing company, having a capacity of 6 lakh units has prepared the following cost sheet:

During the year 2003 the sales volume achieved by the company was 5 lakh units. The company has launched an expansion programme, the details of which are as under:

2. The additional fixed overheads will amount to Rs. 40 lakhs up to 8 lakh units and will increase by Rs. 20 lakhs more beyond 8 lakh units.

3. The cost of investment of expansion is Rs. 80 lakhs, which is proposed to be financed through bank borrowings carrying interest at 15% per annum.

1. Sales can be increased upto 8 lakh units by spending Rs. 10 lakh on special advertisement campaign to explore new market; or

1. The fixed cost has been given at installed capacity of 6 lakh units, however, the present production is 5 lakh units.

Page 369: cost accounting question icwai inter

Output capacity 5 lakh 8 lakh 10 lakhSelling price Rs. Rs. 90 90 80.00Sales revenue Rs. Lakhs 450 720 800.00Variable cost :Material price Rs. Per unit Rs. 25 25 24.75Material cost Rs. Lakh 125 200 247.50Direct labour @ Rs. 10/unit Rs. Lakh 50 80 100.00Factory overhead @ Rs. 10/unit Rs. Lakh 50 80 100.00Selling and distribution cost/unit 5 5 5.25Selling and distribution cost Rs. Lakh 25 40 52.50Total Variable cost Rs. Lakh 250 400 500.00Contribution : Rs. Lakh 200 320 300.00Fixed Expenses : Rs. Lakh Rs. Lakh Rs. LakhFactory overhead 60 60 60Selling and distribution 60 60 60Increase due to expansion -- 40 60Interest @ 15% on Rs. 80 lakhs -- 12 12Depreciation @ 10% on Rs. 80 lakhs -- 8 8Special advertisement -- 10 --Total Fixed cost Rs. Lakh 120 190 200Profit ( Contribution -- Fixed cost ) 80 130 100Break-even point ( Contribution / Sales ) 44% 44% 38%Break-even point in lakh units 3.00 4.75 6.67Break-even point in lakh Rs. 270.00 427.50 533.33

Item Amount in Rs. variationMaterial cost 25,000 100% Variable Labour cost 15,000 100% Variable Power 1,250 80% Semi-variableRepairs and maintenance 2,000 75% Variable Stores 1,000 100% Variable Inspection 500 20% Semi-variableAdministration overheads 5,000 25% Semi-variableSelling overheads 3,000 50% Semi-variableDepreciation 10,000 100% FixedTotal 62,750Cost per unit in Rs. 12.55

You are required to prepare the production cost budget (flexible) at 4000 and 6000 units.Solution: It can be solved on the lines of problem 9 above.Cost per unit for 4000 units is Rs. 13.37 and for 6000 units Rs. 12.0.

Month 1 Month 2Units producedCosts (Other than direct materials and direct labour) Rs. Rs.Salaries 30,000 30,000Power 30,000 39,000Consumable stores 30,000 45,000

Recommendation: Since the profit is maximum at 8 lakh units amounting to Rs. 130 lakhs; This level should be chosen for expansion.

Problem 11 (cwa inter II dec..04)

The cost of an article at a capacity level of 5,000 units is given below. For a variation in capacity above or below this level, the individual expenses vary as given below:

Problem 12 (cwa inter II dec. 05)

Brilliant Ltd. is engaged in production of certain products, 100% capacity being 10,000 units per month. Given below are the information for the just concluded previous two months;

Page 370: cost accounting question icwai inter

Repair 40,000 46,000Indirect shop labour 15,000 22,500Depreciation 25,000 25,000Inspection 10,000 13,000

Solution: Similar to problem 5 above. The answer follows:Capacity 100% 80% 50%Cost of production

Prime cost 280,000 224,000 140,000Variable factory overheads 179,000 152,000 110,500Fixed overheads 55,000 55,000 55,000Total overheads 234,000 207,000 165,500Total cost of production 514,000 431,000 305,500Absorption rate at 100% level Rs. 234/ hour.

10% 20% 30% 40% 50% 60% 70% 80% 90% 100%400 390 380 370 360 350 340 330 320 310

It can market 100% of its output at Rs. 500 per unit provided it incurs the following further expenditure:

(i) It gives gift items costing Rs. 30 per unit of sale;

(iii) it spends Rs. 100,000 on refreshments every month to customers.(iv) It sponsors a television programme every week at a cost of Rs. 20,00,000 per month.

Advise the company on its course of action. Show the supporting cost sheets.Solution:Level of capacity 30% 100%Output in units 30,000 100,000

Per unit Rs. '000 Per unit Rs. '000Works cost 380.0 11,400 310.0 31,000Add: Fixed administrative expenses 5.0 150 1.5 150Cost of production 385.0 11,550 311.5 31,150Add: Fixed marketing expenses 8.3 250 2.5 250Add: Variable distribution expenses 30.0 900 30.0 3,000Add: Special cost -- --Gift items -- -- 30.0 3,000Customer's prizes -- -- 1.0 100Refreshments -- -- 1.0 100Television -- -- 20.0 2,000

423.3 12,700 396.0 39,600Selling price and revenue 550.0 16,500 500.0 50,000Profit 126.7 3,800 104.0 10,400Recommendation:

Rate of production per hour is 10 units. Direct materials costs are Rs. 20 per unit and direct labour costs are Rs. 80 per hour. You are required to compute cost of production segregating fixed, semi-variable and variable costs separately at 100%, 80% and 50% capacity utilization level respectively. Also work out the overhead absorption rate per hour at 100% capacity utilization level. Show your workings.

Problem 13 (ca inter nov.98)

A ltd. company has capacity to produce 100,000 units of product every month. Its works cost at varying levels of production is as under :

Its fixed administrative expenses amount to Rs. 150,000 and fixed marketing expenses amount to Rs. 250,000 per month respectively. The variable distribution cost amounts to Rs. 30 per unit.

(ii) It has lucky draws every month giving the first prize of Rs. 50,000 ; 2nd prize of Rs. 25,000; 3rd prize of Rs. 10,000 and three consolation prizes of Rs. 5,000 each to customers buying the product.

(v) It can market 30% of its output at Rs. 550 per unit without incurring any of the expenses referred to in (i) to (ii) above.

Page 371: cost accounting question icwai inter

Rs./unitDirect materials 30Direct wages 12Factory overheads 30 (50% variable)Selling and administration overheads 18 Two-third fixed)Selling price 120

(i) The capacity will be increased to 12 lakh units.

(iv) The average depreciation rate on the new investment is 15% based on straight line method.

(ii) Sales can be increase to 12 lakh units subject to the following:1.By an overall reduction of Rs. 10 per unit on all the units sold2.By increasing the variable selling and administrative expenses by 8%3.The direct materials cost could go down by 1.5%due to discount on bulk purchase.

Required:(i) Construct a Flexible budget at the level of 6 lakhs, 10 lakhs and 12 lakhs units of production.(ii) Calculate break-even point before and after expansion(iii) Advise the optimum level of output for expansion.Solution: Solve on the lines of solution to problem 10 above. Answer follows:Output level units 6 lakhs 10 lakhs 12 lakhsSales revenue 720 1200 1320Variable cost : 378 630 756.36Fixed cost 189 279 294Break-even point 397.9 587.37 688.53Problem 15 (cs final dec. 08)

Rs.Raw materials 100Labour 30Factory Overheads 30 ( 40% fixed)Administrative Overheads 20 ( 50% fixed)

At 100% capacity the profit rise is to the tune of Rs. 66 lakhs ( 104 -- 38). It is advised to go for special cost and use 100% capacity of production.

Problem 14 (cwa inter June 07)

Futura Ltd. a manufacturing company, having a capacity of 7 lakh units has prepared the following cost sheet: cost in Rs. Per unit

During the year 2006-07 the sales volume achieved by the company was 6 lakh units. The company has launched an expansion programme, the details of which are as under :

(ii) The additional fixed overheads will amount to Rs. 50 lakhs upto 10 lakh units and will increase by Rs. 25 lakh more beyond 10 lakh units.

(iii) The cost of investment of expansion is Rs. 100 lakhs which is proposed to be financed through Bank borrowings carrying interest at 15% per annum.

After the expansion is put through, the company has two alternatives for operating expanded plant as under:

(i) Sales can be increased upto 10 lakh units by spending Rs. 10 lakhs on special advertisement campaign to explore new market. OR

A factory is currently working at 50% capacity and produces 1,000 units. From the following information, You are required to estimate profits of the factory when it works at 60% and 80% working capacity respectively and offer your critical comments.

At 60% working capacity, raw materials cost increases by 2% and selling price falls by 2%. At 80% working capacity, raw materials cost increase by 5% and selling price falls by 5%. At 50% capacity working, the product costs Rs. 180 per unit and is sold at Rs. 200 per unit. The unit cost of Rs. 180 is made up as follows :

Page 372: cost accounting question icwai inter

Solution :Tutorial Notes :

The solution goes as follows :Flexible Budget

Capacity % 50% 60% 80%No. of units 1,000 1,200 1,600Variable cost Rs. Rs. Rs. Rs.Raw materials 100 102 105Labour 30 30 30Factory overheads 18 18 18Administrative overheads 10 10 10Total variable cost per unit 158 160 163Selling price per unit 200 196 190Contribution per unit 42 36 27Total contribution 42,000 43,200 43,200Less Fixed cost : 22,000 22,000 22,000Profit 20,000 21,200 21,200

The greatest pleasure in life is in doing what others say you cannot do.Problem 16: (CWA Inter June 09)

Particulars Rs. Semi-variable expenses at 50% capacityFixed Expenses : Plant maintenanceManagement salaries 210,000 LabourRent and taxes 140,000 Salesmen's salariesDepreciation of Machinery 175,000 Sundry expensesSundry office expenses 222,000Total Fixed expenses 747,000 Total Semi-variable expensesVariable expenses at 50% capacity Rs.Materials 600,000Labour 640,000Salesmen's commission 95,000Total variable expenses 1,335,000

Prepare a flexible budget at 60%, 80% and 100% productive capacity.Solution : This is very easy. You can solve it on the lines of previous problems. Answers are:

60% 80% 100% (in Rs. Lacs)

1. Whenever total cost is given at a particular capacity, it is advisable to separate the variable and fixed components

2. This is an easy question. You should prepare a table with different capacity. You should compute the sales, variable cost and contribution at different capacity and find the profit by deducting fixed cost from contribution. Note that the fixed cost remains constant for different capacity.

3. You should compute the fixed cost. At 50% units the fixed factory overheads is Rs. 12/unit. For 1,000 units this would be Rs. 12,000 and this will remain constant at other capacities as well. Similarly fixed administrative overheads would be Rs. 10,000.

The contribution and profit remains same at 60% and 80% capacity. It is not advisable to increase the production from 60% capacity to 80% capacity. It is advisable to increase production upto 60% capacity.

The following information relates to the production activities of Good Wish Ltd. for 3 months ending 31st December, 2006

It is further noted that semi-variable expenses remain constant between 40% and 70% capacity, increase by 10% of the above figure between 70% and 85% capacity and increase by 15% of the above figure between 85% and 100% capacity. Fixed expenses remain constant whatever the level of activity.

Sales at 60% capacity are Rs. 25,50,000; at 80% capacity Rs. 34,00,000 and at 100% capacity Rs. 42,50,000. All items produced are sold.

Page 373: cost accounting question icwai inter

Sales 25.50 34.00 42.50Variable expenses 16.02 21.36 26.70Semi-variable expenses 4.47 4.92 5.14Fixed expenses 7.47 7.47 7.47Total cost 27.96 33.75 39.31Profit (loss) (2.46) 0.25 3.19 Problem 17: (cwa inter June 2008)

Rs. Rs.Fuel and power 150,000 Indirect wages 80,000Salaries 400,000 Repairs 5,000Stores 40,000 Defectives 20,000Electricity 50,000The budgeted hours for the three products A, B and C are 5, 3 and 8 hours respectively.The overheads increase by 10% for every increase up to 20% in capacity over 100% capacity.Prepare a flexible production overhead budget for the year 2009 with the above data.Solution :Products A B CBudgeted units in lakhs 1.0 1.5 0.5Budgeted hours per unit 5.0 3.0 8.0Total lakh hours 5.0 4.5 4.0 13.5As 100% capacity is 10 lac hours, for 13.5 lac hours the capacity would be 135%

Production overheads budget (fig. in Rs.)Overhead items 100% capacity 135% capacity Fuel and power 150,000 180,000Salaries 400,000 480,000Stores 40,000 48,000Electricity 50,000 60,000Indirect wages 80,000 96,000Repairs 5,000 6,000Defectives 20,000 24,000

745,000 894,000Problem 18: (cwa inter dec. 2008)The following are estimate sales of a company for eight months ending 30.11.07Month April May June July August September OctoberSales units 12,000 13,000 9,000 8,000 10,000 12,000 14,000The company maintains the closing balance of finished goods and raw materials as followsFinished goods : 50% of the estimate sales for next monthRaw materials : Estimated consumption for the next month.Every unit of production requires 2 kg of raw materials costing Rs. 5 per kg.

Solution : Production Budget for half year ending Sept.30, 2007Month April May June July August SeptemberSales units 12,000 13,000 9,000 8,000 10,000 12,000Add: Closing stock 6,500 4,500 4,000 5,000 6,000 7,000

18,500 17,500 13,000 13,000 16,000 19,000Less: Opening stock 6,000 6,500 4,500 4,000 5,000 6,000Estimated production 12,500 11,000 8,500 9,000 11,000 13,000

Raw materials purchase budget for half year ending Sept.30, 2007

A company manufactures three products A, B and C with a target to produce 1 lakh units, 1.5 lakhs units and 0.5 lakh respectively during year 2009. The expenses, overheads at 100% (10 lakh hours) capacity production are budgeted as follows :

Overheads increase by 10% for every increase of 20%. Thus for capacity of 135% the overheads would increase by 20%.

Prepare Production budget in units and Raw materials purchase budget in units and cos of the company for the year ending 30 Sept.2007.

Page 374: cost accounting question icwai inter

Estimated production 12,500 11,000 8,500 9,000 11,000 13,000Consumption @ 2 kg/unit 25,000 22,000 17,000 18,000 22,000 26,000Add: Closing stock 22,000 17,000 18,000 22,000 26,000 26,000Less: Opening stock 25,000 22,000 17,000 18,000 22,000 26,000Purchase 22,000 17,000 18,000 22,000 26,000 26,000Cost @ Rs. 5/kg 110,000 85,000 90,000 110,000 130,000 130,000

Some problems on Cash Budget

Production and Sales ( in units ) Selling price and Costs ( in Rs. / unit )Month Production Sales Selling price / unitApril 2,000 -- Less: Variable cost May 3,000 -- Material June 4,000 1,000 LabourJuly 5,000 2,000 OverheadAugust 5,000 4,000 Contribution / unitSeptember 5,000 5,000Note: 1. Both Production and sales will stabiles at 5.000 units from September onwards.2. Fixed cost are expected to be Rs. 10,000 per month.The following additional information is also given:

2. Suppliers of material have agreed to give one month's credit.3. Labour is to be paid half a month in arrears.4. Variable overhead will be paid during the month following the month in which it is incurred.5. Fixed overheads will be incurred in advance at the beginning of every quarter. 6. Sales will be 50% cash and balance will be on two months credit7. There will be opening cash balance of Rs. 300,000 in hand and bank.

Solution:Working notes : Purchase requirement of material is computed as below: all fig. in '000

Month Purchase Value Rs. PaymentApril 0 2 9 9 7 108 0May 7 3 12 5 9 60 108June 9 4 14 5 10 60 60July 10 5 15 5 10 60 60August 10 5 15 5 10 60 60Sept. 10 5 15 5 10 60 60

Sales and receipts are as follows: Figures in Rs.'000Month Units Cash Debtors

sold Sales Collect.April -- -- --May -- -- --June 1 25 --

Problem 19 (cwa inter II dec.2000)

A Ltd. has just established a small manufacturing unit to manufacture a new product which is expected to have a high margin. The company has made the following estimates of production, sales and costs.

1. An initial stock of material to meet three months requirements will be purchased during April 2000. Further purchases will be made at the beginning of each month to have sufficient stock of material for three months.

Prepare a cash budget for A Ltd. for the six months ending 30 September 2000. Figures should be given monthly and the months, if any, during which additional funds are required should be clearly indicated.

Opening

Balance

Consumption

Requirement

Closing Balance

Page 375: cost accounting question icwai inter

July 2 50 --August 4 100 25Sept. 5 125 50

Labour and Overheads Figures in Rs.'000Month Produc. Wages Wages overhead overhead

due paid due paidApril 2 10 5 10May 3 15 12.5 15 10June 4 20 17.5 20 15July 5 25 22.5 25 20August 5 25 25 25 25Sept. 5 25 25 25 25

Cash Budget for half the year ending September 2000April May June July August

Opening balance 300 265.0 134.5 67.0 (15.50)ReceiptsCash sales -- -- 25 50 100Collection from debtors -- -- -- -- 25Total receipts 300 265 159.5 117.0 125Payments:Purchases 0.0 108.0 60.0 60.0 60.0Wages 5.0 12.5 17.5 22.5 25.0Variable overhead 10.0 15.0 20.0 25.0Fixed overhead 30.0 30.0Total payments 35.0 130.5 92.5 132.5 110.0Closing Balance 265.0 134.5 67.0 (15.50) (0.5)Additional funds are required in the months of August Rs. 15,500 and Sept. Rs. 500.

Based on the following information, prepare a Cash Budget for ABC Ltd.Quarter 1st 2nd 3rd 4th

Rs. Rs. Rs. Rs.Opening cash balance 10,000Collection from customers 125,000 150,000 160,000 221,000Payments:Purchase of material 20,000 35,000 35,000 17,000Other expenses 25,000 20,000 20,000 17,000Salary and wages 90,000 95,000 95,000 109,200Income tax 5,000Purchase of machinery 20,000

Solution:

Problem 20 (cwa inter II june.06)

The company desires to maintain a cash balance of Rs. 15,000 at the end of each quarter. Cash can be borrowed or repaid in multiple of Rs. 500 at an interest of 10% per annum. Management does not want to borrow cash more than what is necessary and wants to repay as early as possible. In any event, loans cannot be extended beyond four quarters. Interest is computed and paid when the principal is repaid. Assume that borrowings take place at the beginning and repayments are made at the end of the quarters.

Page 376: cost accounting question icwai inter

Quarter 1st 2nd 3rd 4thRs. Rs. Rs. Rs.

Opening cash balance 10,000 15,000 15,000 15,325Collection from customers 125,000 150,000 160,000 221,000Total Cash available 135,000 165,000 175,000 236,325Payments:Purchase of material 20,000 35,000 35,000 17,000Other expenses 25,000 20,000 20,000 17,000Salary and wages 90,000 95,000 95,000 109,200Income tax 5,000 -- -- --Purchase of machinery -- -- -- 20,000Total Cash payments 140,000 150,000 150,000 163,200Minimum cash balance required 15,000 15,000 15,000 15,000Total Cash required 155,000 165,000 165,000 178,200Excess (deficit) (20,000) 0 10,000 58,125Borrowings 20,000 0 0 0Repayments 0 0 9,000 11,000Interest payment 0 0 675 1,100Interest computation:For 3rd quarter 9,000 x 0.10 x (9/12) = Rs. 675For 4th quarter 11,000 x 0.10 x 1 = Rs. 1,100

1. Working Capital as on 1 January 2000 has been estimated as under:Rs. Rs.

Cash and Bank 10,900 Debtors 51,400Creditors 42,200 Outstanding expenses 4,000Dividend due 9,700 Tax due 6,400Stock 26,000

2. Budgeted profit statements for the three months are:Jan.00 Feb. 00 Mar-00

Sales 42,000 36,000 34,000Cost of sales 32,700 28,100 26,600Gross profit 9,300 7,900 7,400Admin. Selling and distr. Expenses 6,300 5,400 5,100Net profit before tax 3,000 2,500 2,3003. Budgeted Balances at the end of each month:Stock 24,000 22,000 20,000Debtors 52,000 50,000 47,000Creditors 40,000 39,000 38,000Outstanding expenses 4,000 4,000 4,000Dividend due 9,700 0 0Tax due 6,400 6,400 6,400Depreciation amounting to Rs. 1,700 has been included in the budgeted expenditure of each month.Required :Prepare month wise cash budget for the three months on receipt and payment basis

Cash budget should be prepared column wise i.e. vertically. First receipts and payments of 1st quarter should be prepared, the closing balance of 1st quarter will be opening balance of 2nd quarter and so on.

Problem 21 (cwa final dec.2000)

The following details of estimates are obtained in respect of the retail business of Fancy Ltd. for the months of January to March 2000.

Page 377: cost accounting question icwai inter

Solution :Working notes : Jan Feb Mar.1. Monthly expenses on production :Cost of sales 32,700 28,100 26,600Add: Closing stock 24,000 22,000 20,000Less Opening stock 26,000 24,000 22,000Less Depreciation (being non-cash item) 1,700 1,700 1,700Cash expenses on production for the month 29,000 24,400 22,9002. Payment to Creditors :Opening balance of creditors 42,200 40,000 39,000Add : Cash cost of sales 29,000 24,400 22,900Add : Other expenses 6,300 5,400 5,100Less : Closing balance of creditors 40,000 39,000 38,000Payment to Creditors : 37,500 30,800 29,0003. Receipts from Debtors:Opening balance 51,400 52,000 50,000Add : Credit sales during month 42,000 36,000 34,000Less : Closing balance 52,000 50,000 47,000Receipts from Debtors: 41,400 38,000 37,000

Fancy LimitedCash budget for Jan.2000, Feb.2000 and March 2000

Cash and Bank Opening balance 10,900 14,800 12,300Add : Receipt from Debtors 41,400 38,000 37,000

52,300 52,800 49,300Less : Payment to creditors 37,500 30,800 29,000Less : Dividend paid 0 9,700 0Closing Balance 14,800 12,300 20,300

Nov.01 Dec.01 Janu.02 Feb.02 Mar.02 Apr.02Actual sales in Rs. 80,000 70,000Forecast sales in Rs. 80,000 100,000 80,000 100,00020% of sales is in cash and rest is on credit, payment of which is realised in the third month.The following information are also available:

(ii) Variable expenses is 5% of turnover-time lag for payment half month.(iii) Commission of credit sales @ 5% is payable in the third month.(iv) Rent and other expenses amounting Rs. 3,000 paid every month.(v) Payment for purchase of fixed assets Rs. 50,000 in March,02.(vi) Payment for taxes in April,02 Rs. 20,000.(vii) There will be opening balance of cash of Rs. 25,000.You are required to prepare a Cash budget for the months from January to May 2002.Tutorial Notes:

Solution : Working Notes: All figures in Rs.

Problem 22 (cwa final dec.2001)

The sales forecast for January to May, 2002 and the actual sales for November and December,2001 for Plysales Co. are given under :

(i) Amount of purchase is budgeted at 60% of the Sales Turnover of a month and paid in the third month of purchase.

1. Variable expenses payable @ 5% of turn over. The time lag is half month,. It implies that half of variable expenses is paid in the same month while next half is paid in next month.

Page 378: cost accounting question icwai inter

Computation of Cash and credit sales and commission:Nov.01 Dec.01 Janu.02 Feb.02 Mar.02 Apr.02

Sales 80,000 70,000 80,000 100,000 80,000 100,000Cash sales 16,000 14,000 16,000 20,000 16,000 20,000Credit sales 64,000 56,000 64,000 80,000 64,000 80,000Commission payable* 3,200 2,800 3,200 4,000Payment received 64,000 56,000 64,000 80,000Purchases 48,000 42,000 48,000 60,000 48,000 60,000Payment for purchases 48,000 42,000 48,000 60,000Variable expenses 2.5% of current month and 2.5% of previous month sales.*5% of respective credit sales. Commission for Nov. is obtained in Jan.

Plysales Ltd.Cash Budget for the period from January 02 to May 02

Janu.02 Feb.02 Mar.02 Apr.02 May.02Receipts: Rs. Rs. Rs. Rs. Rs. Opening Balance 25,000 47,050 70,750 42,050 50,550Cash sales 16,000 20,000 16,000 20,000 18,000Collection from debtors 64,000 56,000 64,000 80,000 64,000Total receipts 105,000 123,050 150,750 142,050 132,550Payments :Payment to creditors 48,000 42,000 48,000 60,000 48,000Variable expenses 3,750 4,500 4,500 4,500 4,750Commission 3,200 2,800 3,200 4,000 3,200Rent 3,000 3,000 3,000 3,000 3,000Fixed assets 50,000Taxes 20,000Total payments 57,950 52,300 108,700 91,500 58,950Closing Balance 47,050 70,750 42,050 50,550 73,600

Failure is success if we learn from it.

·       Don't judge those who try and fail. Judge only those who fail and try.

Page 379: cost accounting question icwai inter

Total22

Fixed cost Rs. 5(1-4)9,0005,0005,000

19,00011,400

7,600

;g dgrk gS fd ftl pht ;k ckr dh vki vkRefo'okl ds lkFk mEehn djrs gSa oks vkids thou esa lkdkj gks tkrh gSA vxj vki dke;kc] lsgrean vkSj le`) gksus dh mEehn djrs

The company is presently passing through a period of very lean market demand and operating at 50% capacity and have also selling its product at a discounted price generating a total sales revenue of Rs. 60.000 at that

It is expected that the market scenario will improve in the next year and, on a conservative estimate, the company is likely to operate at 70% capacity level with increased sales revenue of Rs. 120,000.

As an option, the management is considering to close down the operation for one year and restart operation after one year when the market conditions are likely to improve. If closed down for the year it estimated that:

1. Whenever total cost containing fixed and variable portion is given, it should be broken down to fixed and

2.There are two options which are to be considered and compared viz. to continue the current year operations at

3. The question is silent about the profitability of next year operations estimated at 70% capacity, however, the

Page 380: cost accounting question icwai inter

If current year operations are continued, this will lead to a loss of Rs. 9500 as indicated above, this however is

it is recommended that the company should continue to operate at current level and wait for the scenario change likely to take place in next year, which is estimated to fetch a profit of Rs. 30,300 at 70%capacity level.

A newly established manufacturing company has an installed capacity to produce 100,000 units of a consumer product annually. However, its practical capacity is only 90%. The annual capacity utilisation may be substantially lower, as the firm is new to the market and the demand is uncertain. The following budget has been

You are required to prepare budgets at 60%, 70% and 80% levels of capacity utilisation giving the unit variable

Page 381: cost accounting question icwai inter

For the production of 100,000 units, the material required would be Rs. 12.00 lakh (@ Rs. 12 per unit).

A company manufactures and sells two products X and Y using the same raw materials and the same type of labour. It is trying to prepare the annual budget for 2002--03. The marketing department has estimated the average monthly sales of X Rs. 2 lakh and that of Y as Rs. 4 lakh. Further details relating to the two products are

Currently the raw material stock is equal to requirements for 1.5 month average production. However, the closing

Let us take product X first. The annual sales is Rs. 24 lakhs (monthly sale Rs. 2 lakh given) which has 20% of profit element. This gives cost of sales as Rs. 19.20 lakh. The standard cost per unit is Rs. 20, thus the no. of units to be sold is (19.20 / 20) = 96,000. The opening stock is (280,000 / 20 ) = 14,000 units and the closing stock is (200,000 / 20) = 10,000 units. Thus the units to be produced in the year must be (96,000 + 10,000 --

The process loss is given as 8%. It implies that for producing 92 units 100 units are to be introduced. Thus for

Page 382: cost accounting question icwai inter

Work out a flexible budget for overhead expenses on the basis of following data available from the cash records

Page 383: cost accounting question icwai inter

Rate of production per hour is 10 units. Direct material cost is Rs. 2 per unit and direct labour cost is Rs. 8 per

(a) Cost of production at 50%, 80% and 100% capacity respectively showing separately the fixed, semi variable

Page 384: cost accounting question icwai inter

(a) Variable and semi-variable cost (business) Cost of sales and (c) profit - with the help of following

(ii) Semi-variable expenses remain constant between 45% and 64% capacity, increasing by 10% between 65%

Page 385: cost accounting question icwai inter

(i) Production above 16,000 units in a quarter will fetch bonus to workers in addition to regular wages.

Qtr.2 and Qtr.4 qualify for bonus as the production is above 16,000 units

A Ltd. prepares its budget on a quarterly basis. Relevant extracts of the budget prepared by the firm for 2001

(ii) Variable overheads in production overheads vary with production and variable overheads in selling overheads

Page 386: cost accounting question icwai inter

Amount Rs.

2. Production = Sales + Closing stock -- Opening stock; is frequently used in preparation of production budget.

Finished goods in units, equal to half of the budgeted sales of the next month, should be in stock at the end of

1. The closing stock of December is half of budgeted sales of January. Thus opening stock of January is closing stock of Dec. Thus opening stock of any month will be half of budgeted sales of that month.

Page 387: cost accounting question icwai inter

Find out the unit cost of the product under each individual category of expenses at the production levels of 8,000

Page 388: cost accounting question icwai inter

After the expansion is put through, the company has two alternatives for operating expanded plant as under:

During the year 2003 the sales volume achieved by the company was 5 lakh units. The company has launched

2. The additional fixed overheads will amount to Rs. 40 lakhs up to 8 lakh units and will increase by Rs. 20 lakhs

3. The cost of investment of expansion is Rs. 80 lakhs, which is proposed to be financed through bank

1. Sales can be increased upto 8 lakh units by spending Rs. 10 lakh on special advertisement campaign to

Page 389: cost accounting question icwai inter

It can market 100% of its output at Rs. 500 per unit provided it incurs the following further expenditure:

Page 390: cost accounting question icwai inter

(iv) The average depreciation rate on the new investment is 15% based on straight line method.

Page 391: cost accounting question icwai inter

Rs.62,500

247,00072,50065,000

447,000

Answers are:

The following information relates to the production activities of Good Wish Ltd. for 3 months ending 31st

It is further noted that semi-variable expenses remain constant between 40% and 70% capacity, increase by 10% of the above figure between 70% and 85% capacity and increase by 15% of the above figure between 85%

Sales at 60% capacity are Rs. 25,50,000; at 80% capacity Rs. 34,00,000 and at 100% capacity Rs. 42,50,000.

Page 392: cost accounting question icwai inter

November12,000

A company manufactures three products A, B and C with a target to produce 1 lakh units, 1.5 lakhs units and 0.5 lakh respectively during year 2009. The expenses, overheads at 100% (10 lakh hours) capacity production are

Overheads increase by 10% for every increase of 20%. Thus for capacity of 135% the overheads would increase

Prepare Production budget in units and Raw materials purchase budget in units and cos of the company for the

Page 393: cost accounting question icwai inter

Selling price and Costs ( in Rs. / unit )50

1255

28

4. Variable overhead will be paid during the month following the month in which it is incurred.

A Ltd. has just established a small manufacturing unit to manufacture a new product which is expected to have a high margin. The company has made the following estimates of production, sales and costs.

1. An initial stock of material to meet three months requirements will be purchased during April 2000. Further purchases will be made at the beginning of each month to have sufficient stock of material for

Prepare a cash budget for A Ltd. for the six months ending 30 September 2000. Figures should be given monthly and the months, if any, during which additional funds are required should be clearly

Page 394: cost accounting question icwai inter

Rs.'000Sept.(0.5)

12550

175

60.025.025.0

110.064.5

The company desires to maintain a cash balance of Rs. 15,000 at the end of each quarter. Cash can be borrowed or repaid in multiple of Rs. 500 at an interest of 10% per annum. Management does not want to borrow cash more than what is necessary and wants to repay as early as possible. In any event, loans cannot be extended beyond four quarters. Interest is computed and paid when the principal is repaid. Assume that borrowings take place at the beginning and repayments are made at

Page 395: cost accounting question icwai inter

Depreciation amounting to Rs. 1,700 has been included in the budgeted expenditure of each month.Required :Prepare month wise cash budget for the three months on receipt and payment basis

Cash budget should be prepared column wise i.e. vertically. First receipts and payments of 1st quarter should be prepared, the closing balance of 1st quarter will be opening balance of 2nd quarter and so

The following details of estimates are obtained in respect of the retail business of Fancy Ltd. for the

Page 396: cost accounting question icwai inter

May.02

90,000

The sales forecast for January to May, 2002 and the actual sales for November and December,2001

(i) Amount of purchase is budgeted at 60% of the Sales Turnover of a month and paid in the third

1. Variable expenses payable @ 5% of turn over. The time lag is half month,. It implies that half of variable expenses is paid in the same month while next half is paid in next month.

Page 397: cost accounting question icwai inter

May.0290,00018,00072,000

3,20064,00054,00048,000

Don't judge those who try and fail. Judge only those who fail and try.

Page 398: cost accounting question icwai inter

Chapter 12: Standard Costing cwa inter cwa final ca inter ca final

25 3 0 3

Problem 1: (cwa inter dec.05)

Chemical % of total input Standard cost per kg.A 50% 40B 30% 60C 20% 95There is a process loss of 5% during the course of manufacture.The management gives following details for a certain week:

Chemical consumed Quantity purchased and issued Actual cost (Rs.)A 5,200 kg 234,000B 3,600 kg 219,600C 1,700 kg 158,100

Output of finished product :10200 kg. Calculate all the relevant variances.Solution: Tutorial notes:

First of all standard cost of output is to be known.Aish-Abhi Company

Statement showing standard cost of chemical XChemical% of output Quantity in kg Standard Cost / kg.(Rs. Total cost (Rs.)A 50% 0.5 40 20B 30% 0.3 60 18C 20% 0.2 95 19Total input 1.0loss on processing 5% 0.05Output 0.95 57Standard cost of the product in Rs. Per kg = 57 / 0.95 = Rs. 60.(1)Total material cost variance = Standard cost of actual production − Actual cost of actual production = 10200 x 60 − ( 234,000 + 219,600 + 158,100 ) = Rs. 300 (favourable)(2)Total material price variance = Standard price of actual production − Actual price of actual production

Chemical Quantity consumed Actual cost (Rs.) Cost per kg Rs. St.CostA 5,200 kg 234,000 45B 3,600 kg 219,600 61C 1,700 kg 158,100 93Material price varianceChemical St.cost Actual cost Diffe. Quantity VarianceA 40 45 5 5,200 26,000 Adv.B 60 61 1 3,600 3,600 Adv.C 95 93 2 1,700 3,400 Adv.Total price variance 33,000 Adv.(3)Material mix variance:Material mix variance = ( Actual input in standard proportion − Actual input ) x Standard cost of input

,sls nl yksxksa dh lwph rS;kj djsa tks vkids fy;s fofHkUu {ks=ksa esa ennxkj gks ldrs gSA vkt muls ckr djsa ;k feyus tk;saA

A factory manufactures a chemical product with three ingredient chemicals A, B and C as per the standard data given below:

Standard costing is principally based on formulae. The students must memorise various formulae relating to variances thoroughly. The solutions to the problems on standard costing are generally methodical in nature. Once the formula is known, the problem becomes quite easy.

While standard prices of the chemical A,B and C are given as Rs. 40. Rs. 60 and Rs. 95 kg respectively, the actual price is to be computed with the given data.

Page 399: cost accounting question icwai inter

Material mix variance = A (0.50 x 10,500 − 5,200) x Rs. 40 2,000 fav.B (0.30 x 10,500 − 3,600) x Rs..60 (27,000) adv.C (0.20 x 10,500 − 1,700) x Rs. 95 38,000 fav.

Total material mix variance 13,000 fav.(4)Yield varianceYield variance = (Standard yield from actual input − Actual output ) x Standard cost of finished product

= ( 10,500 x 0.95 − 10,200) Rs. 60 (13,500) adv.(5) Usage variance :Material Usage variance = Std. cost of actual output − Std. cost of actual input

= 10,200 x 60 − (5,200 x 40 + 3,600 x 60 + 1,700 x 95) 26,500 fav.Usage variance = Mix variance + Yield variance = 13,000 + 13,500 26,500Total material cost variance = Material price variance + Material usage variance

= 26,500 − 26,200 = Rs. 300 favourableProblem 2:(cwa inter dec.04)

Solution: fj'rksa dks ejus uk nsaA vkt gh fdlh dks Qksu djsaAOverhead expenditure variance = Std. cost of actual overhead − Actual cost of actual overhead

= 1800 x 2 − 4200 = Rs. 600 adv,Variable overhead efficiency variance = (std. time for actual output − Actual time) Std.VOH

= (400 x 5 − 1,800) x Rs. 2 = Rs. 400 fav.Problem 3:(cwa inter june.04)

Rs.Material − 100 kg. @ Rs. 10 per kg. 1,000Labour− 40 hours @ Rs. 20 per hour 800Variable overhead @ Rs. 10 per standard labour hour 400Fixed factory overhead @ Rs. 5 per standard labour hour 2,000The following operating data were taken for May 2005:(i) 500 units were manufactured.(ii) Normal volume is 220 direct labour hours.(iii) 520 kgs of material @ Rs. 11 were consumed.(iv) 190 labour hours @ Rs. 19 were used.(v) Actual variable factory overhead Rs.. 2090(vi) Actual fixed factory overhead−Rs. 1,150You are required to calculate different cost variances.Solution:Material Cost Variances = Std cost of actual output − Actual cost of output

= ( 500 x 10) − (520 x 11) =Rs.720 adv.Material cost variance can also be found out by the following formula:Material cost variance = material price variance + material usage varianceMaterial price variance = Std.price of actual output − actual price of actual output

= (520 x 10 − 520 x 11) = Rs. 520 adv.Material Usage variance = Std. cost of actual output − Std. cost of actual input

= (10 x 500) − (10 x 520) = Rs. 200 adv.Material cost variance = Rs. 520 adv + Rs. 200 adv = Rs. 720 adv. (same as above)Labour cost variance = Std. labour cost of actual output − Actual labour cost of actual output

= (800 x 5 − 190 x 19) = Rs. 390 fav.Labour cost variance can also be found out by using the following formula:Labour cost variance = Labour rate variance + Labour efficiency variance

The standard variable overhead cost of a product is Rs. 10 ( 5 hours @ Rs. 2 / hr) In a certain month it took 1800 hours at a cost of Rs. 4200 to manufacture 400 units. Find the variable overhead expenditure and efficiency variances.

ZED Ltd. has a standard costing system for its single output. Their standard cost for 100 units produced is as follows:

Page 400: cost accounting question icwai inter

Labour rate variance = Actual labour with std. rate − actual labour with actual rate= 190 ( 20 − 19) = Rs. 190 fav.

Labour efficiency variance = std. hours with std. rate − actual hours with std. rate= (200 x 20 − 190 x 20) = Rs. 200 fav.

Labour cost variance = 190 + 200 = Rs. 390 fav.(same as above)variable overhead cost variance = standard cost of voh for actual output − actual cost of actual output= ( 400 x 5 − 2090) = Rs. 90 advProblem 4:(cwa inter dec..04)

Raw materials Percentage of input Cost per kg.in Rs. X 40% 40Y 40% 60Z 20% 85

Note: Loss during the process is 5% of the input having no realizable value.During a certain period 580,000 Kg of finished product was obtained from inputs as per details given below:Raw materials Quantity consumed in kg Cost per kg.in Rs.

X 240,000 38Y 250,000 59Z 110,000 88

Calculate all the relevant variances with details of sub variances relating to price, mix, yield and usage.Solution:Statement showing the calculation for the standard cost of finished product:Raw materials % of input Quantity in kg. Cost per kg.in Rs. total Rs.

X 40% 40 40 1,600Y 40% 40 60 2,400Z 20% 20 85 1,700

Total quantity of input 100 kg. 5,700Less loss in the processing @ 5% 5 kg.Output in kg. 95 kg.Total cost of production in Rs. 5700 Rs. Cost per kg in Rs. (5,700 / 95) 60 Rs.Computation of variances(1)Total material cost varianceTotal material cost variance = Std.cost of output − Actual cost of outputStd. cost of output = 580,000 x 60 = Rs. 348 lakhsActual cost of output Input Cost per kg.in Rs. total

units Rs. Rs. LacsX 240,000 38 91.20Y 250,000 59 147.50Z 110,000 88 96.80

335.50Total cost of material cost variance = Rs. (348.00 − 335.50 ) = 12.50 lakhs (fav.)(2)Material price varianceMaterial price variance = Std.price of actual output − actual price of actual outputMaterial price variance = (Std. price − Actual price ) Actual input consumed

Input Std.price Act. Price Difference Varianceunits Rs. Rs. Rs. Rs. Lacs

X 240,000 40 38 2 4.80Y 250,000 60 59 1 2.50Z 110,000 85 88 (3.0) (3.3)

Material price variance 4.00 (fav.)(3)Material mix variance:Material mix variance = ( Actual input in standard proportion − Actual input ) x Standard cost of input

A product is manufactured by mixing and processing three raw materials X,Yand Z as per the standard data given below:

Page 401: cost accounting question icwai inter

Total input = 600,000 kg.Input in std. proporti actual proportion Differ. Std. cost Variance 6 x 0.40 2.4 2.4 0.00 40 0.006 x 0.40 2.4 2.5 (0.10) 60 (6.00)6 x 0.20 1.2 1.1 0.10 85 8.50

Material mix variance: 2.50 fav.(4) Yield variance :(Actual yield from input − Std. yield from input) x Std. cost of outputYield variance is positive or favourable if actual yield is more than std. yield.Actual yield = 580,000 kg.Standard yield = (600,000 x 0.95 ) = 570,000 kg.Yield variance = (580,000 − 570,000 ) x 60 = Rs. 6.00 lakhs (fav.)(5) Usage variance :Usage variance = Std. cost of output − Std. cost of inputStandard cost of output = 580,000 x Rs. 60 = Rs. 348.00 lakhsStandard cost of input

Input Std.cost Total costX 240,000 40 96.00 Rs. LacsY 250,000 60 150.00 Rs. LacsZ 110,000 85 93.50 Rs. Lacs

Total 339.50 Rs. LacsUsage variance = Rs. ( 348.00 − 339.50) = 8.50 lakhs (fav.)Check : Usage variance = Mix variance + yield variancRs. (6.00 + 2.50) = Rs. 8.50 lakhs (fav)Check : Cost variance = Price variance + Mix variance + Yield variance Cost variance = 4.00 + 2.50 + 6.00 = Rs. 12.50 lakhs fav. (as found above)Problem 5:(cwa inter June 03)

Solution:This is a simple question provided you have all the formulae of variances.First of all standard cost of output is to be known. This will be useful in computing the material cost variance.Computation of standard of product 'Z'.Material Standard ratio Rate (Rs./kg) Amount Rs.A 0.6 20.0 12.0B 0.4 30.0 12.0

1.0 24.0Material cost variance:Material Cost Variances = Std cost of actual output − Actual cost of outputStandard cost of actual out put = 24 x 2,200 = 52,800 Rs.Actual cost of output = (25,000 + 28,000) = 53,000 Rs.Material cost variance = ( 52,800 − 53,000) = Rs. 200 adv.Material Price variance:Material price variance = Std.price of actual output − actual price of actual outputMaterial price variance = (Std. price − Actual price ) Actual input consumed

A = ( Rs. 20 − 25,000 / 1200 ) x 1,200 = Rs. 1,000 (adv)B = ( Rs. 30 − 28,000 / 1,000 ) x 1,000 = Rs. 2,000 (fav.)

Material price variance = 1,000 fav.Material mix variance:Material mix variance = ( Actual input in standard proportion − Actual input ) x Standard cost of inputMaterial mix variance for A = ( 60% of 2,200 − 1,200 ) x 20 = Rs. 2,400 fav.Material mix variance for A = ( 40% of 2,200 − 1,000 ) x 30 = Rs. 3,600 adv.

Raw materials 'A' and 'B' having standard costs of Rs. 20/ kg and Rs. 30/kg are mixed in the standard ratio of 60% and 40% to manufacture 'Z'.

During a particular week 1,200 kg of 'A' costing Rs. 25,000 and 1,000 kg of 'B' costing Rs. 28,000 were mixed to produce 2,200 kgs of 'Z'. Calculate all material cost variances.

Page 402: cost accounting question icwai inter

Material mix variance = Rs. 1,200 adv.Check : Material cost variance = Material price variance + Material mix variance.Problem 6:(cwa inter Dec.03)

Solution:Material cost variance = material price variance + material usage variance

= 4,800 (A) + 4,000 (F) = Rs. 800 (A)Material cost variance = Standard cost of production − Actual cost of production Actual cost of production = 9,600 x 10.50 = 100,800 Rs. Standard cost of actual production = 100,800 − 800 = Rs. 100,000Problem 7:(cwa inter Dec.03)

During the week the firm achieved the following output:Tables 8 nos. Chairs 8 nos. Desks 9 nos.

The annual fixed overhead amounted to Rs. 75,000.1. Fixed production overhead total variance.2. Fixed production overhead expenditure variance.3. Fixed production overhead volume variance.Solution:Standard FOH rate has not been given directly. Standard time for production = 140 hours. Standard overhead for the week = Rs. 70,000.Thus the standard fixed overhead per hour = 70,000 / 140 = Rs. 500.Standard hour of actual production can be computed as follows:Tables = 8 x 4 = 32 hours. Chairs = 8 x 2 = 16 hours. Office desks = 9 x 8 = 72 hours.Standard hour of actual production = 32 + 16 + 72 =120 hours.1. Fixed production overhead total variance.= (Standard FOH − Actual FOH ) of actual production= Rs. 120 x 500 − Rs. 75,000 = Rs. 15,000 adv.2. Fixed production overhead expenditure variance = Budgeted FOH − Actual FOH= Rs. 70,000 − Rs. 75,000 = Rs. 5,000 adv.3. Fixed production overhead volume vari ( Standard FOH of actual production − Budgeted FOH= Rs. 120 x 500 − Rs. 70,000 = Rs. 10,000 adv.Reconciliation : The variance as found above should also satisfy the following formulaStandard FOH of actual production = Actual FOH + Expenditure variance + Volume variance.Actual FOH = 75,000. Expenditure variance = − 5,000, and Volume variance = − 10,000Standard FOH of actual production = 75,000 − 5,000 − 10,000 = 60,000 as calculated as 120 x 500.Problem 8:(cwa inter June 02)

Material Quantity in standard proportion in kg Standard price in Rs. /kgA 5 200B 20 50C 25 20D 50 7Total input 100Loss on processing 5Output in kg. 95During April 2002, 18,500 kg of insecticide was produced incurring actual cost as follows:Chemical A B C DMaterial in kg. 1,010 4,200 4,800 10,200Cost in Rs. 210,000 205,000 100,000 68,000

In a factory 9,600 kg of material at Rs. 10.50/kg were actually consumed resulting in a price variance of Rs. 4,800 (A) and usage variance of Rs. 4,000 (F). The standard cost of actual production is Rs. ………………..

Kolkata furniture manufactures modular tables, chairs and office desks. The standard labour times required per unit of table, chair and desk are 4 hours, 2 hours and 8 hours respectively. The budgeted production per week is 140 standard hours and budgeted fixed overhead is Rs. 70,000.

A certain insecticide is manufactured by mixing four chemicals A,B,C,and D (filter) and processing the same. The standard cost data for the product is as follows:

Page 403: cost accounting question icwai inter

Calculate the following variance duly reconciling them:

Solution:This is formula based question. If you remember the formula, the application is quite easy.(a) Material cost variance: Material Cost Variances = Std cost of actual output − Actual cost of outputStandard cost of actual output can be computed as follows:Material Quantity in standard proportion in k Standard price in Rs. /k AmountA 5 200 1,000B 20 50 1,000C 25 20 500D 50 7 350Total input 100 2,850Loss on processing 5Output in kg. 95 2,850Standard cost of production per kg = Rs. 2,850/ 95 30 Rs.Standard cost of actual output (18,500 kg) = 18,500 x Rs. 30 = Rs.555,000Actual cost of outpChemical A B C D Total

Material in kg. 1,010 4,200 4,800 10,200 Err:522Cost in Rs. 210,000 205,000 100,000 68,000 Err:522

Material Cost Variances = Std cost of actual output − Actual cost of outputMaterial Cost Variances = Rs. 555,000 − 583,000 = Rs. 28,000 (Adv.)( b) Material price variance = Std.price of actual output − actual price of actual output

= Actual output ( Standard price − Actual price )A 1,010 x 200 − 210,000 = 8000 Adv.B 4,200 x 50 − 205,000 5,000 FavC 4,800 x 20 − 100,000 4,000 Adv.D 10,200 x 7 − 68,000 3,400 Fav.Material price variance 3,600 Adv.(c) Material mix variance = ( Actual input in standard proportion − Actual input ) x Standard cost of input

A (20,210 x 5/100 −1,010) x Rs. 200 100 Fav.B (20,210 x 20 / 100 − 4,200) x Rs. 50 7,900 Adv.C (20,210 x 25/100 − 4,800) x Rs. 20 5050 Fav.D (20,210 x 50/100 − 10,200) x Rs. 7 665 Adv.Material mix variance 3,415 Adv.(4) Yield variance (Standard yield from input − Actual yield from input) x Std. cost of output

( 20,210 x 0.95 − 18,500) x Rs. 30 20,985 Adv.(5) Usage variance = Std. cost of output − Std. cost of inputUsage variance = (Standard quantity − Actual Quantity consumed) x Standard priceA (18,500 x 5/95 − 1,010) x Rs. 200 7,263 Adv.B (18,500 x 20/95 − 4,200) x Rs. 50 15,263 Adv.C (18,500 x 25/95 − 4,800 x Rs. 20 1,368 Fav.D (18,500 x 50/95 − 10,200) Rs. 7 3,242 Adv.Total usage variance 24,400 Adv.Reconciliation: Material cost variance = Material price variance + Material mix variance + Material yield variance.Material cost vari Rs. 28,000 Adv. Material price variance 3,600 Adv.

Material mix variance 3,415 Adv.Material yield variance 20,985 Adv.Total 28,000 Adv.

Material usage variance = Material mix variance + Material yield variance Material usage var Rs. 24,400 Adv. Material mix variance 3,415 Adv.

Material yield variance 20,985 Adv.

(a) Material cost variance (b) Material price variance (c) Material mix variance (d) Material yield variance (e) Material usage variance.

Page 404: cost accounting question icwai inter

Total 24,400 Adv.Standard cost of material for actual output Rs. 555,000Less : Favourable variance 0Add: Adverse variance Material price variance Rs. 3,600 Adv.Material mix variance Rs. 3,415 Adv.Material yield variance Rs. 20,985 Adv.Total Rs. 28,000 Adv. 28,000Actual cost actual output 583,000 (same as given)Problem 9:(cwa inter Dec. 01)

Budget Actuals Calculate:Sales (units) 80,000 64,000 (a) the budgeted profit and actual profit Selling price in Rs. per unit 100 110 (b) reconcile the two by working out Raw materials in kg/unit 5.0 5.6 the following variancesPrice of raw materials in Rs./k 8.0 7.5 (i) Sales price varianceDirect labour hours per unit 1.0 0.8 (ii) Sales volume varianceWage rate per hour 20.0 22.0 (iii) Material price varianceFixed overhead per unit 10.0 9.6 (iv) Material usage variance

(v) Labour-rate variance(vi) Labour-efficiency variance(vii) Variable overhead variance

Solution: (viii) Fixed overhead variance.Calculation of various variances :(i) Sales price variance : Actual units sold ( Actual price − Budgeted price)

64,000 x (110 − 100) = 640,000 Fav.(ii) Sales volume variance : Standard margin ( Actual sales unit − Budgeted sales unit)

25 x ( 64,000 − 80,000) = 400,000 Adv.(iii) Material price variance : Actual quantity used ( standard price − Actual price)

64,000 x 5.6 ( 8.0 − 7.5) = 179,200 Fav.(iv) Material usage variance : Standard rate ( Standard quantity in actual output − Actual output)

8 x ( 64,000 x 5.0 − 64,000 x 5.6) = 307,200 Adv.(v) Labour rate variance : Actual hours ( Standard rate − Actual rate)

64,000 x 0.8 ( 20 − 22) = 102,400 Adv.(vi) Labour efficiency variance : ( Standard hours for actual output − Actual hours ) Standard rate

(64,000 x 1 − 64,000 x 0.8) x 20 =256,000 Fav.(vii) Variable overhead variance :(Budgeted overhead for actual output − actual output)

64,000 ( 5.0 − 4.8) = 12,800 Fav.(viii) Fixed overhead variance : Budgeted FOH for actual output − Actual output

64,000 ( 10.0 − 9.60) = 25,600 Fav.Statement showing budgeted and actual profit for the year 2000 − 01Particulars Budgeted ActualSales (units) 80,000 64,000Selling price in Rs. /unit 100 110Less Cost per unitRaw materials 40.0 42.0Direct labour 20.0 17.6Variable overhead 5.0 4.8Fixed overhead 10.0 9.6Total cost per unit 75.0 74.0Profit per unit 25.0 36.0Total profit in Rs. Lakh. 20.0 23.04Statement showing reconciliation of budgeted profit with actual profit

The details of budget and actual of a company manufacturing and selling a single product for the year 2000-01 are as follows:

Page 405: cost accounting question icwai inter

Budget profit Rs. 20,00,000Add: Favourable variances Rs.

Sales price variance 640,000Material price variance 179,200Labour efficiency variance 256,000Variable overhead variance 12,800Fixed overhead variance 25,600 11,13,600

Less: Adverse variancesSale volume variance 400,000Material usage variance 307,200Labour rate variance 102,400 8,09,600

Actual profit 23,04,000 (same as calculated earlier)Problem 10:(cwa inter Dec. 00)The standard cost of a chemical mixture 'AB' is 40% of material A at Rs. 400 per kg.

60% of material B at Rs. 600 per kg.A statement loss of 10% is anticipated in production.The following particulars are available for the month of Dec. 99180 kg of material A has been used at Rs. 360 per kg.220 kg of material B has been used at Rs. 680 per kg.The actual production of AB is 369 kg.Calculate: Material cost variance, price, usage, mix, and yield variance.Solution This can be easily solved on the lines of Problem 5 of this chapter. The answers are given.Material cost variance : (214,400 − 213,200) = Rs. 1,200 AMaterial price variance : (214,400 − 204,000) = Rs. 10,400 AMaterial usage variance : ( 204,000 − 213,200) = Rs. 9,200 F.Material mix variance : ( 204,000 − 208,000) = Rs. 4,000 F.Material yield variance : ( 208,000 − 213,200) = Rs. 5,200 F.Problem 11:(cwa inter June. 00)From the following information about sales: Calculate:(i) Total sales variance (ii) Sales Price variance (iii) Sales volume variance (iv) Sales mix variance (v) Sales quantity variance.Product Standard Actual

Unit Rate Rs. Unit Rate Rs. A 5,000 5 6,000 6B 4,000 6 5,000 5C 3,000 7 4,000 8Solution :Total Sales variance = Standard Sales − Actual Sales =( 70,000 − 95,000) = Rs. 25,000 Fav.Sales Price variance = ( Standard price − Actual Price ) x Actual SalesProduct A = ( 5 − 6 ) x 6,000 6,000 Fav.Product B = ( 6 − 5) x 5,000 5,000 Adv.Product C = ( 7 − 8 ) x 4,000 4,000 Fav.Sales price variance 5,000 Fav.Sales volume variance = ( Standard quantity − Actual quantity ) Standard rateProduct A = ( 5,000 − 6,000 ) x 5 5,000 Fav.Product B = ( 4,000 − 5,000) x 6 6,000 Fav.Product C = ( 3,000 − 4,000) x 7 7,000 Fav.Sales volume variance 18,000 Fav.Sales Mix variance = (Revised standard quantity − actual quantity) Standard PriceProduct A = (6,250 − 6,000) x 5 1,250 Adv.Product B = ( 5,000 − 5,000 ) x 6 0Product C = ( 3,750 − 4,000) x 7 1,750 Fav.Sales Quantity variance = (Standard quantity − Revised Standard quantity) x Standard priceProduct A = ( 5,000 − 6,250 ) x 5 6,250 Fav.

Page 406: cost accounting question icwai inter

Product B = ( 4,000 − 5,000) x 6 6,000 Fav.Product C = ( 3,000 − 3,750 ) x 7 5,250 Fav.Sales Quantity variance 17,500 Fav.Problem 12:(cwa inter June. 06)The summarized budget and actual working results of Gemco ltd. For the year 2005 − 06 are given below;Details Budget Actual

A B C A B C Selling price per unit Rs. 12 16 25 13 16 27Cost per unit Rs. 9 11 20 10 12 21Sales in units 40,000 32,000 24,000 42,000 40,000 22,000Analyze the result and calculate the following :(i) Budgeted profit, actual profit and variance in profit;(ii) Analysis of the variance in profit into the following (a) Price variance (b) Cost variance (c) Sales margin volume variance (d) Sales margin mix variance (e) Sales margin quantity variance Solution:Details Budget Actual

A B C A B C Selling price/unit 12 16 25 13 16 27Cost per unit Rs. 9 11 20 10 12 21Profit per unit Rs. 3 5 5 3 4 6Sales in units 40,000 32,000 24,000 96,000 42,000 40,000 22,000Total actual unit 104,000Total profit 120,000 160,000 120,000 400,000 126,000 160,000 132,000Total actual profit 418,000Variance in profit = ( Budgeted profit − Actual profit ) = ( 400,000 − 418,000 ) = Rs. 18,000 fav.(i) Cost variance: Cost variance = (Std.cost − Actual cost ) Actual output(ii) Price variance: Price variance = ( Std. price − Actual price ) Actual output

Cost variance Price Variance A = ( 9 − 10 ) x 42,000 42,000 Adv. A = ( 12 − 13 ) x 42,000 42,000 fav.B = ( 11 − 12 ) x 40,000 40,000 Adv. B = ( 16 − 16 ) x 40,000 0C = ( 20 − 21 ) x 22,000 22,000 Adv. C = ( 25 − 27 ) x 22,000 44,000 fav.Total cost variance 104,000 Adv. Total price variance 86,000 fav.Sales margin volume variance = ( Budgeted vol. − Actual vol. ) x Standard profitA : ( 40,000 − 42,000 ) x 3 6,000 fav.B : ( 32,000 − 40,000 ) x 5 40,000 fav.C : ( 24,000 − 22,000 ) x 5 10,000 Adv.Sale margin volume variance = Rs. 36,000 fav.Profit variance = Cost variance + Price variance + Volume variance Profit variance = 104,000 adv. + 86,000 fav. + 36,000 fav. = Rs. 1same as aboveSales margin mix variance = (Revised std. qty. − Actual qty.) Standard profitA : ( 40/ 96 x 104,000 − 42,000 ) x 3 4,000 adv.B : ( 32 / 96 x 104,000 − 40,000 ) x 5 26,667 fav.C : ( 24 / 96 x 104,000 −- 22,000 ) x 5 20,000 adv. 2,667 fav.Sales margin quantity variance = ( Revised std. quantity − Budgeted quantity ) x Standard ProfitA : ( 40/ 96 x 104,000 − 40,000 ) x 3 10,000 fav.B : ( 32 / 96 x 104,000 − 32,000 ) x 5 13,333 fav.C : ( 24 / 96 x 104,000 −- 24,000 ) x 5 10,000 fav. 33,333 fav.Reconciliation: Sales margin mix variance 2,667 fav.

Sales margin quantity variance 33,333 fav.Sales margin volume variance 36,000 fav.(same as above)

Problem 13:(cwa inter Dec. 0A chemical is manufactured by combining two standard items of input

A (standard price Rs. 60/kg) and B (Rs. 45/kg) in the ratio 60 : 40. 10% of input is lost during processing. If during a month 1200 kg of the chemical is produced incurring a total cost of Rs. 69,600. The total material cost variance will be ….

Page 407: cost accounting question icwai inter

Solution: Standard cost of output is calculated as follows:Rs.

Material A : 60 kg @ Rs. 60 / kg 3,600Material B : 40 kg @ Rs. 45 / kg 1,800Output ( 100 kg − loss @ 10% ) 90 kg 5,400Standard cost of output Rs. 5400 / 90 = Rs. 60/ kgStandard cost of actual output of 1,200 kg : 1,200 x 60 Rs. 72,000Actual cost actual output Rs. 69,600Material Cost variance: Cost variance = (Std.cost − Actual cost ) Actual outpu Rs. 2400 fav.Problem 14:(cwa inter Dec. 06)

Skilled Semi-skilled UnskilledNo. of workers in the gang 6 8 6Standard rate of wage Rs. / hr. 25 20 16

Skilled Semi-skilled UnskilledNumber engaged 8 6 6Actual rate Rs. / hr 30 24 16Calculate the following variance related to labour (i) cost (ii) rate (iii) efficiency (iv) mix (v) idle time variance.Solution:1. Standard labour cost :Type no. rate Rs./hr Day-hr Cost Rs. Skilled workers 6 25 100 x 8 120,000Semi skilled 8 20 100 x 8 128,000Unskilled 6 16 100 x 8 76,800

324,800Standard gang time 800 hoursStandard gang rate / hr (Rs. 324,800 / 800 ) Rs. 406 / hr.2. Actual Labour cost :Type no. rate Rs./hr Day-hr Cost Rs. Skilled workers 8 30 104 x 8 199,680Semi skilled 6 24 104 x 8 119,808Unskilled 6 16 104 x 8 79,872

399,360

Type no. rate Rs./hr Day-hr Cost Rs. Skilled workers 8 25 104 x 8 166,400Semi skilled 6 20 104 x 8 99,840Unskilled 6 16 104 x 8 79,872

346,112

Type no. rate Rs./hr Day-hr Cost Rs. Skilled workers 6 25 104 x 8 124,800Semi skilled 8 20 104 x 8 133,120Unskilled 6 16 104 x 8 79,872

337,792331,296

Actual hour of working : 104 x 8 − 16 (stoppage due to rains) = 816 hours :@ Rs. 406 / hr(a) Labour Cost Variance : ( 331,296 − 399,360) Rs. 68,064 adv.( Standard cost of actual hours, utilised for completion of work − Actual labour cost)(b) Labour rate variance : ( 346,112 − 399,360) Rs. 53,248 adv.

A building can be constructed by engaging a gang of workers as per the details below, for 100 working days of eight hours each. The standard data:

Actual completion of the work however, took 104 days of eight hour each. This includes 16 hours of stoppages due to heavy rains. The actual number of workers engaged and the actual rates paid are given below

3. Statement labour cost for actual hours of actual gang :

4. Standard labour cost for actual hours of actual gang :

5. Standard labour cost for actual hours utilized for completion of work:

Page 408: cost accounting question icwai inter

(Statement labour cost of actual hours of actual gang − Actual labour cost)(c) Labour efficiency variance ( 346,112 − 331,269) Rs. 14,816 adv.(Std. L.cost of actual hours − Standard cost of actual hours, utilised for completion of work)(d) Labour mix variance : (337,792 − 346,112) Rs. 8,320 adv.(Std. labour.cost of actual hours in std. mix− Standard cost of actual hours)(e) Idle time variance : Idle time x Standard rate = 16 x 406 Rs. 6,496 adv.Summary Rs.Labour cost variance 68,064 adv. Check :Labour rate variance 53,248 adv. Efficiency variance = Mix V. + Idle time V.Labour efficiency variance 14,816 adv. (8,320 + 6,496 ) = 14,816Labour mix variance 8,320 adv.Idle time variance 6,496 adv.Problem 15:(cwa inter II June. 05)In a manufacturing process the following standards apply:Standard prices : Raw materials A : Rs. 10 / kg. B : Rs. 50 / kgStandard mix : 75% of A and 25% of B by weightStandard output ( weight of product as a percentage of weight of raw materials 90%In a particular period actual costs, usages and output were as follows :Input : 4,400 kgs. of A costing Rs. 46,500 and 1,600 Kg. of B,costing Rs. 78,500 Output : 5670 kg. of product. The budgeted output for the period was 7,200 kgs.Required : Compute the material cost variances.Solution:Standard yield from the actual input 6,000 kg (4,400 + 1,600) kg. of raw materia Rs.Material A 75%, 4,500 kg. @ Rs. 10 45,000Material B 25%, 1,500 kg. @ Rs. 50 75,000Output : (6,000 − normal loss of 10%) 5,400 kg. 120,000Standard cost of output = Rs. 120,000 / 5,400Standard cost of actual output of 5,670 kg. = 120000/5400 x 5670 126,000Actual cost of actual output A = Rs. 46,500 and B = Rs. 78,500 125,000Material Cost Variances = Std cost of actual output − Actual cost of output = 126,000 − 125,000Material cost variance 1,000 fav.Material price variance = Std.price of actual output − actual price of actual outputMaterial A = (10 − 46500/4400) x 4400 2,500 adv.Material B = (50 − 78500/1600) x 1600 1,500 fav.Material price variance 1,000 adv.Student should calculate the other variances. The answers are as follows.Usage variance = Rs. 2000 fav. Mix variance = Rs. 4000 adv. Yield variance Rs. 6000 fav.Problem 16:(cwa inter I June. 08)

Calculate the following variance related to labour (i) cost (ii) rate (iii) efficiency (iv) mix (v) idle time variance.Solution : This is similar to problems solved earlier. The answers are given hereunder.(i) Labour wage variance : Rs. 25,600 A; (ii) Labour rate variance : Rs. 16,000 F(iii) Labour eff. variance : Rs. 41,600 A (iv) Mix variance : Rs. 10,800 A (v) Idle time variance : Rs. 16,000 A.

Standards fixed Actual performance for April 00

A group of workers consisting of 30 men above 30 years of age, 15 females above 30 years of age, and 10 youth of age between 20-30 are paid standard hourly rate as follows : Males : Rs. 80/hr , Females : Rs. 60/hr and Youth : Rs. 40/hr.

In a normal week of 40 hours, the group is expected to produce 2,000 units of output. During a week, the group consisting of 40 males, 10 females and 5 youth produced 1,600 units. They were paid wages @ Rs. 70 for males, Rs. 65 for females and Rs. 30 for you per hour. 4 hours were lost due to abnormal idle time.

Problem 17 (cwa inter II June 00)

C Ltd. Produced a popular consumer durable. The details of standards fixed for this product and the actual performance details for April 2000 are given below:

Page 409: cost accounting question icwai inter

Direct materials: 4 kg @ Rs.12/kg Production/ Sales 9,600 unitsDirect labour: 2 hrs @ Rs.15/kg Sales value Rs. 14 lakhsVari. overhead : 2 hrs @ Rs.10/hr Direct materials 42,000 kg Rs. 480,000

Direct labour 18,000 hrs. Rs. 275,000Vari. Overhead Rs. 210,000Fixed overhead Rs. 120,000

Budgeted fixed overhead p.m. for a monthly output of 10,000 units Rs. 100,000Profit margin 25% of sales

No sub-variance under overheads are required.Solution:Tutorial Notes:

The solution goes as follows:Calculation of variances:

For calculation of variance the standard cost and the selling price per unit should be calculated first.Components Rate Rs. Quantity Amount in Rs. Direct material 12 4 kg 48Direct Labour 15 2 kg 30Variable overhead 10 2 hrs 20Fixed overhead per unit 10Total Standard cost 108Profit 1/3 rd of cost 36Standard price 144Sales price variance : ( Standard price − Actual price ) x Actual salesNote: you should not calculate the actual price.Sales price variance : ( 144 − 14,00,000 / 9,600 ) x 9,600 Rs.17,600 fav.Note: If actual sales price is more than the standard price, it is favourable variance.Sales margin volume variance: ( Budgeted qty − Actual qty) Standard margin

= ( 10,000 − 9,600) x 36 Rs.14,400 adv.Material usage variance : ( Standard qty − Actual qty) Standard price

= (9,600 x 4 − 42,000) x 12 Rs. 43,200 adv.Material price variance : (Standard price − Actual price ) x Actual Quantity usedNote: you should not calculate the actual price.Material price variance : (12 − 480,000 / 42,000 ) x 42,000 Rs. 24,000 fav.Note: If actual price of material is less than standard price, it is fav. variance.Labour efficiency variance: (Std time for actual production −- Actual time) x Std. rate( 9,600 x 2 − 18,000 ) x 15 = Rs. 18,000 fav.Labour rate variance : (Standard rate − actual rate ) x actual hours

= ( 15 − 275,000 / 18,000 ) x 18,000 Rs. 5,000 adv.Variance overhead variance : Rs.Standard variable overhead for actual production = Rs. 20 x 9,600 192,000Actual variable overhead incurred 210,000Since actual is more, it is adverse variance 18,000Fixed overhead variance : 9,600 x 10 − 1,20,000 = Rs. 24,000 adv.Summary of variancesVariances Amount in Rs.

Calculate the budgeted profit the actual profit, and reconcile the two in a standard format giving clearly the variances relating to sales, materials, labour and overheads.

Budgeted profit can be computed merely using mental mathematics. The profit margin is 25% of sales i.e. to say profit margin is 1/3 rd of cost. The standard cost is ( 12 x 4 + 2 x 15 + 2 x 10 + 100,000/ 10,000) = Rs. 108. Thus profit is 1/3rd of Rs. 108 i.e. Rs. 36 per unit. For 10,000 units the budgeted profit comes to be Rs. 360,000.

Students are advised to revise their knowledge of variances. The formulae of variances need to be memorized.

Page 410: cost accounting question icwai inter

Sales price variance fav. 17,600Sales volume variance adv. 14,400Material usage variance adv. 43,200Material price variance fav. 24,000Labour efficiency variance 18,000Labour rate variance 5,000Variable overhead variance adv. 18,000Fixed overhead variance adv. 24,000

59,600 104,600Budgeted profit 360,000Add favourable variances 59,600Less adverse variances 104,600Actual profit 315,000

Production and Costs ( Actuals ) Cost variances Rs.Production 10,000 units Material price variance 5,000 (F)Direct materials 105,000 kg. Rs. 520,00 Material usage variance 25,000 (A)Direct labour 19,500 hours Rs. 308,00 Labour rate variance 15,500 (A)Variable overheads Rs. 410,00 Labour efficiency varia 7,500 (F)

Variable overhead varia 10,000 (A) Required : Standard cost data ( both quantity and price ) per unit from the above details.Solution:Material price variance : (Standard price − Actual price ) x Actual Quantity used

5,000 = ( Standard price − 520,000 / 105,000 ) x 105,000Standard price : Rs. 5/ kg.

Material usage variance : ( Standard qty − Actual qty) Standard price( 105,000 x 5 − 10,000 x 5 x Standard quantity ) = 25,000Standard Quantity : 10 kg / unit

Labour rate variance : (Standard rate − actual rate ) x actual hours− 15,500 = (Standard rate − 308,000 / 19,500 ) x 19,500Standard rate : Rs. 15 / hour

Labour efficiency variance: (Std time for actual production −- Actual time) x Std. rate7,500 = ( 10,000 x Standard time − 19,500 ) x 15Standard time : 2 hour / unit

Variable overhead variance : Std. voh cost for actual production − actual variable overhead − 10,000 = Standard variable overhead x 10,000 − 410,000Standard variable overhead : Rs. 40 / unit

Standard cost data are as follows: Rs.Direct material : 10 kg @ Rs. 5 50Direct Labour : 2 hours @ Rs. 15 30Variable Overhead per unit @ Rs. 40 40

120

The summarized budget estimates of ABC Ltd. for the year 2001 are given below along with the actuals:Budgets ActualsRs.lakhs Rs.lakhs

Sales 400 396Cost of sales: Materials 200 209

Labour 60 69Variable Overheads 40 44Fixed overheads 50 57

Total cost 350 379

Problem 18 (cwa inter II dec. 00)

A company operates a standard cost system to control the variable works cost of its only product. The following are the details of actual production, costs and variances for November 2000.

Problem 19 (cwa inter II dec. 02)

Page 411: cost accounting question icwai inter

Profit 50 17

Analyze the variation of profit and present your findings in the following format fig. Rs. LakhsBudgeted profit 50

Fav. Adv.Sales price varianceSales Margin varianceMaterial price variance Material usage variance Labour − rate varianceLabour − efficiency varianceVariable overhead cost varianceFixed overhead variance Net variance 33 Adv.Actual profit 17Solution:

Items Actuals Rise Reduced to budgeted Variance in price price

Sales 396 10% (396x100/110) 360 36 Fav.Material 209 10% (209x100/110) 190 19 Adv.Wages 69 15% (69x100/115) 60 9 Adv.

Material Usage variance = Std. cost of actual output − Std. cost of actual input = [ (360 x 200 / 400) − 190 ] 10 Adv.

Labour efficiency variance = std. hours with std. rate − actual hours with std. rate = [ 360 x 60 / 400] − 60 6 Adv.

Variable overhead variance = Standard cost for actual sales − Actual cost ( 360 x 40 / 400) − 44 8 Adv.

Sales margin ( contribution ) variance :

Fixed overhead variance is simply Rs. 57 lakhs − Rs. 50 lakhs = Rs. 7 lakhs (Adv.)Variance can now be reported as per the given format:Budgeted profit 50

Fav. Adv.Sales price variance 36Sales Margin variance 10Material price variance 19Material usage variance 10Labour − rate variance 9Labour − efficiency variance 6Variable overhead cost variance 8Fixed overhead variance 7Net variance 69 33 Adv.

The company wants a detailed investigation into the fall in profit. During your investigation you find the prices of materials and wage rates have increased by 10% and 15% respectively. There was also an upward revision of selling price by 10%.

The materials, wages and selling price have undergone price changes during the year. The actual items have element of price change in them. First the price variance be computed and the affected items be reduced to the level of budgeted prices to calculate other variances.

For material usage variance, the price rise is to be ignored. Standard ratio of material / sales (200 / 400), as budgeted should be applied to actual output without price rise.

The standard rate of contribution as per budget is 25% i.e. Rs. 100 lakhs on a sale of Rs. 400 lakhs. As the actual sales as per budgeted price is Rs. 360 lakhs the standard contribution should be Rs. 90 lakhs. Contribution lost or the sales margin variance would be Rs. 10 lakhs.(Adv.)

Page 412: cost accounting question icwai inter

Actual profit 17

Rs.Direct materials (6 units at Rs. 3) 18.0Direct labour ( 6 hours at Rs. 2) 12.0Direct expenses 2.0Factory Overheads Variable 1.0

Fixed 1.2Administrative overheads 1.2

35.4Profit per unit 4.6Selling price 40.0Production and sales during the period :7,200 unitsThe following are the variance worked out at the end of the period; (in Rs)Direct materials Fav. Adv.Price variance 8,500Usage variance 2,100Direct labour Rate variance 8,000Efficiency varianc 6400Factory overheadsVariable expenditure variance 800Fixed expenditure variance 800Fixed volume variance 3360Administrative overheadsExpenditure variance 800Volume 3,360

Solution: Vikas LimitedStatement showing standard cost, variance and Actual cost

Standard cost Variance Actual costFav. Adv.

Rs. Rs. Rs. Rs.Direct materials ( 7,200 x 18 ) 129,600Material price variance 8,500Material usage variance 2,100 136,000Direct labour ( 7,200 x 12 ) 86,400Rate variance 8,000Efficiency variance 6,400 88,000Direct Expenses 14,400 14,400Factory overhead : Variable (7,200 x 1) 7,200

Fixed ( 7,200 x 1.2 ) 8,640Variable expenditure variance 800 6,400Fixed expenditure variance 800Fixed volume expenditure variance 3,360 11,200Administrative overhead (7,200 x 1.20 ) 8,640Expenditure variance 800Volume variance 3,360 12,800Total 254,880 10,100 24,020 268,800

Profit and loss account for the period…………..Rs. Rs.

Problem 20 (cwa inter II June 03)

Vikas Ltd. has adopted a Standard Costing System. The standard output for a period is 10,000 units. The standard cost data is given below:

You are required to ascertain the details of costs and prepare the profit and loss account in a statement form for the period, showing actual profit. Reconcile the actual profit with standard profit.

Page 413: cost accounting question icwai inter

Sales revenue ( Rs. 40 x 7,200 ) 288,000Less: Direct materials 136,000

Direct labour 88,000Direct expenses 14,400Factory overheadsVariable 6,400Fixed 11,200Administrative overheads 12,800 268,800Actual profit 19,200Standard profit (7,200 x 4.6 ) 33,120

Reconciliation of standard profit with actual profitStandard profit (7,200 x 4.6 ) 33,120Variance Fav. Adv.

Rs. Rs.Material price variance 8,500Material usage variance 2,100Labour rate variance 8,000Labour efficiency variance 6,400Variable overhead variance 800Fixed overhead Expenditure variance 800Volume variance 3,360Administrative overheadsExpenditure variance 800Volume variance 3360Total 10,100 24,020 (13,920)Actual profit 19,200

Delta Ltd. has a manufacturing division which makes a product to which the following details relate:Rs./ unit

Direct materials : 5 kg. at Rs. 20 100Direct labour : 12 hours at Rs. 20 240Variable overheads : 12 hours at Rs. 10 120

Rs. Direct materials ( 8,500 kg ) 172,000Variable overheads 220,000Direct labour (20,000 hrs.) 420,000Fixed overheads 98,000Total cost 910,000Profit 40,000Sales value 950,0001. Calculate all the variances that occurred during the month ; and 2. Reconcile the actual profit with standard profit, showing the causes of variances.Solution: Delta Ltd. Computation of variances1. Material price variance : (Standard price − Actual price ) x Actual Quantity used

= (20 x 8,500 − 172,000 ) 2,000 adv.2. Material usage variance : ( Standard qty − Actual qty) Standard price

= ( 8,500 / 5 − 1,800 ) x 100 10,000 fav.3. Labour rate variance : (Standard rate − actual rate ) x actual hours

= ( 20 − 420/20 ) x 20,000 20,000 adv.4. Labour efficiency variance = std. hours with std. rate − actual hours with std. rate

Problem 21 (cwa inter II June 04)

Relevant fixed overhead are based at Rs. 100,000 per month and planned output is 2,000 unit per month. The selling price is Rs. 550 per unit. During a recent month when output was 1,800 units, the following actual cost were incurred :( in Rs. )

Page 414: cost accounting question icwai inter

= (standard hours − actual hours ) x standard rate= ( 1,800 x 12 − 20,000 ) x 20 32,000 fav.

5. Variable overhead variance : Std. voh cost for actual production − actual variable overhead = ( 1,800 x 120 − 220,000 ) 4,000 adv.

Variable overhead variance can be segmented into expenditure and efficiency variance as below:5 a. Voh expenditure variance = Actual hours ( Std.rate − actual rate)

= 20,000 x 10 − 220,000 20,000 adv.5 b. Voh efficiency variance = Std. rate x actual production − Actual hrs.x actual production

= 1,800 x 120 − 20,000 x 10 16,000 fav.6. Foh expenditure variance = Budget Fixed overhead −Actual overhead incurred

= 100,000 − 98,000 2,000 fav.7. Foh capacity variance = Budgeted Foh − Foh for actual hours worked

= 100,000 − 83,333* Rs. 16,667 adv.* [ (20,000 x 100,000 ) / ( 12 hrs x 2,000 )] = 83,3338. Foh efficiency variance = Std Foh for actual production − Foh for actual hours

= 100,000 x 1,800 / 2,000 − 83,333 Rs. 6,667 fav.9. Sales margin price variance = ( Std. margin − Actual margin ) on actual sales

Actual selling price per unit 950,000 / 1,800 527.8Standard cost per unit ( 100+240+120+50) 510.00Standard sales margin Rs. 17.8 per unit

Sales margin price variance = 1,800 x 17.78 − 1,800 (550 − 510) 40,000 adv.10. Sales margin volume variance = Standard sales margin ( Standard sales - actual sales)

= 40 x 2,000 − 40 x 1,800 Rs. 8,000 adv.Standard sales margin Rs. 550 − Rs. 510 = Rs. 40 per unit.

Delta Ltd.Reconciliation of profit Rs.

Budgeted profit 2000 x 40 80,000Add: Favourable variances

Material usage variance 10,000Direct labour efficiency variance 32,000Variable overhead efficiency variance 16,000Fixed overhead expenditure variance 2,000Fixed overhead efficiency variance 6,667 66,667

Less: Adverse varianceMaterial price variance 2,000Direct labour rate variance 20,000Variable overhead expenditure variance 20,000Fixed overhead capacity variance 16,667Sales margin price variance 40,000Sales margin volume variance 8,000 (106,667)

Actual profit for the period 40,000

A cost accountant of a company was given the following information regarding the fixed overheads for March' 03:1. Overheads cost variance Rs. 2,8002. Overheads volume variance Rs. 2,0003. Budgeted hours for March 2003, 2,400 hours4. Budgeted overheads for March 2003 Rs. 12,0005. Actual rate of recovery of overheads Rs. 16 per hourYou are required to assist him in computing the following for March 2003.1. Overheads expenditure variance 2. Actual overhead incurred;3. Actual hours for actual production ; 4. Overheads capacity variance;5. Overheads efficiency variance ; 6. Standard hours for actual production Solution:Let us first find out the Standard rate of recovery of overhead rate : Let us say it is SR

Problem 22 (cwa inter II dec. 04)

Page 415: cost accounting question icwai inter

SR = Budgeted Overhead / Budgeted Hours = 12,000 / 2,400 = Rs. 5 per hour.1. Standard hours for actual production :Overhead volume variance = SR ( SH − Budgeted hours )2000(A) = 5 ( SH − 12,000 ) which gives Standard hours = 2,0002. Actual overheads incurred:Overheads cost variance = Std. Variable Overhead − Actual Variable Overhead Standard variable overhead = SH x SR = 2,000 x 5 = Rs. 10,000.Overheads cost variance = Rs. 2,800 adv.Actual Variable Overhead would be Rs. 10,000 + Rs. 2,800 = Rs. 12,800.3. Overhead Expenditure variance :Overheads expenditure variance = Budgeted overhead − Actual overheads Budgeted overheads = Rs. 12,000 Actual overheads = Rs. 12,800Overheads expenditure variance = Rs. 800 (A)Reconciliation: Overhead cost variance = Overhead ( Expenditure + volume ) variance4. Actual hours for actual production : Say AHAH = Actual overheads incurred ( 12,800 ) / Actual recovery rate ( 16 ) = 800 hours.5. Overheads capacity variance = SR (BH − AH) = 5(2400 − 800) = 8,000 Fav.6. Overhead efficiency variance = SR ( SH − AH) = 5 (2000 − 800) = 6,000 Fav.Reconciliation : Volume variance = Capacity variance + Efficiency variance .

Budgeted production of a company is 20,000 units per month. The standard cost sheet is :Direct materials 1.5 kg @ Rs. 6 / kgDirect labour 6 hours @ Rs. 5/hr.Variable overheads 6 hours @ Rs. 4/hr.Fixed overheads Rs. 3 per unit of outputSelling price Rs. 72 per unitActual Details for the monthSales order 21,000 unitsActual sales 18,750 unitsActual production 18,750 unitsDirect materials consumed 29,860 kg at Rs. 5.25/kgDirect labour hours worked 118,125 hours at Rs. 6 per hourActual overheads are Rs. 565,000, out of which a sum of Rs. 40,000 was fixed.(i) Calculate all variances.(ii) Present an operating statement reconciling the budgeted profit with actual profit.Solution:Direct materials variances Usage : Rs. 10,410 A. Price : Rs. 22,395 F.Direct labour variances Efficiency Rs. 28,125 A Rate : Rs. 118,125 AV. overhead variances Efficiency Rs. 22,500 A Expense Rs. 52,500 AF. overhead variances Volume : Rs. 3,750 A Expense Rs. 20,000 ASales variances Volume : Rs. 90,000 A Price : Rs.37,500 A.

Rs.Operating statement : Budgeted profit 120,000

Sales gross margin variance 7,500Standard profit 112,500Price variance 37,500Profit 75,000

Actual Profit and loss account :Sales 18,750 x Rs. 70 1312,500Less : Materials : 156,765, labour : 708,750, Voh: 525,000 Foh : 40,000 1430,515Loss 118,015

The following information has been gathered on an on-going as on 31st Dec.2000.Budgeted cost of work scheduled (BCWS) Rs. 50 lacs

Problem 23 (cwa final Dec.01)

Problem 24 (cwa final Dec.00)

Page 416: cost accounting question icwai inter

Budgeted cost of work performed (BCWP) Rs. 40 lacsActual Cost of Work Performed (ACWP) Rs. 4 lacsBudgeted Cost of Total Work (BCTW) Rs. 100 lacsAdditional Cost of Completion(ACC) Rs. 66 lacsDetermine : (i) Performance Variance (ii) Efficiency Variance (iii) Performance index

(iv) Efficiency Index (v) Estimated Cost of Performance Index.Solution :

(i) Performance Variance = BCWP − BCWS = 40 −50 = − Rs. 10 lakhs(ii) Efficiency Variance = BCWP−ACWP = 40 − 44 = − Rs. 4 lakhs.(iii) Performance Index = BCWP / BCWS = 40 / 50 = 0.80(iv) Efficiency Index = BCWP / ACWP = 40/44 = 0.9091(v) Estimated Cost Performance Index : BCTW/(ACWP+ACC) =100/(44+66) = 0.909

The sales performance of Satya Ltd. a dealer in toy-products for 1996-97 was as follows:Products Tv sets Washing machine Computers TotalUnits 40,000 100,000 180,000 320,000Revenue Rs. 320,000 480,000 500,000 1300,000Standard cost Rs. 200,000 300,000 360,000 Err:522Profit Rs. 120,000 180,000 140,000 Err:522

(i) Sales margin variance is Rs. 33.333.33 (F)(ii) Average standard margin per unit Rs. 1.6667.(iii) Budgeted sales (in units) of washing machines were achieved.(iv) TV sets were sold at the standard selling price.

(v) The std, selling price of TV sets/ unit is as much as a unit each of the other two product lines put together.(vi) Sales price variance is Rs. 60,000 (A).

Solution :Supporting Calculation :(i) Budgeted sales quantity = 320,000 + (33,333.33/1.6667) 300,000 units(ii) Budgeted profit total = 300,000 units x Rs. 1.6667 Rs. 500,000(iii) Computation of Selling price :

Standard selling price of TV sets = Rs. 320,000/40,000 Rs. 8

We have W + C = 8.Adverse sales price variance is given as Rs. 60,000. We have the following equation:40,000 x 8 + 100,000 x W + 180,000 x C = 13,00,000 + 60,000 320,000 + 100,000 x W + 180,000 x (8 − W) = 13,60,000 giving W = Rs. 5 and C = Rs. 3.Standard costs and profit per unit of different products are computed as follows:Products Tv sets Washing machine Computers TotalUnits 40,000 100,000 180,000 320,000Standard cost Rs. 200,000 300,000 360,000 Err:522Standard cost/unit 5 3 2Selling price/unit 8 5 3Profit / unit 3 2 1

Rs.

Problem 25 (cwa final Dec.97)

The company is in the process of producing the budget for 1997−98 and whilst the above actual figures are useful, it would like to know what was in the 1996−97 budget. Unfortunately this information seems to be lost and only one or two crumb of information have come to light i.e. :

Produce the sales budget for 1996−97 showing (a) number of units (b) unit price (c) total cost and (d) total profit for each item and in total. Show supporting calculations.

It is given that the selling price of TV sets is equal to sum of budgeted selling prices of Washing machine and Computer. Let W denote the selling price of Washing machine and C denote the selling price of Computer.

Budgeted sales (in units) of washing machines were achieved. Let T and C denote budgeted sales units of TV and Computers:

Page 417: cost accounting question icwai inter

Budgeted profit of all sales is given as 500,000Less : Budgeted profit of washing machines 200,000Budgeted profit of TV and Computer( 3T + 1C ) 300,000

Total Budgeted quantity of all products in units 300,000 as computed above.or We can write T + C + 100,000 (washing machines) 300,000

T + C = 200,000 and We also have 3T + 1C = 300,000Solving We have T = 50,000 units and C = 150,000 units.

We can now present the Sales Budget for 1996−97 as follows:Products Tv sets Washing machine Computers TotalSelling price in Rs. 8 5 3Standard cost in R 5 3 2Profit Rs./unit 3 2 1Quantity 50,000 100,000 150,000 Err:522Sales Revenue Rs 400,000 500,000 450,000 1350,000Standard cost in R 250,000 300,000 300,000 Err:522Profit Rs. 150,000 200,000 150,000 Err:522Problem 26 (ca final June 09)

Standard cost per unit of X : Material price variance : Rs. 9,800 (Adverse) Materials : 50 kg @ Rs. 40/k.g Material usage variance : Rs. 4,000 (Favourable). Actual production : 100 units Actual material cost : Rs. 42/kgCalculate the actual quantity of material used during the month April, 2009. Solution : Solve yourself.

Actual quantity of material used during the month of April = 4900 Kg.Problem 27 (ca final June 09)Following is the sales budget for the first six months of the year 2009 in respect of PQR Ltd. :

Month Jan. Feb. March April May JuneSales (units) 10,000 12,000 14,000 15,000 15,000 16,000

Each unit of finished product requires two types of materials as detailed below : Material X : 4 kgs @ Rs. 10/kg Material Y : 6 kgs @ Rs. 15/kg

Budgeted direct labour hour per unit of finished product is 3/4 hour. Budgeted direct labour cost for the first quarter of the year 2009 is Rs. 10,89,000. Actual data for the quarter one, ended on March 31, 2009 is as under :Actual production quantity : 40,000 unitsDirect material cost(Purchase cost based on materials actually issued to production)Material X : 1,65,000 kgs @ Rs. 10.20/kgMaterial Y : 2,38,000 kgs @ Rs. 15.10/kgActual direct labo 32,000 hoursActual direct labour cost : Rs. 13,12,000Required :(a) Prepare the following budgets :

(i)         Following details relating to product X during the month of April, 2009 are available :

Material price variance = AQ used (Standard price per kg - Actual price per kg )

Finished goods inventory at the end of each month is expected to be 20% of budgeted sales quantity for the following month. Finished goods inventory was 2,700 units on January 1. 2009. There would be no work-in-progress at the end of any month.

Material on hand on January 1, 2009 was 19,000 kgs of material X and 29,000 kgs of material Y. Monthly closing stock of material is budgeted to be equal to half of the requirements of next month's production.

(i)          Monthly production quantity budget for the quarter one.(ii)         Monthly raw material consumption quantity budget from January, 2009 to April, 2009.(iii)        Materials purchase quantity budget for the quarter one.

Page 418: cost accounting question icwai inter

(b) Compute the following variances : (i) Material cost variance (ii) Material price variance (iii) Material usage variance (iv) Direct labour cost variance (v) Direct labour rate variance (vi) Direct labour efficiency variance.Solution :(i) Statement showing monthly production quantity Budget for the quarter.

Jan Feb Mar

Sale (in unit) 10,000 12,000 14,000Add : Closing stock 2,400 2,800 3,000Less : Opening stock 2,700 2,400 2,800

9,700 12,400 14,200

(ii) Monthly Raw materials consumption quantity budget from January 2009 to April 2009Particula Raw materials Total

X Y Kg.Jan 9700 x 4 38,800 9700 x 6 58,200 97,000Feb 12400 x 4 49,600 12400 x 6 74,400 124,000Mar 14200 x 4 56,800 14200 x 6 85,200 142,000Apr 15000 x 4 60,000 15000 x 6 90,000 150,000

205,200 307,800(iii) Material purchase quantity budget for the quarter

Raw materials X :Consumption Closing stock Opening stock Purchase quantity

kg. kg. kg. kg.Jan 38,800 24,800 19,000 44,600Feb 49,600 28,400 24,800 53,200March 56,800 30,000 28,400 58,400Raw material Y

Jan 58,200 37,200 29,000 66,400Feb 74,400 42,600 37,200 79,800March 85,200 45,000 42,600 87,600Computation of Material variance

SP X SQ SP X SM SP X AQ used AP x AQ usedX 10 x 1,60,000 10 x 1,61,200 10 x 1,65,000 10.20 x 1,65,000Y 15 x 2,40,000 15 x 2,41,800 15 x 2,38,000 15.10 x 2,38,000 52,00,000 52,39,000 52,20,000 52,76,800

Solve yourself on the lines of solved problems above. The answers are supplied below:Material cost variance Rs. 76,800 adverse Material price variance Rs. 56,800Material usage variance Rs.20,000 adverse Direct labour cost variance Rs. 112,000Direct labour rate variance Rs. 32,000 adverse Direct labour efficiency variance Rs. 80,000Problem 28 (ca final may 08)The following information has been extracted from the books of Goru Enterprises which

Actual output = 9,000 unitsDirect wages paid = 1,10,000 hours at Rs.22 per hour, of which5,000 hours, being idle time, were not recorded in productionStandard hours = 10 hours per unitLabour efficiency variance = Rs. 3,75,000 (A)Standard variable Overhead = Rs. 150 per unitActual variable Overhead = Rs. 16,00,000You are required to calculate:(i) Idle time variance (ii) Total variable overhead variance(iii) Variable overhead expenditure variance (iv) Variable overhead efficiency varianceSolution :

Particulars

Production

is using standard costing system:

Page 419: cost accounting question icwai inter

Solve yourself on the lines of solved problems above. The answers are supplied below:Actual output = 9,000 units Idle time = 5,000 hours Production time (Actual) = 1,05,000 hoursStandard hours for actual production = 10 hours / unit x 9,000 units = 90,000 hours.Labour efficiency variance = 3,75,000 (A)i.e. Standard rate x (Standard Production time – Actual production time) = 3,75,000(A).Standard rate = 375,000 / ( 105,000 − 90,000) = Rs. 25(i) Idle time variance = 5,000 hours x 25 Rs. / hour = 1,25,000. (A)(ii) Total Variable Overhead Variance = 2,50,000 (A)(iii) Variable Overhead Expenditure Variance = (Standard Rate x Actual Hours) – (Actual Rate x Actual Hours)

= (15 x 1,05,000) – 16,00,000 = Rs. 25,000 (A)Variable Overhead Efficiency Variance

= Standard Rate x (Standard Hours for actual output – Actual hours for Actual output)15 (90,000 – 1,05,00 = Rs. 225,000 (A)

Small people always try to discourage you, just ignore them.Problem 29: (CWA Inter June 09)The standard process cost card for a processed item is as under :

Rs. Per kg of Finished productDirect material 2 kgs @ Rs. 10 per kg 20Direct labour 3 hours @ Rs. 20 per hour 60Fixed overhead 90Total 170Budgeted output for the period is 1000 kgs.Actual production and cost data for a month are as under :Actual production (on equivalent production basis)Material 1,400 Kgs Labour 1,140 Kgs Overheads 1,140 KgsDirect material : 2900 kgs = Cost Rs. 32,000Direct labour : 3300 hrs = Cost Rs. 68,000Fixed overhead : Rs. 88,000You are required to work out the following variances :(i) Material price and usage variances ; (ii) Labour rate and efficiency variances and (iii) Fixed overhead budget variance.Solution :This is simply formula based question and can be easily solved on the lines of previous problems.Answers are as follows : Material price variance Rs. 3,000 Adverse

Material usage variance Rs. 1,000 AdverseLabour rate variance Rs. 2,000 AdverseLabour efficiency variance Rs. 2,400 FavourableFixed overhead budget variance Rs. 2,000 Favourable

Problem 30: (CWA Inter June 08)

Rs.Direct material 3 units @ Rs. 1.50 4.50Direct labour 3 hours @ Rs. 1.50 3.00Direct expenses 0.50Variable Overheads Factory 0.25Fixed overheads Factory 0.30Administrative Overheads 0.30Total Cost 8.85Profit 1.15Selling price 10.00

The following variances are worked out at the end of the period. Figures in Rs. Favourable Adverse

Shree Balaji Textiles Limited follows standard costing system. The standard output for a month is 20,000 units and standard cost and profit / unit are as follows :

The actual production and sales for the period was 14,400 units. There has been no price revision by the government during the period.

Page 420: cost accounting question icwai inter

Direct material Price 4,250Usage 1,050

Direct labour Rate 4,000Efficiency 3,200

Factory Overheads Variable expenditure 400Fixed expenditure 400Fixed volume 1,680

Administrative Overheads Expenditure 400Volume 1,680

You are required to reconcile the actual profit with the standard output.Solution :

1. All adverse variances to cost will be added to the cost because they show increase in cost over the standard cost.For the actual production of 14,400 units, compute all the revenues and costs.

Rs./unit For 14,400 units Add DeductAdv. var. Fav.var

Direct material 3 units @ Rs. 1.50 4.50 64,800 4,250 1,050Direct labour 3 hours @ Rs. 1.50 3.00 43,200 4,000 3,200Direct expenses 0.50 7,200 0 0Variable Overheads Factory 0.25 3,600 400Fixed overheads Factory 0.30 4,320 1,680 400Administrative Overheads 0.30 4,320 400+1680

Sales revenues 10.00 144,000ProfitStandard profit (20,000 x Rs. 1.15)Reconciliation of actual profit with budgeted profit:Particulars Favourable AdverseSales volume (20,000 – 14,400) x 1.15 6,440Material price variance 4,250Material usage variance 1,050Labour rate variance 4,000Labour efficiency variance 3,200Factory overheadExpenditure variance (fixed) 400Expenditure variance (variable) 400Volume variance (fixed) (20,000 – 14,400) x 0.30 1,680Admin. Overhead expenditure variance 400Admin. Overhead volume variance 1,680

5,050 18,450Budgeted profit 23,000Add : Favourable variance 5,050Less : Adverse variance 18,450Actual profit 9,600Problem 31: (CWA Inter Dec.08)The following information was obtained from the records of a manufacturing unit using standard costing system:

Particulars Standard ActualProduction 4,000 units 3,800 unitsWorking days 20 21Fixed overheads Rs. 40,000 Rs. 39,000Variable overheads Rs. 12,000 Rs. 12,000

Solution : Answers are as follows :(a) Rs. 600 A (b) Rs. 1000 F (c) Rs. 2,000 A (d) Rs. 4,000 A (e) Rs. 2,000 A.

Calculate : (a) Variable overhead variance (b) Fixed overhead expenditure variance (c) fixed overhead volume variance (d) fixed overhead efficiency variance (e) fixed overhead calendar variance.

Page 421: cost accounting question icwai inter

·       vxj yksx vki ij gal ugha jgs gSa ;k vkidh vkykspuk ugha dj jgs gSa rks vki dqN [kkl dj Hkh ugha jgs gSa-

Page 422: cost accounting question icwai inter

Total

31

(1)Total material cost variance = Standard cost of actual production − Actual cost of actual production

(2)Total material price variance = Standard price of actual production − Actual price of actual production

,sls nl yksxksa dh lwph rS;kj djsa tks vkids fy;s fofHkUu {ks=ksa esa ennxkj gks ldrs gSA vkt muls ckr djsa ;k feyus

A factory manufactures a chemical product with three ingredient chemicals A, B and C as per the standard data

Standard costing is principally based on formulae. The students must memorise various formulae relating to variances thoroughly. The solutions to the problems on standard costing are generally methodical in nature.

While standard prices of the chemical A,B and C are given as Rs. 40. Rs. 60 and Rs. 95 kg respectively, the

Page 423: cost accounting question icwai inter

Yield variance = (Standard yield from actual input − Actual output ) x Standard cost of finished product

Overhead expenditure variance = Std. cost of actual overhead − Actual cost of actual overhead

The standard variable overhead cost of a product is Rs. 10 ( 5 hours @ Rs. 2 / hr) In a certain month it took 1800 hours at a cost of Rs. 4200 to manufacture 400 units. Find the variable overhead expenditure and efficiency

ZED Ltd. has a standard costing system for its single output. Their standard cost for 100 units produced is as

Page 424: cost accounting question icwai inter

During a certain period 580,000 Kg of finished product was obtained from inputs as per details given below:

Calculate all the relevant variances with details of sub variances relating to price, mix, yield and usage.

A product is manufactured by mixing and processing three raw materials X,Yand Z as per the standard data

Page 425: cost accounting question icwai inter

First of all standard cost of output is to be known. This will be useful in computing the material cost variance.

Raw materials 'A' and 'B' having standard costs of Rs. 20/ kg and Rs. 30/kg are mixed in the standard ratio of

During a particular week 1,200 kg of 'A' costing Rs. 25,000 and 1,000 kg of 'B' costing Rs. 28,000 were mixed to

Page 426: cost accounting question icwai inter

In a factory 9,600 kg of material at Rs. 10.50/kg were actually consumed resulting in a price variance of Rs. 4,800 (A) and usage variance of Rs. 4,000 (F). The standard cost of actual production is Rs. ………………..

Kolkata furniture manufactures modular tables, chairs and office desks. The standard labour times required per unit of table, chair and desk are 4 hours, 2 hours and 8 hours respectively. The budgeted production per week is

A certain insecticide is manufactured by mixing four chemicals A,B,C,and D (filter) and processing the same. The

Page 427: cost accounting question icwai inter

Material Cost Variances = Std cost of actual output − Actual cost of output

(c) Material mix variance = ( Actual input in standard proportion − Actual input ) x Standard cost of input

(a) Material cost variance (b) Material price variance (c) Material mix variance (d) Material yield variance (e)

Page 428: cost accounting question icwai inter

The details of budget and actual of a company manufacturing and selling a single product for the year 2000-01

Page 429: cost accounting question icwai inter

The summarized budget and actual working results of Gemco ltd. For the year 2005 − 06 are given below;

A (standard price Rs. 60/kg) and B (Rs. 45/kg) in the ratio 60 : 40. 10% of input is lost during processing. If during a month 1200 kg of the chemical is produced incurring a total cost of Rs. 69,600. The total material cost

Page 430: cost accounting question icwai inter

Calculate the following variance related to labour (i) cost (ii) rate (iii) efficiency (iv) mix (v) idle time variance.

Rs. 406 / hr.

A building can be constructed by engaging a gang of workers as per the details below, for 100 working days of

Page 431: cost accounting question icwai inter

Calculate the following variance related to labour (i) cost (ii) rate (iii) efficiency (iv) mix (v) idle time variance.

(iii) Labour eff. variance : Rs. 41,600 A (iv) Mix variance : Rs. 10,800 A (v) Idle time variance : Rs. 16,000 A.

A group of workers consisting of 30 men above 30 years of age, 15 females above 30 years of age, and 10 youth of age between 20-30 are paid standard hourly rate as follows : Males : Rs. 80/hr , Females : Rs. 60/hr and

In a normal week of 40 hours, the group is expected to produce 2,000 units of output. During a week, the group consisting of 40 males, 10 females and 5 youth produced 1,600 units. They were paid wages @ Rs. 70 for males, Rs. 65 for females and Rs. 30 for you per hour. 4 hours were lost due to abnormal idle time.

Page 432: cost accounting question icwai inter

For calculation of variance the standard cost and the selling price per unit should be calculated first.

Calculate the budgeted profit the actual profit, and reconcile the two in a standard format giving clearly the

Budgeted profit can be computed merely using mental mathematics. The profit margin is 25% of sales i.e. to say profit margin is 1/3 rd of cost. The standard cost is ( 12 x 4 + 2 x 15 + 2 x 10 + 100,000/ 10,000) = Rs. 108. Thus profit is 1/3rd of Rs. 108 i.e. Rs. 36 per unit. For 10,000 units the budgeted profit comes to be Rs. 360,000.

Page 433: cost accounting question icwai inter

The summarized budget estimates of ABC Ltd. for the year 2001 are given below along with the actuals:

A company operates a standard cost system to control the variable works cost of its only product. The following

Page 434: cost accounting question icwai inter

The company wants a detailed investigation into the fall in profit. During your investigation you find the prices of materials and wage rates have increased by 10% and 15% respectively. There was also an upward revision of

The materials, wages and selling price have undergone price changes during the year. The actual items have element of price change in them. First the price variance be computed and the affected items be reduced to the

For material usage variance, the price rise is to be ignored. Standard ratio of material / sales (200 / 400), as

The standard rate of contribution as per budget is 25% i.e. Rs. 100 lakhs on a sale of Rs. 400 lakhs. As the actual sales as per budgeted price is Rs. 360 lakhs the standard contribution should be Rs. 90 lakhs.

Page 435: cost accounting question icwai inter

Actual cost

Vikas Ltd. has adopted a Standard Costing System. The standard output for a period is 10,000 units. The

You are required to ascertain the details of costs and prepare the profit and loss account in a statement form for

Page 436: cost accounting question icwai inter

Relevant fixed overhead are based at Rs. 100,000 per month and planned output is 2,000 unit per month. The selling price is Rs. 550 per unit. During a recent month when output was 1,800 units, the following actual cost

Page 437: cost accounting question icwai inter

A cost accountant of a company was given the following information regarding the fixed overheads for March' 03:

Page 438: cost accounting question icwai inter

(v) The std, selling price of TV sets/ unit is as much as a unit each of the other two product lines put together.

The company is in the process of producing the budget for 1997−98 and whilst the above actual figures are useful, it would like to know what was in the 1996−97 budget. Unfortunately this information seems to be lost and

Page 439: cost accounting question icwai inter

Material usage variance : Rs. 4,000 (Favourable).

Finished goods inventory at the end of each month is expected to be 20% of budgeted sales quantity for the following month. Finished goods inventory was 2,700 units on January 1. 2009. There would be no work-in-

Page 440: cost accounting question icwai inter

(vi) Direct labour efficiency variance.

AP x AQ used10.20 x 1,65,00015.10 x 2,38,000

adverseadverseadverse

Page 441: cost accounting question icwai inter

(iii) Variable Overhead Expenditure Variance = (Standard Rate x Actual Hours) – (Actual Rate x Actual Hours)

Small people always try to discourage you, just ignore them.

(i) Material price and usage variances ; (ii) Labour rate and efficiency variances and (iii) Fixed overhead budget variance.

FavourableFavourable

Shree Balaji Textiles Limited follows standard costing system. The standard output for a month is 20,000 units and standard

The actual production and sales for the period was 14,400 units. There has been no price revision by the government during

Page 442: cost accounting question icwai inter

1. All adverse variances to cost will be added to the cost because they show increase in cost over the standard cost.

68,00044,000

7,2003,2005,6006,400

134,400144,000

9,60023,000

The following information was obtained from the records of a manufacturing unit using standard costing system:

Calculate : (a) Variable overhead variance (b) Fixed overhead expenditure variance (c) fixed overhead volume variance (d)

Page 443: cost accounting question icwai inter

vxj yksx vki ij gal ugha jgs gSa ;k vkidh vkykspuk ugha dj

Page 444: cost accounting question icwai inter

Chapter 13: Activity based costing cwa inter cwa final ca inter ca final

4 4 7 1

CustomersA B C D E

Cases sold 4,680 19,688 136,800 71,550 8,775List selling price in Rs. 108 108 108 108 108Actual selling price in Rs. 108.0 106.2 99.0 104.4 97.2No. of purchase orders 15 25 30 25 30No. of customers visits 2 3 6 2 3No. of deliveries 10 30 60 40 20Kilometers traveled per delivery 20 6 5 10 30No. of expedited deliveries 0 0 0 0 1Its five activities and their cost drivers are:Activity Cost drivers rate in Rs.Order taking 750 per orderCustomer visits 600 per visitDeliveries 5.75 per km.Product Handling 3.75 per case soldExpedited deliveries 2,250 per expedited deliveryRequired:

Solution:Tutorial Notes :

4. This is an easy question and you should attempt to solve before seeing the solution.

·       d`rKrk dk vH;kl djsaA gj ml pht ds izfr d`rKrk eglwl djsa tks vkids ikl gSA vkids ikl lsgr vkSj [kqf'k;ksa dh ck<+ vk tk;sxhAProblem 1 (CA PE-II Nov.03)

Alpha limited has decided to analyze the profitability of its five new customers. It buys bottled water at Rs. 90 per case and sells to retail customers at a list price of Rs. 108 per case. The data pertaining to five customers are:

(i) Compute the customer-level operating income of each of the five customers now being examined. Comment on the result.

(ii) What insights are gained by reporting both the list selling price and the actual selling price for each customers?

(iii) What factors Alpha limited should consider in deciding whether to drop one or more of five customers?

1. For computing operating income, operating revenues and operating expenses must be computed and the difference between the two would represent the operating income. The operating revenues are from sales. Two selling prices viz. List selling price and actual selling price are given in question. The difference between the list price and actual price is the discount allowed to customers. You can obtain the revenues directly by using actual selling price but it is strongly recommended that you should show revenues as per list price and actual price and should clearly show the discount allowed to customers. This is what is required in part (ii) of the question.

2. Cost of operating activities can easily be known because quantity of activity and rate of its cost drivers are known. You should reduce the time required in various calculations by using figures in lacs.

3. Profitability of 5 customers are to be analyzed and compared. The comparison can be done on following parameters (i) total profit to the firm by each one of them (ii) profit as % of sales (iii) discount availed by them (iv) whether discount given to them is in tune with profit earned from them.

Page 445: cost accounting question icwai inter

The solution goes as follows :Statement showing revenues, cost of goods sold and operating income

CustomersParticulars A B C D ECases sold 4,680 19,688 136,800 71,550 8,775List selling price in Rs. 108 108 108 108 108Revenue at list selling price Rs. 505,440 2126304 14774400 7727400 947700Actual selling price/ Case 108.0 106.2 99.0 104.4 97.2Revenue at actual selling price Rs. 505,440 2090866 13543200 7469820 852930Discount given to customers 0 35438 1231200 257580 94770Cost of goods sold @ Rs. 90/case 421,200 1771920 12312000 6439500 789750Cost of operating activities:No. of order 15 25 30 25 30Order taking cost @ Rs. 750/order 11,250 18,750 22,500 18,750 22,500No. of customer visits 2 3 6 2 3customer visit cost @ Rs. 600 1,200 1,800 3,600 1,200 1,800No. of deliveries 10 30 60 40 20Kilometers traveled per delivery 20 6 5 10 30Total km. traveled 200 180 300 400 600Delivery cost @ Rs. 5.75/km. 1,150 1,035 1,725 2,300 3,450Product handling cost @ Rs. 3.75/case s 17,550 73,830 513,000 268,313 32,906Expedited delivery cost @ Rs. 2250/exd.del. 2,250Total cost of operating activities 31,150 95,415 540,825 290,563 62,906Summary of revenue, cost of goods sold and operating income:Revenue at list selling price Rs. 505,440 2126304 14774400 7727400 947700Less discount 0 35438.4 1231200 257580 94770Revenue at actual selling price Rs. 505,440 2090866 13543200 7469820 852930Cost of operating activities:Order taking cost @ Rs. 750/order 11,250 18,750 22,500 18,750 22,500customer visit cost @ Rs. 600 1,200 1,800 3,600 1,200 1,800Delivery cost @ Rs. 5.75/km. 1,150 1,035 1,725 2,300 3,450Product handling cost @ Rs. 3.75/case s 17,550 73,830 513,000 268,313 32,906Expedited delivery cost @ Rs. 2250/exd.del. 2,250Total cost of operating activities 31150 95415 540825 290563 62906Cost of goods sold @ Rs. 90/case 421200 1771920 12312000 6439500 789750Total cost 452350 1867335 12852825 6730063 852656Profit 53,090 223,531 690,375 739,758 274Profit % 3.11% 13.09% 40.44% 43.34% 0.02%Sales % 2.07% 8.55% 55.36% 30.54% 3.49%Discount given to customers/Rs. Per cas 0 1.8 9.0 3.6 10.8Comment on the result:

Part B: Insight gained by reporting both the list and the actual price:

Part C: Factors to be considered for dropping one or more customers:

1.The customer D is giving maximum profit and customer E is giving minimum profit. Although C contributes 55.36% of sales, it accounts for only 40% of profit, it is because C receives Rs. 9 per case discount while D receives only Rs. 3.60 per case as discount.

2.The profit share of E is only 0.02%, and it is receiving maximum discount of Rs. 10.8. It must be investigated that what prompted management to give highest discount to a customer lowest in contribution towards profit.

The difference between list price and actual price represents the discount. It can be studied that whether there exists a relationship between discount and sales. It is observed that more the discount, more the sales except in case of E.

Page 446: cost accounting question icwai inter

What is the cost of dropping the customer?What is impact of dropping on the production, profit and reputation of the firm?Alternatively :

CustomersParticulars A B C D ECases sold 4,680 19,688 136,800 71,550 8,775List selling price in Rs. 108 108 108 108 108Revenue at list selling price Rs. Lacs 5.05 21.26 147.74 77.27 9.48Actual selling price/ Case Rs./unit 108.0 106.2 99.0 104.4 97.2Revenue at actual selling pricRs. Lacs 5.05 20.91 135.43 74.70 8.53Discount given to customers 0.00 0.35 12.31 2.58 0.95Cost of goods sold @ Rs. 90 Rs. Lacs 4.21 17.72 123.12 64.40 7.90Cost of operating activities:No. of order 15 25 30 25 30Order taking cost @ Rs. 750/order 11,250 18,750 22,500 18,750 22,500No. of customer visits 2 3 6 2 3customer visit cost @ Rs. 600 1,200 1,800 3,600 1,200 1,800No. of deliveries 10 30 60 40 20Kilometers traveled per delivery 20 6 5 10 30Total km. traveled 200 180 300 400 600Delivery cost @ Rs. 5.75/km. 1,150 1,035 1,725 2,300 3,450Product handling cost @ Rs. 3.75/case sold 17,550 73,830 513,000 268,313 32,906Expedited delivery cost @ Rs. 2250 2,250Total cost of operating activities 31,150 95,415 540,825 290,563 62,906

Total cost of operating activities in Rs.lacs.0.31 0.95 5.41 2.91 0.63

Summary of revenue, cost of goods sold and operating income:Revenue at list selling price 5.05 21.26 147.74 77.27 9.48Less discount 0.00 0.35 12.31 2.58 0.95Revenue at actual selling price 5.05 20.91 135.43 74.70 8.53Cost of operating activities:Order taking cost @ Rs. 750/order 0.11 0.19 0.23 0.19 0.23customer visit cost @ Rs. 600 0.01 0.02 0.04 0.01 0.02Delivery cost @ Rs. 5.75/km. 0.01 0.01 0.02 0.02 0.03Product handling cost @ Rs. 3.75/case sold 0.18 0.74 5.13 2.68 0.33Expedited delivery cost @ Rs. 2250/exd.del. 0.00 0.00 0.00 0.00 0.02Total cost of operating activities 0.31 0.95 5.41 2.91 0.63Cost of goods sold @ Rs. 90/case 4.21 17.72 123.12 64.40 7.90Total cost 4.52 18.67 128.53 67.30 8.53Profit 0.53 2.24 6.90 7.40 0.00Profit % 3.11% 13.09% 40.44% 43.34% 0.02%Sales % 2.07% 8.55% 55.36% 30.54% 3.49%Discount given to customers/Rs. Per case 0 1.8 9.0 3.6 10.8

Dropping customers should always be last resort opted by any organization. Future potential should be considered before dropping any customer. Least or low profitable customers of today may become most or high profitable customer of tomorrow.

Note: You can substantially reduce your labour and gain time if you use figures in lacs as shown below: You are advised to follow this in the examination and make a note that figures have been used in lacs to save time.

Problem 2 (CA PE-II Nov.04)

MNP suits is a ready-to-wear suit manufacturer. It has four customers: two wholesale channel customers and two retail channel customers.

Page 447: cost accounting question icwai inter

MNP has developed the following activity based costing system:Activity Cost driver Rate in Rs. in 2004Order processing No. of purchase orders 1,225 per orderSales visits No. of customer visits 7,150 per visitDelivery regular No. of regular deliveries 1,500 per deliveryDelivery-rushed No. of rushed delivery 4,250 per delivery

The following data is available for year 2003Item Whole sale customers Retail customers

W H R T Total no. of orders 44 62 212 250Total no. of sales visits 8 12 22 20Regular Deliveries 41 48 166 190Rush Deliveries 3 14 46 60Average no. of suits per order 400 200 30 25Average selling price 700 800 850 900Required:1.Calculate the customer-level operating income.

Solution:

Customer wise Profitability Analysis Fig. in Rs.Particulars Whole sale customers Retail customers

W H R T Total no. of orders 44 62 212 250Average no. of suits per order 400 200 30 25Total suits ordered 17,600 12,400 6,360 6,250Revenue at list price of Rs. 1000/suit in lacs. 176.0 124.0 63.6 62.5Average price per suit 700 800 850 900Revenue at average price of Rs.in lacs. 123.2 99.2 54.06 56.25Discount at list price Rs. In lacs. 52.8 24.8 9.5 6.3Cost of goods sold@ Rs.550/suit in Rs. Lac. 96.8 68.2 34.98 34.375

Item Whole sale customers Retail customersW H R T

Total no. of orders 44 62 212 250Ordering cost @ Rs. 1225/order 53,900 75,950 259,700 306,250Total no. of sales visits 8 12 22 20Cost of sales visits @ Rs.7150 57,200 85,800 157,300 143,000Regular Deliveries 41 48 166 190Cost of regular deliveries @ Rs. 1500 61,500 72,000 249,000 285,000Rush Deliveries 3 14 46 60Cost of rushed deliveries @ Rs. 4250 12,750 59,500 195,500 255,000

List selling price in Rs.per suit is 1,000 and average cost per suit is Rs. 550. The CEO of MNP suits wants to evaluate the profitability of each of the four customers in 2003 to explore opportunities for increasing profitability of his company in 2004.

2. What do you recommend to CEO of MNP suits to do to increase the company's operating income in 2004.

3. Assume MNP suits' distribution channel costs are Rs. 17,50,000 for its wholesale customers and Rs. 10,50,000 for the retail customers. Also, assume that its corporate sustaining costs are Rs. 12,50,000. Prepare income statement of MNP suits for 2003.

Tutorial notes: Whenever the list and average selling prices are given; the message is clear. Show the quantum and impact of discount. This question is simple and easy. Student should solve it and compare his solution with the solution given below.

Problem 3 (cwa inter II June 05)

Page 448: cost accounting question icwai inter

Particulars Hours per unit Materials VolumeLabour Machine per unit Rs. units

hours hoursProduct P 0.5 1.5 20 750Product Q 1.5 1.0 12 1,250Product R 1.0 3.0 25 7,000

Further analysis shows that the total of production overheads can be divided as follows:Costs Set-up machine material handling inspection Total production overhead

% 35 20 15 30 100

Product no. of set ups no. of material handling no. of inspectionsP 75 12 150Q 115 21 180R 480 87 670

Required:(i) Calculate the cost per unit for each product using conventional methods;(ii) Calculate the cost per unit for each product using ABC activity based costing principles ;(iii) Comment on the reasons for any difference in the costs in (i) and (ii) above.Solution:(i) Statement showing cost per unit of Products P, Q and R by Conventional method

Foamstar LimitedLabour Amount Machine Amount Materials Totalhours @ Rs. 6 hours @ Rs.28 per unit Rs.

Product P 0.5 3 1.5 42 20 65Product Q 1.5 9 1.0 28 12 49Product R 1.0 6 3.0 84 25 115(ii) Cost of products on the basis of ABC Activity Based Costing:

Particulars Machine Volume total machine Cost of overheadshours units hours @ Rs. 28 per machine hour

Product P 1.5 750 1,125 31,500Product Q 1.0 1,250 1,250 35,000Product R 3.0 7,000 21,000 588,000

23,375 654,500Distribution of overhead cost of Rs. 654,500 to different activities

Costs Set-up machine material handling inspection Total prodn. ohead % 35 20 15 30 100

Rs. 654,500 229,075 130,900 98,175 196,350 654,500Apportionment of cost of overheads to products P,Q and RActivities Ratio Total costs P Q R

Rs. Rs. Rs. Rs.Set-ups 75:115:480 229,075 25,643 39,319 164,113Machining 1125:1250:21000 130,900 6,300 7,000 117,600Material handling 12:21:87 98,175 9,818 17,181 71,177Inspection 150:180:670 196,350 29,453 35,343 131,555

654,500 71,213 98,842 484,445

Foamstar Ltd. makes three main products using broadly the same production methods and equipment for each. A conventional product costing system is used at present, although Activity Based Costing is being considered. Details of three products for a typical period are :

Direct labour costs Rs. 6 per hour and production overheads are absorbed on machine hour basis. The rate for the period is Rs. 28 per machine hour.

The following activity volumes are associated with the product line for the period as a whole. Total activities for the period:

First of all total overheads is to be computed. The overhead absorption rate is furnished as Rs. 28 per machine hour. The total machine hours and overheads are calculated as follows:

Page 449: cost accounting question icwai inter

No. of units 750 1,250 7,000Production overheads per unit 95 79 69Statement showing Product cost per unit based on ABC principles:Particulars P Q R

Rs. Rs. Rs.Materials 20 12 25Direct Labour 3 9 6Production overheads 95 79 69Product cost per unit in Rs. 118 100 100Product cost per unit in Rs.(conventional method) 65 49 115(iii) Reasons for difference:

(i) The set up costs have been distributed based on no. of set-ups.(ii) The material handling costs have been distributed based on no. of material movements.(iii) The inspection cost is also distributed on the basis of no. of inspections.

0.2% of Astha Insurance policyholder's claims;+ 0.6% of other party's property claims and + 0.8% of total personal injury claims.Data from two recent automobile accident claims are as under :

Particulars Automobile Claim Automobile ClaimNo. A/ 893405 No. A/ 578341

Rs. Rs.Policy holder Claim 4,500 23,600Other Party Claim 0 3,400 Personal Injury Claim 12,400 0Total Claim Amount 16,900 27,000Required:

(ii) How would you recommend that Astha Insurance estimate the cost of processing claims?Solution: Try to solve this by using calculator, only in 5 minutes.Statement showing processing cost using account analysis and activity analysis:

Astha Insurance LimitedCost of processing using Activity AnalysisParticulars Automobile Claim Automobile Claim In Rs.

No. A/ 893405 No. A/ 578341Amount Rate Cost Amount Rate Cost

Policy holder Claim 4,500 0.20% 9.0 23,600 0.20% 47.2Other Party Claim 0 0.60% 0.0 3,400 0.60% 20.4 Personal Injury Claim 12,400 0.80% 99.2 0 0.80% 0.0Total estimated cost of processing : 108.2 67.6

1. In conventional method, the overheads are charged at a flat rate of Rs. 28 per machine hour which is illogical.

2. In ABC method, the production overheads have been charged based on the activities which is more logical.

Problem 4 (cwa inter II dec 05)

Astha Insurance ltd. processes a variety of insurance claims for losses, accidents, thefts and so on. Account analysis has estimated the variable cost of processing each claim at 0.5%(0.005) of the Rupee value of the claim. This estimate seemed reasonable because higher claims often involve more analysis before settlement.

To control processing cost better, however, Astha Insurance conducted an activity analysis of claims processing. The analysis suggested that more appropriate cost drivers and behaviour for automobile accident claims are :

(i) Estimate the cost of processing each claim using data from accident analysis and then the activity analysis;

Page 450: cost accounting question icwai inter

Cost of processing using Account AnalysisTotal Claim Amount 16,900 0.50% 84.5 27,000 0.50% 135.0(ii) For this part see the advantages and limitations of ABC analysis.

Activity Cost driver Capacity CostPower Kilowatt hours 50,000 kwh Rs. 200,000Quality inspection No. of inspections 10,000 inspections Rs. 300,000

Product Kilowatt hours Quality InspectionsM 10,000 3,500S 20,000 2,500T 15,000 3,000

Required: 1. Compute the costs allocated to each product from each activity.2. Calculate the cost of unused capacity for each activity.

Solution: Working notes:1. Cost of power per kilowatt hour = Rs. 200,000 / 50,000 i.e. Rs. 4 per kwh2. Cost of quality inspection is Rs. / unit = Rs. 300,000 / 10,000Statement of cost allocation to each product from each activityProduct Kwh Rate Rs. Amount QI Rate Rs. Amount Total Rs. M 10,000 4 40,000 3,500 30 105,000 145,000S 20,000 4 80,000 2,500 30 75,000 155,000T 15,000 4 60,000 3,000 30 90,000 150,000

45,000 180,000 9,000 270,0002. Calculation of the cost of unused capacity for each activity. Rs.Power @ Rs. 4/ unit for (50,000 − 45,000) i.e. 5000 units = Rs. 4 x 5,000 20,000Quality inspection @ Rs. 30 for (10,000 − 9,000) i.e.1000 inspections Rs.30 x1000 30,000Total cost of unused capacity in Rs. 50,000

(i) General supermarket Chains (ii) Drugstore Chains (iii) Chemist ShopsThe following data for April, 2004 in respect of RST ltd. has been reported:

Drugstore ChemicalChains Shops

Average revenue per delivery Rs. 84,975 28,875 5,445Average cost of goods sold per delivery Rs. 82,500 27,500 4,950No. of deliveries 330 825 2,750

The company plans to use activity based costing for analyzing the profitability of its distribution channels.

The activity analysis of RST ltd. is as under:Activity area Cost DriverCustomer purchase order processing Purchase orders by customersLine-item ordering Line-items per purchase orderStore delivery Store deliveriesCartons dispatched to stores Cartons dispatched to a store per day

Problem 5 (ca inter may 04)

MST ltd. has collected the following data for its two activities. It calculates activity cost rates based on cost driver capacity.

The company makes three products M, S and T. For the year ended March 31,04, the following consumption of cost drivers was reported:

3. Discuss the factors the management consider in choosing a capacity level to compute the budgeted fixed overheads cost rate.

Problem 6 (ca inter may.04)

RST ltd. specializes in the distribution of pharmaceutical products. It buys from the pharmaceutical companies and resells to each of the three different markets.

Gen. Super. Chains

In the past, RST ltd. has used gross margin percentage to evaluate the relative profitability of its distribution channels.

Page 451: cost accounting question icwai inter

Shelf-stocking at customer store Hours of shelf-stocking

Activity area Total cost in April 04 in Rs. Total units BaseCustomer purchase order processing 220,000 5,500 ordersLine-item ordering 175,560 58,520 line itemsStore delivery 195,250 3,905 store deliveriesCartons dispatched to stores 209,000 209,000 cartonsShelf-stocking at customer store 28,160 1,760 hoursOther data for April 2004, include the following:

Drugstore ChemicalChains Shops

Total no. of orders 385 990 4125Average no. of line items per order 14 12 10Total no. of store deliveries 330 825 2750Average no. of cartons shipped per store delivery 300 80 16Average no. of hours of shelf-stocking per delivery 3 0.6 0.1Required:

2. Compute the April,2004 rate per unit of the cost allocation base for each of the five activity areas.

Solution: 1. Computation of Gross Margin Percentage for each channelsGen. supermarket Drugstore ChemicalChains Chains Shops

Average revenue per delivery Rs. 84,975 28,875 5,445No. of deliveries 330 825 2750Total revenue in Rs. Lacs 280.42 238.22 149.74Average cost of goods sold per delivery 82,500 27,500 4,950Total cost in Rs. Lacs 272.25 226.88 136.13Gross margin (Revenue − Cost) 8.17 11.34 13.61Gross margin percentage 2.91% 4.76% 9.09%

2. Computation of cost per unit of each of the five activitiesActivity area Cost in Rs. Units Cost/ unit Rs. Customer purchase order processing 220,000 5,500 orders 40Line-item ordering 175,560 58,520 line items 3Store delivery 195,250 3,905 store deliveries 50Cartons dispatched to stores 209,000 209,000 cartons 1Shelf-stocking at customer store 28,160 1,760 hours 16

3. Computation of operating cost of each distribution channel:General supermar. Drugstore ChemicalChains Chains Shops

Total no. of orders 385 990 4125Total Order processing cost @ Rs. 40 per order 15,400 39,600 165,000Average no. of line items per order 14 12 10Total no. of line items for orders (385 x 14) (990x12) (4125x10)

The April 2004 operating costs (other than cost of goods sold) of RST ltd. are Rs. 827,970. These operating costs are assigned to five activity areas. The cost in each area and the quantity of the cost allocation basis used in that area for April 04 are as follows:

Gen. Super. Chains

1. Compute for April 2004, gross margin percentage for each of its three distribution channels and compute operating income of RST limited.

3.Compute the operating income of each distribution channel in April 2004 using the activity based costing information. Comment on the results. What new insights are available with the activity based costing information ?

Page 452: cost accounting question icwai inter

5,390 11,880 41,250Total Line items ordering cost @ Rs. 3/order 16,170 35,640 123,750Total no. of store deliveries 330 825 2750Total store deliveries cost @ Rs. 50/delivery 16,500 41,250 137,500Average no. of cartons shipped per store delivery 300 80 16Total cartoon dispatch cost @ Rs. 1/cartoon 99,000 66,000 44,000Average no. of hours of shelf-stocking per delivery 3 0.6 0.1Total hours for shelf stocking (ave.hrs x total delivery) 990 495 275Cost shelf stocking @ Rs. 16/delivery 15,840 7,920 4,400Total Operating cost in Rs. 162,910 190,410 474,6504. Operating Income Statement of each distribution channel in April 04 using ABC approach

General supermar. Drugstore ChemicalChains Chains Shops

Gross margin (Revenue − Cost) in Rs. Lacs 8.17 11.34 13.61Less Total Operating cost in Rs. Lacs 1.63 1.90 4.75Operating Income 6.54 9.44 8.87Total revenue in Rs. Lacs 280.42 238.22 149.74% of operating income 2.33% 3.96% 5.92%5.Comment on Results:

General supermar. Drugstore ChemicalChains Chains Shops

Total revenue in Rs. Lacs 280.42 238.22 149.74Total Operating cost in Rs. 162,910 190,410 474,650% of operating cost 0.58% 0.80% 3.17%

Particulars Soft drinks Fresh PackageProduce Food

Revenues Rs. 7,93,500 21,00,600 12,09,900Cost of goods sold Rs. 6,00,000 15,00,000 9,00,000Cost of bottles returned Rs. 12,000 0 0No. of purchase orders place no. 360 840 360No. of deliveries received no. 300 2190 660Hours of shelf-stocking time hours 540 5400 2700Item sold no. 1,26,000 11,04,000 3,06,000Family stores also provides the following information for the year 2002-03

Activity Description of activity Cost Rs. Cost allocation base

Bottle returns Returning of empty bottles 12,000

Ordering Placing order for purchase 156,000

Delivery Physical delivery 252,000 3,150 deliveries

Shelf stocking Stocking on store self and ongoing restocking. 172,800

Customer support Assistance to customers including check out. 307,200 15,36,000 items sold.

Required:

ABC in the above case reveals that the chemical shop uses the resources in larger share than the other two distribution channels.

Problem 7 (ca inter may.03)

Family store wants information about the profitability of individual product lines: Soft drinks, Fresh produce and Package food. Family store provides the following data for the year 2002−03 for each product line:

Direct tracing to soft drink line

1,560 purchase orders

8,640 hours of shelf stocking

Page 453: cost accounting question icwai inter

3. Comment of your answer on 1 and 2 above.Solution:First of all let us calculate total support cost and then allocate it on the basis of cost of goods sold.

Activity Ordering Delivery Total Rs. 12,000 156,000 252,000 172,800 307,200 900,000

Total cost of goods sold (600,000 + 1500,000 + 900,000) 30,00,000% of support cost with cost of goods sold = 9/30 = 30%1. Statement of operating income and its % with revenues of each product line:Particulars Amount in Rs. Soft drinks Fresh Package Total

Produce FoodRevenues 7,93,500 21,00,600 12,09,900 41,04,000Cost of goods sold 6,00,000 15,00,000 9,00,000 30,00,000Support cost @ 30% of cost of goods sold 1,80,000 4,50,000 2,70,000 9,00,000Total cost 7,80,000 19,50,000 11,70,000 39,00,000Operating income 13,500 1,50,600 39,900 2,04,0002. Cost of each activity and cost driver rate:Activity Total cost Rs. Cost allocation base Cost driver rate Rs./unitOrdering 156,000 1,560 orders 100Delivery 252,000 3,150 deliveries 80Shelf stocking 172,800 8,640 hours 20Customer support 307200 1536000 items sold 0.203. Statement of operating income and its % with revenues of each product line:(ABC basis)

Soft drinks Fresh Package Total Produce Food

Rs. Rs. Rs. Rs.Bottle return costs 12,000 0 0Ordering cost (360:840:360) 36,000 84,000 36,000 156,000Delivery cost (300:2190:660) 24,000 175,200 52,800 252,000Shelf stocking cost (540:5400:2700) 10,800 108,000 54,000 172,800Customer support cost (126000:1104,000:306000) 25,200 220,800 61,200 307,200Total support cost 108,000 588,000 204,000 888,000Cost of goods sold 600,000 1500,000 900,000 3000,000Total Cost in Rs. Lakhs 7.08 20.88 11.04 39.00Total revenue in Rs. Lakhs 7.935 21.006 12.099 41.040Operating income in Rs. Lakhs 0.855 0.126 1.059 2.040Operating income as % of revenue 10.8% 0.6% 8.8% 5.0%Comments:

2. The ABC information provides more parameters for pricing decision purposes.3. This system is more credible because it distinguishes with different activities more precisely.

1. Family store currently allocates support cost (all cost other than cost of goods sold) to product lines on the basis of cost of goods sold of each product line. Calculate the operating income and operating income as a % of revenues for each product line.

2. If family store allocates support costs (all costs other than cost of goods sold) to product lines using an activity based costing system, calculate the operating income and operating income as % of revenues for each product line.

Bottle returns

Shelf stocking

Customer support

1. The ABC system correlates the use of resources with the product lines in a more precise manner than traditional costing system. For example in the case of Family store, the soft drinks provide higher % of operating income, worded differently, Soft drinks consume less resources than Fresh produce or Package food.

Problem 8 (ca inter may.06)

Page 454: cost accounting question icwai inter

Activity based Account Usage by customerCosting per transaction Rs. X Y Z

Deposits/Withdrawals with teller 125 40 50 5Deposits/Withdrawals with ATM 40 10 20 16

25 0 12 60Bank Cheques written 400 9 3 2Foreign currency drafts 600 4 1 6Inquiries about Account balance 75 10 18 9Average Premier Account balance Rs. 55,000 40,000 12,50,000

Required:(i) Compute the 2005-06 profitability of the customers X,Y and Z Premier account at ABC Bank.

(iii) What changes would you recommend for ABC Bank's Premier Account ?Solution: This is very easy. Try to solve it before you see the solution.

Customer wise Profitability analysis with Activity-based approachActivity based Account Usage by customerCosting per transaction Rs. X Y Z

Deposits/Withdrawals with teller 125 40 50 5Total cost of transaction of every customer (40x125) (50x125) (5x125)

5,000 6,250 625Deposits/Withdrawals with ATM 40 10 20 16Total cost of transaction of every customer (10x40) (20x40) (16x40)

400 800 640Deposits/Withdrawals monthly basis 25 0 12 60Total cost of transaction of every customer 0 (12x25) (60x25)

0 300 1,500Bank Cheques written 400 9 3 2Total cost of transaction of every customer (9x400) (3x400) (2x400)

3,600 1,200 800Foreign currency drafts 600 4 1 6Total cost of transaction of every customer (4x600) (1x600) (6x600)

2,400 600 3,600Inquiries about Account balance 75 10 18 9Total cost of transaction of every customer (10x75) (18x75) (9x75)

750 1,350 675Customer Cost per year 12,150 10,500 7,840

ABC Bank is examining the profitability of its Premier Account, a combined Saving and Cheque account. Depositors receive a 7% annual interest on their average deposit. ABC Bank earns an interest rate spread of 3%( the difference between the rate at which it lends money and rate it pays to depositors) by lending money for home loan purpose at 10%.

The Premier Account allows depositors unlimited use of services such as deposits, withdrawals, cheque facility and foreign currency drafts. Depositors with Premier Account balances of Rs. 50,000 or more receive unlimited free use of services. Depositors with minimum balance of less than Rs. 50,000 pay Rs.1000-a-month service fee for their Premier Account.

ABC Bank recently conducted an activity-based costing study of its services. The use of these services in 2005−06 by three customers is as follows:

Deposits/Withdrawals with prearranged monthly basis

Assume Customer X and Z always maintains a balance above Rs. 50,000, whereas Customer Y always has balance below Rs. 50,000.

(ii) What evidence is there of cross-subsidization among the three Premier Account ? Why might ABC Bank worry about this cross subsidization, if the Premier Account product offering is Profitable as a whole?

Page 455: cost accounting question icwai inter

Income from customer:Average Premier Account balance 55,000 40,000 12,50,000Interest @ 3% p.a. on average balance 1,650 1,200 37,500Fee @ Rs. 1000/month if balance is below Rs. 50,000 12,000Total benefit from customers 1,650 13,200 37,500Customer benefit/cost (10,500) 2,700 29,660

(iii) The changes which I recommend for ABC Bank's Premier Account are as follows:

Equipment Y Equipment ZBudgeted production volume units 2,500 3,125Direct materials cost Rs./ unit 300 450Direct labour costY: 3 hours @ Rs. 150 per hourZ: 4 hours @ Rs. 150 per hour 450 600ABC Ltd.'s overheads of Rs. 12,42,500 can be identified with three major activities:

Orders processed Machine hours worked Inspection hoursY 350 23,000 4,000Z 250 27,000 11,000

Total 600 50,000 15,000Required:

(ii) There is evidence of cross-subsidizing because customer Z is most profitable and is neutralizing the negative income from X. Y is paying for the services. Y can be advised to deposit Rs. 10,000 more to avoid the service charges of Rs. 12,000. The rational of charging Rs. 1000-a-month service charge if the account balance is below Rs. 50,000 on average basis, needs to examined in detail.

(ii) It is true that the Premium Account is profitable to the extent of Rs. (29,660 + 2,700 − 10,500) i.e. Rs. 21,860 This is subject to the condition that the sample of these three customers represents the entire population in a reasonable and realistic manner. This profitability is solely dependent of single performer i.e. customer Z. More the Z's, more the profitability.

1. The base of above analysis must be broadened. How many customers are like Z and how many like X and Y? More observations are necessary for decision making.

2. The free service system for customer maintaining a minimum balance may be dispensed with. All services may be rendered on chargeable basis. The cross subsidization as indicated above does not reflect efficiency on the part of the bank. To encourage more deposits, interest rate may be correlated with the amount deposited and time length of deposit.

3. The deposit/withdrawal with teller machines costs Rs. 125 each, while with ATM, it costs Rs.40. Small transaction say below Rs. 5000 may not be allowed with teller. Efforts must be made to encourage the customer to use ATM more frequently.

4. The demand of home loan is being met or there is some deficit or surplus on this account. If deficit, some incentive schemes for encouraging customers to deposit more can be taken up. Special attention must be paid to customer like Z.

Problem 9 (ca inter may.07)

ABC Ltd. manufactures two types of machinery equipments Y and Z and applies/absorbs overheads on the basis of direct-labour hours. The budgeted overheads and direct labour hours for the month of December 2006 are Rs. 12,42,500 and 20,000 hours respectively. The information about Company's product is as follows:

Order processing : Rs. 2,10,000; Machine processing : Rs. 8,75,000 and Product inspection : Rs. 1,57,000. These activities are driven by number of orders processed, machine hours worked and inspection hours, respectively. The data relevant to these activities is as follows:

Page 456: cost accounting question icwai inter

(iv) Discuss how an activity-based costing might benefit ABC ltd.Solution:

The manufacturing cost of equipment Y and Z would be computed as follows:Equipment Y Equipment Z

Direct materials cost Rs./ unit 300 450Direct labour costY: 3 hours @ Rs. 150 per hour 450 600Z: 4 hours @ Rs. 150 per hourOverheads @ Rs. 62.125/hour 96.38 248.50Total cost Rs. Per unit 846.38 1,298.50(ii) Estimation of cost-driver rate:Activity Cost driver Overhead cost No. of cost drivers Rate Rs. / unit

Rs. no.Order processing no. of orders 210,000 600 350.0Machine processing hours 875,000 50,000 17.5Inspection hours 157,500 15,000 10.5Overheads costs are to be distributed on the basis of cost driver level.Activity Overhead cost Ratio Y Z

Rs. Rs. Rs.Order processing 210,000 350:250 122,500 87,500Machine processing 875,000 23:27 402,500 472,500Inspection 157,500 4:11 42,000 115,500

567,000 675,500Budgeted volume 2,500 3,125Overhead rate 226.80 216.16Manufacturing cost of Y and Z with Activity-based costing approach:

Equipment Y Equipment ZDirect materials cost Rs./ unit 300.00 450.00Direct labour costY: 3 hours @ Rs. 150 per hour 450.00 600.00Z: 4 hours @ Rs. 150 per hourOverhead cost 226.80 216.16Total cost as per activity based costing 976.80 1,266.16Total cost as per traditional costing 846.38 1,298.50

(iv) Discuss how an activity-based costing might benefit ABC ltd.

(i) Assuming use of direct labour to absorb/apply overheads to production, compute the unit manufacturing cost of equipments Y and Z, if the budgeted manufacturing volume is attained.

(ii) Assuming use of activity-based costing, compute the unit manufacturing cost of equipments Y and Z, if the budgeted manufacturing volume is attained.

(iii) ABC Ltd.'s selling prices are based heavily on cost. By using direct labour hour as an application base, calculate the amount of cost distortion(under-costed or over-cost) for each equipment.

(i) If budgeted manufacturing volume is attained, The total direct labour hours would be 20,000 hours as given. The overheads amount to Rs. 12,40,500. Thus the overheads may be absorbed as a predetermined rate of Rs. 12,40,500/ 20,000 i.e. Rs. 62.125 per hour.

In traditional costing system, the low volume product Y is over-cast and high volume product is under-cost. This is due to limitation of traditional costing system.

The ABC ltd. should go for activity based costing because it can avoid cost distortion as shown above. The pricing decisions of this company are mainly dependent of cost and ascertain of cost with greater reasonableness would justify the decision.

Problem 10 (cwa inter II June 07)

Page 457: cost accounting question icwai inter

Products A B C DOutput units 120 100 80 120Cost per unit Rs.

Direct materials Rs. 40 50 30 60Direct labour Rs. 28 21 14 21Machine hours per unit 4 3 2 3

The production overheads during the period are as follows: Rs. Factory work expenses 20,860Set-up costs 10,500Stores receiving 7,200Inspection quality control 4,200Material handling and dispatch 9,240

Information in these activity cost pools and their drivers is given belowActivity cost pools Cost DriversFactory work expenses Machine-hoursSet-up costs Number of production runsStores receiving Requisitions raisedInspection quality control Number of production runsMaterial handling and dispatch Number of orders executed

Requirements :1. Total cost of each product assuming the absorption of overhead on machine-hour basis.2. Total cost of each product assuming the absorption of overhead on activity based costing.3. Show the difference between 1 and 2 above and comment.Solution:

The machine-hours required for all the four productsProducts A B C D TotalOutput units 120 100 80 120Machine hours per unit 4 3 2 3Machine hours required 480 300 160 360 1,300 hours The production overheads during the period are as follows: Rs.

Factory work expenses 20,860Set-up costs 10,500Stores receiving 7,200Inspection quality control 4,200Material handling and dispatch 9,240Total Overheads to be absorbed 52,000Total machine-hours available 1,300 hrs.Overhead absorption rate Rs. 40 per machine hour.

Statement showing Cost of different products : Absorption of overheads on machine-hour basis.Products A B C DOutput units 120 100 80 120

Fasteners ltd. produces and sells four products A,B,C and D. Details of the four products and relevant information are given below for week ended 28th April 2007.

The four products are similar and are usually produced in production runs of 20 units and sold in batches of 10 units.

The production overhead is currently absorbed by using a machine-hour rate and the company wishes to introduce activity based costing ( ABC) system and has identified major cost pools for production overheads and their associated cost drivers.

The number of requisitions raised on the stores was 20 for each product and number of orders executed was 42, each order being for a batch of 10 of a product.

Assuming that overheads are absorbed on machine hour basis, we should first work out the machine hour rate:

Page 458: cost accounting question icwai inter

Cost per unit Rs. Direct materials Rs./unit 40 50 30 60Direct labour Rs./unit 28 21 14 21Machine hours per unit 4 3 2 3Overheads @ Rs. 40 per machine hour 160 120 80 120Total cost per unit Rs./unit 228 191 124 201Total cost Rs. Rs. 27,360 19,100 9,920 24,120 80,500

If overheads are to be absorbed on cost driver basis, cost per unit of each cost driver must be worked out.

The production overheads during the period are as follows: Rs. Overheads Amount Cost Drivers No. Cost/unit of driversFactory work expenses 20,860 Machine hours 1,300 16Set-up costs 10,500 Production runs 21 500Stores receiving 7,200 Requisitions 80 90Inspection quality control 4,200 Production runs 21 200Material handling and dispatch 9,240 Orders 42 220Products A B C DOutput units 120 100 80 120

No. of production runs @ 20 units per run 6 5 4 6Overheads absorption per unitFactory work expenses@ Rs. 16 per machine hourNo. of machine hours per unit 4 3 2 3Factory overhead expenses/unit 64 48 32 48Set-up costs @ Rs. 500/runNo. of production runs @ 20 units per run 6 5 4 6Set-up costs for all units 3,000 2,500 2,000 3,000Set-up cost per unit 25 25 25 25Stores receiving @ Rs.. 90 per requisitionsNo. of requisitions 20 20 20 20Costs of stores receiving for all units 1,800 1,800 1,800 1,800Store receiving cost per unit 15 18 22.5 15Inspection quality control @ Rs. 200 per production runNo. of production runs 6 5 4 6Cost of inspection quality control for all units 1,200 1,000 800 1,200Inspection quality control per unit 10 10 10 10Handling / dispatch cost @ Rs. 220/ orderNo. of orders @ 10units per order 12 10 8 12Cost of orders for all units 2,640 2,200 1,760 2,640Handling / dispatch cost per unit 22 22 22 22Total overhead Cost per unit Rs. 136 123 111.5 120Cost per unit Rs. Direct materials Rs./unit 40 50 30 60Direct labour Rs./unit 28 21 14 21Total overhead Cost per unit Rs. Rs./unit 136 123 111.5 120Unit cost Rs. 204 194 155.5 201Total cost Rs. 24,480 19,400 12,440 24,120Comparative statement showing cost/unit and total cost under two methods:Products A B C DTotal cost per unit 228 191 124 201Unit cost 204 194 155.5 201Difference 24 3 31.5 0Total cost Activity based costing 24,480 19,400 12,440 24,120 80,440Total cost Rs. Absorption basis 27,360 19,100 9,920 24,120 80,500Difference Rs. 2,880 300 2,520 0

Page 459: cost accounting question icwai inter

A company produces four products viz. P,Q,R and S. The data relating to production activity are as under :

Products Production Quantity

P 1,000 10 1 6 0.5Q 10,000 10 1 6 0.5R 1,200 32 4 24 2.0S 14,000 34 3 18 3.0

Production overheads are as under : Rs. 1. Production applicable to machine oriented activity; 149,7002. Overheads relating to ordering materials 7,6803. Set up costs 17,4004. Administrative overheads for spare parts 34,3805. Material handling costs 30,294The following further information have been compiled :

no. of material orders no. of spare partsP 3 3 6 6Q 18 12 30 15R 5 3 9 3S 24 12 36 12Required:

2. Using the concept of activity based costing, compute the factory cost per unit of each product.Solution:Working notes for total cost drivers:

product units hr./unit Total orders handlings partsP 1,000 0.5 500 3 3 6 6Q 10,000 0.5 5,000 18 12 30 15R 1,200 2.0 2,400 5 3 9 3S 14,000 3.0 42,000 24 12 36 12

49,900 50 30 81 36Selection of cost drivers:Activity: Rs. Cost driver no. per cost driver1. machine oriented activity; 149,700 Machine hrs. 49,900 32. ordering materials 7,680 No. of orders 30 2563. Set up costs 17,400 no. of setups 50 3484. spare parts 34,380 no. of spare parts 36 9555. Material handling costs 30,294 no. of handlings 81 374Apportionment of overheads to products on basis of cost drivers:Products Rate Rs./unit P Q R S 1. machine oriented activity; 3 1,500 15,000 7,200 126,0002. ordering materials 256 768 3,072 768 3,0723. Set up costs 348 1,044 6,264 1,740 8,3524. spare parts 955 5,730 14,325 2,865 11,4605. Material handling costs 374 2,244 11,220 3,366 13,464Total Rs. 11,286 49,881 15,939 162,348

The traditional costing methods apportions the overheads on machine-hour basis which is not logical. The activity based costing is more logical and precise than the traditional method and hence the difference.

Problem 11 (cwa final Dec.01)

Material cost / unit Rs.

Labour hrs/unit

Labour cost / unit Rs.

Machine hrs/unit

No. of setups

no. of times material handled

1. Select a suitable cost driver for each item of overhead expense and calculate the cost per unit of cost

No. of setups

Page 460: cost accounting question icwai inter

no. of units produced 1,000 10,000 1,200 14,000factory overhead per unit 11.29 4.99 13.28 11.60Material 10 10 32 34Labour 6 6 24 18Total factory cost per unit 27.29 20.99 69.28 63.60

Cost of production (same for both the products) Rs.Direct materials 125Direct labour 24Direct Expenses (sub-contract charges) 36Overheads (400% of direct labour) 96Total Cost 281

Unit Product A Product B TotalQuantity Sold No. 124,000 23,150Unit sale price 300 290Total sales realisation 43913500Cost of sales as above 41349150Margin 2564350

(a)

(b)

Prepare a revised profitability statement to find out if the Marketing manager's proposals is viable.Solution: Amount is calculated in thousand rupees.Computation of total overheads

Unit Product A Product BQuantity Sold No. 124,000 23,150Labour/unit Rs. 24 24Overheads (400% of labour) Rs. 96 96Total overheads 11,904 2,222Gross Total of overheads 14,126The cost activities relating to subcontracting is 75% of total overheads :Overheads relating to sub-contracting (75% of 14126) 10,595 3,532 1766( Gross total of overhead less overheads of subcontracting : 14,126 − 10,595 = 3,532)

Apportionment of overheads to products on basis of activities Rs.'000Product A (5/7 x 105,94,800) 7,568Product B (2/7 x 105,94,800) 3,027

10,595

Problem 12 (cwa final Dec.01cost audit)

Precision Auto comp Ltd. manufactures and sells two automobile components A and B. Both are identical with slight variation in design. Although the market for both the products is the same, the market share of the company for product A is very high and that of product B very low. The Company's accountant has prepared the following profitability statement for the two products:

The company's marketing manager, after attending a workshop on activity-based-costing, challenges the accountant's figures. The nearest competitor's prices for the two products are Rs. 330 and Rs. 275 per unit respectively and, if the company can match the competitor's prices, it can sell 75000 nos. each of the two products. The production manager confirms that he can produce this product mix with the existing facilities. The management engages you as consultant, and the following facts have been identified by you :

product A undergoes 5 operations and product B undergoes two operations by sub-contractors, although the total sub-contract charges are same for both the products and

75% of the overheads is accounted for by three major heads relating to sub-contracting operations, viz, ordering, inspection and movement of components, to and from the sub-contractor's works

According to Market manager the expected sales of both the products are same and equal to 75,000 units. If activities form the basis of apportionment of overheads of Rs. 105,94,800, this should be apportioned to product A and B in the ratio of 5:2 because they undergo operations in that ratio.

Page 461: cost accounting question icwai inter

Taking sub-contracting activities as cost drivers, the revised profitability statement is as under:Unit Product A Product B Amount Rs. '000

Quantity Sold No. 75,000 75,000Direct material cost @ Rs. 125/unit 9,375 9,375Direct labour cots @ Rs. 24/unit 1,800 1,800Direct expenses @ Rs. 36/unit 2,700 2,700Apportioned overheads 7,568 3,027Balance overheads (1:1) ( 3532/2 ) 1,766 1,766Total cost 23,209 18,668 41,877Sales price Rs. 330 275Sales revenue Rs. thousand 24,750 20,625 45,375Margin 1,542 1,957 3,499Existing margin 2,564Extra margin with market manager's proposal 935

Cost Unused Quantity Activity rate Rs.**driver capacity demanded* Fixed Variable

Direct materials units 0 2,000 0.0 3.0Direct labour direct labour hours 0 400 0.0 7.0Set-ups set-up hours 0 25 50.0 8.0Machining machine hours 6000 4000 4.0 1.0* This represents the amount of resources demanded by the special order being considered

Required:

(b) Suppose that the set-up activity had 50 hours of unused capacity. How does this affect the analysis?

Solution:Tutorial Notes :1. All variable costs are relevant for this special order and need to taken into account.

Total overheads are Rs. 14126 thousand, the balance after apportionment is (14126 − 10595) = Rs. 3532 th to be distributed to two products equally.

Recommendation: The marketing manager's proposal is recommended to be accepted in the light of extra margin to the tune of Rs. 935,000 than present level. The cost statements are recommended to be prepared on Activity-based-costing.

Problem 13 (cwa final June 03)

XYZ Ltd. having idle capacity received an offer to sell 2,000 units of one of its products to a new customer in a geographical region not normally serviced by the company. The offering price is Rs. 10 per unit. The product normally sells for Rs. 14. The activity based accounting system provides the following information:

** Fixed activity rate is the price that must be paid per unit activity capacity. The variable activity rate is the price per unit of resources for resources acquired as needed

Although the fixed activity rate for set-ups is Rs. 25 per hour, any expansion of this resource must be required in blocks. The unit purchase for set-ups is 100 hours of set-up servicing. Thus any expansion of set-up activity must be done 100 hours at a time. The price per hour is the fixed activity rate.

(a) Compute the change in income for XYZ ltd., if the order is accepted. Comment on whether the order should be accepted of not, (in particular, discuss the strategic issues).

2. The machine hours are available as 6000. 4,000 hours are required for this order which is well within the capacity, the fixed component of Rs. 4/ hour need not be considered in the appraisal of this order.

Page 462: cost accounting question icwai inter

Solution goes as follows:The various costs pertaining to this order are as follows: Rs. Direct materials cost 2,000 x Rs. 3 6,000Direct labour cost 400 x Rs. 7 2,800Set-ups 100 x Rs. 50 + 25 x Rs. 8 5,200Machining 4,000 x Rs. 1 4,000Total cost of special order 18,000Total revenue to be received 2,000 x Rs. 10 20,000Additional revenue of this order 2,000

The special order may be accepted as the machine hours are available in spare quantity, provided

3. Special orders should not be accepted if these are likely to be proved counter-productive in the long run.

Part (b)

Product X Product YNumber of units produced 30 30Material moves per product line 5 15Direct labour hour per unit 200 200What cost would be allocated to product x under activity based costing?Solution:Total moves in material handling (5 + 15)Percentage moves for product x 5/20 = 25%Material handling cost to be allocated to Product X

25% of Rs. 60,000 = Rs. 15,000 = Rs. 500/unit

Rs. Lakhs

3. The set-up hours will have to be required to the extent of 100 hours although the requirement is just 25 hours. Thus variable activity rate of Rs. 8/hour will be applicable for 25 hours and fixed activity rate of Rs. 50/hour will be applied to 100 hours.

4. It needs to be considered beforehand as what will happen to the addition in fixed activity in set-up activity requiring Rs. 5000 (Rs. 50/hr x 100 hours). If there is likelihood of such order coming regularly in future, such additions may be worth while otherwise the additions need to evaluated in the light of enhanced income, if any, due to this special order.

1. It does not affect the regular sales. The company is suffering from idle capacity only in respect of machine hours and not in respect of materials, labour and set-ups. The contribution granted by these vectors must be examined before any acceptance of this order.

2. The company is entering new geographical region via this order, what is market of this product in that region? Whether the existing distribution channels will or can accommodate this order. This must be examined.

4. What is the nature of additional set-up hours? If the fixed costs required for this order, remain a continuous burden on the company, this order should not be accepted.

If the company has 50 sparable set-ups, there will be no need to acquire additional 100 hour block requiring Rs. 5,000. The order should be undoubted accepted as this will enhance the revenue to an extent of Rs. 7,000.

Problem 14 (cwa final dec. 05)

A company manufactures two products using common material handling facility. The total budgeted material handling cost is Rs. 60,000. The other details are:

Problem 15 (ca final may 02)

ABC electronics makes audio player model 'AB 100'. It has 80 components. ABC sells 10,000 units each month at Rs. 3,000 per unit. The cost of manufacturing is Rs. 2,000 per unit or Rs. 200 lakhs per month for the production of 10,000 units. Monthly manufacturing costs incurred are as follows :

Page 463: cost accounting question icwai inter

Direct materials costs 100.0Direct manufacturing labour costs 20.0Machining costs 20.0Testing costs 25.0Rework costs 15.0Ordering costs 0.2Engineering costs 19.8

200.0

The following additional information is available for 'AB 100'.(i) Testing and inspection time per unit is 2 hours.(ii) 10% of 'AB 100' manufactured are reworked.(iii) It currently takes 1 hour to manufacture each unit of 'AB 100'.

(iv) ABC places two orders per month for each component. A different supplier supplies each component.

Activity Description Cost driver Cost per unit of cost driver1. Machining cost Machining components Machine hours of Rs. 200

capacity2. Testing costs Testing hour Rs. 125

3. Rework costs Correcting and fixing errors and defects Rs. 1,500 per unit

4. Ordering costs Ordering of components Rs. 125 per order

5. Engineering cosDesigning of products and processes

(i) The member of components will be reduced to 50.(ii) Direct materials costs to be lower by Rs. 200 per unit.(iii) Direct manufacturing labour costs to be lower by Rs. 20 per unit.(iv) Machining time required to be lower by 20 per cent.(v) Testing time required to be lower by 20 per cent.(vi) Rework decline to 5 per cent.(vii) Machining capacity and engineering hours capacity remain the same.ABC currently out sources the rework on defective units.Required:(i) Compare the manufacturing cost per unit of 'AB 100' and 'AB 200'.

(ii) Determine the immediate effect of design change and pricing decision on the operating income of ABC..

Solution :

Labour is paid on piece rate basis. Therefore, ABC considers direct manufacturing labour cost as variable cost.

ABC has identified activity cost pools and cost drivers for each activity. The cost per unit of the cost driver for each activity cost pool is as follows:

Testing components and finished products. (each unit of 'AB 100' is tested individually)

Units of 'AB 100' reworked

No. of orders

Engineering hours

Rs. 1,980 per engg. hourOver a long-run horizon, each of the overhead costs described above vary with chosen cost drivers. In

response to competitive pressure ABC must reduce the price of its product to Rs. 600 and to reduce the cost by Rs. 400 per unit. ABC does not anticipate increase in sales due to price reduction. However, if it does not reduce price it will not be able to maintain the current sales level.

Cost reduction on the existing model is almost impossible. Therefore, ABC has decided to replace 'AB 100' by a new model 'AB 200', which is a modified version of 'AB 100'. The expected effect of design modifications are as follows:

Ignore Income tax. Assume that the cost per unit of each cost driver for 'AB 100' continues to apply to 'AB 200'.

Page 464: cost accounting question icwai inter

Tutorial notes :

3. Same logic should be adopted for computing ordering cost per unit. Do it yourself.

The solution goes as follows :Working Notes: 1. Machining hours and cost for 'AB 200' :

Rs.Cost per unit per machine hour for 'AB 100' 200Cost per unit per machine hour for 'AB 200' 160

Rs.Cost per unit per testing hour for 'AB 100' 200Cost per unit per testing hour for 'AB 200' 160

3. Rework cost per unit : Rs.Rework units 5% of 10,000 units 500Rework cost @ Rs. 1,500/unit 750,000Rework cost per unit 750,000 / 10,000 7.5

4. Computation of Ordering cost per unit2 orders per month are placed for each component and no. of components in 'AB 200' is 50.

No. of orders : 50 x 2 = 100. Ordering cost is Rs. 125 per order.Rs.

Ordering cost per month (100 x 125) 12,500Ordering cost per unit (12,500 / 10,000) 1.25

5. Computation of cost per unit Rs. Lakhs Rs./ unitDirect materials costs 100.0 1,000Direct manufacturing labour costs 20.0 200Machining costs 20.0 200Testing costs 25.0 250Rework costs 15.0 150Ordering costs 0.2 2Engineering costs 19.8 198

200.0 2,000Answer (i) Comparison of Manufacturing cost per unit

AB 100 AB 200Rs. Rs.

Direct materials costs 1,000 800Direct manufacturing labour costs 200 180Machining costs 200 160

1. What is machine hour and testing hour required for 'AB 100' and 'AB 200'? Read the question again and find out. Show this by way of working note.

2. Rework cost is to borne by the units sold. 5% of 10,000 units i.e. 500 units require to be reworked @ Rs. 1,500 per unit. The reworking cost works out to Rs. 750,000 which will be distributed to 10,000 units. Thus rework cost per unit would be Rs. 7.50. This is your working note.

4. Part (ii) of the question relates to saving in cost due to new model 'AB 200'. The revenue loss is Rs. 400 per unit for 10,000 units which works out to Rs. 40 lakhs. Savings are on account of (i) Direct materials (ii) Direct manufacturing labour costs (iii) Rework costs. The difference of revenue loss and cost saving is effect on operating income.

5. This question is an easy one and you should try to solve it yourself before seeing the solution which goes as follows.

In modified model 'AB 200', the saving of time is 20%. It currently takes one hour to produce one unit of 'AB 100'. Thus it will take 0.80 hour to produce one unit of 'AB 200'. Saving in time and consequently in cost would be 20%.

2. Testing and inspection time of 2 hour is reduced by 20% in modified model. Thus testing cost in modified model would be 80% of current model.

Page 465: cost accounting question icwai inter

Testing costs 250 200Rework costs 150 75Ordering costs 2 1.25Engineering costs 198 198

2,000 1,614Answer (ii) Effect of design change and pricing change on operating income of ABC

no. of units Rs. Rs. Lacs1. Revenue loss due to reduction in price 10,000 400 (40)2. Saving in cost :Direct materials costs 10,000 200 20.0Direct manufacturing labour costs 10,000 20 2.0Rework costxs (5% of 10,000 units) 500 1,500 7.5Total savings 29.5Net loss 11Conclusion :

1. The operating income per month will be reduced by Rs. 10.50 lakhs.

Problem 16 ( CWA Inter June 08)A company manufactures two products using common facilities. The following cost data for a month is presented to you:

A BUnits produced 1,000 2,000Direct labour hours per unit 2 3Machine hours per unit 6 1.5Set up of machines 15 50Orders 18 70Machine activity expenses Rs. 3 lacsSet up relating expenses Rs. 30,000Orders relating expenses Rs. 35,000Calculate the overheads per unit absorbed using activity based costing approach.Solution :Machine expenses will be absorbed as Total expenses/ total hoursSet up expenses will be absorbed as Total expenses/ total set upsOrders relating expenses will be absorbed as Total expenses / total ordersTotal machine hours : 1000 x 6 + 2,000 x 1.5 = 9,000 hoursTotal set ups : 15 + 50 = 65Total orders : 18 + 70 = 88Overheads absorption rates are : Machine activity 300,000 / 9,000 Rs. 34 per hour

Set ups 30,000 / 65 Rs. 462 per set upOrders 35,000 / 88 Rs. 398 per order

Overheads absorbed by products A and B are as per the following table :A B Total amt.

Quantity rate Rs. Amount Quantity rate Rs. AmountMachine hours 6,000 34 204,000 3,000 34 102,000 306,000Set ups 15 462 6,930 50 462 23,100 30,030Orders 18 398 7,164 70 398 27,860 35,024Overheads absorbed 218,094 152,960 371,054The actual overheads is Rs. 365,000. The difference is due to approximation.

2. The reduction in machining time, testing time is hard to materialize in practical terms. The existing facilities are not tuned to such changes. The effect might take some time to manifest.

       Be Brave, if not, pretend to be, No one will notice the difference.

·       vehj cuuk vklku ugha gS ij xjhch esa ;k de iSlksa esa xqtj clj djuk Hkh dkSu lk vklku gS

Page 466: cost accounting question icwai inter

Total

16

Page 467: cost accounting question icwai inter

A company manufactures two products using common facilities. The following cost data for a month is presented to you:

Page 468: cost accounting question icwai inter
Page 469: cost accounting question icwai inter
Page 470: cost accounting question icwai inter

Chapter 14: Transfer Pricing cwa inter cwa final ca inter ca final Total5 5 0 3 13

There are basically two methods for fixation of transfer; Cost based price and market based pricing

The possible cost per unit of Division Y's finished product is given below: Rs.Other purchased component 500Component to be supplied by division X 120Other variable overheads 320Fixed overheads 180Total cost per unit 1,120

(ii) Would it be an economic advantage for Division Y to buy @ Rs. 180 ?Solution:

If division Y procures the component @ Rs. 180, its variable cost becomes: Rs.

vkidks bl nqfu;k ds gj O;fDr ds fy;s ogh dkeuk djuk gS tks vki vius fy;s pkgrs gSa vFkkZr lsgr] [kq'kh vkSj thou dh lkjh fu;kersasaAA Transfer price is that notional value at which goods and services are transferred between divisions in decentralised organization. fixation of transfer price is essential for measurement of performance of a division.

In cost based pricing, Actual cost of production, Full cost plus a mark up, standard cost, marginal cost, are basic parameters to arrive at transfer price.

Under market pricing methods, the market price will be determined according to the market price prevailing in the market price. As no selling efforts are required, usually selling and distribution overheads should be deducted and price thus arrived at should be charged as transfer price.

It may also happen that the selling division may notice the price offered by the buying division is less than the price at which the product can be sold in the open market. Alternatively the buying division may find that it can procure the product from the market at price lower than that charged by selling division. In all such conflicts, it is in the interest of the organisation that the buying division and selling division should negotiate the price. Such prices are known as 'negotiated price'.

Opportunity cost pricing: This pricing recognises the minimum price that the selling division is ready to accept and the minimum price the buying division is ready to pay. The transfer price is fixed between these two limits.

Problem 1 (cwa inter II June 03)

A company AB Ltd. has two divisions viz. X and Y. Division X manufactures a special type of electrical component and Division Y sells a finished product for which it requires one component per unit from X division. Division X sells the component in the external market @ Rs. 180 per unit and division X is also working at almost its full capacity. The variable cost of manufacturing per component is Rs. 102.

Division Y is now operating at 50% capacity. It has received a special order for its product. Division Y is keen to get this order. Division Y will meet the special order at a price of Rs. 1,200 per unit and it offers a price of Rs. 120 per component to Division X.

(i) As a manager of Division X, what decision would you like to take regarding the offer by Division Y for the supplies @ Rs. 120 ?

(i) Since Division X is operating at full capacity and can sell its product @ Rs. 180 per unit, any price below Rs. 180 would result in contribution loss to that extent. Obviously when division X can sell the product to others @ Rs. 180 per unit, it should refuse the offer price of Rs. 120 by division Y.

(ii) The price structure of Y's finished product contains fixed cost on per unit basis which is incorrect. The fixed cost is a sunk cost for this special order and therefore should be kept out of decision making process.

Page 471: cost accounting question icwai inter

Other purchased component all fig. in Rs. 500Component to be supplied by division X 180Other variable overheads 320Total variable cost per unit 1,000Contribution at selling price of Rs. 1200 per unit 200

Required :(i) Should the manager of division M transfer its products at Rs. 22 to division N ?(ii) What is the lowest price that the division M should accept ?Solution:Tutorial notes:

The solution goes as follows:Comparative profitability statement of Division M under different options:Particulars Option 1 Option 2 Option 3

Sell at Rs. 25 Sell at Rs.25 Do not transferTransfer at Rs. 22

Sales revenue: Rs. In lakhs Rs. In lakhs Rs. In lakhsMarket sales of 50,000 units. 12.5 12.5 12.5Transfer of 10,000 units. 2.5 2.2 0.0Total revenue 15.0 14.7 12.5Variable cost @ Rs. 15/unit 9.0 9.0 7.5Fixed cost 3.0 3.0 2.6Total cost 12.0 12.0 10.1Profit 3.0 2.7 2.4Total Investment 12.0 12.0 10.0Return of investment 25% 22.50% 24%

Thus even when division Y pays Rs. 180 for the component to division X, it manages to receive Rs. 200 per unit as contribution, it should accept this price.

Problem 2 (cwa inter II June 03)

Royal industries Ltd. has two divisions M and N. Division M manufactures product A-15 which it sells in outside market as well as to division N which processes it to manufacture Z-25. The manager of division N has expressed the opinion that transfer price is too high. The two divisional managers are about to enter into discussions to resolve the conflict, and manager of division M to supply him with some information prior to discussions.

Division M has been selling 50,000 units to outsiders and 10,000 units to division N, all at Rs. 25 per unit. It is not anticipated that these demands will change. The variable cost is Rs. 15 per unit and the fixed costs are Rs. 3 lakhs. Divisional investment in assets is Rs. 12 lakhs.

The manager of division M anticipates that division N will want a transfer price of Rs. 22. If he does not sell to division N, Rs. 40,000 of fixed cost and Rs. 200,000 of assets can be avoided. The manager of division M would have no control over the proceeds from the sales of the assets and is judged primarily on rate of return.

The performance of division is judged by return on investments. There are three options available with division M. First option is to sell and transfer the product at a uniform selling price of Rs. 25. The second price is to sell @ Rs. 25 and transfer @ Rs. 22 to division N. The third option is to sell 50,000 units @ Rs. 25 and do not transfer and reduce Rs. 40,000 on fixed cost and Rs. 200,000 on investments.

The parameter for judging the performance of division M is on RIO basis. The ROI is highest at 25% with option 1 which is to sell and transfer at a uniform price of Rs. 25 per unit. Thus manager of division M should not accept to transfer at Rs. 22 per unit to division N as it brings down its ROI from 25% to 22.50%.

Page 472: cost accounting question icwai inter

Total revenue required is computed as follows: Rs. LakhsReturn of investment of Rs. 12.0 lakhs @ 24% 2.88Variable cost @ Rs. 15/unit of 60,000 units (50,000 + 10,000) 9.00Fixed cost 3.00Total revenue required 14.88Less: Revenue from sell in the market of 50,000 unit @ Rs. 25/unit 12.50Revenue required from transfer of 10,000 units 2.38Transfer price of 10,000 units in Rs. 23.80The lowest price acceptable to division M is Rs. 23.80.

Rs. Purchased parts−outside vendors 180Purchased parts−from division DA 40Other variable costs 100Fixed overheads including administration 80

400

Required :

(iii) Suggest an alternative transfer price and show how could it lead to goal congruence?Solution:

Contribution analysis for company as a whole:Particulars Sold externally Transfer to DB in Rs. Selling price 60 440Less: Variable cost Component from division DA 34 34

(ii) The lowest price that the manager of division M can accept to transfer to division N is the price which maintains the ROI of division M at 24% (do not transfer option).

Problem 3 (cwa inter II June 05)

Novelty ltd. has two divisions− division DA and division DB. Division DA is currently operating at full capacity. It has been asked to supply its product to division DB. Division DA sells its product to its regular customers for Rs. 60 each.

Division DB, currently operating at 50% capacity, is willing to pay Rs. 40 each for the component produced by division DA ( this represents the full absorption cost per component at division DA). The component will be used by division DB in supplementing its main products to conform to the need of special order. As per the contract terms of sale, the buyer calls for reimbursement of full cost to division DB, plus 10%.

Division DA has a variable cost of Rs. 34 per component. The cost per unit of division DB subsequent to the buying part from division DA is estimated as follows:

The company uses return on investment ROI in the measurement of performance of division manager.

(i) As manager of division DA would you recommend sales of your product to division DB at the stipulated price of Rs. 40 ?

(ii) Would it be in the overall interest of the company for division DA to sell its product to division DB?

(i) The division DA is operating at full capacity and selling its product at a market selling price of Rs. 60 per unit. Any price below Rs. 60 would lead to corresponding loss of contribution and profit of division DA. As the performance of division is measured in terms of ROI, any price below Rs. 60 would lower the ROI. Thus sales at price of Rs. 40 (resulting in a loss of profit of Rs. 20 per unit for DA) to division is not recommended at all.

(ii) The buyer is ready to pay to division, its full cost and 10%. The full cost is Rs. 400. Thus the selling price for division DB is Rs. 440.

Page 473: cost accounting question icwai inter

Purchased from outside − 180Other variable cost − 100Total variable cost 34 314Total profit 26 126

Thus the cost structure of the product would be : Rs. Purchased component from division DA 60Purchased from outside 180Other variable cost 100Fixed overhead administration 80Total cost per unit 420Add: Profit @ 10% 42Total selling price 462Contribution ( Rs. 80 + Rs. 42 ) 122

Rs. Fixed assets 500,000Current assets (other than debtors) 300,000Debtors 200,000Annual fixed cost of the division 800,000Variable cost per unit of the product 10Budgeted volume of production per year in units 400,000Desired return on investment 28%You are required to determine the transfer price for the Division A.Solution:You should try to solve this problem mentally. The mental process may be as follows;

Thus individually the component yields a contribution of Rs. 26 per unit if sold in the market at Rs. 60 per unit while if supplied at a variable cost of Rs. 34 per unit to division DB, the contribution to the company goes up to Rs. 126. Thus the transfer must take place, in the overall interest of the company, as it would enhance the existing profit by Rs. 100 per unit.

(iii) The dual price policy for fixing transfer price is most appropriate in such a case. The transfer price of division DA should be at its selling price i.e. Rs. 60 per unit which the division is otherwise getting. The full absorption cost of Rs. 40 of the product to division DA, should be the transfer price for division DB.

Problem 4:(cwa inter II June 2006)

A company fixes the inter-divisional transfer prices for its products on the basis of cost plus an estimated return on investment in its divisions. The relevant portion of the budget for the Division A for the year 2006−07 is given below:

Total assets in the division is 10 lakhs, 28% of this is 2.8 lakhs. Thus return required on investment is Rs. 2.8 lakhs from 4 lakh units giving Re.0.70 per unit as return required. The variable cost per unit is given as Rs. 10 and the fixed cost per unit is 800,000 / 400,000 i.e. Rs. 2. Thus transfer price should be fixed at Rs. ( 0.70 + 10 + 2 ) Rs. 12.70 per unit.

Problem 5:(cwa inter II dec.2006)

Magatron Ltd. is having production shops which are cost centers. Each shop charges other shops for material supplied and services rendered.

The shops are motivated through goal congruence, autonomy and management efforts. Magatron ltd. is having a welding shop and painting shop. The welding shop welds annually 750,000 purchased items with other 1500,000 shop made parts into 120,000 assemblies.

The assemblies are having variable cost of Rs.19 each and are sold in the market at Rs. 24 per assembly. Out of the total production, 80% is diverted to painting shop at same price ruling in the market. Welding shop incurs a fixed cost of Rs. 500,000 per annum.

Page 474: cost accounting question icwai inter

Requirements:1. Find out profit of individual cost centers and overall profitability of the company.

Give reasons for your recommendations.

Solution: Rs. In lacs.1. The profitability statement of both the shops is given below:

Welding shop Painting shopQuantity Rate Rs. Value (lac) Quantity Rate Rs. Value (lac)

Sales to painting shop 80% 96,000 24 23.04 96,000 50 48.024,000 24 5.76

Total sales 28.80 48.00Less Variable cost 120,000 19 22.80 96,000 40 38.40Contribution 6.00 9.60Less Fixed cost 5.00 6.00Profit 1.00 3.60Overall profit for the company = Rs. 4.60 lac.

The variable cost of the unit of Painting shop can also be computed as:

Variable cost = Purchase price from the market + Own variable cost

Variable cost = Rs. 20 + ( Rs. 40 − transfer price from welding shop Rs.24) = Rs. 36.00

Profitability statement when Painting shop buys from open market @ Rs. 20/unitWelding shop Painting shop

Quantity Rate Rs. Value (lac) Quantity Rate Rs. Value (lac)Sales 24,000 24 5.76 96,000 50 48.00Less Variable cost 24,000 19 4.56 96,000 36 34.56Contribution 1.20 13.44Less Fixed cost 5.00 6.00Profit (loss) (3.80) 7.44Overall profit for the company = (7.44 − 3.80) = Rs. 3.64 lacs.3. Profitability statement when transfer price is Rs. 20

Welding shop Painting shopQuantity Rate Rs. Rs. Lacs Quantity Rate Rs. Value (lac)

Sales 24,000 24 5.76 96,000 50 48.00Transferred to painting shop 96,000 20 19.20Total sales 120,000 24.96 96,000 48.00Less Variable cost 120,000 19 22.80 96,000 36 34.56Contribution 2.16 13.44Less : Fixed cost 5.00 6.00Profit (loss) (2.84) 7.44Overall profit for the company = (7.44 −2.84) = Rs. 4.60 lacs

Comparative Statement of three alternativesWelding shop Painting shop Overall profit

Profit under alternative 1 1.00 3.60 4.60

The painting shop is having a fixed cost of Rs. 600,000 and its cost of painting including transfer price from welding shop comes to Rs. 40 per unit. The painting shop sells all units transfer to it welding shop at Rs. 50 per assembly.

2. Recommend course of action, if painting shop wishes to purchase its full requirement either from open market @ Rs. 20 per assembly or from welding shop at Rs. 20 per assembly.

2. The painting shop buys @ Rs. 24/unit from the welding shop. It is buying from the open market @ Rs. 20/unit. Thus variable cost in this case will go down from Rs. 40/unit to Rs. 36/unit.

Page 475: cost accounting question icwai inter

Profit under alternative 2 (3.80) 7.44 3.64Profit under alternative 3 (2.84) 7.44 4.60

Division A Division BComponent units no. 50,000Finished units no. 50,000Total Variable cost Rs. 250,000 600,000Fixed cost Rs. 150,000 200,000The fixed costs are separable for each division.

Circumstance 1

Circumstance 2

Circumstance 3

Solution: Circumstance 1

Circumstance 2 Transfer price = variable cost and contribution of A + Profit of C.

Circumstance 3 Transfer price = variable cost and fixed cost of A + variable cost of B.Transfer price = Rs. 250,000 + Rs.150,000 + Rs. 600,000 = Rs. 10,00,000 for 50,000 units.Thus transfer price would be Rs. 20 per unit.

Recommendation: Although profit under alternative 1 and 3 are same, alternative 3 is recommended because this ensures optimum utilisation of resources of painting shop. The loss of Rs. 2.84 lacs of welding shop is due to its fixed cost of Rs. 5 lacs. The overall goal congruence policy of the company is followed.

Problem 6:(cwa final june.2000)

D ltd. has two divisions. Division A manufactures a component which is used by Division B to produce a finished product. For the next period, output and costs have been budgeted as follows:

You are required to advise on the transfer price to be fixed for Division A's component under the following circumstances :

Division A can sell the component in a competitive market for Rs. 10 per unit. Division B can also purchase the component on the same price from the open market.

Same situation as above but assume that Division B currently buys the component from an external supplier at the market price of Rs. 10 and there is a reciprocal agreement between the external supplier and another Division C, within the group. Under this agreement the external supplier agrees to buy one product unit from Division C, at a profit of Rs. 4 per unit to that division, for every component which Division B buys from the supplier.

If the transfer from Division A to Division B is made at full cost, what is the minimum price that Division B could sell the finished product for, and would this price be in the best interest of the organisation as a whole ?

If component is transferred to Division B from Division A, The transfer price should cover the variable cost of A and contribution for A. Thus Rs. 10/unit should be the transfer price for this case.

Thus transfer price should be fixed at Rs. 5 + Rs. 5 + Rs. 4 = Rs. 14. As Division B can get this component at Rs. 10 from the open market, this transfer price would naturally be not acceptable to Division B and it will continue to buy the component from the open market.

It is not prudent to fix price on total cost basis. In this case the fixed cost of Division A is treated as variable cost for Division B which is not in the best interest of the organisation as a whole. The variable cost should form the basis of fixation of price.

The variable cost per unit of Division A is Rs. 5 and that of Division B is Rs. 12. Thus Rs. 17 per unit be the price which is in the best interest of the organisation as a whole. Fixed cost should be recovered through contribution but it should not form the groundwork for fixation of price.

Problem 7:(cwa final dec..2001)

Page 476: cost accounting question icwai inter

Products D E F Market price / unit Rs. 240 230 200Variable cost / unit Rs. 168 150 140Direct labour hours / unit 6 8 4Sales units 4,000 2,500 1,500Division B requires 1000 units of product E for use in one of its product.Required :

(i) 38,000 direct labour hours (ii) 56,000 direct labour hours .Solution: First of all let us see whether the labour hours are limiting or not.Products D E F Direct labour hours / unit 6 8 4Sales units 4,000 2,500 1,500Total hours required 24,000 20,000 6,000 50,000

Products D E F Market price / unit Rs. 240 230 200Variable cost / unit Rs. 168 150 140Contribution / unit Rs. 72 80 60Direct labour hours / unit 6 8 4Contribution per hour 12 10 15Ranking 2nd 3rd IstAllocation of hours in order of ranking 24,000 8,000 6,000Thus E can be given only 8,000 hours which are adequate to produce 1000 units.

In the case when available direct labour hour is 56,000, this is no longer a limiting factor.

Present direct labour hours 50,000Addition demand for 1000 units 8,000Total hours required 58,000Available hours 56,000Shortfall 2,000

Transfer price would then be aggregate of Rs. Opportunity cost of 2000 hours @ Rs. 10/unit 20,000For variable cost @ Rs. 150 for 1000 units 150,000Transfer price for 1000 units 170,000Transfer price per unit 170

A company is organized into two production departments A and B. Department A manufactures three products as per data given below:

Determine the transfer price which Division A should charge for its supplies of product E to Division B if the capacity of Division A is :

In the case when available direct labour hour is 38,000, this becomes limiting factor. Contribution analysis per limiting factor has to be done to decide the ranking of the products.

These 1000 units can otherwise be sold in the market because the demand exists, the transfer price would be the market price i.e. Rs. 230 per unit.

Additional demand of 1000 units over and above the existing demand will require 8000 more hours of direct labour.

Contribution of 2000 labour hours will have to be sacrificed for meeting the demand of Division B. Opportunity cost would come into the scene.

Problem 8:(cwa final June 06)

Page 477: cost accounting question icwai inter

Estimated Profit Statement of Fairdeal ltd. for the current quarter (July-Sept. 06) in Rs.'000Wholesale Retail Company

External sales 1,200 t @ Rs. 150 180 0 180120,000 pcs @ Rs. 2.67 0 320 320Internal transfer 1,200 t @ Rs. 90 108 (108) 0Variable cost 2,400 t @ Rs. 58.33 (140) (140)120,000 pcs @ Rs.0.50 0 (60) (60)Fixed cost (100) (40) (140)Profit 48 112 160

Solution:

Let us find the Pv ratios for both the divisions as per the estimated profit statement:W R Rs. '000

External Sales 180 320Variable cost 70 70Variable cost at R 60Contribution 110 190Pv ratio 61.1% 59.4%

Pv ratio For W = ( T − 70 ) / T and For S = (320 − T − 60)/ 320 We thus get the following:(T− 70) / T = (320−T−60)/320 Which gives T = Rs. 122,600.

This is transfer price for transfer of 1,200 tones of material from W to S.Transfer price per tonne = Rs. 122,600 / 1,200 = Rs. 102.17/tonne.Statement showing the Pv ratio of W and S divisions with transfer price of Rs. 102.17/tonne

W S Company Rs.'000 Rs.'000 Rs.'000

Sales 0 320 320Transfer 122.6 122.6 0

'When goods are passed between divisions of an organisation, a certain transfer price is needed so that no sub-optimal or dysfunctions results ensue………one way to attain this objective is to aim at the same contribution margin ratio (P/V ratio on the goods subject to internal transfer for both the transferor division and the transferee division.''

The Managing Director of Fairdeal Limited has just attended a lecture on Transfer pricing in a seminar organized by ICWAI. The above is a quote from the said lecture.

Fairdeal has two divisions, Wholesale (W) and Retail (R). As per the existing rule, W sells its products @ Rs. 150 per tonne in the external market but supplies identical product @ Rs. 90 per tonne to R. R uses the transferred material as captive consumption and produces finished goods in the ratio of 1 tonne input : 100 pieces of output. R sells all its output in the external market. W insists that R be charged at market price, to which R does not agree.

The MD of the Fairdeal Ltd. requests you to find the transfer price of the goods that his wholesale division supplies to the Retail division, conforming to the principle laid out in the above quote.

For your information the Accountant of Fairdeal Ltd. furnishes you with an estimated Profit Statement of the company as under:

Note: Your answer should (i) highlight the need for a change in the ruling transfer price and (ii) show that the suggested transfer price meets the MD's requirement.

The MD is reasonable when he requires the PV ratio of W and S divisions same. At present the difference in selling price and transfer price for division W is the problem between the two divisions.

As is seen from the above, the Pv ratios of both the divisions are different. As directed by the MD the transfer price should be such to equate the Pv of both the divisions. Let T be the transfer price at which the Pv's equate.

Page 478: cost accounting question icwai inter

Variable cost 70 60 130Contribution 52.6 137.4 190Pv ratio 42.90% 42.90% 59.40%

Bright LightRs. Rs.

Direct materials 10 5Direct labour and variable overhead @ Rs. 35/hour 140 35

C assembles 'Tight' by using imported component. The annual fixed cost of division C amounts to Rs. 4,00,000.

The product cost data is as under:Imported component : Rs. 300 Direct materials : Rs. 40 Selling price : Rs. 700Direct labour and variable overhead @ Rs. 25/hour Rs. 250

(i) Division C imports 5,000 components for producing 'Tight'.(ii) Division C substitutes 'Bright' by drawing 'Bright' from division A at Rs. 300 per unit.(iii) Same condition as in (ii) above but Division A supplies 'Bright' at Rs. 250 per unit.Solution:Contribution from the three products is as follows: Bright Light TightSelling price Rs./unit 300 60 700Variable cost Rs./unit 150 40 590Contribution Rs./unit 150 20 110Production time required per unit in hours 4 1

Available hours in division A = 100,000. Hours required for permanent customer : 15,000 x 4 = 60,000 hours.Balance Hours available : 40,000. Possible production of Light : 40,000 units.(i) Profit Statement when Division C imports the component:

Division A Division C CompanyContribution from 15,000 units of Bright 2,250

40,000 units of Light 8005,000 units of Tight 550

Less : Fixed cost 2,000 400Profit 1,050 150 1,200(ii) Profit statement when Division A supplies Bright @ Rs. 300/unit

The overall Pv ratio of the company remains unaltered while the Pv ratios of both the divisions is equal, as required by the MD.

Problem 9:(cwa final June 07)

A company has two manufacturing divisions, A and C. Each division operates as an independent profit centre. A, which produces two components 'Bright' and 'Light', has a capacity of 100,000 hours per annum. The annual fixed cost of A amount to Rs. 20 lacs. The product cost data is as under:

A has a permanent customer for the purchase of 15,000 units of Bright per annum, at a selling price of Rs. 300 per unit. The balance capacity is devoted to the production of Light, for which there is unlimited sales potential at a price of Rs. 60 per unit.

With a view to minimizing dependence of imported components, the possibility of using 'Bright' was explored. It was found that the import substitution was possible with slight modification, which will take two extra hours per unit of 'Tight' in Division C. The production of 'Tight' is estimated at 5,000 units per annum.

You are requested to present the division-wise profit statement and the company's profit as a whole on the basis of following conditions:

Page 479: cost accounting question icwai inter

Division A Division C CompanyContribution from rate Rs./unit Rs.'000 Rs.'000 Rs.'000

15,000 units of Bright 150 2,2505,000 units of Bright from C 150 750

20,000 units of Light 20 4005,000 units of Tight 60 300

Less : Fixed cost 2,000 400Profit / Loss 1,400 (100) 1,300

Division A Division C CompanyContribution from rate Rs./unit Rs.'000 Rs.'000 Rs.'000

15,000 units of Bright 150 2,2505,000 units of Bright from C 100 500

20,000 units of Light 20 4005,000 units of Tight 110 550

Less : Fixed cost 2,000 400Profit / Loss 1,150 150 1,300

Fixed overheads : Rs. 4,00,000. Variable cost : Rs. 1/unitAverage Assets : Sales Debtors : Rs. 2,00,000. Stocks : Rs. 6,00,000.

Plant and other asset ; Rs. 4,00,000

Solution:25% is required on assets employed in Division P.Total assets employed in division P = Rs. 12 lakh. The return required would be Rs. 3 lakh.Sales required to achieve this profit = Fixed cost + Variable cost + profit = Rs. 9 lakhs.The selling price would be Rs. 9 lakhs/ 2 lakh units = Rs. 4.50/unit.Comparative statement of two options available to division B (amount in Rs. 000Sales Transfer to B No transfer to B

The balance hours available in division A : 40,000. If division A supplies 5,000 additional units of Bright to division C, hours required would be 5,000 x 4 = 20,000. Thus balance 20,000 hours would be available to produce C @ 1hr/unit.

The contribution from Tight is Rs. 110/unit as computed above. If Tight is assembled in division C in two extra hours of machining @ Rs. 25/hr, the contribution per unit will be Rs. 110 − Rs. 50 = Rs. 60/unit.

(iii) Since the transfer price is Rs. 50 less than in the case (ii) above, the profit of Division A will be reduced by Rs. 50 x 5000 = Rs. 250,000. The contribution of division C will be increased by Rs. 250,000. The following table will be applied.

Problem 10:(cwa final June 04)

Division P of Wider Horizons Ltd. has been given a budgeted target of selling 2 lakhs components, 'Santran 84' that it manufactures, at a price which would result in a return of 25% on the average assets employed by it. The following figures are relevant:

However, the marketing department of the company finds out by a survey that the maximum number of Santran 84 that the market can take at the proposed price is only 1,40,000 units.

Fortunately, Division B of the company is willing to purchase the balance 60,000 units. The Manager of Division P is willing to sell to Division B at a concessional price of Rs. 4 per unit. But the Manager of Division B is ready to pay Rs. 2.25 only per unit, as he feels he can himself make Santran 84 in his division at that price. Rather than sell to Division B at Rs.2.25, the Manager of Division P feels he will restrict the activity of his division to the manufacture and sales of 1,40,000 units only. But this could reduce Rs.80,000 in stocks, Rs. 1,20,000 of plant and other assets and Rs.40,000 in selling and administrative expenses.

You are required to work out the various computations and show that selling 60,000 Santran 84 to Division at Rs. 2.25 per unit would be in the overall interest of the company.

Page 480: cost accounting question icwai inter

140,000 units x Rs. 4.50 630 63060,000 units x Rs. 2.25 135 0

765 630Fixed overheads 400 360Variable overheads @ Rs..1/unit 200 140

600 500Profit 165 130Capital employed 1,200 1,000ROI 13.75% 13%ROI is greater if Santran 84 is transferred to B at Rs. 2.25/unit. Transfer to B is preferable.

Printing Division Component DivisionRs.'000 Rs.'000

Sales Revenue 6,000 6,400Manufacturing cost:

Component 1,050 0Other direct materials, direct labour and variable overhead 1,680 1,920

Fixed Overhead 480 704Total manufacturing overhead 3,210 2,624Gross margin 2,790 3,776Variable marketing expenses 270 384Fixed marketing and administrative overhead 855 704Non-manufacturing cost 1,125 1,088Operating profit 1,665 2,688

Also, for an increase in capital employed by Rs. 2 lakhs the enhancement in profit is Rs. 35,000 which is 17.5%. This shows that selling to B at Rs. 2.25/unit is in the overall interest of the company.

Problem 11:(ca final may 07)

Hardware Ltd. manufactures computer hardware products in different divisions which operate as profit centers. Printer division makes and sells printers. The Printer Division's budgeted income statement, based on a sales volume of 15,000 units is given below. The Printer Division's Manager believes that sales can be increased by 2,400 units, if the selling price is reduced by Rs. 20 per unit from the present price of Rs.400 per unit, and that, for this additional volume, no additional fixed costs will be incurred.

Printer Division presently uses a component purchased from an outsider supplier at Rs. 70 per unit. A similar component is being produced by the Components Division of Hardware Ltd. and sold outside at a price of Rs.100 per unit. Components Division can make this component for the Printer Division with a small modification in the specification, which would mean a reduction in the direct material cost for the Components Division by Rs. 1.5/unit. Further, the Component Division will not incur variable selling cost on the units transferred to the Printer Division. The Printer Division's Manager has offered the Component's Division's Manager a price of Rs.50 per unit of the component.

Component Division has the capacity to produce 75,000 units, of which only 64,000 can be absorbed by the outside market.

The current budgeted income statement for Components Divisions is based on a volume of 64,000 units considering all of it as sold outside.

(i) Should the Printer Division reduce the price by Rs. 20 per unit even if it is not able to procure the components from the Component Division at Rs. 50 per unit?

(ii) Without prejudice to your answer to part (i) above, assume that Printer Division needs, 17,400 units and that, either it takes all its requirements from Component Division or all of it from outside source. Should the Component Division be willing to supply the Printer Division at Rs. 50 per unit?

Page 481: cost accounting question icwai inter

Support each of your conclusions with appropriate calculations.Answer :(i) The first part of the answer is very easy. Rs./unit Rs.'000Present Sales of 15,000 units @ Rs. 400 amount to 400 6,000Component 70 1,050Other direct materials, direct labour and variable overhead 112 1,680Variable marketing expenses 18 270 3,000Contribution with present price 3,000Sales if price is reduced by Rs. 20, 17,400 units @ Rs. 380 6,612Component 70 1,218Other direct materials, direct labour and variable overhead 112 1,949

Variable marketing expenses 18 313.2 3,480Incremental contribution 3,132Contribution with reduced price 132

Alternatively: Rs.Contribution margin of sales increase (Rs. 180 x 2,400) 432,000Loss of contribution due to reduce price (Rs. 20 x 15,000) 300,000Incremental contribution 132,000

Rs.'000Loss of contribution due to loss of sale to outsiders @ Rs. 64 for 6,400 units 409.6Contribution from sales to Printer Division Rs.21.50 for 17,400 units 374.1Decrease in contribution 35.5No, the Component Division should not sell all 17,400 units to Printer Division for Rs. 50.(iii) Increase in contribution if the component is transferred within the company, Rs.'000

(3480 − 3132) 348.0Contribution to the C.Division 374.1Total increase in contribution : 722.1Less: Contribution lost by C.Division 409.6Net increase in contribution : 312.5

Problem 12 (ca final may 01)A company is engaged in the manufacture of edible oil. It has three divisions as under:(i) Harvesting oil seeds and transportation thereof to the oil mill.(ii) Oil Mill, which processes oil seeds and manufacture edible oil.

(iii) Without prejudice to your answer to part (ii) above, assume that Printer Division needs 17,400 units, Would it be in the best interest of Hardware Ltd. for the Components Division to supply the components to the Printer Division at Rs. 50?

Thus reduction in price from Rs. 400 to Rs. 380 will result in incremental contribution of Rs. 1,32,000, the Printer Division should institute the Rs. 20 price reduction.

(ii) Presently the Component Division is selling 64,000 units to outside market fetching Rs. 64/unit as contribution. It can still produce 11,000 units more and If it decides to sell 17,400 units to Printer Division, it will have to sacrifice the contribution of 6,400 units as it can produce only 75,000 components.

thus from the above calculation it is evident that, the net increase in contribution would be Rs. 312,500 if all the 17,400 components are supplied to P.Division @ Rs. 50/unit by C.Division. The cost saving would be more than the contribution lost, and it would be in the best interest of Hardware Ltd.

(iii) Marketing division, which packs the edible oil in 2 kg containers for sale at Rs. 150 each container.

Page 482: cost accounting question icwai inter

The cost data for each division for the period are as under :

Harvesting Division:

Variable cost per kg. of oil seed Rs. 2.50

Fixed cost per kg. of oil seed Rs. 5.00

Oil Mill Division :

Variable cost of processed edible oil Rs. 10.0 per kg.

Fixed cost of processed edible oil Rs. 7.50 per kg.

Marketing Division

Variable cost per can of 2 kg. of oil Rs. 3.75

Fixed cost per can of 2 kg. of oil Rs. 8.75

Required :

(1) Shared contribution in relation to variable costs ; and(2) Market Price(iii) Which Transfer price method will each divisional manager prefer to use ? Solution :Tutorial Notes :

1. Computation of the profit for the company is very easy. Please compute this in 5 minutes.

Statement of overall profit of the company H Division O Division M Division Rs.

Output in Kg. 2,000 1,000 1,000Rate per kg in Rs. 2.5 10.0 (3.75/2)Variable cost (output x rate) 5,000 10,000 1,875 16,875Fixed cost per kg. in Rs. 5.0 7.5 (8.75/2)Fixed cost 10,000 7,500 4,375 21,875Total Cost Rs. 15,000 17,500 6,250 38,750Sales revenue (500 cans x Rs. 150) 75,000Profit 36,250

Computation of shared contribution in relation to variable costsTotal Contribution (75,000 − 16,875) 58,125

The Oil Mill has a yield of 1000 kgs of oil from 2,000 kg of oil seeds during a period. The Marketing Division has a yield of 500 cans of edible oil of 2 kg. each from 1,000 kg. of oil. The net weight per can is 2 kg. of oil

The fixed costs are calculated on the basis of the estimated quantity of 2,000 kg. of oil seeds harvested, 1,000 kg. of processed oil and 500 cans of edible oil packed by the aforesaid divisions respectively during the period under review.

The other oil mills buy the oil seeds of same quality at Rs. 12.50 per kg. in the market. The market price of edible oil processed by the oil mill, if sold without being packed in the marketing division is Rs. 62.50 per kg. of oil.

(i) Compute the overall profit of the company of harvesting 2,000 kg. of oil seeds, processing it into edible oil and selling the same in 2 kg. cans as estimated for the period under review.

(ii) Compute the transfer prices that will be used for internal transfers from (1) Harvesting Division to Oil Mill Division and (2) from Oil Mill Division to Marketing Division under the following pricing methods :

2. For computation of transfer price on the basis of shared contribution is the easiest. The total contribution is divided by total variable cost and is transferred to the respective divisions in the ratio of their variable costs. The transfer price is the total of variable cost of the division plus shared contribution. You should also do this in five minutes.

Page 483: cost accounting question icwai inter

Total Variable cost 16,875Shared contribution of Harvesting Division (58,125 / 16,875) x 5,000 17,222Shared contribution of Oil Division (58,125 / 16875) x 10,000 34,444Shared contribution of Marketing Division (58,125 / 16,875) x 1,875 6,458

Computation of Transfer price under following pricing methods

1. Shared Contribution in relation to Variable cost :

Transfer price for Harvesting division to Oil Mill Division:

= Variable cost of Harvesting division + Shared contribution of Harvesting division

= Rs. 5,000 + Rs. 17,222 = Rs. 22,222

Transfer price for Oil Mills division to Marketing Division:

=

= Rs. 22,222 + Rs. 10,000 + 34,444 = Rs. 66,6662. Market Price Method :Transfer price for Harvesting division to Oil Mill Division:

= Market price of 2,000 kg. of oil seeds transferred to Oil Mill division= 2,000 x Rs. 62.50 = Rs. 25,000

Transfer price for Oil Mills division to Marketing Division:= Market price of 1,000 kg. of edible oil= 1,000 kg x Rs. 62.50 = Rs. 62,500

Statement of Profitability of Divisions H division O division M division

Shared Contribution Method : Rs. Rs. Rs.Transfer price 22,222 66,666 75,000*Less : Transfer price 22,222 66,666

22,222 44,444 8,334Less : Variable cost 5,000 10,000 1,875Less : Fixed cost 10,000 7,500 4,375

15,000 84,166 6,250Profit 7,222 -17,500 2,084* 500 cans @ Rs. 150/canMarket Price MethodTransfer price 25,000 62,500 75,000Less : Transfer price 25,000 62,500Less : Variable cost 5,000 10,000 1,875Less : Fixed cost 10,000 7,500 4,375Total Cost 15,000 42,500 68,750Profit 10,000 20,000 6,250Summary of resultsProfit under shared contribution method 7,222 26,944 2,084Profit under Market price method 10,000 20,000 6,250

Transfer price for Harvesting division to Oil Mill division + Variable cost of Oil Mill division + Shared contribution of Oil Mill division

(iii) Decision : Divisional Manager of Harvesting division would prefer the use of market price method for transferring 2,000 kg. of oil seeds to Oil Mill division because the profit of his goes up from Rs. 7,222 to Rs. 10,000 over the share contribution method.

Page 484: cost accounting question icwai inter

Problem 13 (ca final may 08)

The following figures are presented:Division A Division B

Rs. Per Unit s. Per Unit

Direct material cost 50 24 Other than AMaterial A, if transferred from Division A 144Material A, if purchased from outside 160Direct labour 25 14Variable production overhead 20 2Variable selling overhead 13 26Selling price in outside market 160 300Selling price to B 144 ─ 144Selling price to S Ltd. ─ 250 250Other Information:

Product A is available in the outside market at Rs. 160 per unit from competitors.

A (units) B (units)Manufacturing capacity 20,000 28,000 20,000 28,000Demand in external market 18,000 26,000 18,000 26,000S Ltd.’s demand ─ 6,000 or zeroAssume that Divisions A and B will have to operate during the year.What is the best strategy for: (i) Department A?

(ii) Department B, given that A will use its best strategy?(iii) For X Ltd. As a whole?

Solution :Best strategyA = Maximize Production; Sell maximum no. of units @ 52 / unit (outside)Contribution = Rs. 52 x 18,000 = Rs. 936,000Sell the remaining units 2,000 x rs 49 = 98,000 Total Contribution for A Rs. 10,34,000Best strategy for B:Maximize contribution / unit by selling outside and procuring from A 90 / unit Contribution x 2,000 unitsBalance units can yield contribution of either 74/ unit for outside or Rs. 50 / unit to S Ltd.Production Capacity = 28,000.units.

The Manager of Oil Mill division would prefer to use share contribution method over the market price method because its use would increase its profit by Rs. 6,944 (Rs. 26,944 − Rs. 20,000). Similarly Marketing division manager would be showing higher profit to the tune of Rs. 4,167 (Rs. 6,250 − Rs. 2,083) by using market price method.

(a) X Ltd. has two divisions, A and B, which manufacture products A and B respectively. A and B are profit centers with the respective Divisional Managers being given full responsibility and credit for their performance.

To make one unit of B, one unit of component A is needed. If transferred from A, B presently takes product A at Rs.144 per unit, with A not incurring variable selling overheads on units transferred to B.

B can sell its product B in the external market at Rs. 300 per unit, whereas, if it supplied to X Ltd.’s subsidiary, S Ltd., it supplies at Rs. 250 per unit, and need not incur variable selling overhead on units transferred to S Ltd. S Ltd. requires 6,000 units and stipulates a condition that either all 6,000 units be taken from B or none at all.

Choose to get 2,000 units from A, sell 6,000 units to S and 20,000 to outside. Make 28,000 units @ full capacity. Total Contribution Rs19,60,000.

Page 485: cost accounting question icwai inter

If A and B are allowed to act independent of the group synergy,Make A transfer all output to B. Sell 6,000 units of B to S and 22,000 units to outside

O God, give me the serenity to accept the things I cannot change, the courage to change the things I can, and wisdom to know the difference.

Page 486: cost accounting question icwai inter
Page 487: cost accounting question icwai inter
Page 488: cost accounting question icwai inter

Chapter 15 : Ratio Analysis cwa inter cwa final ca inter ca final

16 5 3 1

Problem 1: CWA Inter.June 05

Solution:

Quick ratio 1.8 = (80,000 - Stock)/ 40,000 Stock = Rs. 8,000Problem 2: CWA Inter.June 05

Current ratio : 2.5 Liquid ratio 1.6 Inventory turnover ratio on cost of sales : 8Credit allowed 1.5 months. Net working capital Rs. 300,000Gross profit on sales 20%

Prepare balance sheet of the company as on 31.03.05. Show your workings.Solution: Tutorial notes:

(I) Students are advised to memories the formulae of ratio analysis to become expert in this chapter.

(ii) Debtors are assumed on sales, may also be assumed on cost of sales ignoring the profit element.(iii) All sales are assumed as credit sales.Calculations for various balance sheet figures are shown below:Current ratio = 2.5 = Current Assets/ Current liabilitiesCurrent Assets = 2.5 Current liabilitiesNet working capital = Current Assets − Current Liabilities = 300,0002.5 Current liabilities − Current liabilities = 300,000 which gives Current liabilities as Rs. 200,000Current Assets = 2.5 Current liabilities = 2.5 x 200,000 = 500,000Liquid ratio = Quick ratio = Acid test ratio = ( Current Assets − Stock)/ Current liabilities( Current Assets − Stock) / Current liabilities = 1.6(500,000 − stock) / 200,000 = 1.6 which gives stock =180,000Inventory turnover ratio on cost of sales =1.80 = Cost of sales / StockCost of sales = 180,000 x 1.80 = Rs. 14,40,000Gross profit on sales = 20%Sales − 20% of sales = Cost of sales = 14,40,000 which gives Sales = Rs. 18,00,000Credit allowed = 1.5 months = ( average debtors / credit sales) x 12All sales are assumed to be credit sales. Credit sales equal to Rs.18,00,000 as calculated aboveAverage debtors = 18,00,000 x 1.5 / 12 = 225,000Current Assets = Cash + Debtors + Stock500,000 = Cash + 225,000 + 180,000 which gives Cash = 95,000

Net worth = 0.8 Net worth + 300,000 which gives net worth as 15,00,000

Net worth = Share capital + Reserve & Surplus. Reserve & Surplus are given as 50% of share capital.

15,00,000 = Share capital + 0.50 Share capital which gives Share capital = 10,00,000

·       vki Mj] ruko] fpark] uQjr rFkk cnyk ysus tSlh xyr Hkkoukvksa vkSj fopkjksa ds dkj.k vius g`n;] QsQM+s] yhoj vkSj vU; vaxksa dh lkekU; y; dks xM+cM+ dj ldrs gSA vius eu esa lnHkko] lsgr vkSj 'kkafr ds fopkjksa dh txg nsa] blls vkids 'kjhj dh lewph dk;Ziz.kkyh lkekU; gks tk;sxh vkSj vki The current ratio of BM Ltd. Is 2:1. while quick ratio is 1.80:1. If the current liabilities are Rs.40,000, What will be the value of stock?

Current ratio is 2:1 and current liabilities are Rs. 40,000, this gives current assets equal to Rs. 80,000. The quick ratio is the ratio of current assets less stock to the current liabilities.

Star Enterprises Ltd. Indicates the following financial ratios and performance figures for the year ending 31.03.2005

Company's fixed assets are equivalent to 80% of net worth i.e. share capital plus reserve & surplus, while the latter amount to 50% of share capital.

Net worth = Fixed Assets + Working capital. Fixed assets are given as 80% of net worth. Working capital is given as Rs. 300,000

Page 489: cost accounting question icwai inter

Reserve & Surplus = 500,000Fixed assets are 80% of net worth = 0.80 x 15,00,000 = 12,00,000The Balance sheet as on 31.03.05 is shown below:

Star Enterprises Ltd.Balance sheet as on 31.03.05 fig. in Rs. '000

Liabilities AssetsShare capital 1,000 Fixed Assets 1,200Reserves & Surplus 500 Current assetsCurrent liabilities 200 Stock 180

Debtors 225Cash 95 500

1,700 1,700Problem 3: (cwa inter dec.05)

Solution:

The average debtors = Credit sales / Collection periodCredit Sales Collection period Average Debtors120 lakhs One month Rs. 10.0 lakhs108 lakhs Half month Rs. 4.50 lakhsFall in debtors Rs. 5.50 lakhsProblem 4: (cwa inter dec.04)

Solution:Collection period = Average debtors / Credit sales = 5 / 60 year or 1 month.

Solution:Students should try to solve this mentally without using pen and calculator.Current assets = 10 x 2.2 = 22 lakhs. Liquid ratio is given as 0.8 which implies (22 − Stock ) = 0.8 x 10 which gives stock as Rs. 14 lakhs.

The following information are furnished by a company in regard to its working as on 31.03.03 :Capital and Reserves Rs.28 lakhsNet working capital Rs.2.80 lakhsCurrent ratio 2.4Liquid ratio 1.8Inventory turnover based on cost of sales 8Gross profit on sale 20%Credit allowed 1.5 monthsReserve amount to 40% of share capital. All sales are credit sales.Current assets consist of stock, debtors and cash only.Prepare balance sheet of the company as on 31.03.03.Solution:

…………….. Ltd.Balance sheet as on 31.03.03 Fig. in Rs. Lakhs

Liabilities AssetsShare capital 20.00 Fixed Assets 25.20Res. & Surplus 8.00 Investments

The average period of credit allowed by a company which has an annual credit sales of Rs. 120 lakhs in one month. By reducing the period of credit to half-a-month, sales fall to Rs. 108 lakhs. Find the fall in average amount of debtors.

The budgeted annual sales of a firm is Rs. 80 lakhs and 25% of the same is cash sales. If the average amount of debtors of the firm is Rs. 5 lakhs, find the average collection period of credit sales.

Problem 5:( cwa inter June 03)

If the current ratio and liquid ratio of a firm are 2.2 and 0.8 respectively and its current liabilities are Rs. 10 lakhs, the value of stock held by the firm is Rs. …………..

Problem 6: (cwa inter June 03)

Students are advised to solve this problem by themselves on the lines of solution problem 2 given above. Note that Total liabilities = Fixed assets + Current assets.

Page 490: cost accounting question icwai inter

Secured loans Current assets, loans and advancesUnsecured loans Stock 1.60Current liabilities Debtors 2.00Sundry creditors 2.00 Cash 1.20

30.00 30.00

The annual credit sales of a firm amounts to Rs. 12,80,000 and the debtors amount to Rs. 1,60,000.

Find the debtors turnover and average collection period.Solution:Debtors turnover = Credit sales / Average debtors = 12.80 / 1.60 = 8Collection period = 365 / 8 = 45.75 say 45 days

The production unit of a company has a capacity ratio of 0.8 and an activity ratio 1:2.What is its efficiency ratio ?Solution:Capacity ratio = ( Actual hours worked / Budgeted hours ) Activity ratio = ( Output in standard hours / Budgeted hours)Efficiency ratio = ( Output in standard hours / Actual hours worked ) ER = AR / CR = 1.2 / 0.8 = 1.50

Operation leverage = Contribution / ( Contribution − Fixed cost )Last year Current year1.25 = C/ C −F 1.20 = D / D − 1.25 F C = 5 F D = 7.5 F

Ratio D / C = 7.5 / 5 = 1.50 or Contribution has increased by 50%.

Solution: Last year Current yearAverage debtors in Rs. Lakhs 10 25Average collection period in month 1 2Sales (Debtors x 12 / Collection period) Rs. In lakhs 120 150Increase in sales in lakhs 30Increase in contribution with contribution / sales ratio as 40% = 40% of Rs. 30 lakhsIncrease in contribution Rs. 12 lakhs.

The following accounting information and financial ratios of Zenith Ltd. pertain to the year ended 31.3.06Paid up capital Rs. 200,000Plant and machinery Rs. 500,000Total annual sales Rs. 20 lakhsSales return 20% of salesAnnual credit sales 80% of net sales Gross profit margin 25%Current ratio 2Inventory turnover (cost of sales) ratio 4Fixed assets turnover ratio (net sales) 2Average collection period 73 daysBank credit to trade credit 2

Problem 7: (cwa inter Dec 03)

Problem 8: (cwa inter June 00)

Problem 9: (cwa inter June 00)

A company has an operation leverage of 1.20 against 1.25 during the previous year. If the current fixed cost is 25% more than that of previous year, to what extent the contribution earned by the firm changed over the previous year ?

Problem 10: (cwa inter June 00)

The average period of credit allowed by a company to its customers last year was one month and the average amount of debtors was Rs. 10 lakhs. To increase sales and profitability the company doubled the period of credit during the current year. As a result the average amount of debtors increased to Rs. 25 lakhs. If the company has a contribution / sales ratio of 40% ; what additional contribution has been earned by the company during the current year ?

Problem 11: (cwa inter June 06)

Page 491: cost accounting question icwai inter

Cash to inventory 1:15Total debt to Current liabilities 3You are required to prepare Balance sheet of Zenith ltd. As on 31.3.06. Show workings.Solution :(i) Net sales = Sales − Sales return = 20 − 20% of 20 = 20 − 4 = Rs. 16 lakhs.(ii) Credit sales 80% of net sales = 80% of Rs. 16 lakhs = Rs. 12.80 lakhs.(iii) Gross profit = 25% of net sales = 25% of Rs. 16 lakhs = Rs. 4 lakhs(iv) Cost of sales = 75% of net sales = Rs. 12 lakhs.(v) Inventory turnover ratio = 4; Inventory = Cost of sales / 4 = Rs. 12 lakhs / 4 = Rs. 3 lakhs.(vi) Receivable collection period 73 days. Receivable turnover = 365 / 73 = 5(vii) Receivables = Credit sales / 5 = Rs. 12.80 lakhs / 5 = Rs. 2.56 lakhs.(viii) Cash : inventory = 1 : 15 ; Inventory being 300,000 thus cash = Rs. 20,000(ix) Total current assets = inventory + receivables + cash = 300,000 + 256,000 + 20,000 = Rs. 576,000(x) Current ratio = Current assets / Current liabilities = 2 ; Current liabilities = 576,000/2 = Rs. 288,000(xi) Current liabilities = Bank credit + trade credit = 3 trade credit. = Rs. 288,000(xii) Trade credit = Rs. 96,000 and Bank credit = Rs. 192,000.(xiii) Total debt = Current liabilities x 3 = Rs. 288,000 x 3 = Rs. 864,000(xiv) Fixed assets turnover on net sales = 2 hence Fixed assets = 16/2 = Rs. 8 lakhs.(xv) Other fixed assets = fixed assets − plant and machinery = 8 − 5 = Rs. 3 lakhs.(xvi) Total assets = Fixed assets + current assets = 8 + 5.76 = Rs. 13.76 lakhs.(xvii) Reserves and surplus = Total assets − Share capital − Total debts = 13.76 − 2.0 − 8.64 = Rs. 3.12 lc

Zenith Ltd.Balance sheet as on March 31, 2006

Liabilities Rs. Assets Rs.Share capital 200,000 Plant and machinery 500,000Reserve and Surplus 312,000 Other fixed assets 300,000Long term debts 576,000 Current AssetsCurrent liabilities Inventory 300,000Bank credit 192,000 Receivables 256,000Trade credit 96,000 Cash 20,000

13,76,000 13,76,000

Solution: Inventory = half month sale = 120 / 24 = Rs. 5 lakhs.Average collection period is one month. Average debtors equal to one month sale i.e. Rs. 10 lakhs.Current assets = Current liabilities x Current ratio = 9 x 2 = Rs. 18 lakhs.Current assets = Cash + Debtors + Stock = Cash + 10 + 5 = 18 i.e. Cash = 3 lakhs.

The company is examining the following alternative working capital policies:Policy Investment in current assets Estimated sales EBIT

Rs. Crore Rs. Crore Rs. CroreConservative 11.475 31.365 3.1365Moderate 9.945 29.325 2.9325Aggressive 6.63 25.5 2.55You are required to calculate the following for each policy:(i) Rate of return on total assets (ii) Net working capital position (iii) Current assets to fixed assets ratio(iv) Discuss the risk-return trade off of each policy.

Problem 12: (cwa inter Dec 06)

A company has an annual sales of Rs. 120 lakhs entirely on credit. It keeps an average inventory sufficient to meet sales demand of half a month and gives its customers one month credit. Its c. average Rs. 9 lakhs.The company maintains cash to have a current ratio of 2. The cash balance is .....

Problem 13: (ca pe II may 03)

An engineering company is considering its working capital investments for the year 2003−04. The estimated fixed assets and current liabilities for the next year are Rs. 6.63 crore and Rs. 5.967 crore respectively. The sales and earning gs before interest and taxes (EBIT) depend on investment in its current liabilities−particularly inventory and receivables.

Page 492: cost accounting question icwai inter

Solution: fig. in croresBasic data are tabulated below: Conservative Moderate Aggressive

1 Current assets 11.475 9.945 6.632 Fixed assets 6.63 6.63 6.633 Total assets 18.105 16.575 13.264 Current liabilities 5.967 5.967 5.9675 Estimated sales 31.365 29.325 25.56 Estimated EBIT 3.1365 2.9325 2.557 Current ratio [(1)/ (4)] 1.92 1.67 1.11

Computation of required parameters(i) Rate of return on total assets [(6) / (3)] 17.32% 17.69% 19.23%(ii) Net working capital (crores) [(1) − (4)] 5.508 3.978 0.663(iii) Current assets to fixed assets ratio [(1) / (2)] 1.73 1.50 1.00

From the following data for the year ended 31st Dec. 2000:

Calculate (i) inventory turnover ratio of the two items and (ii) stock holding in terms of number of days.

Material A Material B Rs. Rs.

Opening stock 1.1.2000 20,000 18,000Purchase during the year 104,000 54,000Closing stock 31.12.2000 12,000 22,000Solution:Material consumed (A) 112,000 50,000Average stock (B) 16,000 20,000Inventory turnover ratio (A/B) 7.00 2.50Stock holding ( 365 / Inventory turnover) in days 52.14 146.00

Using the following information complete the balance sheet given below:Total debt to net worth : 1:2. Total Assets Turnover : 2 and Gross Profit : 30%Average collection period taking one year as 360 days : 40 days. Acid test ratio − 3 : 4Inventory turnover ratio based on cost of sales and year end inventory : 3

Balance sheet as on……………………..Liabilities Equity Share Capital 200,000 Assets Plant and machinery −

Reserve and Surplus 300,000 and other fixed assets − Current liabilities − Current assets −

Inventory Accounts receivables Cash in hand and bank

Solution: Try your self. The answer is as follows.Balance sheet as on…………………….. fig. in Rs.

Liabilities Equity Share Capital 200,000 Assets Plant and machinery 212,500 Reserve and Surplus 300,000 and other fixed assets Current liabilities 250,000 Current assets

Inventory 350,000 Accounts receivables 166,667 Cash in hand and bank 20,833

750,000 750,000

Solution: The monthly rate of interest is 2%. The discount rate would be (1 − 100 / 102 ) = 1.93%

(iv) The net working capital or current ratio is a measure of risk. Less the current ratio; less is the working capital. The rate of return on total assets is a measure of return. The expected risk and return are minimum in the case of conservative investment policy and maximum in case of aggressive policy.

Problem 14: (cwa inter I June 01)

Problem 15: (cwa inter II Dec.2000)

Problem 16: (ca pe II nov.2004)

A firm is considering offering 30-day credit to its customers. The firm likes to charge them an annualized rate of 24%. The firm wants to structure the credit in terms of a cash discount for immediate payment. How much the discount rate have to be ?

Page 493: cost accounting question icwai inter

The summarized balance sheet of R.K.Enterprises as on 31.3.2002 is given below:Liabilities Rs. Assets Rs.Equity Capital 40,00,000 Fixed assets net 28,00,000Reserve and Surpluses 6,00,000 Cash 2,40,000Sundry Creditors 14,00,000 Debtors 18,60,000

Inventory 11,00,00060,00,000 60,00,000

Other Information: Rs.Sales 52,00,000Less:Cost of production of goods sold 36,00,000Less: Cost of selling and administration 12,50,000Net Profit 3,50,000

The current industry average of important ratios is given below:Current ratio 2.1Liquid ratio 1.4Inventory turnover (Sales / Inventory ) 10.2Net profit / Sales 9.0Debtors turnover 8.0Sales / Total assets 2.0

Calculate the above ratio for R.K.Ente., compare the same with the inventory average and comment.Solution:Current ratio = Current assets / Current liabilities = ( 2.4 + 18.6 + 11) / 14 2.3Liquid ratio = Current assets − Stock / Current liabilities = ( 2.4+18.6−11) 14 1.5Debtors turnover = Sales / Debtors = 52 / 18.60 2.8Inventory turnover (Sales / Inventory ) = 52 / 11 4.7Net profit / Sales = 3.50 / 52 6.7Sales / Total Assets = 52 / 60 0.86Comparative table of ratios of R.K.Enterprise and Industry standards

Industry R.K.Enterprises CommentsCurrent ratio 2.1 2.3 SatisfactoryLiquid ratio 1.4 1.5 SatisfactoryDebtors turnover 8 2.8 Far below the standard. Debtors be reduced.Inventory turnover ratio 10.2 4.7 Far below the standard. Inventory be reduced.Net profit / Sales 9 6.7 Poor profitability Sales / Total Assets 2 0.86 Poor utilisation of assets.

Share capital 199,500 Average collection period 2 monthsWorking capital 45,000 Current ratio 1.5Gross margin 20% Quick ratio 0.9Inventory turnover ratio 6.0 Reserves and surplus to cash 3.0Solution: Solve yourself. Answer follows:

XYZ Ltd. Balance sheet as on 31.12.99

Liabilities Rs. Assets Rs.Share capital 199,500 Fixed assets 195,000Reserves and surplus 40,500 Inventory 54,000Bank loan 60,000 Debtors 67,500Trade creditors 30,000 Cash 13,500

330,000 330,000

Problem 17: (cwa inter II June 2002)

Problem 18: (cwa final dec 2000)

The assets of XYZ Ltd. consist of Fixed and Current assets while its Current liabilities comprise of Bank credit and trade credit in the ratio of 2:1. From the following figures relating to the company for the year 1999 prepare its Balance sheet showing details of working:

Problem 19: (cwa final June 2000)

Page 494: cost accounting question icwai inter

(iii) The finished goods were valued at full production cost.(iv) The cash on hand and at Bank on 31.3.2000 was Rs. 12 lakhs.(v) The company raised additional capital of Rs. 100 lakhs on 1.2.2000.

(x) The following ratios are available:(a) Current ratio : 2 (b) Debtors turnover : 6 times (c) Creditors turnover :6

Required : Prepare P/L account for the year ended 31.3.2000 and Balance sheet on that date .Solution :

5. Once the current liabilities known, the current assets become known with the help of current ratio.

Transportation Ltd. was incorporated on 1.4.99 with a paid up capital of Rs. 200 lakhs. The company had the capacity to manufacture 4000 nos. of motorized Ricksha Vans per annum. The operating statements of the company revealed the following information and ratios :

(i) For the year ended 31.3.2000 the Company produced 3000 Motorized Vans and sold 2500 nos. at a price of Rs. 24,000 per van.

(ii) Cost of production consisted of Raw materials, direct labour, manufacturing overheads, and Depreciation. Direct labour was 35% of the production cost.

(vi) The company paid an interim dividend at 10% on 31.10.1999, A further dividend of 10% provided out of profits on 31.3.2000. No dividend was payable on the additional capital raised.

(vii) A long term loan of Rs. 200 lakhs at 20% rate of interest was obtained on 1.4.99. The loan is to be repaid in five annual installments. The company paid the interest as well as the first installment of loan repayment of Rs. 40 lakhs on 31.3.2000.

(viii) The company invested a sum of Rs. 200 lakhs in Govt. Bonds on 1.6.99 carrying an interest of 12%.The interest was received at the end of every month. All monies were duly received.

(ix) The total assets ( net fixed assets, investment and current assets) of the company as on 31.3.2000 equaled the Sales turnover of the year.

(d) Interest coverage ratio :4 (e) Debt service coverage ratio : 1.75 (f) Profit after tax : 10% of sales (g) Raw materials turn over (based on closing stock ) : 4 times

The question of this magnitude poses the problem of how and where to begin ? It is a full marks question. You can start with the first additional information. The various elements of profit and loss account and balance sheet are given at various locations in the question and you are to pick up and join the pieces to frame the balance sheet.

You should revise the various formulae of ratio analysis. Some clues are given below to help you to solve the question.

1. Sales turnover can be easily computed. Total assets are equal to sales turnover. Total liabilities are equal to total assets.

2. Profit after tax is given indirectly. Interim and final dividends both are given. What is PAT minus total dividend ? This is reserve and surplus. One item of balance sheet becomes known here.

3. Tax rate is not mentioned anywhere. The PAT is given. PBIT is indirectly given. PBIT / Interest is Interest Coverage ratio which is given as 4. Interest is calculated @ 20% on investment of Rs. 200 lakhs. The PBIT becomes known. PBIT less interest as calculated earlier is PBT. The difference between PBT and PAT is tax paid during the year.

4. Have a look on the liabilities side of the balance sheet. Total liabilities, share capital, reserve and surplus, loans, all are known. What is the balancing figure ? The balancing figure will denote the current liabilities.

6. Depreciation is included in the cost of production as mentioned in the question. Think where the depreciation is hidden ? Remember and revise the Debt service coverage ratio formula. Use it to find the depreciation.

Page 495: cost accounting question icwai inter

11. Cost of production known. Cost of goods sold = Cost of production − Cost of finished goods.

Please do not see the solution immediately. Try it. Solution goes as follows:Transportation Ltd.

Profit and loss account for the year ended 31.3.2000 (Rs. In lakhs)Sales 600

Less : Cost of goods sold :Raw materials consumed Raw materials purchased 240Less: Closing stock 48 192Direct labour 168Manufacturing overheads 80Depreciation 40Less: Closing stock of finished goods 80 400

Gross profit 200Less : Selling and Administrative overheads 60Add : Investment income ( 200 x 12% x 10/12) 20Net profit before interest and tax PBIT 160Less : Interest @ 20% on Rs. 200 lakhs 40Profit before tax PBT 120Less ; Tax 60PAT 60

Balance sheet as on 31.3.2000Liabilities Rs. lakhs Rs. lakhs Assets : Rs. lakhs Rs. lakhsShare capital 300 Fixed assets Reserve and Surplus 20 Gross Block 200Loan 200 Depreciation 40 160Less : Installment paid 40 160 Investment 200Current liabilities : Current assets :Sundry creditors 40 Stock of raw materials 48Provision for tax 60 Finished goods 80Proposed Dividend 20 120 Debtors 100

Cash at Bank 12 240600 600

Working notes : Rs. In lakhs1. Sales Turnover (2,500 x Rs. 24,000) 6002. PAT is 10% of Sales turnover of Rs. 600 lakhs. 603. Interest @ 20% of Rs. 200 lakhs. 404. PBIT : Interest x Interest coverage ratio = 40 x 4 1605. PBT = PBIT − Interest = (160 − 40) 120

7. As indicated in the question, the total current liabilities consist of provision for tax, proposed dividend and sundry creditors. Note that dividend paid is not a liability but proposed dividend is. Sundry creditors become known. Creditors turnover ratio is given as 6 which will give the credit purchase of raw material.

8. Purchase of raw materials = raw materials consumed + closing stock. Raw materials turnover ratio on closing stock is 4. Thus raw materials consumed is 4 x Closing stock. Thus raw materials consumed as well as closing stock become known.

9. Debtor turnover ratio is 6. Debtors will be 1/6 of sales. Total current assets are known. Other items of current assets like cash at bank, stock of raw materials and debtors are known. What is signified by the balancing figure ? This is cost of closing stock of finished goods. But closing stock is 500 vans because 3000 vans were produced and 2500 vans were sold. Thus cost of 500 vans is known. This will give you cost of production.

10. Direct labour is 35% of cost of production. Manufacture overheads = Cost of production − raw materials consumed − direct labour − depreciation.

12. Gross profit = Sales − Cost of goods sold. Selling and admin. expenses = Gross profit + Investment income − PBIT.

Page 496: cost accounting question icwai inter

6. Tax = PBT − PAT = ( 120 − 60 ) 607. Investment income @ 12% of 10 months on Rs. 200 lakhs. 208. Total Assets = Sales Turnover = Total liabilities 6009. Reserve and surplus = PAT − Interim dividend − final dividend 2010. Loan as on 31.3.2000 ( 200 − 40) 16010. Current liabilities = Total liabilities − (Share capital + loan + reserve) 12012. Current assets : 2 x Current liabilities 24013. Sundry Debtors : Sales / 6 10014. Sundry creditors : Current liabilities − provision for tax − proposed dividend 4015. Credit purchase : 6 x Creditors 240

17. Stock of finished goods = Total current assets − ( debtors + cash + raw materials stock) 8018. Production is 3000 vans. Closing stock of 500 vans costing Rs. 80 lakhs.

Cost of production = 80 x (3000 / 500) 48019. Debt coverage ratio = (PAT + dep. + interest ) /( Loan installment + interest) = 1.75

PAT = 60, Interest = 40, Installment = 40 hence Depreciation = 4020. Manufacture overheads = Cost of production − ( raw materials consumed + labour +depre.)

Direct labour is 35% of cost of production. Manufacturing overheads 8021. Cost of goods sold = Cost of production − Closing stock of finished goods. 40022. Gross profit = Sales − Cost of goods sold 20023. Selling and manufacturing overheads = GP + Investment income − PBIT 6024. Net Fixed assets = Total assets − Current assets − Investment 160

12% term loan Rs. 150 lakhs, Working capital : Borrowing from Bank at 15% Rs. 100 lakhs

Public deposit at 11% Rs. 45 lakhs

Calculate the change in interest coverage ratio after the additional borrowing and comment.

Solution : Interest charges = EBIT / Interest charges Rs. Lakhs

Current New

EBIT 45.00 (45x1.16) 52.20

Current interest charges = (18 + 15 + 4.95) 37.95

New interest charges after the fresh borrowing (37.95 + 16% x 50) 45.95

Interest coverage ratio (EBIT/Interest) 1.19 1.14

The additional burden of Rs. 50 lacs @ 16% will bring down the Interest coverage ratio from 1.19 to 1.14.

The following information and financial ratios of PQR Ltd. relate to the year ended 31st December, 2006:

Accounting Information :

Gross profit : 15% of Sales and Net profit : 8% of sales

Raw materials consumed : 20% of works cost and Direct wages : 10% of works cost

Stock of raw materials : 3 months' usage and Stock of finished goods : 6% of works cost.

Financial Ratios : Debt collection period : 60 days. Fixed asset to sales ratio − 1:3

Fixed assets to Current assets − 13 : 11 Current ratio : 2 : 1

Long term loans to current liabilities : 2 : 1 and Capital to Reserves and Surplus : 1:4.

Solution : Tutorial Notes

16. Raw materials turnover is 4. The raw materials consumed would be 4 x Closing stock. Closing stock is 240/5 = 48 lakhs. Raw materials consumed is 48x4 = Rs. 192 lacs.

Problem 20: (ca final may 03)

A company is presently working with an earning before interest and taxes EBIT of Rs. 45 lakhs. Its present borrowings are :

The sales of the company is growing and to support this the company proposes to obtain additional borrowing of Rs. 50 lakhs expected to cost 16%. The increase in EBIT is expected to be 16%.

Problem 21: (ca inter may 07)

If value of fixed assets as on 31st dec, 2005 amounted to Rs. 26 lakhs, prepare a summarized Profit and loss account of the company for the year ended 31st dec,06 and also the balance sheet as on that date.

Page 497: cost accounting question icwai inter

On the above lines, Solve this by yourself. Brief solution follows:Profit and loss account of PQR Ltd.For the year ended Dec.31, 2006 fig. Rs.'000

To Direct materials 1,326 By Sales 7,800Direct wages 663Overheads (bal. fig.) 4,641Gross Profit 1,170

7,800 7,800Selling and distribution 546 By Gross Profit 1,170Net Profit 624

1,170 1,170Balance sheet as at Dec.31,2006

Share Capital 300 Fixed assets 2,600.0Reserves and surplus 1,200 Stock of raw materials 331.5Long Term loans 2,200 Stock of finished goods 397.8Current liabilities 1,100 Debtors 1,300.0

Cash 170.74,800 4,800.0

You are given the following figures :Current ratio : 2.50; Liquid ratio : 1.50 ; Net Working capital : Rs. 300,000; Gross profit : 25%Stock turnover ratio : (cost of sales / closing stock) : 6 times.Fixed asset turnover ratio (on cost of assets) : 2 times.Average debt collection period : 2 months;Fixed asset / Net worth : 0.08Reserve and surplus / Capital : 0.05Draw up the balance sheet and show your workings.Solution : This can be solved on lines of earlier problems. Answers are given below.

Balance sheet of Bakwaas Ltd. as on ……………..

Liabilities Rs. Lakhs Assets Rs. LakhsShare capital 50.0 Fixed assets 6.0Reserve and surplus 25.0 Current assets

Stock 2.0Current liabilities 2.0 Debtors 2.5

Bank and Cash 0.5Balancing figure 66.0

77.0 77.0

The summarized balance sheet of Penguin India Ltd. at 31st March 2007 was as followsRs. Lakhs Rs. Lakhs

A close look of the question can indicate where to begin. Sales has not been given. Fixed assets have been given and ratio of sales to fixed assets is given. Sales, gross profit and net profit can be known. Sales less gross profit is works cost, thus raw materials consumed and direct wages can be easily known.

Current assets are known as the ratio of Fixed assets and current assets is given and fixed assets are also given. Current ratio is given which will give current liabilities that in turn will give Long term loans.

Collection period and sales are known. This will supply debtors amount. The collection period is given as 60 days; the year could be assumed to be of 360 days to facilitate easy calculation.

The difference between gross profit and net profit may be taken as selling and distribution expenses. Similarly the balancing figure of trading account may be taken as overheads.

Problem 22: (cwa inter June 2008)

Tutorial notes ; In this problem, you will find that the balance sheet has unequal amount on its two sides. This may confuse you and you may recheck your calculation. You can add some balancing figure and present the balance sheet.

Problem 23: (cwa final June 07)

Page 498: cost accounting question icwai inter

Equity shares (Rs. 10 each) 200 Fixed Assets 1,5357% Preference share (Rs. 100 par) 100 Current Assets 590Share premium account 110Profit and loss account 655 (260) 3309% debentures 800 Net current assets

1,865 1,865Additional Information

2. Dividend indicated on Preference Share is 11%. The current price of Preference Shares is Rs. 78.

4. Corporate tax 35% and Dividend Tax 10%Requirements : 1. Calculate Gearing ratio for Penguin Ltd. using (a) Book Value (b) Market Value

Solution : Gearing ratio = (Prior charge capital) / (Prior charge capital + Equity)1. Gearing ratio with book value:Prior charge capital consists of (i) 9% debentures (ii) 7% preference shareEquity consists of (i) Equity shares (ii) Share premium account (iii) Profit and loss account Gearing ratio with book value ( 800 + 100) / ( 800 + 100 + 100 + 110 + 655) = 48.26%Another acceptable alternative of Gearing ratio = Prior charge capital / Equity = 900/965 = 93.2%2. Gearing ratio with market value No. lakhs Market rate Rs./ share Value Rs. LakhsEquity Shares 20 75 1,500Preference Shares 1 78 78Debentures 8 80 640Gearing ratio with market value = ( 640 + 78) / (640 + 78 + 1500) = 32.3%Another acceptable alternative of Gearing ratio = 640 /1500 = 47.9%2. Determination of cost of specific sources:Cost of equity : For computing let us compute growth rate firstGrowth = EPS − DPS ( 1 + 0.10) / 75 = 6.67%Cost of equity = ( 10/75 ) + 0.0667 = 0.20 or 20%Cost of Preference Shares = [11 + (11 + 0.10)] / 78 = 0.1551 = 15.5%Cost of Debentures : After tax cost of debentures = 9 ( 1 − 0.35) / 80 = 0.731 = 7.3%Determination of Weighted Cost of Capital based on market valueSource of Capital Market value Rs. Cost in % Total costEquity shares 1500 20.00% 300.00Preference Shares 78 15.50% 12.09Debentures 640 7.30% 46.72

2,218 358.81Weighted average cost of capital = 358.81 / 2218 = 16.2%

Problem 24 (cwa final dec.06)GNL Limited has maintained the following relationships in recent years:

1 Gross profit to sales 40%2 Net profit to net sales 10%3 Selling expenses to net sales 20%4 Books debts turnover 8 per annum5 Inventory turnover 6 per annum6 Quick ratio 27 Current ratio 38 Assets turnover ( sales basis) 2 p.a.9 Total assets to intangible assets 20

10 Accumulated depreciation to cost of fixed assets 1/3

Creditors falling due within one year

1. The current price of Equity Shares is Rs. 75 ex-dividend. The projected EPS and DPS for the current year are Rs.16 and Rs.10 respectively.

3. The debenture interest has also been paid recently and Debentures are currently trading at Rs. 80 per Rs. 100 nominal.

2. Calculate the Company's Weighted Average Cost of Capital using respective market values as weighting factors.

Page 499: cost accounting question icwai inter

11 Books debts to sundry creditors for goods 1.512 Shareholder's funds to working capital 1.613 Total debts to shareholder's funds 0.5

Quick assets comprise 15% cash, 25% marketable securities and 60% book debts.

You are required to prepare :(i) Balance sheet as on March 31, 2006(ii) Profit and loss a/c for the year ended March 31, 2006. Ignore taxation.Show necessary workings.Solution :

GNL LimitedBalance sheet as on March 31,2006

Liabilities Rs. Lacs Assets Rs. LacsShare capital Fixed assets 29.25

Cost 9.75 19.5025.0 Less: Depreciation

Investments2.5 (marketable securities)

Reserves and surplus Current assets Share premium 2.5 Stock 12Profit and loss a/c 10.0 Book debts 15Secured loans Cash 3.75

Prepaid 0.5 31.250.75 Intangible assets 3.00

Current liabilitiesSundry creditors for goods 10.0Outstanding expenses 2.5

53.25 53.75Profit and loss a/c

To Cost of goods sold 72 By Sales 120To Selling expenses 24To Admin expenses 11.25To Interest 0.75To Net profit 12

120 120Working notes : Computations are in Rs. Lacs1. 1.20 = Net profit = 10% of sales Sales = 12.0 Rs.lacs2. Cost of goods sold is 60% of sales = Rs. 72.0 lacs.3. Selling expenses = 20% of sales = Rs. 24.0 lacs.4. Administrative expenses as balancing figure.5. Book debts = Sales / 8 = Rs. 15.0 lacs.6.Cash = 60/15 of Book debts = Rs. 3.75 lacs.7. Marketable securities 15 x (25/60) = Rs. 6.25 lacs.8. Total Quick assets = Rs. 25.0 lacs. 9. Quick ratio is 2. Current liabilities would Rs. 12.5 lacs.10. Total assets = sales / 2 = Rs. 60 lacs.11. Intangible assets = Total assets / 20 = Rs. 3.0 lacs12. Alter paying dividend of pref. shares of Rs. 30,000, the balance of profit is Rs. 11.70 lacs.13. Eps is 4.68, number of shares 11.70 / 4.68 = 25,000.

During 2005 - 06, the company earned Rs. 1200000 or Rs. 4.68 per equity share, the market value of one equity share was Rs. 78. The capital consisted of equity shares issued at a premium of 10% and 12% preference shares of Rs. 100 each. Interest was Rs. 75,000 in 2005-06; many years ago, the company had issued 10% debs due for redemption in 2007. During 2005−06, there was no change in the level of inventory, Book debts, debentures and shareholder's funds. All purchases and sales were on account. Preference dividend paid in 2005-06, in full was Rs. 30,000.

Issued and Subscribed 250000 equity shares of Rs. 10 each fully paid

12% Preference shares of Rs. 100 each fully paid

10% debentures due for redemption in 2007

Page 500: cost accounting question icwai inter

Problem 25: (CWA Final Dec. 08)Complete the balance sheet and sales information in the table below for Godrej Industries using the following financial data :Debt ratio : 50% ; Quick ratio : 0.80 ; Total Asset Turnover : 1.5 ; Days sales outstanding : 36.5 daysLiabilities Rs. Assets Rs.Accounts Payable CashLong-term Debt 60,000 Accounts receivablesCommon Stock InventoriesRetained earnings 97,500 Fixed assetsTotal Liabilities and Equity Total assets 300,000Sales Cost of goods soldSolution:Liabilities Rs. Assets Rs.Accounts Payable 90,000 Cash 27,000Long-term Debt 60,000 Accounts receivables 45,000Common Stock 52,500 Inventories 90,000Retained earnings 97,500 Fixed assets 138,000Total Liabilities and Equity Err:522 Total assets 300,000Sales 450,000 Cost of goods sold 337,500

Many of life's failure are people who had not realised how close they were to success when they gave up.

·       euq"; ogh gksrk gS tks og fnuHkj lksprk jgrk gS- blds vfrfjDr oks gks Hkh dSls ldrk gS-

Page 501: cost accounting question icwai inter

Total

25

Page 502: cost accounting question icwai inter
Page 503: cost accounting question icwai inter

Complete the balance sheet and sales information in the table below for Godrej Industries using the following financial data :

euq"; ogh gksrk gS tks og fnuHkj lksprk jgrk gS- blds vfrfjDr oks gks Hkh dSls ldrk gS-

Page 504: cost accounting question icwai inter

Chapter 16 : Learning Curve cwa inter cwa final ca inter ca final Total

3 5 0 2 10

Solution: time in hoursBatch No. of Cumulative Average Cumulat. Batch

units units Time Time Time1 1 1 10.00 10.00 10.0 For first unit2 1 2 8.00 16.00 6.0 For 2nd unit3 2 4 6.40 25.60 9.6 for 3rd and 4th unit4 4 8 5.12 40.96 15.4 for 5th,6th,7th and 8th units

Thus time taken for production of 8 units with 80% learning curve would be 40.96 hours.Cost of production for 8 units Fig. in Rs..Particulars AmountMaterial @ Rs. 200 per unit 1600.0Labour @ Rs. 15 per hour for 40.96 hours 614.4Overheads @ 150% of labour cost 921.6Total cost of production : 3136.0Profit @ 25% on cost ( 20% on sales ) 784.0Total sales value for 8 units 3920.0Selling price per unit 490.0The firm should quote Rs. 490 per unit as selling price.

1. What is the price per unit to be quoted for the trial order ?

Solution: time in hoursBatch No. of Cumulative Average Cumulat. Batch

units units Time Time Time1 1 1 10.00 10.00 For first unit 10.02 1 2 8.00 16.00 For 2nd unit 6.03 2 4 6.40 25.60 for 3rd and 4th unit 9.64 4 8 5.12 40.96 for 5th,6th,7th and 8th units 15.36

40.96

·       vius ckjs esa vkidh D;k lksp ;k jk; gS \ vkids thou dk gj fgLlk mlh lksp dh vfHkO;fDr gSA vkidh LQwfrZ] tks'k] 'kjhj] foRrh; fLFkfr] nksLr rFkk lkekftd izfr"Bk mlh lksp dk izfrfcac gSA vki vius ckjs esa tks Hkh lksp j[krs gSa oks vkids thou ds gj {ks= esa O;Dr gksrh gSAProblem 1 (cwa inter II Dec.2000)A company wants to manufacture a new product against order. The initial trials showed that the first unit would take 10 hours @ Rs. 15 per hour and that the operations would be subjected to a learning curve of 80%. The cost of material per unit is Rs. 200 and the overheads amount to 150% of labour cost. The first order received is for 8 units of the product. What price should be quoted to get a margin of 20% of sales ?

Life will give you pain all by itself, you have to create joy for yourself. Feel good right now and smile.

Problem 2 (cwa inter II June 2002)

A firm received a trial order for manufacture and supply of four units of a certain component. The cost of material required per unit is Rs. 80. The first unit required 10 hours of labour @ Rs. 10/hour and the manufacturing process is subject to 80% learning effect. Overheads are absorbed at 150% of labour and the firm takes 20% on the price quoted.

2. Assuming that the firm is agreeable to pass on the benefit of learning to the customer, what price per unit should be quoted if a second order for 4 units is received by the firm ?

Page 505: cost accounting question icwai inter

Trial order Repeat orderNo. of units to be manufactured 4 4Cost of material @r Rs. 80 320 Rs. 320 Rs. Labour @ Rs. 10 / hour 25.6 hr. 256 15.36 hr 153.6Overheads @ 150% of labour cost 384 230.4

960 704Add: Profit 20% of sales or 25% of cost 240 176Price to be quoted in Rs. for four units 1200 880Price to be quoted in Rs. for per unit 300 220

Solution: Let us first calculate whether overtime is required at all.It is given that time required to complete the first machine is 10 days.Total time required for 15 machines 99.4 days

14 machines 93.7 days

13 machines 88.0 days

Total time available is 23 days per month for 4 months i.e. 92 days.

Comparative statement of both the options is given below :Option 1: No overtime and penalty is paid for delayed machines: Rs. LacsDirect materials @ Rs. 2.40 lac/ machine for 15 machines 36.00Direct labour @ Rs. 10,000 for 99.4 days 9.94Overheads @ Rs. 12,000/day for 92 days 11.04Penalty for 2 machines @ Rs. 40,000/machine 0.80Total cost 57.78Sales revenue @ Rs. 4 lacs per machine 60.00Profit 2.22Option 2 : Work overtime to avoid penaltyDirect materials @ Rs. 2.40 lac/ machine for 15 machines 36.00Direct labour @ Rs. 10,000 for 92 days 9.20Overtime labour @ Rs. 20,000 for 7.4 days 1.48

From the above analysis, it is evident that the first four units of trial order takes 25.6 hours for manufacture and the next four units of repeat order will take 15.36 hours. The results are tabulated below:

Problem 3 (cwa final June 2000)A company has accepted an order for making 15 items of a specialized machine at a price of Rs. 4 lacs each. The delivery is to be completed within 4 months. The company works 23 days a month and the normal direct wages per day amount to Rs. 10,000. However, in case of need, the company can work overtime up to 8 days during the said period at double the normal rate of wages.

Overheads amount to Rs. 12,000 per normal working day but no overheads are charged on overtime working days. The material cost is Rs. 240,000 per machine. The company has estimated that it will take 10 working days to manufacture the first machine. The company is expected to experience a learning effect of 90% ( b = 0.152). The contract stipulates a penalty of Rs. 40,000 per machine delivered beyond the schedule of 4 months.

You are required to calculate the costs and advised the company whether it is preferable to work only during the normal working days and pay penalty for any delayed delivery of the machines or to work overtime to avoid paying penalty.

15 x 10 x 15 −0.152

14 x 10 x 14 −0.152

13 x 10 x 13 −0.152

Thus 13 machines can be supplied in time. For the balance 2 machines either normal working hours with penalty or overtime working without penalty is to be selected.

It is very easy to compare the two options and select the one without doing any complex calculation. In both the options the direct materials cost and the overheads cost are same and therefore become irrelevant for decision making. If overtime is not worked the penalty would be Rs. 80,000 and if overtime is worked the extra wages paid would be for 7.4 days @ Rs. 10,000 per day i.e. Rs. 74,000. Thus working overtime will result in saving of Rs. 6,000.

Page 506: cost accounting question icwai inter

Overheads @ Rs. 12,000/day for 92 days 11.04Total cost 57.72Sales revenue @ Rs. 4 lacs per machine 60.00Profit 2.28

Rs. LacsOption 1: No overtime and penalty is paid for delayed machines: 2.22Option 2 : Work overtime to avoid penalty 2.28Extra profit with working overtime 0.06

figures in rupeesDirect materials 3,000,000Direct labour 2,000,000 8000 hrs. @ Rs. 250Tooling costs 480,000Variable overheads 900,000 Proportional to direct labourGeneral overheads 1,200,000 Proportional to direct labourTotal 7,580,000Solution ;The learning curve index is not given. It is to be calculated as follows:Index b = log (learning %) / log 2 = log 0.85 / log 2 = − 0.2345.Cumulative units to be produced for this order = 400 (first lot) + 900 = 1,300 units.Per batch consists of 400 units hence aggregate 3.25 batches (1300/400) to be produced.Average hours for 3.25 batches as per the learning formula;A = time for initial lot x (no. of batch) − 0.2345 = 8,000 x 3.25 −0.2345 6,068 hoursTotal time in hours for 3.25 batch = 3.25 x 6,868 19,721 hoursLess time for initial batch 8,000 hoursIncremental time required for incremental 900 units 11,721 hoursAverage hours for one unit of incremental production of 900 units 13.02 hoursCost per unit : Cost per unit of second lot of 900 units Rs. Direct materials (30,00,000 / 400) 7,500Direct labour (13.02 Rs. 250) 3,255Tooling costs (absorbed in first lot) 0Variable overheads (45% of direct labour) 1,465General overheads (60% of direct labour) 1,953Total cost 14,173Selling price in Rs. (14,173/0.70) 20,247Profit @ 30% of Selling price 6,074

A company with two production departments has set the following standards for the forthcoming year:

DepartmentsS W

Direct labour hours available per period 6,000 4,000Standard wage rate per hour (Rs.) 6 5Expected learning curve 80% 70%Standard variable overhead per hour (Rs.) 9 5

Problem 4 (cwa final Dec. 2001)Dynamo, a manufacturer of aircraft parts, has been asked to bid for 900 units of a particular type of component. Four months ago the company had produced a first lot of 400 units for another customer. The cost details of this first lot are given below:

Repeated assembly of this type of component experiences a learning effect of 85%. The cost benefit of this will be reflected in the bid price. Dynamo follows a policy of setting the selling price to earn 30% gross margin. Tooling cost have been fully recovered from the first lot sold. Determine the selling price per unit for the second lot indented.

Problem 5 (cwa final Dec.97)

Page 507: cost accounting question icwai inter

Standard Fixed overheads per hour (Rs.) 12 8Direct labour hours required for first 100 units 18 9

Level of output (units) 100 200 800Prices of direct materials per unit of output (Rs.) 72.0 64.8 54.0

It is the policy of the company to add profit margin as under quoting the prices :Departments % on total labour & overhead costS 25%W 15%Subcontract work 5% on subcontract price

Required :

Solution:Tutorial Notes :

2. What is meant by ' when there are no capacity constraints ?' Where will it be relevant ?

1. Learning curve impact on direct labour hours: DepartmentsS total W total

Learning curve % 80% time 70% timeAverage time for total 100 units 18.0 1,800 9.0 900Average time for total 200 units 14.4 2,880 6.3 1,260Average time for total 400 units 11.5 4,608 4.4 1,764Average time for total 800 units 9.2 7,373 3.1 2,470

2. Available Hours and overtime requirements: S W Direct labour hours available 6,000 4,000

The direct materials are introduced in Department S. The company is able to negotiate the following prices for purchase of direct materials during the year:

Overtime, if required, is paid at time and a half. The overhead rates as given above does not include overtime premium.

The company has received a special order. Special tooling costs of the order amount to Rs. 1,200. If this order is for 200 units or less, it will be executed in the period which has a workload of 3840 direct labour hours in Department S and 2100 direct labour hours in Department W. For the work which is done in Department W, a subcontract price of Rs. 50 per unit is quoted by an associate company.

(i) If the company decides to get the work executed entirely within the company, what price, on cost plus basis, should be quoted for the order, if it consists of (a) 100 units (b) 200 units.

(ii) Assuming that the initial order placed by the customer is for 200 units, what lowest price should be quoted for a repeat order of 600 units ? Assume that this order will be executed when there are no capacity constraint.

(iii) State the output level at which the company should close down Department W to get the work executed through subcontractors.

1. The problem is where and how to begin the solution ? The learning % has been mentioned for both the departments. You can begin with this. First find out the total and incremental time required to produce 200 and 800 units. You can also commence the solution by computing the hours available in each department for production by deducting the existing workload from the total hours available.

3. The direct labour hours required for first 100 units have been given as 18 and 9 for dept. S and W respectively. Whether this is for 100 units or for one unit, It is not clear from the question. Looking to capacity and other parameters given in the question it can be concluded that it is on per unit basis.

4. Dept. W can be closed when the price offered by the subcontractor is lower than in-house production. You should prepare the cost of labour and overheads for different levels of production of 100, 200, 400, and 800 units and compare this with the offer presented by the subcontractor.

Page 508: cost accounting question icwai inter

Existing workload as given 3,840 2,100Balance hours available 2,160 1,900Hours required for 100 units 1,800 900Hours required for 200 units 2,880 1,260

3. Cost of Production : Department S

100 units (per unit) 200 units (per unit)Direct labour 18 hrs. x Rs. 6 108.0 108 x 80% 86.4Variable overheads 18 hrs. x Rs. 9 162.0 162 x 80% 129.6Fixed overheads 18 hrs. x Rs. 12 216.0 216 x 80% 172.8Overtime premium (50% of Rs. 6) 0.0 (720 x 3)/200 10.8

486.0 399.64. Cost of Production : Department W

100 units (per unit) 200 units (per unit)Direct labour 9 hrs. x Rs. 5 45.0 45 x 70%Variable overheads 9 hrs. x Rs. 5 45.0 45 x 70%Fixed overheads 9 hrs. x Rs.8 72.0 72 x 70%

162.05. Determination of Selling price 100 units (per unit) 200 units (per unit)

Direct materials 72.0 64.8Special tools (1200/100) 12.0 6.0Direct labour Dept. S 486.0 399.6Direct labour Dept. W 162.0 113.4

732.0 583.8Add: ProfitDept. S 25% of Rs. 486 121.5 25% of Rs. 399.6 99.9Dept. W 15% of Rs. 162 24.3 15% of Rs. 113.4 17.0Recommended Selling price Rs. 877.8 700.7

Answer part 2: If the initial order is for 200 units and capacity is no constraint:

Department S. 200 units (per unit) 800 units (per unit) Rs.

Direct Materials 64.8 54.0

Direct labour 86.4 86.4 x 0.8 x 0.8 55.3Variable overheads 129.6 129.6 x 0.8 x 0.8 82.9Department W.Direct labour 31.5 31.5 x 0.7 x 0.7 15.4Variable overheads 31.5 31.5 x 0.7 x 0.7 15.4

Rs. 343.8 Rs. 223.1Rs.

Cost of 800 units 800 x Rs. 223.1 178,480Cost of 200 units 200 x Rs. 343.8 68,760Incremental cost of 600 units 109,720Cost per unit (109,720/600) 182.9The company can quote Rs. 182.9 per unit for additional 600 units.Answer 3:

Thus if order is for 100 units no overtime premium would be required for either departments. If order is for 200 units overtime will be required for department S for (2,880 – 2,160) = 720 hours. The rate of overtime premium is 50% of labour rate i.e. Rs. 6/hr. Thus the overtime premium for 200 units would be Rs.6 x 50% x 720 = Rs. 2,160 i.e. Rs. 10.80 (2,160/200) per unit.

In this case no tooling costs need be considered as it is already recovered, as capacity is no constraint, the fixed cost element is also to be ignored for quotation purpose.

Page 509: cost accounting question icwai inter

Consider the direct labour and variable overhead for dept. W at different output as under:At 100 units Rs. 90 per unitAt 200 units Rs. 63 per unitAt 400 units Rs. 63 x 0.70 Rs. 44.4 per unitAt 800 units Rs. 63 x 0.70 x 0.70 Rs. 30.9 per unit

Required : Find out the time per unit of product to be used for quotation.Solution: You must have scientific calculator to solve this question.

We get the average time for the cumulative 200 units Y = 30 x 200 − 0.2345 = 8.66 hoursAverage time for the cumulative 199 units Y = 30 x 199 − 0.2345 = 8.67 hoursTotal time required for 200 units 200 x 8.56 1732.0 hoursTotal time required for 199 units 199 x 8.67 1725.33 hoursTime taken for 200th unit 6.67 hours

Solution: No. of units Total time Incremental timeNo. of Cumulative Average Cumulat. Batchunits units Time Time Time

1 1 4,000 4,000 4,000 For first unit1 2 3,600 7,200 3,200 For 2nd unit2 4 3,240 12,960 5,760 for 3rd and 4th unit4 8 2,916 23,328 10,368 for 5th,6th,7th and 8th units

Total time taken will be equal to 10,368 hours.

Direct Materials 5,000Direct labour 800 hrs @ Rs.5 4,000Overheads 150% of Direct labour 6,000

15,000Profit mark up 3,000Selling price 18,000

He has asked the company to clarify the following :(i) If he pays Rs. 18,000 for the first boat, what price would he have to pay for the second boat?

(ii) Can the company quote the same unit price for two boats, if he orders two at the same time?

The offer price of the subcontractor is Rs. 50 per unit. The company can consider it favorable upto a level of 200 units of production. Beyond this level the offer should not be accepted and it is better to continue with in-house production.

Problem 6 (cwa final Dec.06)A company is preparing a quotation for a new product. The time taken for the first unit is 30 hours. The company expects 85% learning curve (index is − 0.2345). The company desires that the quotation should be based on the time taken for the final output within the learning period which is expected to end after the company has produced 200 units.

Using the learning curve formula Y = a X b

Problem 7 (cwa final June 07)If the time taken to produce the first unit of a product is 4000 hours, What will be the total time taken to produce the 5th and 8th units of product, when a 90% learning curve applies ?

Problem 8 (cwa inter II June 99)Marine Engineering Ltd. builds pleasure boats made of fiber glass. The firm has now launched a new type of boat for the use of children. The cost data and selling price of the first boat built by the firm are given below

An 80% learning curve is expected to apply to this type of production. The Company's policy is to fix selling price at cost plus 20%. A customer has expressed interests in buying the boat but thinks that Rs.18,000 is too high a price to pay. He might want to buy two or even four of these boats during next six months.

Page 510: cost accounting question icwai inter

Solution 80% Learning curve timings if first unit takes 10 hours are as followsBatch No. of Cumulative Average Cumulative Batch

units units Time Time Time1 1 1 10.00 10.00 10.0 For first unit2 1 2 8.00 16.00 6.0 For 2nd unit3 2 4 6.40 25.60 9.6 for 3rd and 4th unit4 4 8 5.12 40.96 15.4 for 5th,6th,7th and 8th units

Thus time taken for production of 8 units with 80% learning curve would be 40.96 hours.Average cost per boat for different levels of production by applying learning curveNo. of boats 1 2 4 5Cost fig. in Rs.Direct materials 5,000 5,000 5,000 5,000Direct labour in hrs 800 640 512 409.6Direct labour @ Rs. 5 4,000 3,200 2,560 2,048Overheads 150% of direct labour 6,000 4,800 3,840 3,072Total cost 15,000 13,000 11,400 10,120Profit mark up (20%) 3,000 2,600 2,280 2,024Selling price 18,000 15,600 13,680 12,144 (1) Price to be paid for the second boat: Rs.Price of first two boats @ Rs. 15,600 31,200Less : Price paid for the first boat 18,000Price to be paid for the second boat: 13,200(2) Unit price per boat for the 1st and 2nd boats taken together 15,600(3) Average price for four boats Figures in Rs. 13,680Hence total price to be paid for them 54,720Less : Price paid for the first two boats (15,600 x 2) 31,200

Price to be paid for 3rd and 4th boats 23,520Unit price for 3rd and 4th boats (23,520/2) 11,760

(4) (i) Unit price to be paid per boat if four boats were ordered in one lot 13,680(4) (ii) Unit price to be paid per boat if eight boats were ordered in one lot 12,144

Order No. of fire-alarm systemsFirst 100Second 60Third 40

The firm estimates the following cost per unit for the first order:Direct materials Rs. 500Direct labourDeptt. A (Highly automatic) 20 hours at Rs. 10 per hourDeptt. B (Skilled labour) 40 hours at Rs. 15 per hourVariable overheads : 20% of direct labourFixed Overheads absorbed :

Deptt. A Rs. 8 per hour

(iii) If he buys two boats now at the same unit price, what would be the price for the third and fourth boats, if he orders them separately later on?

(iv) Can the company quote a single unit price for (1) four boats (2) eight boats, if they are all ordered now? Assuming there is no other customer now, what replies will the company give? Show workings.

Problem 9 (ca final may 2001)An electronic firm which has developed a new type of fire-alarm system has been asked to quote for a prospective contract. The customer requires separate price quotations for each of the following possible orders:

Page 511: cost accounting question icwai inter

Deptt. B Rs. 5 per hour

X 1.0 1.3 1.4 1.5 1.6 1.7 1.8 1.9 2.0Y 100.0 91.7 89.5 87.6 86.1 84.4 83 81.5 80.0

X represents the cumulative total volume produced to date expressed as a multiple of the initial order.

Y is the learning curve factor, for a given value of X, expressed as a percentage of the cost of the initial order.

Solution :

Answer (1) Computation of Price for the first order of 100 units Rs.

Direct materials 500Direct labourDeptt. A (Highly automatic) 20 hours at Rs. 10 per hour 200Deptt. B (Skilled labour) 40 hours at Rs. 15 per hour 600 800Variable overheads : 20% of direct labour 160Fixed Overheads absorbed :

Deptt. A Rs. 8 per hour 160Deptt. B Rs. 5 per hour 200 360

Total Cost 1,820Add : Mark up @ 25% 455Selling price per unit 2,275

(2) Computation of Price for the second order of 60 units Learning effect will be experienced only in deptt. B.

Cumulative output ( 100 + 60) = 160 units i.e. 1.6 times for which learning is 86.1%.Therefore total hours for 160 units = 160 units x 40 x 0.861 = 5,510 hoursTherefore total hours for 60 units = time for 160 units less time for 100 unitsTime for 160 units 5,510 hoursTime for 100 units (100 x 40 hours of deptt. B) 4,000 hoursTime for 60 units 1,510 hoursTime for one unit of second order (1510/60) 25.17 hours

Computation of Selling price for the second order of 60 units Rs.

Direct materials 500Direct labourDeptt. A 20 hrs @ Rs. 10 200Deptt. B 25.17 hrs @ Rs. 15 377 577Variable overheads :20% of direct labour 115Fixed Overheads absorbed :

Deptt. A Rs. 8 per hour 160Deptt. B Rs. 5 per hour 126 286

Total cost 1,478Add : 25% Mark up 370Selling price per unit 1,848

(2) Computation of Price for the third order of 40 units Cumulative output : 100 + 60 + 40 = 200 unitsCumulative output is 2 times the initial order for which learning is 80% from the tableTotal hours for 200 units = 200 x 40 x 0.80 6,400 hours

Determine a price per unit for each of the three orders, assuming the firm uses a mark up of 25% on total costs and allows for an 80% learning curve. Extract from 80% Learning curve table:

Deptt. A is highly automatic hence learning curve is not applicable to it. Learning effect will be effective only in respect of deptt. B.

Page 512: cost accounting question icwai inter

Less : Total hours for 160 units as computed earlier 5,510 hoursTime for 40 units 890 hours Time for 1 unit in hour = 890 / 40 = 22.25 hours.

Computation of Selling price for the second order of 60 units Rs.

Direct materials 500Direct labourDeptt. A 20 hrs @ Rs. 10 200Deptt. B 22.25 hrs @ Rs. 15 334 534Variable overheads :20% of direct labour 107Fixed Overheads absorbed :

Deptt. A Rs. 8 per hour 160Deptt. B Rs. 5 per hour 111 271

Total cost 1,412Add : 25% Mark up 353Selling price per unit 1,765

Problem 10 (ca final may 2008)

Direct material 30 Rs. / unit Direct labour (1 hour / unit) 20 Rs. / unitVariable overhead 10 Rs. / unit Fixed overheads at maximum capacity is Rs. 1,50,000.

price acceptable for an order of 15,000 units for a prospective client?

Solution : 5000 units 20,000 unitsDirect materials 1.50 6.00Direct labour 1.00 2.56Variable overhead 0.50 2.00Total variable cost 3.00 10.56Fixed cost 1.5 1.5Total Cost 4.50 12.06Total cost Rs. / unit 90 60.3Selling price 100

Rs.Total cost for 20,000 units ( 12.06 − 4.50) = Rs. 7.56 lacs. 1206000Revenue received for first 5000 units 500,000Selling price for 15,000 units must be quoted to cover the cost 706,000Selling price to cover the cost = 706,000 / 15,000 47.1

Problem 11 (CWA Final June 05)

The supplier's cost break-up for this is :Materials : Rs. 2,500 ; Labour : Rs. 1,000 ; Overheads : Rs. 1,500; Total cost : Rs. 5,000.Profit @ 15% : Rs. 750; Selling Price : Rs. 5,750.

M Ltd. Manufactures a special product purely carried out by manual labour. It has a capacity of 20,000 units. It estimates the following cost structure:

It is estimated that at the current level of efficiency, each unit requires one hour for the first 5,000 units. Subsequently it is possible to achieve 80% learning rate. The market can absorb the first 5,000 units at Rs.100 per unit. What should be the minimum selling

Small problems small achievements. Big problems big achievements. What do you select ?

We have agreed to pay 15% mark up price on cost to Reliabl & Company, our new supplier of 120 nos. of TV. table, to be delivered @ 10 tables in the 2nd week of each month. The tables will be produced in twelve batches to fit in the production schedule. Production time for a batch is 2 days.

Page 513: cost accounting question icwai inter

Solution : Tutorial Notes 1. The learning % has been given but learning index is not given. You need scientific calculator to solve this problem.2. The learning index (for 90%) = log (0.90) / log (2) = – 0.152

= 685.40

(Y = Average time taken by X units and a is the time taken by the first unit.)With the 90% learning effect on labour hours, the price for the 120 tables will be: (in Rs.)Materials 2,500 x 120 300,000Labour 685.4 x 120 82,248Overheads 1,500 x 120 180,000

562,248Mark up 15% on cost 84,337

646,585Less: Payment for first 40 tables @ Rs. 5,750 230,000Balance payable 416,585Problem 12 (CWA Final June 06) A company is asked to quote for a special order to be delivered ex-works.Direct internal costs per unit of output are :For a total of : 100 : Rs. 18 each; 200 : Rs. 18 less 10% discount; 400 : Rs. 18 less 20% discount.The work will be done in departments F and G.

Department F Department GVariable (Rs.) 1.0 1.0Fixed (Rs.) 3.5 2.0Based on a budgeted level of (a) 3,000 direct labour hours per period (b) 2,000 direct labour hours per period.

Department F : 20% ; Department G : 10% ; Direct material : 2% and Sub-contractor's work : 2%.

You are required to calculate :(a) the price per unit of an order of 100 units if made entirely in the company;(b) the price per unit of an order of 200 units if made entirely in the company;

After having paid for four deliveries of 40 units of table, we learn that this new type table, we learn that this new type table to subject to a learning curve effect of 90% on labour hours. By negotiations, the supplier now agrees to make price correction for all the 120 items. What amount shall we pay to the supplier upon delivery of the balance 80 units?

3. the average time taken is given by the formula : Y = a X index Y = 1000 x 12 – 0.152

Department F employs highly skilled operations paid at Rs. 2.50 per hour. Each unit of output requires 6 direct labour hours of work for the first 100 units. However, experience has shown that an 80% learning curve can be expected to operate.

Department G employs skilled operators paid at Rs. 2.0 per hour. Each unit of output requires 3 direct labour hours of work for the first 100 units. Here, too an 80% learning curve is expected.

Overtime is in either department is paid at time and a half. No premium for overhead is included in standard manufacturing overhead. Standard manufacturing overhead per direct labour hour is as follows:

The special order will require special tooling of Rs. 300 which is chargeable to the customer. If the order received is for 100 or 200 units, the work will have to be done in period no. 8, which for department F, is already loaded with 2,200 direct labour hours of work. Department G, however, will be working at only around 55% capacity.

On special orders of this type, it is the company's practice to add the following margins in cost in arriving at selling prices :

An outside sub-contractor has offered, irrespective of the size of order, to do the work of department G on this order for a price of Rs. 8 per unit, including collection from and delivery to the works.

(c) a separate price per unit for an extra 200 units subsequent to the order for 200 in (b) above, thus bringing the total order to 400 units; (N.B. you are to assume that)

(i) this additional order for the extra 200 units would be done when there are no capacity limitations in the either departments;

Page 514: cost accounting question icwai inter

(ii) the materials supplier would give the full discount for the 400 units.

Solution : See problem 5 above. Key answers are supplied.(a) price per unit : Rs. 88.26 (b) Price per unit : Rs. 72.74.(c) Differential cost of 200 units : Rs. 8,992 (23,301 + 200 + 40 – 14,549). Rs./ unit : 44.96.

Problem 13 (CWA Final June 04)

Budgeted costs are : Direct materials Rs. 2,000 per unit.

Variable production overhead : 60% of direct wages.

You are required to :

→ → → → → → → → → →→ → → → → → → → → →Solution :

Year Prodn. Cum.prod. Basic man Learning Cum. Hrs. Cum.annualunits units hours (2335 x c) Curve (d x e) hours

(a) (b) (c) (d) (e) (f) (h)1 20 20 46,700 38% 17,807 17,8072 40 60 140,100 27% 37,505 (37,505 – 17,807) 19,6983 40 100 233,500 23% 53,028 (53,028 – 37,505) 15,5234 20 120 280,200 21% 59,991 (59,991 – 53,028) 6,963

Year Prodn. Hours Wages @ Rs. 5.5 Material Overhead Fixed overhead 1 20 (500x20) 10,000 55,000 40,000 33,000 20,0002 40 (500x40) 20,000 110,000 80,000 66,000 20,0003 40 (500x40) 20,000 110,000 80,000 66,000 20,0004 20 (500x20) 10,000 55,000 40,000 33,000 20,000

60,000Evaluation of the project under NAP method: (On average annual hours)

Year Sales Cost Cash flow Pv factor Pv

(d) the change in unit selling price that would result from using the outside sub-contractor instead of department G for an order of (i) 100 units (ii) 200 units (iii) 400 units.

(d) Change in price : (i) for 100 units : reduce by Rs. 1.74 (ii) for 200 units : increase by Re.0.24 (iii) for 400 units : increase by Rs. 1.82.

A company is considering an investment in a project that will produce one type of product whose total budgeted sales are 120 units at a selling price of Rs. 8,500 per unit.

Sales are to be 20 units in year 1, 40 units in each of year 2 and year 3, and the balance in year 4. Production will be in line with sales.

Direct wages : the first unit will take 2,335 man-hours of work at a wage rate of Rs. 5.50j per man-hour. An 80%learning curve is expected to operate.

Fixed overhead relating to the product is Rs. 47,500 per annum, including depreciation on new plant to be purchased for the project which is charged on a straight line method over its life of 4 years. The new plant will cost Rs. 110,000 payable on day 1 of year 1. It will have no residual value. On investment of this type, the company requires a return of 16% DCF on the fixed asset investment only.(a) recommend, with supporting calculations, whether the company should proceed with the project if either the following bases of calculation were used over the 4 years:- (i) the total quantity of 120 units produced and sold; or (ii) a year-by-year production / sale schedule.

(b) recommend, with your reasons which of bases (a) (i) or (a) (ii) above should be used as the criterion for the company's decision. Ignore tax and inflation.

Note : An 80% learning curve on ordinary graph paper would show the following relationship between the X-axis and Y-axis (cumulative average cost of elements subject to the learning curve) :

Annual hours are required to be calculated to compute the cost per year, on the basis of which year-to-year production / sales schedule can be prepared for answer a (ii).

Annual hours = Average hour for all the units produced x units produced during the year. Average hour for all the units = 59,991 / 120 = 500 hrs.

Page 515: cost accounting question icwai inter

0 (110,000) 1.000 (110,000)1 170,000 148,000 22,000 0.862 18,9642 340,000 276,000 64,000 0.743 47,5523 340,000 276,000 64,000 0.641 41,0244 170,000 148,000 22,000 0.552 12,144

848,000 9,684 NPV is positive. Recommended.Evaluation of the project under NPV method: (On cumulative hours)

Year Prodn. Hours Wages @ Rs. 5.5 Material Overhead Fixed overhead 1 20 17,807 97,939 40,000 58,763 20,0002 40 19,698 108,339 80,000 65,004 20,0003 40 15,523 85,377 80,000 51,226 20,0004 20 6,963 38,296 40,000 22,978 20,000

59,991Year Sales Cost Cash flow Pv factor Pv

0 (110,000) 1.000 (110,000)1 170,000 216,702 (46,702) 0.862 (40,257)2 340,000 273,343 66,657 0.743 49,526 3 340,000 236,603 103,397 0.641 66,277 4 170,000 121,274 48,726 0.552 26,897

847,922 (7,556) NPV is negative. Not recommended.

Method a (ii) is to be relied upon and the project is not recommended for acceptance.

Method a (i) shows NPV of project as positive while method a (ii) shows the NPV to be negative. Method a (ii) is more accurate as it takes into account the learning effect into account. Learning curve effect implies that '' more a job is repeated, the more experience is gained by the workers. Time taken in earlier units is more than those units which are completed in later period."'

·       le; cgqr lh leL;kvksa dks lqy>k nsrk gS- le; ml leL;k dks Hkh lqy>k ldrk gS tks vkt vkidks fpafrar dj jgh gS-

Page 516: cost accounting question icwai inter

A company wants to manufacture a new product against order. The initial trials showed that the first unit would take 10 hours @ Rs. 15 per hour and that the operations would be subjected to a learning curve of 80%. The cost of material per unit is Rs. 200 and the overheads amount to 150% of labour cost. The first order received is for 8 units of the

A firm received a trial order for manufacture and supply of four units of a certain component. The cost of material required per unit is Rs. 80. The first unit required 10 hours of labour @ Rs. 10/hour and the manufacturing process is subject to 80% learning effect. Overheads are absorbed at 150% of labour and the firm takes 20% on the price

on the benefit of learning to the customer, what price per unit should be

Page 517: cost accounting question icwai inter

From the above analysis, it is evident that the first four units of trial order takes 25.6 hours for manufacture and the

A company has accepted an order for making 15 items of a specialized machine at a price of Rs. 4 lacs each. The delivery is to be completed within 4 months. The company works 23 days a month and the normal direct wages per day amount to Rs. 10,000. However, in case of need, the company can work overtime up to 8 days during the said

Overheads amount to Rs. 12,000 per normal working day but no overheads are charged on overtime working days. The material cost is Rs. 240,000 per machine. The company has estimated that it will take 10 working days to manufacture the first machine. The company is expected to experience a learning effect of 90% ( b = 0.152). The contract stipulates a penalty of Rs. 40,000 per machine delivered beyond the schedule of 4 months.

You are required to calculate the costs and advised the company whether it is preferable to work only during the normal working days and pay penalty for any delayed delivery of the machines or to work overtime to avoid paying

Thus 13 machines can be supplied in time. For the balance 2 machines either normal working hours with penalty or

It is very easy to compare the two options and select the one without doing any complex calculation. In both the options the direct materials cost and the overheads cost are same and therefore become irrelevant for decision making. If overtime is not worked the penalty would be Rs. 80,000 and if overtime is worked the extra wages paid would be for 7.4 days @ Rs. 10,000 per day i.e. Rs. 74,000. Thus working overtime will result in saving of Rs. 6,000.

Page 518: cost accounting question icwai inter

A company with two production departments has set the following standards for the forthcoming year:

Dynamo, a manufacturer of aircraft parts, has been asked to bid for 900 units of a particular type of component. Four months ago the company had produced a first lot of 400 units for another customer. The cost details of this first lot are

Repeated assembly of this type of component experiences a learning effect of 85%. The cost benefit of this will be reflected in the bid price. Dynamo follows a policy of setting the selling price to earn 30% gross margin. Tooling cost have been fully recovered from the first lot sold. Determine the selling price per unit for the second lot indented.

Page 519: cost accounting question icwai inter

The direct materials are introduced in Department S. The company is able to negotiate the following prices for

Overtime, if required, is paid at time and a half. The overhead rates as given above does not include overtime

The company has received a special order. Special tooling costs of the order amount to Rs. 1,200. If this order is for 200 units or less, it will be executed in the period which has a workload of 3840 direct labour hours in Department S and 2100 direct labour hours in Department W. For the work which is done in Department W, a subcontract price of

(i) If the company decides to get the work executed entirely within the company, what price, on cost plus basis, should

(ii) Assuming that the initial order placed by the customer is for 200 units, what lowest price should be quoted for a repeat order of 600 units ? Assume that this order will be executed when there are no capacity constraint.

(iii) State the output level at which the company should close down Department W to get the work executed through

1. The problem is where and how to begin the solution ? The learning % has been mentioned for both the departments. You can begin with this. First find out the total and incremental time required to produce 200 and 800 units. You can also commence the solution by computing the hours available in each department for production by

3. The direct labour hours required for first 100 units have been given as 18 and 9 for dept. S and W respectively. Whether this is for 100 units or for one unit, It is not clear from the question. Looking to capacity and other parameters

4. Dept. W can be closed when the price offered by the subcontractor is lower than in-house production. You should prepare the cost of labour and overheads for different levels of production of 100, 200, 400, and 800 units and

Page 520: cost accounting question icwai inter

200 units (per unit)31.531.550.4

113.4200 units (per unit)

Thus if order is for 100 units no overtime premium would be required for either departments. If order is for 200 units overtime will be required for department S for (2,880 – 2,160) = 720 hours. The rate of overtime premium is 50% of labour rate i.e. Rs. 6/hr. Thus the overtime premium for 200 units would be Rs.6 x 50% x 720 = Rs. 2,160 i.e. Rs.

In this case no tooling costs need be considered as it is already recovered, as capacity is no constraint, the fixed cost

Page 521: cost accounting question icwai inter

(i) If he pays Rs. 18,000 for the first boat, what price would he have to pay for the second boat?

The offer price of the subcontractor is Rs. 50 per unit. The company can consider it favorable upto a level of 200 units of production. Beyond this level the offer should not be accepted and it is better to continue with in-house production.

A company is preparing a quotation for a new product. The time taken for the first unit is 30 hours. The company expects 85% learning curve (index is − 0.2345). The company desires that the quotation should be based on the time taken for the final output within the learning period which is expected to end after the company has produced 200

If the time taken to produce the first unit of a product is 4000 hours, What will be the total time taken to produce the 5th

Marine Engineering Ltd. builds pleasure boats made of fiber glass. The firm has now launched a new type of boat for the use of children. The cost data and selling price of the first boat built by the firm are given below

An 80% learning curve is expected to apply to this type of production. The Company's policy is to fix selling price at cost plus 20%. A customer has expressed interests in buying the boat but thinks that Rs.18,000 is too high a price to

Page 522: cost accounting question icwai inter

for 5th,6th,7th and 8th units

(iv) Can the company quote a single unit price for (1) four boats (2) eight boats, if they are all ordered now? Assuming

An electronic firm which has developed a new type of fire-alarm system has been asked to quote for a prospective contract. The customer requires separate price quotations for each of the following possible orders:

Page 523: cost accounting question icwai inter

Y is the learning curve factor, for a given value of X, expressed as a percentage of the cost of the initial order.

Determine a price per unit for each of the three orders, assuming the firm uses a mark up of 25% on total costs and

Deptt. A is highly automatic hence learning curve is not applicable to it. Learning effect will be effective only in respect

Page 524: cost accounting question icwai inter

M Ltd. Manufactures a special product purely carried out by manual labour. It has a capacity of 20,000 units. It

It is estimated that at the current level of efficiency, each unit requires one hour for the first 5,000 units. Subsequently it is possible to achieve 80% learning rate. The market can absorb the first 5,000 units at Rs.100 per unit. What should

We have agreed to pay 15% mark up price on cost to Reliabl & Company, our new supplier of 120 nos. of TV. table, to be delivered @ 10 tables in the 2nd week of each month. The tables will be produced in twelve batches to fit in the

Page 525: cost accounting question icwai inter

1. The learning % has been given but learning index is not given. You need scientific calculator to solve this problem.

Based on a budgeted level of (a) 3,000 direct labour hours per period (b) 2,000 direct labour hours per period.

After having paid for four deliveries of 40 units of table, we learn that this new type table, we learn that this new type table to subject to a learning curve effect of 90% on labour hours. By negotiations, the supplier now agrees to make price correction for all the 120 items. What amount shall we pay to the supplier upon delivery of the balance 80 units?

Department F employs highly skilled operations paid at Rs. 2.50 per hour. Each unit of output requires 6 direct labour hours of work for the first 100 units. However, experience has shown that an 80% learning curve can be expected to

Department G employs skilled operators paid at Rs. 2.0 per hour. Each unit of output requires 3 direct labour hours of

Overtime is in either department is paid at time and a half. No premium for overhead is included in standard

The special order will require special tooling of Rs. 300 which is chargeable to the customer. If the order received is for 100 or 200 units, the work will have to be done in period no. 8, which for department F, is already loaded with 2,200 direct labour hours of work. Department G, however, will be working at only around 55% capacity.

On special orders of this type, it is the company's practice to add the following margins in cost in arriving at selling

An outside sub-contractor has offered, irrespective of the size of order, to do the work of department G on this order

(c) a separate price per unit for an extra 200 units subsequent to the order for 200 in (b) above, thus bringing the total

(i) this additional order for the extra 200 units would be done when there are no capacity limitations in the either

Page 526: cost accounting question icwai inter

→ X 1 2 10 20 30 40 50 60 70 80 90 100 110 120

→ Y % 100 80 47.65 38.13 33.46 30.50 28.39 26.77 25.48 24.40 23.50 22.71 22.03 21.41

Total cost148,000276,000276,000148,000848,000

(d) the change in unit selling price that would result from using the outside sub-contractor instead of department G for

(d) Change in price : (i) for 100 units : reduce by Rs. 1.74 (ii) for 200 units : increase by Re.0.24 (iii) for 400 units :

A company is considering an investment in a project that will produce one type of product whose total budgeted sales

Sales are to be 20 units in year 1, 40 units in each of year 2 and year 3, and the balance in year 4. Production will be

Direct wages : the first unit will take 2,335 man-hours of work at a wage rate of Rs. 5.50j per man-hour. An

Fixed overhead relating to the product is Rs. 47,500 per annum, including depreciation on new plant to be purchased for the project which is charged on a straight line method over its life of 4 years. The new plant will cost Rs. 110,000 payable on day 1 of year 1. It will have no residual value. On investment of this type, the company requires a return of

(a) recommend, with supporting calculations, whether the company should proceed with the project if either the following bases of calculation were used over the 4 years:- (i) the total quantity of 120 units produced and sold; or (ii) a

(b) recommend, with your reasons which of bases (a) (i) or (a) (ii) above should be used as the criterion for the

Note : An 80% learning curve on ordinary graph paper would show the following relationship between the X-axis and

Annual hours are required to be calculated to compute the cost per year, on the basis of which year-to-year production

Annual hours = Average hour for all the units produced x units produced during the year. Average hour for all the units

Page 527: cost accounting question icwai inter

Total cost216,702273,343236,603121,274847,922

Method a (i) shows NPV of project as positive while method a (ii) shows the NPV to be negative. Method a (ii) is more accurate as it takes into account the learning effect into account. Learning curve effect implies that '' more a job is repeated, the more experience is gained by the workers. Time taken in earlier units is more than those units which are

Page 528: cost accounting question icwai inter
Page 529: cost accounting question icwai inter
Page 530: cost accounting question icwai inter
Page 531: cost accounting question icwai inter
Page 532: cost accounting question icwai inter
Page 533: cost accounting question icwai inter
Page 534: cost accounting question icwai inter
Page 535: cost accounting question icwai inter
Page 536: cost accounting question icwai inter
Page 537: cost accounting question icwai inter
Page 538: cost accounting question icwai inter
Page 539: cost accounting question icwai inter